Wikipedia:Auskunft/Archiv/2014/Woche 37

aus Wikipedia, der freien Enzyklopädie
Zur Navigation springen Zur Suche springen


Dies ist ein Archiv der Seite Wikipedia:Auskunft. Es enthält alle Abschnitte, die in der Kalender-Woche 37 im Jahr 2014 begonnen wurden.

Möchtest du in einer neuen Diskussion auf dieses Archiv verweisen? Nutze folgenden Link und ersetze ________ durch den Namen des Abschnittes. Die Nummer im Inhaltsverzeichnis gehört nicht dazu:

[[Wikipedia:Auskunft/Archiv/2014/Woche 37#________]]


← vorherige Woche Gesamtarchiv nächste Woche →

Strom, fließt.

Moin,

unser Praktikant hat gerade ein Layout für eine Schaltung gemacht (Platine mit EAGLE entworfen). Legt er das dem Chef vor, der kuckt und sagt:"Alles prima, alles da, funktioniert. Nur hier musst Du was ändern!" und weist auf eine im Winkel von 135° abknickende Leiterbahn, also spitz zurück. Fragt der Praktikant:"OK, aber warum denn?" Die Erklärung war... unbefriedigend. Da der Strom fließe, habe er es nicht so gerne, um enge Kurven zu verlaufen, wie in einer Wasserleitung gäbe es dann Verluste. Ich fragte dann noch etwas genauer nach, fand die Erklärung irgendwie esoterisch, die Analogie überstrapaziert, das mochte Prof. Dr. dann wiederum nicht hören; es lief auf ein "Das ist eben so!" hinaus. Daher: Ist es erstens so, daß spitze Winkel bei Leiterbahnen vermieden werden sollen und zweitens, wenn ja, warum ist das so? Gr., redNoise (Diskussion) 16:59, 8. Sep. 2014 (CEST)

Fleißt da Wechselstrom? Wenn ja wie Hochfrequent? --Mauerquadrant (Diskussion) 17:11, 8. Sep. 2014 (CEST)
Sihe auch: http://www.mikrocontroller.net/articles/Richtiges_Designen_von_Platinenlayouts
(BK) Nein, das wird ein I/O-Board + ein wenig Stromversorgung ist auch noch unterwegs. Gr., redNoise (Diskussion) 17:18, 8. Sep. 2014 (CEST)
Sünde Nr. 2. --Pp.paul.4 (Diskussion) 17:23, 8. Sep. 2014 (CEST)
Das geht hier ja schneller als... Pp.Paul.4, dem von Dir angegebenen Link bin ich gerade schon begegnet, dennoch besten Dank. Auch dem Nicht-mal-IP-Vorposter meinen Dank, dort stieß ich auf den Link. Ich habe verstanden und werde meinen Chef belehren ;-) Gr., redNoise (Diskussion) 17:29, 8. Sep. 2014 (CEST)
Archivierung dieses Abschnittes wurde gewünscht von: --Eike (Diskussion) 18:15, 8. Sep. 2014 (CEST)

Kann man dem Islam der Status einer Religion aberkennen?

Angestrebt ist ein Verbotsverfahren beim Bundesverfassungsgericht, ohne in Konflikt mit der Religionsfreiheit zu kommen.--BrigITTIchen (Diskussion) 21:57, 8. Sep. 2014 (CEST)

Nein, das geht nicht, siehe Art. 4 GG. --Rôtkæppchen₆₈ 22:06, 8. Sep. 2014 (CEST)
Für das Verbot einzelner islamischer Religionsgesellschaften (Vereine, Kirchen etc) sind die Innenministerien der jeweligigen Bundesländer zuständig. Über ein Vereinsverbotsverfahren können aber nur einzelne Religionsgesellschaften verboten werden, nicht die Religion an sich. Laut Vereinsgesetz (Deutschland) ist das Bundesverfassungsgericht nur für das Verbot von Parteien zuständig. --Rôtkæppchen₆₈ 22:10, 8. Sep. 2014 (CEST)
Verboten werden kann keine Weltanschauung, weil die Gedanken frei sind. Aber deren außere Darstellung liese sich verbieten. Man könnte prinzipiell den Besitz des Koran oder das Ausrufen von Allahu Akbar bestrafen (wie es etwa bei Kinderpornos oder Sieg Heil der Fall ist). Geht man hier weit genug, käme es einem Verbot des Islam nahe (wie es beim Nationalsozialismus der Fall ist).--Antemister (Diskussion) 22:25, 8. Sep. 2014 (CEST)
Ein derartiges Verbot hätte keinerlei Chance, wäre sehr schnell vom Bundesverfassungsgericht kassiert. Jeder Richter oder Innenminister, der ein derartiges Verbot erlässt, würde damit große berufliche Probleme bekommen. --Rôtkæppchen₆₈ 22:42, 8. Sep. 2014 (CEST)
Nun ja. Eine wirklich dumme Frage. Islam, eine Weltreligion. Mit der gleichen Naivität könnte man fragen, ob man dem Christentum "den Status einer Religion aberkennen" kann. --94.217.19.6 23:12, 8. Sep. 2014 (CEST)
Man könnte fragen ob man %RELIGION% den Status einer Religion aberkennen könnte. Religionen sind aber grundgesetzlich geschützt. Also nein. Es gibt aber durchaus Grenzfälle, siehe S*********y#Religionscharakter. --Rôtkæppchen₆₈ 23:26, 8. Sep. 2014 (CEST)
Gemeint sein dürfte Scientology#Religionscharakter. --88.130.114.41 23:51, 8. Sep. 2014 (CEST)
Archivierung dieses Abschnittes wurde gewünscht von: Offensichtliche Trollfrage. --AndreasPraefcke (Diskussion) 22:32, 8. Sep. 2014 (CEST)

Abrufstatistik (Seitenaufrufe)

Ähm ja, hat das Statistikprogramm seit dem 2. September die Arbeit eingestellt ? --Beademung (Diskussion) 15:13, 9. Sep. 2014 (CEST)

Hinweis: Für Fragen speziell zur Wikipedia gibt es ein eigenes Diskussionsforum: Fragen zur Wikipedia. Siehe Wikipedia:Fragen zur Wikipedia#Abrufstatistik ist tot. XenonX3 – () 15:20, 9. Sep. 2014 (CEST)

Meinst Du http://stats.grok.se? Versuchs mal mit https://tools.wmflabs.org/wikiviewstats/de/ . 217.230.89.210 15:20, 9. Sep. 2014 (CEST)

Funktioniert, Danke für die Hilfe. -- Beademung (Diskussion) 15:49, 9. Sep. 2014 (CEST)
Archivierung dieses Abschnittes wurde gewünscht von: --Eike (Diskussion) 16:04, 9. Sep. 2014 (CEST)

Grammatik feat. Bayerischer Rundfunk

Hallo. Vor einigen Tagen habe ich den Artikel zu Marmor, Stein und Eisen bricht gelesen. Dort werden auch die Auseinandersetzungen um die Titelzeile - "bricht" statt "brechen" - erklärt. Dann heißt es: (...) Wegen dieses grammatikalischen Fehlers durfte das Lied im Bayerischen Rundfunk lange Zeit nicht gespielt werden." Nun die Frage: Kann das wirklich stimmen? Die Geschichte klingt zwar sehr lustig, aber doch auch ziemlich unwahrscheinlich. Klar, es geht um den BR, aber trotzdem... Ich habe schon selbst nach einem Beleg gesucht, aber über Google bisher nichts finden können, abgesehen von der Behauptung, dass es so gewesen sein soll. Zweifeln läßt mich vor allem, dass in keinem Artikel ein konkretes Zitat genannt wird. Es wird auch kein Verantwortlicher des BR erwähnt, der diese Aussage gemacht haben soll. Wenn es wirklich eine offizielle Begründung in diese Richtung gegeben hat, würde ich sie nur zu gerne mal lesen. Könnt Ihr mir helfen? Vielen Dank schon jetzt.--Guglhupf88 (Diskussion) 08:36, 8. Sep. 2014 (CEST)

Solange eine starke Aussage nicht belegt ist, darf man sie auch gerne herausnehmen und in der Disk. parken.... GEEZER… nil nisi bene 08:39, 8. Sep. 2014 (CEST)
So, ist drin. Dam Dam; Dam Dam... GEEZER… nil nisi bene 08:48, 8. Sep. 2014 (CEST)
Diese Geschichte höre ich auch schon seit über 20 Jahren. Allerdings handelt es sich noch nicht einmal um einen Fehler (Duden-Grammatik, §§ 1602(ii), 1605 ff.). Solche Geschichten hört man über den Bayerischen Rundfunk ja öfters: Manchmal sind sie historisch (Beispiele), manchmal aus den historischen Tatsachen extrapolierte Potenz wie möglicherweise in diesem Falle. --93.219.62.94 09:11, 8. Sep. 2014 (CEST)
Danke für die schnelle Ergänzung im Artikel. Als Referenz reicht der Welt-Artikel sicher, zumal sich der Autor ja offensichtlich gut im Leben Drafi Deutschers auskennt. Trotzdem kann ich mir immer noch vorstellen, dass es eine urban legend ist, die mittlerweile einfach zu häufig erzählt wurde, um sie noch zu überprüfen. Wahrscheinlich müsste ich schon beim BR selbst anrufen, um ganz sicher zu gehen.--Guglhupf88 (Diskussion) 09:27, 8. Sep. 2014 (CEST)
Ich gestehe, dass ich bei der Referenzierung ein Kribbeln im ... Kopf habe. Das findet sich so massiv in Blogs etc. dass es für einen Journalisten ein leckeres Gewürz in einem Nachruf ist. Es wäre solider, wenn man eine zeitnahe Ref. finden könnte.
Schreibst du den BR an - oder soll ich es - mit der gesamten Wucht der akademischen Titel - tun? ;-) GEEZER… nil nisi bene 09:35, 8. Sep. 2014 (CEST)
Anrufen würde nicht helfen. Selbst wenn da noch jemand wäre, der das noch wüsste und es auch zugeben würde, wäre es als Referenz für die Wikipedia ungeeignet. Ein Bestätigungsschreiben des BR wäre auch nicht ganz so einfach als valider Beleg zu verwenden, aber schon wertvoller.
Falsch an der Meldung ist meiner Ansicht nach nicht der Zusammenhang mit dem angeblichen Grammatikfehler, sondern das Wort "Verbot". Der Bayrische Rundfunk spielte das Lied eben einfach nicht. Ein Verbot wäre das nur dann, wenn irgendein Moderator es hätte spielen wollen und es ihm untersagt worden wäre, oder wenn es eine allgemeine Anweisung gäbe, genau dieses Lied nicht zu spielen, oder wenn es eine "schwarze Liste" gegeben hätte, auf der das Lied stand. Für all das gibt es nicht den Hauch eines Belegs, all das wird ja noch nicht einmal behauptet. Es wird nur - von zahlreichen Autoren, Wikipedia-Autoren eingeschlossen - eine jahrzehntealte Floskel wiederholt. Als Fakt hat das m.E. im Artikel nichts verloren, und der Beleg ist in meinen Augen ungeeignet, da er keine Quelle ist, sondern auch nur eine Wiederholung der Behauptung ohne eigene Quellenangabe. --Snevern 09:43, 8. Sep. 2014 (CEST)
Nur zur Klarstellung: Es geht mir gar nicht so sehr um den Beleg im WP-Artikel. Dieses Detail ist ja nur eine nette Anekdote, kann man nehmen oder nicht. Wenn es wirklich eine urban legend ist, dann finde ich allerdings schon, dass das im Eintrag aufgeklärt werden sollte. Was mich verwundert ist, dass diese Geschichte so häufig auftaucht, manchmal als Beweis für den bösen öffentlich-Rechtlichen Rundfunk, der alles zensieren will. Es klingt immer so, als gäbe es da etwas offizielles, wie es ja auch bei Jeanny der Fall war. Immer wird die Titelzeile als Auslöser des "Verbots" angegeben, dabei gibt es doch genug Gründe, ein Lied nicht zu spielen. Dem Musikredakteur gefiel die Melodie/der Text/das Stück allgemein nicht. Vielleicht war dem BR auch einfach nur der Mittelteil zu "fetzig". Möglich ist ja alles. Ich habe leider noch nie derartige Anfragen an Rundfunkstationen gestellt, deshalb wäre ich sehr froh, wenn das jemand für mich machen könnte.--Guglhupf88 (Diskussion) 10:11, 8. Sep. 2014 (CEST)
Ob die BRAVO-Redaktion ein Archiv hat (und wenn ja: Welche Formulierung wurde da verwendet?). Drafi D. wurde da intensivst gecovered - als er auf dem Weg nach oben war ... und auch auf dem Weg nach unten.
GEEZER… nil nisi bene 10:19, 8. Sep. 2014 (CEST)
Gibt's nicht ein Forum für "Bravo"-Sammler, wo um die Durchsicht der Jahrgänge 66 und 67 gebeten werden könnte? Eine Anfrage im 'Bild'-Archiv ist in Bearbeitung. --Aalfons (Diskussion) 19:21, 8. Sep. 2014 (CEST)
Eine Anfrage läuft bezüglich eines potenziellen BRAVO-Archivs. Aber bei den Anfragen und nach der Mail wird man dauernd geduzt... ungewohnt, wenn man sonst nur bei DIE ZEIT nachfragt... GEEZER… nil nisi bene 19:44, 8. Sep. 2014 (CEST)
Einige Bibliotheken in Deutschland haben die Zeitschrift "Bravo" (ISSN 0406-9595) mehr oder weniger vollständig archiviert, teilweise auch auf Mikrofilm, siehe ZDB. Offenbar erschien 1999 eine Mikrofilmausgabe der Jahrgänge 1956 - 1973, vorhanden z.B. in der Bayerischen Staatsbibliothek. Gestumblindi 22:19, 8. Sep. 2014 (CEST)

Allgemeine Einführung in das Römische Meßbuch

Bei der Frage, ob man beim Orate fratres stehen oder sitzen sollte, stolperte ich über den Artikel Allgemeine Einführung in das Römische Meßbuch; dort lese ich, daß es bereits seit 2002 eine dritte Auflage gibt, die die AEM unter dem Namen Grundordnung des Römischen Meßbuches ablösen solle. Nun also meine Frage: Wie ist das zu verstehen? Gilt da nicht sowieso das lateinische Original? Und wieso läßt man sich nun schon 12 Jahre Zeit mit der Übersetzung? Muß ich nun zum Orate fratres sitzen (AEM) oder stehen (GORM)? Kundige vor! :-) --93.219.62.94 08:59, 8. Sep. 2014 (CEST)

Nach §390 der Vorab-Publikation kann die Bischofskonferenz „Gebärden und Körperhaltungen der Gläubigen“ im einzelnen abweichend festlegen. Also gilt bis zum Erscheinen der deutschen Fassung der 3. Ausgabe die deutsche Fassung der 2. Ausgabe, nicht die lateinische Fassung der 3. Ausgabe. --Pp.paul.4 (Diskussion) 09:53, 8. Sep. 2014 (CEST)
Zu der Frage, warum man sich so lange Zeit läßt, ließe sich manches schreiben, indes hülfe es nicht weiter. Wir stehen beim Orate fratres auf, egal ob auf Deutsch oder Latein. Zum Gabengebet erhebt man sich sowieso, wieso also nicht gleich? (Als Faustregel kann gelten, wenn einer sagt: „Lasset uns beten“, steht man immer auf).--Turris Davidica (Diskussion) 10:11, 8. Sep. 2014 (CEST)
Herzlichen Dank für die Antworten! Ist es also so, daß die Bischofskonferenz sozusagen Abweichungen von den Bestimmungen der 3. Auflage in Form der Bestimmungen der 2. Auflage vorgeschrieben hat? Das ist ja sehr interessant! Also sind die vielen anderen abweichenden Bestimmungen der GORM ebenfalls noch nicht in Kraft, es gilt weiterhin die AEM? --93.219.62.94 11:13, 8. Sep. 2014 (CEST)
Die Bestimmungen der GORM können im deutschen Sprachraum erst in Kraft treten, wenn das Meßbuch auch tatsächlich erscheint, das trifft zu. Da die Bestimmungen zur 2. Auflage zeitlich natürlich vor denen der 3. Auflage erlassen worden sind, kann man es jedoch nicht so ausdrücken, daß die Bischöfe zuvor Abweichendes erlassen hätten, sie kannten beim Erlaß die Zukunft ja auch nicht besser als jeder andere. iMHO gibt es aber keine Verpflichtung, auf Unlogischem beharren und sich wie ein dressierter Goldfisch verhalten zu müssen. Steh zum Orate fratres einfach auf, dann wirst du sehen, daß andere deinem Beispiel folgen. Als der hl. Josef ins Hochgebet aufgenommen wurde, war er am selben Abend drin, bevor noch irgendein liturgisches Institut auch nur etwas dazu sagen konnte ;) --Turris Davidica (Diskussion) 12:41, 8. Sep. 2014 (CEST)
Bei uns auch, aber er ist dann nach ungefähr zwei Wochen auch wieder verschwunden. :-( Meine Unsicherheit rührte ja daher, daß bei uns etwa ein Drittel der Leute aufstehen, der Rest bleibt sitzen. Da "Orate fratres" ja nichts anderes als ein verlängertes "Oremus" ist, fand ich es logisch, aufzustehen, wollte aber mich aber gern vergewissern. Nun schließe ich mich also der rebellischen Minderheit an. :-) Vielen Dank für Deine kompetenten Antworten! --93.219.62.94 14:25, 8. Sep. 2014 (CEST)
Offenbar auch im englischen Sprachraum kompliziert... --Concord (Diskussion) 22:57, 8. Sep. 2014 (CEST)
Die Sache ist liturgiewissenschaftlich heikel. Kein Problem früher, solange alles vor der Präfation die Gemeinde wenig anging. Dann stand man eben nach dem (laut Vorschrift genau deswegen betont deutlich gesprochenen "per saecula saeculorum" auf. Wer das Suscipiat betete (Messdiener, Konzelebranten oder nur der Priester selbst), stand ja ohnehin schon. Jetzt beißt sich's halt: Macht man das Aufstehen zum Zeichen einer klaren Trennung von Gabenbereitung und Präfation (mit eigener Begrüßung "Dominus vobiscum"), oder steht man schon zum Suscipiat auf? Freilich ist das Suscipiat ja auch kein Gebet als Anrufung Gottes schlechterdings, sondern ist in der 3. Person Konjunktiv gehalten, also eher eine Art "Wohl geling's!" an die Adresse des Zelebranten. Persönlich bleibe ich (wie der Rest der Gemeinden, bei denen ich an Gottesdiensten teilnehme) traditionell beim Suscipiat sitzen, aber ich würde keine Ideologie daraus machen. Hauptsache, man steht nicht erst beim Sursum corda auf ("Die Herzen, die Herzen!" ;) Grüße Dumbox (Diskussion) 23:56, 8. Sep. 2014 (CEST)
Was die Schwierigkeit beim Suscipiat angeht, gebe ich dir recht, glücklicherweise stehe ich als Meßdiener da ohnehin und brauche mir nichts zu überlegen. Weshalb ich aber oben von einer Faustregel sprach „Betet, Brüder und Schwestern, daß mein und euer Opfer Gott dem allmächtigen Vater gefalle“ kommt „Lasset uns beten“ schon sehr nahe, insofern würde ich da aufstehen. Ich kenne allerdings einen Konvent Nonnen (wo der Zelebrant eigentlich sagen müßte: „Betet, Schwestern…“), der ganz verinnerlicht sitzen bleibt und betet, das paßt dann irgendwie auch (wie auch knien passen würde), und da stehe ich als einzelne dann auch nicht auf. --Turris Davidica (Diskussion) 10:41, 9. Sep. 2014 (CEST)
Nach Lektüre von Concords Link: die machen es aber auch kompliziert. Zur Inzens (egal durch wen) steht man ohnehin und schmeißt sich dann natürlich nicht zwischendurch wieder auf die Bank, rauf und unter… ;) --Turris Davidica (Diskussion) 10:53, 9. Sep. 2014 (CEST)

Betrugsversuch?

Gestern habe ich über ein Internetreisebüro eine Romreise gebucht, die knapp 650 Euro kostet. Die Bezahlung sollte wie üblich per Einzug von der Kreditkarte erfolgen. Bei der Eingabe bekam ich vom System keine besondere Meldung. Heute morgen kam nun per Mail eine Buchungsbestätigung von einem Reisebüro in Paris, die auf die mir gestern ausgegebene Reservierungsnummer Bezug nahm.

Zugleich erhielt ich jedoch eine Mail vom selben Reisebüro -- ebenfalls mit Angabe der Reservierungsnummer --, in der mir dargelegt wurde, daß der Einzug fehlgeschlagen sei, was wohl am überschrittenen Einzugslimit der Kreditkarte liege. Ich solle den Betrag stattdessen per Erlagschein auf ein Pariser Konto überweisen und die Quittung "mit offiziellem Stempel" per Fax an eine Pariser Nummer senden.

Ich rief meinen Bankberater an und bat darum, den Kreditrahmen für diesen Monat auf 1000 Euro zu erhöhen, damit das Reisebüro es noch einmal versuchen könne. Der jedoch war erstaunt und meinte, das Limit liege bei 2500 Euro und sei natürlich längst nicht ausgeschöpft. Wenn das Limit überschritten gewesen sei, hätte ich sogleich bei der Buchung eine automatische Meldung bekommen müssen.

Frage: Handelt es sich möglicherweise um einen Betrugsversuch? --93.219.62.94 11:03, 8. Sep. 2014 (CEST)

Es kann schon sein das was bei der Transaktion fehlgeschlagen ist. Wichtig ist das du jetzt zuerst mal abklärst ob bei deiner Karte/Konto was abgebucht wurde. Wenn nein, wie vom reisebüro vorgeschlagen vorgehen. Wenn ja, Polizei einschalten und auch sofort versuchen Karten-Abbuchung rückgängig zu machen, denn dann ist da was ober faul. --Bobo11 (Diskussion) 11:14, 8. Sep. 2014 (CEST)
Ruhe bewahren und dokumentieren (Screenshot der Abbuchung, die beiden Buchungsnummern etc. an die Hauptadresse senden). Wir hatten das gleiche auch schon mal und nach 4 Tagen hatten sie ihren internen Fehler gefunden und ausgebügelt. Im Leben beruhen 94 % aller Schräglagen auf Blödheit, nur 6 % sind Gemeinheit. GEEZER… nil nisi bene 11:23, 8. Sep. 2014 (CEST)
Vielen Dank für die Antworten! Es ist tatsächlich noch keine Buchung abgegangen, ich weiß aber auch nicht, wie lange es normalerweise dauert, bis so was im Onlinebanking sichtbar ist. Ich habe nun auf die Mail geantwortet, daß ich momentan nicht mit einer Quittung dienen kann, denn meine Bank ist am andern Ende des Landes. Weiters habe ich darum gebeten, die Daten über ein sicheres Formular noch einmal eingeben zu dürfen, so was wird ja möglich sein. Ob das eine Dummheit war, weiß ich noch nicht. Mal sehen. :-) Was mich auch stutzig machte, war, daß man mir eine Frist bis heute nachmittag um drei gegeben hat, um zu reagieren. Vielleicht sollte ich diese Frist einfach verstreichen lassen und die Buchung noch einmal versuchen, wenn es das Angebot dann noch gibt? --93.219.62.94 11:37, 8. Sep. 2014 (CEST)
Kann auch sein, dass es eine zeitliche Überschneidung gab. Spätestens nach drei Werktagen sollte die Überweisung ausgeführt sein. Beim Onlinebanking ist jede Buchung einen Werktag später zu sehen. Also sobald Du den Kontoauszug hast, solltest Du zum Reisebüro marschieren. Dann verlangst Du eine Buchungsbestätigung. Zusatzfrage an alle: Gibt es einen Rücktrittsgrund wegen schweren Vertrauensverlust von Gesetz wegen? Grundlage von Treu und Glauben nicht mehr gewährleistet? --93.133.191.248 12:43, 8. Sep. 2014 (CEST)
Bei einem Fernabsatzvertrag hast du ein Widerrufsrecht. --88.130.114.41 13:18, 8. Sep. 2014 (CEST)
Bei einer Reisebuchung normalerweise nicht, siehe Widerrufsrecht#Gültigkeit, letzter Absatz. --Komischn (Diskussion) 14:12, 8. Sep. 2014 (CEST)
Nun habe ich eine um 14.20 Uhr versendete Mail bekommen, daß ich mich doch bitte bis spätestens 15 Uhr telephonisch in Paris melden soll, andernfalls wird meine Buchung gelöscht. Telefonnummer stand latürnich nicht dabei -- scheint ja 'ne tolle Klitsche zu sein, das... --93.219.62.94 14:28, 8. Sep. 2014 (CEST)
Kreditkarte kann schon etwas laenger dauern bis sich bei dir sichtbar wird, aber die Geschaeftsgebahren kommen mir sehr seltsam vor. Ich wuerde verstreichen lassen und woanders buchen (und Kreditkarte genau im Auge behalten, falls sie es spaeter mit was anderem nochmal versuchen). --192.91.60.11 14:45, 8. Sep. 2014 (CEST)
Du hast ja vermutet, dieser Anbieter habe versucht dich zu betrügen und du fragst, wie du ihn los wirst. Mail mit Antwortfrist 40 Minuten ist auch wieder so ein unseriöses Ding. Seh ich es richtig, dass du auf diesen Anbieter eh keinen Bock mehr hast? Dann freu dich, wenn er "die Buchung löscht", such dir nen anderen Anbieter und gut ist. Wobei ich mich frage, aus welchem Grund der Anbieter sich jetzt vom Vertrag lösen will? Einfach so kann der das nämlich auch nicht. Wenn du nachher einen anderen Anbieter gefunden hast und der ist teurer, kannst du u.U. sogar einen Schadenersatzanspruch gegen den ersten Anbieter haben. --88.130.114.41 14:55, 8. Sep. 2014 (CEST)
Ich habe auch erst recht spät gelernt, dass Buchungen über eine Debit-Kreditkarte unter Umständen erst 2 Wochen später oder zum Monatsende auftauchen, angeblich unter anderem weil Unternehmen Vorgänge sammeln, um Gebühren zu sparen. Davor dachte ich, eine Debit-Karte wird immer sofort belastet und wenn auf dem zugehörigen Konto nix drauf ist, scheitert der Vorgang - meistens ist dies ja auch so.
Wie dem auch sei, per Erlagschein würde ich da nichts überweisen - das Geld kann man nicht mehr zurückholen, oder? --92.202.108.79 00:07, 9. Sep. 2014 (CEST)

War es wirklich so, dass die Menschen in Bayern zur Erstausstrahlung Anfang der 1970er den Film nicht sehen konnten? Also, wer das Erste einschaltete sah einfach nur schwarz? --112.198.82.120 14:27, 8. Sep. 2014 (CEST)

Nein, der sah dann wahrscheinlich eine andere Sendung, Schnee, Testbild oder ein Pausenbild mit Senderkennung ("Solang der alte Peter" bzw. "Wir sind die blöden Bayern"). Ich tippe auf das erstere. :-) --93.219.62.94 14:34, 8. Sep. 2014 (CEST)
Es war ein Rennfahrerfilm. Gruss --Nightflyer (Diskussion) 14:36, 8. Sep. 2014 (CEST)
Und zwar den finnischen Dokumentarspielfilm Benzin im Blut Benutzerkennung: 43067 18:47, 8. Sep. 2014 (CEST)
Das war doch der Film, in dem das Rennauto in eine Schweineherde raste... wurde auch außerhalb Bayerns ausgestrahlt, wer weiß was da der Bayrische Rundfunk zeigte, vielleicht eine Reportage über Marmorstein-Bergwerke. --Aalfons (Diskussion) 19:38, 8. Sep. 2014 (CEST)

Lawson-Kriterium

Bei dem Artikel über das Lawson-Kriterium bleibt mir noch eine Frage. Warum nimmt der Quotient aus der thermischen Energie und der Verlustleistung die Dimension einer Zeit an? Ich bräuchte die Antwort für meine Hausarbeit über Fusionsreaktoren, bei der ich die gesamte Rechnung herleiten will. Kann mir jemand ein kurze Erklärung, vielleicht auch mit Einheiten etc., liefern? Danke schon mal im Voraus!

--93.208.112.223 16:01, 8. Sep. 2014 (CEST)

Thermische Energie hat die Dimension einer Energie (surprise!), Leistung hat Dimension Energie durch Zeit. Der Quotient hat also die Dimension Bruchrechnen. --Wrongfilter ... 16:18, 8. Sep. 2014 (CEST)

Das ist mir schon klar, aber ich wollte das mit den Einheiten beweisen, dass es auch tatsächlich eine Zeit ist, also dass Sekunden oder ähnliches herauskommen. --93.208.112.223 16:41, 8. Sep. 2014 (CEST)

Energie misst man in Joule oder Wattsekunden, Leistung in Watt. Wenn Du also Wattsekunden durch Watt dividierst, bleiben Sekunden übrig. Falls Du das nicht glaubst, frage einen West-, Ost- oder Nordfriesen. Die sind Experten für Watt. --Rôtkæppchen₆₈ 16:58, 8. Sep. 2014 (CEST)
Wir hätten hierfür z.B. den Wahren Wattwurm :) --Jogo.obb (Diskussion) 18:28, 8. Sep. 2014 (CEST)

ähhhh, schon Ferienende?

Hat die Schule in Hessen schon begonnen? Ich hatte vorhin eine Meldung bekommen, ich hätte in der Schule gefehlt, aber ich höre ständig im Radio, dass am 15. September Schulanfang ist, also kann das doch irgendwie nicht sein? -- Liliana 16:30, 8. Sep. 2014 (CEST)

Herzlichen Glückwunsch, du hast den ersten Schultag verpasst. Wie kann es sein, dass man nicht weiß, wann die Ferien aufhören? --88.130.114.41 16:34, 8. Sep. 2014 (CEST)
Am Besten in Zukunft weniger SWR3 hören ;) --Studmult (Diskussion) 16:51, 8. Sep. 2014 (CEST)
Da muss ich mal für den SWR eine Lanze brechen. Der hat heute morgen korrekt berichtet, dass die Schule wieder los ging (SWR1 RP und SWR3) vielleicht war es ja aber der hr? Oder Liliana hat nicht richtig zugehört, für die ABC-Schützen geht die Schule erst morgen los. Aber ich schließe mich der IP an, wie kann man das nur vergessen? Gruß kandschwar (Diskussion) 17:33, 8. Sep. 2014 (CEST)
Nein, es war nicht der Württembergische Staatsrundfunk und es war auch nicht die Regentonne, sondern es war sunshine live. Warum ich nicht geahnt habe dass das nicht für mein Bundesland gilt - spätestens seit der Wahlwerbung mit "Für ein starkes Baden-Württemberg" wo ich mich schon gefragt habe wieso das mich in Hessen etwas angehen sollte wenn Baden-Württemberg stark ist weiß ich nicht. -- Liliana 18:43, 8. Sep. 2014 (CEST)

Und jetzt schreibt die Schölerin bitte noch 100 Mal: "Ich gebe im Feld "Betreff" einen aussagekräftigen Titel ein." :-) --178.8.249.238 17:54, 8. Sep. 2014 (CEST)

Ich finde "ähhhh" toll. Besonders nach 37000 Wikiedits und mit globalen sysop-Rechten. --mfb (Diskussion) 18:35, 8. Sep. 2014 (CEST)
Ich hoffe ja mal, dass Liliana nicht als Lehrerin vermisst wurde. :-) --Schraubenbürschchen (Diskussion) 21:58, 8. Sep. 2014 (CEST)

Konjugation von "schmelzen"

Gerade landete ich aus naheliegendem Anlaß im Abschnitt Kernschmelze#Die Bezeichnung China-Syndrom und stieß dort auf den Satz: "Häufig wird die Herkunft des Ausdrucks damit erklärt, dass die Volksrepublik China von den USA aus betrachtet, nach populärer Meinung ungefähr auf der entgegengesetzten Seite der Erde (Antipode) liegt (was tatsächlich nicht der Fall ist, da sich beide Staaten nördlich des Äquators befinden) und man meint, dass sich der geschmolzene Reaktorkern in Richtung China tief in die Erde hineinschmilzt." Das kam mir spanisch vor, und ich war nicht nur der festen Überzeugung, daß es "hineinschmelzt" heißen müßte, sondern sogar schon drauf und dran, das im Artikel zu ändern, besann mich aber noch und blickte zunächst mal ins Wiktionary, wo ich nur "schmelzen, schmilzt, schmolz, geschmolzen" fand.

Ich dachte bisher immer:

  • Das Eis schmilzt, schmolz, ist geschmolzen.
  • Das Feuer schmelzt das Eis, schmelzte es, hat es geschmelzt.

Ist da in meinem Kopf was falsch verdrahtet? --93.219.62.94 19:41, 8. Sep. 2014 (CEST)

Vielleicht ist in deinem Kopf etwas geschmolzen? Wenn ich geschmelzt lese, schüttelt es mich... ;-) Uwe Dedering (Diskussion) 19:50, 8. Sep. 2014 (CEST)
Nicht veraltet, sondern "veraltend" (Vorgang also noch nicht abgeschlossen). Einen Unterschied zwischen schmelzt und schmilzt im Sinne von aktiv und passiv gibt es aber nicht.
Das Wort steht auf der Duden-Liste für rechtschreiblich schwierige Wörter. --Snevern 20:01, 8. Sep. 2014 (CEST)
Der Duden listet die Variante als veraltet. --mfb (Diskussion) 19:55, 8. Sep. 2014 (CEST)
Die Unterscheidung von starkem Verb für Aktiv und schwachem Verb für Passiv ist aber dem Duden und mir auch unbekannt. --Optimum (Diskussion) 19:59, 8. Sep. 2014 (CEST)
Nur Substantive und Adjektive werden dekliniert, bei Verben spricht man von Konjugation. Scnr, habs geändert. --Φ (Diskussion) 20:00, 8. Sep. 2014 (CEST)
Nicht veraltet, sondern "veraltend" (Vorgang also noch nicht abgeschlossen). Einen Unterschied zwischen schmelzt und schmilzt im Sinne von aktiv und passiv gibt es aber nicht.
Das Wort steht auf der Duden-Liste für rechtschreiblich schwierige Wörter. --Snevern 20:01, 8. Sep. 2014 (CEST)
Gerade kam ich auf die Idee, auf der Dudenseite mal etwas runterzuskrollen; da werden Bedeutung 1 und 2 unterschieden:
  1. unter dem Einfluss von Wärme flüssig werden, zergehen; Perfektbildung mit »ist«; Herkunft mittelhochdeutsch smelzen, althochdeutsch smelzan (starkes Verb), eigentlich = weich werden, zerfließen
  2. durch Wärme flüssig machen, zergehen lassen; Perfektbildung mit »hat«; Herkunft mittelhochdeutsch, althochdeutsch smelzen (schwaches Verb), ursprünglich Kausativ zu schmelzen (1)
Dann spreche ich also mittelhochdeutsch. Sachen gibt's...
Zusatzfrage: Warum gilt dieses Wort dem Duden als rechtschreiblich schwierig?! --93.219.62.94 20:20, 8. Sep. 2014 (CEST)
"Die Liste beruht auf der langjährigen Erfahrung der Dudenredaktion, auf statistischen Analysen des Dudenkorpus, auf der Auswertung dessen, was Nutzerinnen und Nutzer in die Suche dieser Website eingeben, sowie auf den Ergebnissen der relevanten (Fach)publikationen." ([1]) --Snevern 20:33, 8. Sep. 2014 (CEST)
Ach so ist das... --93.219.62.94 20:59, 8. Sep. 2014 (CEST)
(BK)Das ist vergleichbar mit löschen, erschrecken oder hängen, je nachdem, ob es transitiv (= schwach) oder intransitiv (= stark) gebraucht wird. Die schwache Form von schmelzen ist Kausativ der starken. Adelung kennt noch beides. Selbst der Grimm meint noch: "in dem deutschen schmelzen sind verschiedene verba zusammengefallen, die gesondert zu behandeln sind. da nämlich das alte ablautende verb smeltan im germanischen intransitive bedeutung angenommen hat, so ist für die transitive bedeutung, die das wort nach ausweis des griech. und sanskr. ursprünglich hatte, eine schwache causativbildung *smaltjan geschaffen, die im mhd. noch durch den vocal der stammsilbe von dem ersteren schmelzen geschieden war. der unterschied der flexion ist auch im nhd. erhalten und deckt sich im allgemeinen mit dem der bedeutung, doch nicht ohne gelegentliche vertauschungen und übergänge". Pfeifer schreibt: "Dazu das schwache Kausativum ²schmelzen Vb. ‘(durch Wärmeeinwirkung) flüssig machen’, ahd. (um 800), mhd. smelzen, mnd. smelten. Jedoch gehen die starken Flexionsformen (schmilzt, schmolz, geschmolzen) allmählich auch in den trans. Gebrauch über, und schwache Bildungen (schmelzt, schmelzte, geschmelzt) sind seit dem 19. Jh. selten."--IP-Los (Diskussion) 20:36, 8. Sep. 2014 (CEST)
Ist meine Unterscheidung oben also richtig oder falsch? --93.219.62.94 20:59, 8. Sep. 2014 (CEST)
Sie ist richtig: sein + geschmolzen vs. haben + geschmelzt (häufiger jedoch hier ebenfalls heute geschmolzen, das Wb. der dt. Gegenwartssprache schreibt z. B. "das Gold wurde geschmolzen, die Sonne hat das Eis geschmolzen, [die Arbeiter] schmelzten 26,31 Tonnen Stahl je Stunde – Tageszeitung 1966", Grimm bemerkt: "imperf. ich schmelzte. perf. ich habe geschmelzet Frisch 2, 207a. doch wird in der neuern sprache (19. jahrh.) dieser unterschied vielfach verwischt; namentlich sind die schwachen formen des perf. und part. jetzt etwas auszer gebrauch gekommen (s. z. b. die belege aus Ludwig).").--IP-Los (Diskussion) 22:06, 8. Sep. 2014 (CEST)

Frage zu Halogenbirne

Das Halogenleuchtmittel an meiner GY6.35 Schreibtischlampe ist defekt. Ich habe zwei zur Auswahl. Ein mal

"http://www.osram.de/osram_de/produkte/lampen/halogenlampen/halostar/halostar-starlite/index.jsp"

und ein mal

http://www.osram.de/osram_de/produkte/lampen/halogenlampen/halostar/halostar-eco/index.jsp

Letzteres Leuchtmittel behauptet, bei 35 Watt so fast so hell wie 50 Watt zu sein, die 35W Variante behauptet 860 Lumen zu haben, während die klassische Variante 900 Lumen hat.

Theoretisch ist das gut möglich. Allerdings zweifle ich an den Angaben. Die Glühwendeltemperatur der 35 Watt Lampe wird mit 2900K angegeben, die der 50 Watt Lampe mit 3000K. Nach allem was ich weiß, kann in diesem Fall die 35 Watt Lampe niemals auch nur annähernd so hell sein wie die 50 Watt Lampe. Wöllte man einen Wolframdraht bei 35 Watt Leistung so hell leuchten lassen wie bei 50 Watt, dann müsste die Wendeltemperatur wesentlich über der 50 Watt Lampe liegen.

Mit Schreibtischlampen kenne ich mich nicht sehr aus, dafür aber mit KFZ Haogenlampen. Bei diesen kann man unisono feststellen, dass, um so höher die Lichtausbeute bei gegebener Leistungsaufnahme, um so höher muss die Glühwendeltemperatur sein und um so niedriger ist die Lebenserwartung. Das hat etwas mit dem Schmelzpunkt von Wolfram zu tun, an dem sich die Hochleistungsleuchtmittel nahe bewegen, nämlich 3695K. Während Longlife-Leuchtmittel weit darunter betrieben werden, d.h. nicht so heiß werden, und dafür weniger Liehchtausbeute haben, dafür aber länger leben.

Oben genannte Datenblätter widersprechen dem. Hat jemand eine Erklärung dafür? --95.112.151.15 02:52, 9. Sep. 2014 (CEST)


Die Helligkeit hängt nicht nur von der Temperatur, sondern auch von der Größe der leuchtenden Fläche ab. -- Janka (Diskussion) 04:56, 9. Sep. 2014 (CEST)
Das ist absolut klar und widerspricht nicht meiner Argumetation. --95.112.179.129 05:01, 9. Sep. 2014 (CEST)
Energiesparende Halogenleuchtmittel haben eine Infrarotreflektionsbeschichtung. Damit wird weniger Energie zur Erzielung einer bestimmten Farbtemperatur benötigt als ohne. --Rôtkæppchen₆₈ 06:48, 9. Sep. 2014 (CEST)
Noch ein wichtiger Hinweis: Beim Austausch des Leuchtmittels musst Du unbedingt das Typenschild des Halogentrafos beachten. Hat das neue Leuchtmittel zu wenig Leistung, so steigt die Betriebsspannung und das Leuchtmittel stirbt vorzeitig durch Überspannung. Hat das neue Leuchtmittel zu viel Leistung, so steigt der Strom durch den Halogentrafo. Bei zu großem Strom kann dieser den vorzeitig durch Hitze sterben. --Rôtkæppchen₆₈ 07:06, 9. Sep. 2014 (CEST)

Groß-/Kleinschreibung von Begriffen der Baule, Elfenbeinküste

Hallo, ich stelle mir die Frage, ob es eine offizielle Regelung bezüglich der Groß-/Kleinschreibung nichtdeutscher Begriffe in der deutschen Sprache gibt, nach der ich mich bei der Verwendung von Begriffen der Baule richten kann. In einer Bildbeschreibung begegnen mir zwar "Blolo Blo" und "Asie Usu" in der Großschreibung, jedoch habe ich den Begriff "Blolo" noch nicht in einer deutschen Publikation finden können, weshalb ich da keinen Ahnhaltspunkt habe. Die englische Literatur verwendet alle drei Begriffe der Baule und weitere in diesem Kontext immer die Kleinschreibung. Ich würde mich über Rat freuen. Viele Grüße --Julius1990 Disk. Werbung 02:58, 9. Sep. 2014 (CEST)

Spontan würde ich sagen, wie im Deutschen: Substantive groß, Adjektive klein, ... Googeln zeigt nicht viele Treffer im Deutschen, von denen manche es in Anführungsstrichen und klein schreiben. --Eike (Diskussion) 08:44, 9. Sep. 2014 (CEST)
Die Engländer passen es an ihre Gewohnheiten an und schreiben Substantive (mit wenigen Ausnahmen) daher klein. Du machst es ebenso – und schreibst groß. Näheres hier. 194.25.103.254 08:59, 9. Sep. 2014 (CEST)
Würde mich bei solchen Fällen an GoogleBooks halten (und dort nach Verwendung in deutschen Texten suchen). Bei beiden Beispielen oben würde ich Kleinschreibung und zum Absetzen (WP Fremdwort-Formatierung) Kursivschrift verwenden. GEEZER… nil nisi bene 09:11, 9. Sep. 2014 (CEST)
Ist nach ins Deutsche übernommenen Fremdwörtern oder nach zitierten fremdsprachlichen Baule-Wörtern gefragt? Im ersten Fall muss großgeschrieben werden, im zweiten Fall wie in der Orginalsprache klein. Im ersten Fall richtet sich das Genus nach den Regeln für Fremdwörter, im zweiten Fall nach denen der Originalsprache, sofern es dort ein Genus gibt. Außerdem sollte die Wikipedia-Typografie für fremdsprachige Wörter (und Wikipedia:Fremdwortformatierung) beachtet werden. Beispiele:
  • Ich schreibe an einem Computer.
  • Engländer bezeichnen ein elektronisches Bürogerät als computer.
  • Engländer bezeichnen elektronische Bürogeräte als computers.
  • Bei Ovid besitzt der Begriff der arbor Ergänzung: Genitiv von die arbor, falsch ist: des arbor (Baum) eine besondere Bedeutung. (Das lateinische arbor ist weiblich: die arbor.) --BlackEyedLion (Diskussion) 09:18, 9. Sep. 2014 (CEST)
Es sind keine ins Deutsche übernommene Wörter, sondern die Baule-begriffe, die ich halt im Text verwenden möchte. Auf die Orginalsprache habe ich leider keinen wirklichen Zugriff, weil ich allein über englische Literatur Zugang dazu habe und die deutsche Literatur zienmlich überschaubar ist. Julius1990 Disk. Werbung 10:30, 9. Sep. 2014 (CEST)
Amtssprache ist dort Französisch. Dann könntest du die Begriffsschreibung in dieser Sprache (und im Vergleich mit Englisch) verwenden. Auch wieder - da nicht eingedeutscht - vorwiegend Kleinschreibung. GEEZER… nil nisi bene 10:48, 9. Sep. 2014 (CEST)

Steigflugwinkel

(oder wie das heißt): Neulich habe ich beobachtet, dass startende Starrflügeler (normale Jets) erst sehr steil von der Startbahn abheben (gut 45°, dann aber seeehr gemächlich steigern, d.h. den Winkel verringern. Wieso? Je schneller man auf Reiseflughöhe ist, desto mehr Krafstoff spart man ja. Mir ist klar, dass sich das die meisten Piloten nicht aussuchen können (Vorgabe des Towers), aber jetzt mal grundsätzlich gefragt. Meine einzige Vermutung ist, dass der Windwiderstand in der unteren Atmosphäre insgesamt höher ist und daher mehr Kerosin verbraucht (wenn man nicht ganz so steil steigt). --217.189.212.50 10:52, 9. Sep. 2014 (CEST)

Steigflug gelesen? Ich vermute der Artikel könnte deine Frage (die mir nicht ganz klar ist) beantworten. :-) --Nattr (Diskussion) 11:09, 9. Sep. 2014 (CEST)
Auch Englisch: Ab Backward stick movement forces the tail down and the nose up. GEEZER… nil nisi bene 11:13, 9. Sep. 2014 (CEST)
Die Frage: Wieso wird der Steigflug vom Abheben bis zum Erreichen der Reiseflughöhe nicht konstant mit dem gleichen Winkel geflogen? Im Artikel finde ich dazu nichts. Daher mal auf blöd was dazugeschrieben.... --217.189.212.50 11:30, 9. Sep. 2014 (CEST)
Zitat Artikel: Aus Lärmschutzgründen und aus Gründen des "Freimachens" von Luftraum starten Verkehrsflugzeuge von großen Flughäfen oft mit steilem Neigungswinkel, etwa um 20° (abhängig von den Vorgaben des Herstellers, der Beladung, der Windverhältnisse und des Lotsen). Der Steigwinkel wird in der Regel verringert, sobald die Flughafenumgebung verlassen wurde.
--192.91.60.11 11:52, 9. Sep. 2014 (CEST) <- das habe ich ja gerade reingeschrieben :-))) --217.189.212.50 16:06, 9. Sep. 2014 (CEST)
Im Artikel steht noch was zu den Risiken, die werden bei geringerem Winkel kleiner, die Vogelschlaggefahr ist dagegen bei größerem Winkel kleiner, da kürzere Strecke im gefährdeten Luftraum. Wahrscheinlich wird der Winkel erst reduziert wenn man die Vögel unter sich gelassen hat. --Jogo.obb (Diskussion) 12:44, 9. Sep. 2014 (CEST)

Wie heisst dieser Schmetterling

Servus, ich suche den Namen dieses Schmetterlings. Wäre cool, wenn irgendwer mir sagen könnte, wie er heisst. Vielen Dank im Voraus, CDV --178.1.137.165 12:47, 9. Sep. 2014 (CEST)

en:Bhutanitis lidderdalii. XenonX3 – () 12:49, 9. Sep. 2014 (CEST)

RFID Personalausweis

Ich bin gerade dabei mittels eines USB RFID-Kartenlesers (kann ISO/IEC 14443-A/B und ISO 15693) mifare-Karten auszulesen, z.B. dort auch die Kartenseriennummer. Wenn ich anstelle von mifare-Karten meinen neuen Personalausweis an den Leser halte bekomme ich auch eine Seriennummer zurückgeliefert, lustigerweise bei jedem Lesevorgang aber immer eine andere. Absicht oder triggere ich mit der mifare-Funktion irgendwie eine zufällige Response an? BTW, welchem DIN/ISO/IEC Standard entspricht eigentlich die RFID-Funktion auf dem Perso? 82.207.169.99 15:54, 9. Sep. 2014 (CEST)

Der Perso gibt nicht seine Seriennummer zurück, sondern versucht, eine sichere Verbundung aufzubauen. Die RFID-Kommunikation mit dem Personalausweis muss gesichert sein, damit niemand den Perso abhören kann. --Rôtkæppchen₆₈ 16:11, 9. Sep. 2014 (CEST)
Personalausweis_(Deutschland)#Der_elektronische_Personalausweis_.28nPA.29 sagt ISO/IEC 14443.--Simius narrans 16:14, 9. Sep. 2014 (CEST)
Ja, die Zufallsnummer ist Absicht, damit keiner Bewegungsprofile erstellen kann. --Tbhgeo (Diskussion) 16:29, 9. Sep. 2014 (CEST)
Siehe auch en:German identity card und en:Basic access control. --Rôtkæppchen₆₈ 16:31, 9. Sep. 2014 (CEST)

Was könnte das sein?

ich habe ein kleines Werbegeschenk bekommen, das einer Lieferung von Zahnbürstenaufsätzen aus China beilag. Weiß jemand, was für ein Zweck das Gerät hat? Das Ding kst ca. 4 cm lang. [2], [3] --95.112.179.129 17:01, 9. Sep. 2014 (CEST)

Ich halte das für einen Touchscreen-Pen, siehe z.B. hier. --Magnus (Diskussion) 17:09, 9. Sep. 2014 (CEST)
Danke. Ich wede alt. --95.112.179.129 17:11, 9. Sep. 2014 (CEST)
Archivierung dieses Abschnittes wurde gewünscht von: Yoursmile (Diskussion) 08:53, 10. Sep. 2014 (CEST)

Angst vor Symmetrien?

Gibt es nachgewiesene Fälle der Angst vor Symmetrien oder ist dies nur eine Art Internet Hoax? Wenn ja, wie drückt sich diese Angst aus? (nicht signierter Beitrag von 188.100.84.1 (Diskussion) 22:47, 9. Sep. 2014 (CEST))

Ich fürchte die Liste der Phobien ist eine Art Internethoax, vor der man eine Art Angst kriegen kann... --87.176.78.161 23:00, 9. Sep. 2014 (CEST)
Fürchte dich nicht, denn ... The Encyclopedia of Phobias, Fears, and Anxieties, Third Edition kennt sie als symmetrophobia. GEEZER… nil nisi bene 23:13, 9. Sep. 2014 (CEST)
Archivierung dieses Abschnittes wurde gewünscht von: GEEZER… nil nisi bene 12:13, 10. Sep. 2014 (CEST)

Gelbe Gefahr Verwirrung

Ein kleines Geschenk für GG ;) -- HilberTraumd, m13:02, 10. Sep. 2014 (CEST)
Hier die Variante für den Großverbraucher mit 28 Fächern --Rôtkæppchen₆₈ 13:18, 10. Sep. 2014 (CEST)

Ich möchte diese Website verlinken, sehe aber jedes "d" gelb markiert. Habe ich wieder meine Pillen vergessen - oder sehen das andere Besucher ebenso? Wenn ja: Wie kann ich die Seite neutral zeigen? GEEZER… nil nisi bene 10:36, 10. Sep. 2014 (CEST)

Mach die Suche in Firefox wieder zu ;-) --212.71.197.50 10:49, 10. Sep. 2014 (CEST)
Offenbar die Pillen gegen Ikterus intermittens ordinatri vergessen. Seite sieht in Firefox 31.0 normal aus. --Pp.paul.4 (Diskussion) 10:57, 10. Sep. 2014 (CEST)
@212.71.197.50: Im Firefox sind die Ds wenn schon, dann magenta, wenn Du nach d suchst und in der Suchleiste “Highlight All” anklickst. Im Internet Explorer werden die Ds gelb, wenn Du nach d suchst und in der Suchleiste den Textmarker anklickst. --Rôtkæppchen₆₈ 11:01, 10. Sep. 2014 (CEST)
Merkwürdig. Ich hatte nicht nach "d" gesucht. Suchbegriffe waren "Managerkrankheit Wirtschaftswunder". Mittlerweile ist die Site entgelbt (ich habe einen Schreenshot mit den gelben "d"'s ...). GEEZER… nil nisi bene 11:10, 10. Sep. 2014 (CEST)
Archivierung dieses Abschnittes wurde gewünscht von: GEEZER… nil nisi bene 11:10, 10. Sep. 2014 (CEST)

"Ihr Schuhwerk ist sehr geschmackvoll. Darf ich...."

"...mein Glied in Sie einführen?" Vielleicht kann mir jemand helfen, den Film zu finden, aus dem dieses Zitat stammt. Ich suche den Film schon länger und finde ihn nicht mehr. Es geht um Aliens, die sich (aus mir nicht mehr bekannten Beweggründen) unter die Menscheit mischen wollen. Dazu lernen sie in Kursen, wie man sich als Mensch verhält und mit ihnen fortpflanzt. Obiges Zitat ist ein Anmachspruch für Alienmänner, der ihnen nach Auffassung der Aliens garantiert zum Erfolg bei menschlichen Frauen verhelfe... naja :) Der Film muss von vor 2009 sein, eher noch älter (B-Movie-Optik, wenn ich mich recht erinnere). Das Orakel habe ich schon befragt. Gibts irgendwo noch Listen von Sci-Fi-Filmen, die ich durchstöbern könnte? Danke. --78.51.91.49 13:59, 10. Sep. 2014 (CEST)

Mein Google spuckt „Good Vibrations – Sex vom andern Stern“ als allerersten Treffer aus. --Rôtkæppchen₆₈ 14:03, 10. Sep. 2014 (CEST)
Oh. Das ist er, aber warum spuckt mein google das nicht aus? --78.51.91.49 14:12, 10. Sep. 2014 (CEST)
Vielleicht hast du's mit Anführungsstrichen eingegeben? Der Wortlaut scheint nicht ganz zu stimmen. --Eike (Diskussion) 14:16, 10. Sep. 2014 (CEST)
(nach BK) Komisch. Wenn ich meine google-Suche wie üblich zu Anonymisierung über eine Zwischenseite laufen lasse und mit deaktivierten Skripten + Anführungsstrichen arbeite, bekomme ich das Ergebnis nicht. Erst im Ersatzbrowser mit voller Nutzerdatensammlung spuckt google mir das auch aus. --78.51.91.49 14:18, 10. Sep. 2014 (CEST)
Ohne Anführungsstriche sollte es so oder so kommen, denke ich. --Eike (Diskussion) 14:22, 10. Sep. 2014 (CEST)
Lag daran, dass das Zitat falsch war. Fußbekleidung muss ich suchen, dann kommt's.
"Ihre Fußbekleidung ist sehr geschmackvoll." "Danke." Darf ich meinen Penis in Sie einführen?" "Ja." So schnell kann es gehen :) --78.51.91.49 14:28, 10. Sep. 2014 (CEST)
Wäre es möglich, dass deine Zwischenseite Anzügliches herausfiltert? Wenn ich das "Glied" in meiner Suchanfrage ersatzlos weglasse ("ihr Schuhwerk ist sehr geschmackvoll darf ich mein in sie einführen"), finde ich auch nichts passendes, mit kann ich mir die passenden Treffer aus mehreren aussuchen. Prinzipiell setzt zwar auch Google dir andere Suchergebnisse vor wenn es dich z.B. woanders vermutet, was mit solchen Proxy-Lösungen der Fall sein dürfte, aber das sollte bei solch spezifischen Suchanfragen eigentlich nicht ins Gewicht fallen (anders als wenn du beispielsweise nach Produkten suchst und Google dir dann lauter Shops und Reviews aus einem anderen Land liefert). --YMS (Diskussion) 14:29, 10. Sep. 2014 (CEST)
Ich benutze startpage und habe den Familienfilter ausgeschaltet. Ich war mir wohl zu sicher bei dem genauen Wortlaut, wie rotkaeppchen angedeutet hat. Zuerst hatte ich es auch mit 'Penis' statt 'Glied' probiert. Kein Wunder... --78.51.91.49 15:14, 10. Sep. 2014 (CEST)
@78.51.91.49:@Eike sauer: Ich hab es ganz in Kleinbuchstaben, ohne Satzzeichen und ohne umschließende Anführungszeichen eingegeben, wie man das eben so macht, wenn man den exakten Wortlaut nicht kennt und es keine Eigennamen sind. --Rôtkæppchen₆₈ 14:32, 10. Sep. 2014 (CEST)
Bis auf die Anführungsstriche drumrum wird das eh ignoriert. [4] --Eike (Diskussion) 14:38, 10. Sep. 2014 (CEST)
Dann haben die das mit den Eigennamen irgendwann geändert und das mir entgangen. --Rôtkæppchen₆₈ 14:41, 10. Sep. 2014 (CEST)
Archivierung dieses Abschnittes wurde gewünscht von: Merci. 78.51.91.49 15:26, 10. Sep. 2014 (CEST)

Wortkomposita

Gibt es irgendeine etymologische Erklärung, warum es zwar Bundestag heißt, aber nicht Landestag, aber stattdessen Landtag? Ganz konsequent müsste man doch auch Landestag sagen können. --112.198.82.120 12:44, 8. Sep. 2014 (CEST) --112.198.82.120 12:44, 8. Sep. 2014 (CEST)

Ich würde da auf einen Fugenlaut tippen. Der wird nicht streng systematisch verwendet, eher so nach Gefühl, wie es sich besser spricht oder anhört. Das es hier uneinheitlich ist, könnte noch historische Gründe haben. Rainer Z ... 13:23, 8. Sep. 2014 (CEST)
Siehe auch Tagtraum, aber Sommernachtstraum. Bereits Jean Paul hat an solchen Uneinheitlichkeiten des Fugen-s Anstoß genommen und, den Lehren des "Deutschmanns" Christian Heinrich Wolke folgend, in Nachauflagen seiner Romane jeden Ratsherrn zum Ratherrn gemacht. Sein Biograf Günter de Bruyn bedauert das sehr, denn die Flegeljahre und der Komet blieben Fragment, vielleicht auch, weil der Autor so viel Energie und Zeit in seinen Einheitlichkeitswahn steckte.--Mautpreller (Diskussion) 15:29, 8. Sep. 2014 (CEST)
Landestag gibts in derselben Bedeutung wie Landtag, allerdings wohl deutlich seltener, siehe z. B. hier. --Abderitestatos (Diskussion) 17:47, 8. Sep. 2014 (CEST)
Hinzu kommt: Ohne Fugen-S würde der Bund-Tag wie bunt-Tag klingen, also phonetisch mehrdeutig sein; für Lant-Tag ist der Sinn phonetisch eindeutig. --Zerolevel (Diskussion) 12:05, 9. Sep. 2014 (CEST)
Das dürfte (zumindest früher) weniger eine Rolle gespielt haben, siehe Bundsteg, Bundweite, Bundschuh; der Grimm kennt neben bundbrüchig (so auch noch im DDR-Duden, 15. Aufl. [1963], S. 96) sogar Varianten wie Bundesgenoß vs. Bundgenoß, Bundesgeselle vs. Bundgeselle.--IP-Los (Diskussion) 16:54, 9. Sep. 2014 (CEST)

Korridor in der Luft und MH 370

Der Flug MH 370 ist nun ein halbes Jahr her. Im Zuge der Diskussion (und zwar auch gleich zu Beginn) über dessen Verschwinden wird angenommen, dass er in einem Korridor geflogen ist. Meine Frage dabei ist, woher man denn weiß, dass ein Korridor genommen wurde. Da der Verlauf des Fluges sowieso schon mysteriös ist, warum sollte er sich dann an die „Regeln“ halten und in einem solchen Korridor fliegen? Da es in der Luft zudem keine Begrenzungen oder Führungen (wie es Schienen oder Straßen sind) gibt, hätte das Flugzeug jeden x-beliebigen Weg wählen können. Auch werden oft der nördliche und der südliche Korridor genannt. Warum kommt aus dem nördlichen ein Ping-Signal, wenn der Flieger im Süden war? Grüße, --Urgelein (Diskussion) 15:15, 8. Sep. 2014 (CEST)

Man hat Signallaufzeiten zum Satelliten gemessen, und die lassen sich durch einen bestimmten Abstand erklären - von dem man aber zunächst nicht weiß, ob er nach Süden oder nach Norden führte. Wo der Satellit war, könntest du aus der Zeichnung erraten... aber es ist ja auch eingezeichnet. --Eike (Diskussion) 15:29, 8. Sep. 2014 (CEST)
(BK) Das Flugzeug hat wohl nach seiner geplanten Landezeit diesen geostationaeren Satelliten angepingt. Durch die Auswertung der Signallaufzeit dieses Pings konnte ein Abstand des Flugzeugs zum Satelliten bestimmt werden, was auf der Erdoberflaeche einen kreisfoermigen "Korridor" um diesen Satelliten ergibt. Durch die begrenzte Reichweite konnte sich das Flugzeug aber nur in einem Bogensegment des Kreises befinden: Ausgehend vom Startpunkt (Kuala Lumpur) in nordlicher Richtung fuer 5000 km dem Kreis folgend ("noerdlicher Korridor"), oder in suedlicher Richtung fuer 5000 km dem Kreis folgend ("suedlicher Korridor"). --Nurmalschnell (Diskussion) 15:35, 8. Sep. 2014 (CEST)

Das heißt also, wenn ich das richtig verstehe, dass die Maschine nicht entlang des Korridors geflogen ist, sondern dass sie einen Kurs geflogen ist, der diesen Bogen gekreuzt hat bzw. der irgendwo an der Kreuzung mit diesem Bogen zum Absturz geführt hat. --178.8.249.238 18:04, 8. Sep. 2014 (CEST)

Die Grafik basiert nur auf dem letzten, also genau einem Ping. Das Flugzeug sollte da irgendwo auf den roten Linien gewesen sein. Über welche Route es da hinkam, lässt sich nicht sagen, wenn man nicht auch die Daten anderer Pings kennt. Dass es überhaupt zwei verschiedene Korridore gibt, kann ich mir nur so erklären, als dass im Mittelstück auch der zweite eingezeichnete Satellit (Inmarsat3-F3) das Signal hätte empfangen müssen, aber eben nicht hat. Dass die Korridore (vor allem der nördliche) auf der Grafik noch deutlich über den "ungefähren maximalen Flugradius" hinausragen, ist eine Ungenauigkeit in der Grafik, oder übersehe ich da was (von eventuellen Strömungsunterschieden mal abgesehen, die dann ja aber auch an anderen Stellen Einfluss haben müssten)? Wenn ich mit meinem Treibstoffvorrat in jede andere Richtung nur noch etwa 5250 Kilometer weit fliegen kann, dann kann ich schliesslich auch nach Nordwesten nicht plötzlich doch 6000 Kilometer weit fliegen... --YMS (Diskussion) 18:22, 8. Sep. 2014 (CEST)
Richtig, das Signal kam aus dem Korridor; wie das Flugzeug dorthin gelangte, ist unbekannt, ebenso wie unbekannt ist, wo genau es sich befand. -die Reichweite eines Flugzeugs in der Luft lässt sich nicht auf ein paar zig Kilometer genau eingrenzen; sie hängt von Geschwindigkeit, Flughöhe, Windstärke und -richtung usw. ab. Nur die maximale Reichweite unter günstigsten Bedingungen lässt sich einigermaßen eingrenzen - das sind in etwa die beiden Endpunkte der Korridore. Theoretisch, wenn das Flugzeug hin und her oder im Kreis geflogen wäre, hätte es sich auch auf dem fehlenden Stück Kreisbogen befinden können. --Snevern 18:28, 8. Sep. 2014 (CEST)
Nein, denn dann wäre es gleichzeitig noch vom Satelliten über dem Pazifik erfasst worden. --Studmult (Diskussion) 20:47, 8. Sep. 2014 (CEST)
Nein, das Flugzeug wurde nicht von dem Satelliten "erfasst", sondern das Inmarsat System an Bord des Flugzeugs peilte den Satelliten an. Das System peilt immer nur genau einen Satelliten an ([5]), und im Falle von MH370 war das Inmarsat 3 F1. Die Lücke zwischen den Korridoren ist auch nicht symmetrisch zum nächsten Satelliten (Inmarsat 3 F3), sondern symmetrisch zur letzten gemeldeten Position des Flugzeugs. --Snevern 23:04, 8. Sep. 2014 (CEST)

Ist nicht auch der Text in der Legende falsch ("die Hüllkurve zeigt an, wie weit das Flugzeug anschließend noch mit seiner Tankfüllung fliegen konnte")? Ist es tatsächlich so, dass der Sprit nach dem Ping nur noch diese kurze Strecke gereicht hätte? Von jedem Punkt der Korridor-Linie aus? --193.18.240.18 14:29, 10. Sep. 2014 (CEST)

Vom letzten Signal aus hätte das Flugzeug noch weiterfliegen können, aber wo auf der Kreisbahn sich das Flugzeug zum Zeitpunkt des letzten Signals befand, kann man anhand dieser Daten nicht sagen. Da, soweit ich mich erinnere, die Signale einmal pro Stunde kamen, hätte das Flugzeug also noch bis maximal eine Stunde weiterfliegen können. Für eine Stunde Flugzeit erscheint mir die Hülle zu schmal, aber da gibt es vielleicht zusätzliche Informationen, die ich nicht habe.
Was ich mich dagegen frage, ist folgendes: Es gab nicht nur genau ein Signal, es gab mehrere Stunden lang jede Stunde eines. Für jedes dieser Signale müsste man so einen Radius (oder dazu passende "Korridore") errechnen können, und da das Flugzeug zweifellos nicht exakt den Korridor entlanggeflogen ist, müssten sich also mehrere Radien ergeben. Zusammen mit der zeitlichen Abfolge der Signale könnte man also mindestens sagen, ob die Radien größer oder kleiner wurden, ob sich das Flugzeug also der Position des Satelliten näherte oder sich davon entfernte. Davon ist aber, soweit ich das sehe, nirgendwo die Rede. --Snevern 15:56, 10. Sep. 2014 (CEST)

Sicherheitsgurte in Flugzeugen

Wo steht das geschrieben (international und/oder Deutschland), dass diese Vorführung vor dem Flug bzgl. Sicherheitsgurte, Notausgänge, Sauerstoffmasken, Rettungsweste und dergleichen notwendig ist? --217.189.212.50 17:59, 9. Sep. 2014 (CEST) Vielleicht kommt man doch mal darüber hinweg, die Erläuterung zur Bedienung des Beckengurtes wegzulassen (idiotensicher zu bedienen und vom Prinzip sowieso fast dasselbe wie 99,8% der Fluggäste ihn ja vom Auto kennen). Übrigens wird nie auf die Assistenztaste (rot, mit Personenpiktogramm) hingewiesen, die wäre bei akuter Erkrankung ja wichtig.

Die Antwort bitte auch in Safety Instruction Demonstration einfügen, selbst wenn sie (gesichert) "Die machen das aus freien Stücken, um nicht nach einem Unglück blöd dazustehen" lauten sollte. --YMS (Diskussion) 18:07, 9. Sep. 2014 (CEST)
Wenn jemand das in Wikidata richtig mit Pre-flight Safety Instruction Demonstration verknüpft hätte, statt ein neues Item [6] aufzumachen, könnte man auch die zuständigen Behörden finden. Hier, was Wunder, die FAA allen voran. --Eingangskontrolle (Diskussion) 18:26, 9. Sep. 2014 (CEST)
Steht im Joint Aviation Requirements JAR-OPS 1 (Pre-flight safety demonstration). Dort unter Pflichten der Flugzeugbesatzung, verantwortlich dazu ist der leitende Flugbegleiter. Siehe Swiss International Air Lines Safety Video Weiter Auskunft gibt das Portal:Luftfahrt , Assistenztaste kannst du auch drücken wenn du eine Wisky haben möchtest. --77.211.216.238 18:29, 9. Sep. 2014 (CEST)
Die bislang fehlenden Interwiki-Links bzw. das Erstellen des einsamen Items sind Wikidata bzw. dem Bot nicht anzukreiden, da der Artikel schon vier Jahre ohne Interwikilinks existierte ehe es Wikidata überhaupt gab (und weil das Lemma so auch in keiner Wikipedia verwendet wird, war es auch keine ganz triviale Aufgabe, die entsprechenden Artikel anderswo zu finden). --YMS (Diskussion) 19:47, 9. Sep. 2014 (CEST)
„Assistenztaste kannst du auch drücken wenn du eine Wisky haben möchtest.“ << is'n Brüller! kurz vor der Bewußtlosigkeit, vor einem Herzanfall oder einem Bauchaorten-Aneurysma etc. soll man den Schalter nicht nehmen?! Schau mal in einer B747 mitten in der Nacht zwischen London und Tokio, dass Du mit letzter Kraft jemanden vom Bordpersonal bei dem Lärmpegel herbeihauchen kannst. Bestimmt erwartest, Du, dass jeder ein großes Megaphon am Mann hat. --217.189.212.50 00:12, 10. Sep. 2014 (CEST)

Obwohl Hossein Ali Montazeri die Hinrichtungen kritisierte, passierte ihm relativ wenig. Wie konnte er eigentlich einer Strafe entkommen? Oder war Montazeri als Ayatollah in einer etwas privilegierten Position und genoss gewisse Unantastbarkeit? Khomeini hätte den durchaus richtig kaltstellen können. --112.198.82.168 20:16, 9. Sep. 2014 (CEST)

"Großayatollah Hussein Ali Montazeri genoss bis zu seinem Tod hohe Anerkennung in der iranischen Bevölkerung. Zu seinen Befürwortern zählten überwiegend junge, reformorientierte Studenten, aber auch viele religiös-konservative Iraner." <= Machtpolitische Kosten/Nutzen-Rechnung. Du hast jemanden, den die Leute mögen. Nun sagt der auch ab und zu Krempel, den du selber aber nicht magst. Was ist dir wichtiger - bei deinen eigenen Absichten? "OK, er darf weitermachen, aber kurz vor dem Ende trete ich ihm dafür noch in die .... " GEEZER… nil nisi bene 09:59, 10. Sep. 2014 (CEST)

Maler: Keller-Reutlingen, Paul Wilhelm, 2.2.1854 - 10.1..1920

--80.137.217.22 21:13, 9. Sep. 2014 (CEST)

Da gibt es scheinbar zwei Paul Wilhelms, die auch noch beide Maler waren. Wir haben bislang nur einen Artikel über diesen hier: Paul_Wilhelm_(Maler). Über den von dir angesprochenen haben wir im Moment noch keinen Artikel, er wird aber wie in Wikipedia:Relevanzkriterien#Bildende_Künstler gefordert im Virtuellen Katalog Kunstgeschichte erwähnt und ist somit relevant. Wenn du Informationen über ihn hast, kannst du unter dem Lemma Paul Wilhelm (Maler, 1854) einen Artikel erstellen! Sei mutig! :-) --88.130.77.236 21:50, 9. Sep. 2014 (CEST)
In Auktionen sind Werke nachgewiesen: Paul Wilhelm Keller, genannt "Keller-Reutlingen". 1854 Reutlingen - 1920 München. Maler mit nicht besonders auffälligen Familiennamen verwendeten häufig als Namenszusatz ihren Geburtsort, um nicht verwechselt zu werden. Quellen über Auktionen seiner Werke lassen sich über Google finden. --80.139.156.124 23:42, 9. Sep. 2014 (CEST)

Kunstinstallation in Finnland gesucht

Es geht um die Insel Korkeasaari (Helsinki) mit einem Zoo. Dort muss es eine Kunstinstallation geben, die Skulpturen in Form von Schweineköpfen aus Stein und Metall enthält - als politische Kritik. Kennt jemand den Künstler und Bildquellen dieser Arbeit?

--80.139.156.124 21:13, 9. Sep. 2014 (CEST)

Was ist die Quelle dieser Information? Null Resultate nach 10 Min. Suche. GEEZER… nil nisi bene 22:24, 9. Sep. 2014 (CEST)
Vielleicht ist ja en:Posankka gemeint. --Rôtkæppchen₆₈ 22:36, 9. Sep. 2014 (CEST)
Hm, ich bin schon des öftern im Zoo von Helsinki gewesen aber eine solche Statue wäre mir nicht aufgefallen. --Bobo11 (Diskussion) 22:40, 9. Sep. 2014 (CEST)
Wenn der Fragesteller nicht ausdrücklich das Material genannt hätte, hätte ich vermutet, dass es etwas mit dem alljährlichen Sandskulpturenwettbewerb Art Meets Sand oder dem alljährlichen Eisskulpturenwettbewerb Art Meets Ice zu tun hat. Es wäre vielleicht hilfreich zu erfahren, woher die Information stammt. --Jossi (Diskussion) 10:55, 10. Sep. 2014 (CEST)
Wo auf der Insel soll das sein? Sie ist ja nun nicht gerade riesig. --Pölkky 15:39, 10. Sep. 2014 (CEST)

Eintrag des "WLE-Bahnmuseum-mobil" in die WIKIPEDIA

Hallo,

mein Artikel über das WLE-Bahnmuseum-mobil ist gelöscht worden. Bitte teilen Sie mir mit, wo der Fehler lag.

Bin gerne bereit, einen Mentor als Hilfe anzunehmen. Er müßte sich in Ost-Westfalen auskennen. Es gibt zahlreiche Zeitungsartikel, die über die Existens des Museums seit 1985 geschrieben haben. Auch Fotos der Anfangszeit können vorgelegt werden. Außerdem ist es in der Gründungs- Satzung des Vereins verankert worden.

Vielen Dank für Ihre Bemühungen.

MfG W. Zylka WLE-Bahnmuseum-mobil.de

--WLE Bahnmuseum mobil (Diskussion) 21:39, 9. Sep. 2014 (CEST)
Hinweis: Für Fragen speziell zur Wikipedia gibt es ein eigenes Diskussionsforum: Fragen zur Wikipedia.

Also diesen Entwurf gibt es noch, der ist aber von 2013. Aus dem Entwurf geht nicht klar hervor, was das eigentlich ist. --mfb (Diskussion) 21:47, 9. Sep. 2014 (CEST)
Siehe Relevanzcheck - das erfüllt nicht die Relevanzkriterien, ein Artikel dazu ist also nicht erwünscht. --mfb (Diskussion) 21:50, 9. Sep. 2014 (CEST)
(BK)Deine Artikelbaustelle befindet sich unter Benutzer:WLE Bahnmuseum mobil/WLE Bahnmuseum mobil. Dort kannst Du sie bei Vorliegen von Wikipedia:Relevanz zu einem Wikipedia:Artikel ausbauen und dann in den Artikelnamensraum entlassen. Im Wikipedia:Mentorenprogramm erfährst Du, wie Du erinen passenden Mentor findest. --Rôtkæppchen₆₈ 21:51, 9. Sep. 2014 (CEST)
Um herauszufinden, welche Ansprüche an Artikel über Eisenbahnmuseen gestellt werden, sollte der OP sich durch die Liste von Eisenbahnmuseen und die dort verlinkten Artikel durcharbeiten. -- Janka (Diskussion) 21:53, 9. Sep. 2014 (CEST)
Ich würde den Artikelgegenstand aber nicht als Museum einordnen, sondern als Wanderausstellung. --Rôtkæppchen₆₈ 22:23, 9. Sep. 2014 (CEST)

Bilddatierung

1917 oder 1915 entstanden? Mann oder Frau? Wer ist verletzt?

Woher weiß der Artikel Kriegsgefangene dass dieses Bild 1915 aufgenommen wurde? Und warum ist sich der Beschreibungstext auf Commons unsicher bzgl des Geschlechts des Soldaten? Wer ist verletzt? --Mythographus (Diskussion) 00:18, 10. Sep. 2014 (CEST)

Na ja, der historisch korrekte Weg zur Datierung wäre: nach dem 1. August 1914 und vor dem 27. Februar 1916 (Publikation, siehe Bildbeschreibung auf Commons). Der Beschreibungstext auf Commons ist sich vermutlich unsicher, weil die Vorstellung von Soldatinnen in der kaiserlich russischen Armee nicht gerade Allgemeingut ist... Nach dieser englischsprachigen Webseite gab es allerdings mehr als 6000 russische Kombattantinnen. Die Soldatin ist verletzt (so jedenfalls der Begleittext in der Publikation); ihr linkes Auge ist sichtbar zugeschwollen. Noch Fragen? --Concord (Diskussion) 01:35, 10. Sep. 2014 (CEST)
Danke. Ich hatte die unterschiedlichen Angaben irritierend gefunden und wollte auf einen möglichen Widerspruch aufmerksam machen, gerade weil Artikel und Beschreibungsseite unterschiedliche Angaben machen. Ich hatte mich bzgl. der Verletzung von der Person im Hintergrund irritieren lassen. In anderen Sprachen schien mir die Beschreibung einen männlichen Soldaten zu bezeichnen. --Mythographus (Diskussion) 01:54, 10. Sep. 2014 (CEST)
Tut es ja auch, bzw. sind die fremdsprachigen Texte, soweit ich sie verstehe, zum Teil wirr. Dazu kann man als Ursache annehmen, dass Benutzer:1970gemini zuerst den richtigen deutschsprachigen Text aus der Original-Bildbeschriftung (in der unbeschnittenen Version des Bildes noch zu sehen) und erst danach mit dieser Änderung die fremdsprachigen Texte aus einer automatischen Übersetzung mit Google Translate heraus eingefügt hat. Da Google Translate zuerst vom Deutschen ins Englische und dann vom Englischen in die weiteren Sprachen übersetzt, geht das grammatische Geschlecht (engl. soldier) und der Sinn auf diesem Weg völlig verloren, ein sogenannter Babelfish-Unfall. --Pp.paul.4 (Diskussion) 03:06, 10. Sep. 2014 (CEST)
Ah, das erklärt es natürlich. Ich weise ihn mal drauf hin. --Concord (Diskussion) 05:27, 10. Sep. 2014 (CEST)
Ich habe die Bildbeschreibung geändert. Das mathematische Datum "< 1917" habe ich ersetzt durch ein Datum in Worten: "published 27 February 1916". --Pp.paul.4 (Diskussion) 10:22, 10. Sep. 2014 (CEST)
Jetzt verstehe ich es erst richtig – denke ich: Benutzer:1970gemini hatte die fremdsprachigen Bildbeschreibungen womöglich aus den fremdsprachigen Bildverwendungen in den fremdsprachigen Wikipedias extrahiert und sich damit auf die mehr oder minder gute Qualität der Vorübersetzungen durch andere Wikipedia-Autoren verlassen. Google Translate liefert ja gar kein Tschuwaschisch zurück. --Pp.paul.4 (Diskussion) 14:01, 10. Sep. 2014 (CEST)
Ich bin der Ansicht, dass 1916 hier entfällt. Wenn dies ein Privatfoto ist, dürfen wird, wenn wir die heutige Entwicklungszeit verwenden, von mindestens zwei Wochen ausgehen. Der Fotograf wird nicht umgehend nach der Aufnahme nach Hause gegangen sein um es dort umgehend zu entwickeln und es in einer Zeitung zu publizieren. Ich gehe nicht davon aus, dass es bei den Lübeckische Anzeigen zuerst erschienen ist. In jener Zeit hätten sie mehr über den Urheber vermerkt. Abgesehen davon waren die Lübecker ()die komplette 81. IB, 17. RD) nicht an der Ostfront und die Lauenburgischen Jäger und das Lübeckische Kinderregiment (215) erst später. Ebenso glaube ich 1914 streichen zu können. Dass das Bild nicht von heute auf morgen nach Lübeck gekommen ist halte ich für ebenso wahrscheinlich, wie dass es nicht über ein Jahr dafür gebraucht haben wird. --1970gemini 14:32, 10. Sep. 2014 (CEST)

Win 32:Malware-gen

--178.200.165.210 11:12, 10. Sep. 2014 (CEST)

[7] Am besten neu aufsetzen bzw. PC aus Backup wiederherstellen. Alles andere ist für die Tonne. --88.130.92.213 11:17, 10. Sep. 2014 (CEST)
Computervirus! Gruß Thogru Sprich zu mir! 11:18, 10. Sep. 2014 (CEST)

Wathe - Fischerei

In einem Bericht aus dem Jahr 1692 heißt es:

  • Fischerei mit der Wathe auf der Spree

Die Wathe dürfte ein Netz sein? Die Befragung des großen Netzes bringt kaum brauchbare Ergebnisse. Kennt jemand diesen alten Begriff genauer und hat einen Tipp, wie man den kurz und knapp in einem Klammerzusatz erklären könnte? --Lienhard Schulz Post 22:24, 10. Sep. 2014 (CEST)

Bei Grimms gibts folgenden Eintrag. Grüße   • hugarheimur 22:33, 10. Sep. 2014 (CEST)
Super. Besten Dank und Gruß --Lienhard Schulz Post 22:37, 10. Sep. 2014 (CEST)
Archivierung dieses Abschnittes wurde gewünscht von: Yoursmile (Diskussion) 07:49, 11. Sep. 2014 (CEST)

Fehler im Artikel : der Dunkle Turm

Hallo zusammen,

im folgenden Beitrag: http://de.wikipedia.org/wiki/Der_Dunkle_Turm

gibts ein Fehler im Abschnitt "Bezüge zum Zyklus". Im buch "der Anschlag" reist Jake Epping ins Jahr 1958 und nicht ins Jahr 1954. Quersumme hin oder her. Da ich das Buch der Anschlag öfters gelesen habe (und es im eurem Wikipedia-Eintrag zum Anschlag Buch auch richtig drin steht), ist dieser Fehler mir aufgefallen...

VG Ingo

--217.110.57.2 17:19, 11. Sep. 2014 (CEST)
Hinweis: Für Fragen speziell zur Wikipedia gibt es ein eigenes Diskussionsforum: Fragen zur Wikipedia.

Auch Du bist Schöpfer und Verwalter unseres Wikiversums. Ändere es! Dieser Quersummenkrempel mit der 19 - stimmt das ? GEEZER… nil nisi bene 17:48, 11. Sep. 2014 (CEST)
Für Fragen zu den einzelnen Artikeln sind die jeweiligen Artikeldiskussionen da, hier also Diskussion:Der Dunkle Turm. --Rôtkæppchen₆₈ 17:51, 11. Sep. 2014 (CEST)

erledigt: https://de.wikipedia.org/w/index.php?title=Der_Dunkle_Turm&diff=133936671&oldid=132944898. Zum "Quersummenkrempel" siehe http://wiki.stephen-king.de/index.php/19_/_99 --Vsop (Diskussion) 18:39, 11. Sep. 2014 (CEST)

Na bitte! Das Jahr ist 1958 und der Quersummenkrempel ist nur Krempel und hat da - beim Jahr - nichts zu suchen. DUiiG. GEEZER… nil nisi bene 19:12, 11. Sep. 2014 (CEST)
Archivierung dieses Abschnittes wurde gewünscht von: ... oder? GEEZER… nil nisi bene 19:12, 11. Sep. 2014 (CEST)

Wer war A. A. Komarewa?

А. А. Комарева in Kyrillisch. Wird im Artikel Russischer Nihilismus mit einem Werk erwähnt. Ich werde weder bei Worldcat noch bei der russischen Nationalbibliothek fündig. Wer kann helfen? Grüße --Chricho ¹ ² ³ 20:47, 11. Sep. 2014 (CEST)

Alexandra Komarova wird manchmal die Autorinnenschaft von "Odna iz mnoghikh. Iz zapisok nigilistki" zugeschrieben. Hier steht viel über sie drin; immer unter Komarowa und Komarova suchen, da gibt's einiges an Material. Das Buch ist wohl von Petersburg 1881. --Aalfons (Diskussion) 21:27, 11. Sep. 2014 (CEST) Die hat das wohl wirklich geschrieben, habe die Lit. nur sehr flüchtig durchgesehen.
Ah, danke, also ein Tippfehler. Und das Buch hatte ich nicht gefunden, da die Härtezeichen heutzutage weggelassen werden. Dann finde ich es auch.[8] Danke! --Chricho ¹ ² ³ 22:04, 11. Sep. 2014 (CEST)
Siehe auch hier --92.106.184.230 22:05, 11. Sep. 2014 (CEST)
Archivierung dieses Abschnittes wurde gewünscht von: --Chricho ¹ ² ³ 22:04, 11. Sep. 2014 (CEST)

Anschluss Elektroherd

Nabend :-) Einen E-Herd in einer Mietwohnung ans Stromnetz zu frickeln muss laut Elektroherd gemäß der Niederspannungsanschlussverordnung durch eine Fachkraft durchgeführt werden. So weit so gut so schwammig. Mit der Fachkraft ist vermutlich ein gelernter Elektriker gemeint (oder sogar Meister?). Wie siehts mit nem Elektrikergesellen aus, der schon 10 Jahre in Rente ist? Habe leider in den Artikeln oder der allwissenden Müllhalde nix konkretes gefunden :-( Grüße und Danke, --Strange (Diskussion) 22:15, 8. Sep. 2014 (CEST)

Diese Verordnung bezieht sich nur auf den Anschluss von Häusern, Industrieanlagen etc an das öffentliche Elektrizitätsnetz. Zum Anschluss eines Elektroherdes genügt eine EUP, das schließt Elektrikergesellen, die schon 10 Jahre in Rente sind, mit ein. --Rôtkæppchen₆₈ 22:23, 8. Sep. 2014 (CEST)
Danke für die Auskunft :-) Dann steht allerdings im Artikel: Elektroherd in Abschnitt Anschluss Murx. Ich hatte von einigen Mietern was von einer Nachweispflicht gehört, dass der Anschluss durch einen Fachbetrieb gemacht werden müsste und im Moment hält das beim Küchenbau etwas auf ;-) Thx. --Strange (Diskussion) 22:45, 8. Sep. 2014 (CEST)
Sorry, Rotkaeppchen, aber das stimmt nicht. In § 13 (2) NAV heißt es ausdrücklich: „Die Arbeiten dürfen außer durch den Netzbetreiber nur durch ein in ein Installateurverzeichnis eines Netzbetreibers eingetragenes Installationsunternehmen durchgeführt werden.“ Und das bezieht sich auf die „elektrische Anlage hinter der Hausanschlusssicherung“, nicht auf den Anschluss der Hausanlage ans Netz. Elektrotechnisch unterwiesene Personen dürfen keine Installationen vornehmen (steht so auch in unserem Artikel). --Jossi (Diskussion) 23:22, 8. Sep. 2014 (CEST)
Direkt der Satz danach: Mit Ausnahme des Abschnitts zwischen Hausanschlusssicherung und Messeinrichtung einschließlich der Messeinrichtung gilt Satz 4 nicht für Instandhaltungsarbeiten. Ist das Anschliessen vom Herd also eine Instandhaltung? Ich behaupte mal ja. --Simius narrans 23:50, 8. Sep. 2014 (CEST)
Würde ich spontan nicht sagen. Instandhaltung dem Wortsinn nach ist die Sicherstellung, dass der vorher bereits bestehende, funktionsfähige Zustand erhalten bleibt oder bei Ausfall wiederhergestellt wird. Ihn erstmalig herzustellen ist dagegen Installation. --88.130.114.41 23:57, 8. Sep. 2014 (CEST)
Mit diesem Gedankengang war ich schon beim posten d'accord. --Simius narrans 00:02, 9. Sep. 2014 (CEST)
Ähm, und was heisst das jetzt für mein aktuelles Problem bezüglich Anschluss und pensioniertem Fachmann? --Strange (Diskussion) 00:26, 9. Sep. 2014 (CEST)
Ein Elektrikergeselle hat seine Ausbildung absolviert und dürfte damit eine Fachkraft sein. Wenn Du jetzt in einer Elektrofirma deswegen anrufst kommt auch nicht der Meister persönlich. Seine Qualifikation verliert er vermutlich auch nicht durch die Rente, Du evtl. aber den Nachweis durch Rechnung gegenüber dem Vermieter wenn die Bude dann doch abbrennt.--Simius narrans 00:42, 9. Sep. 2014 (CEST)
Faktisch kannst du davon ausgehen, dass ein Elektrogeselle, auch ein pensionierter, einen Elektroherd ordnungsgemäß anschließen kann; rechtlich zugelassen ist es nicht, denn die NAV spricht von einem „Installationsunternehmen“, nicht von einem Elektriker. An der Schule, an der ich früher gearbeitet habe, war der Hausmeister ausgebildeter Elektriker (mit Gesellenbrief); trotzdem durfte er nicht einmal eine kaputte Steckdose selber auswechseln, damit musste ein Elektroinstallationsbetrieb beauftragt werden. Was du jetzt aus dieser Sachlage machst, bleibt dir überlassen. --Jossi (Diskussion) 01:13, 9. Sep. 2014 (CEST)
Danke. Ich denke „Installationsunternehmen“ ist vermutlich das Stichwort. --Strange (Diskussion) 02:27, 9. Sep. 2014 (CEST)
Ja, wobei man das E-Unternehmen über die Akkreditierung eines Betriebes fragen kann, den so lt. §7 darf der Netzbetreiber die Arbeiten überwachen, was er in aller Regel delegiert. Es ist Dir aber klar, dass dieses Gesetz primär den Photovoltaik-Guerilleros »gewidmet« ist.
Ralph Diermann: Mini-Kraftwerke: Angriff der Photovoltaik-Guerilleros, SPON 23. September 2013
Das Einspeisen bezieht sich auf die Summe der Ströme, wobei einer Parallelschaltung von Sicherungen gleichkommt. Wenn nur ein Herd angeschlossen wird, kann das nicht passieren, außer der Elektriker hat im Sicherungskasten dieselbe Phase für 2 Leitungen benutzt, was in Mehrfamilienhäusern möglich werden könnte kann. Dem Laien ist das keines Falls zuzumuten. Nur sind im einen oder anderen Schwellenland, der ein oder andere 11-jährige in der Lage, unter Spannung einen Lichtschalter zu montieren. Soviel zum Fachkräftemangel und unserem weltbesten Schul- und Ausbildungssystem. --Hans Haase (有问题吗) 08:26, 9. Sep. 2014 (CEST)
Das hat mit der ursprünglichen Frage genau gar nichts zu tun. Die Vorschrift, dass Elektroinstallationen nur durch zugelassene Fachbetriebe durchgeführt werden dürfen, war schon in der AVBEltV von 1979 enthalten (noch ältere Normen habe ich jetzt nicht überprüft), als noch kein Mensch an Photovoltaik dachte. Die Begründung ist im vorliegenden Fall schlicht die, dass jedes Herumfummeln an 380 bzw. 400 V Drehstrom hochgradig lebensgefährlich ist. --Jossi (Diskussion) 10:24, 9. Sep. 2014 (CEST)
Das ist erst gefährlich, wenn keine Sicherung dahinter sitzt. Sonst ist es nichts anders als 3 Steckdosen an drei Sicherungen in einer. Das was am Herdanschluss anliegt führt ununterbrochen, jedoch über Sicherungen an die Steckdosen im Haus. Wirklich gefährlich wird es, wenn die Sicherungen nicht gekoppelt sind. Dann könntest Du den N schon einmal überlasten. Das wurde, als grün-gelb und blau noch gemeinsam grau waren tatsächlich gefährlich. Danach kam noch der FI-Schutzschalter dazu. Bei allem weiteren machen wir uns mit Unwissen zusehr zum Entwicklungsland auf hohem Niveau. Dies zeigte sich im Jahr 1999 in der Automobilbranche, auf den Patentämtern bereits etwas früher, Stichwort: Hybridantrieb. --Hans Haase (有问题吗) 15:17, 9. Sep. 2014 (CEST)
Wenn Du den Herd hast, hat der PE N und diverse L-Anschlüsse. Diese werden unter den Phasen verteilt. Dies füllt die Handbücher der Küchengeräte. Vom Herdanschluss läuft die Leitung direkt zum Sicherungskasten. Der Installateur muss nur darauf achten wie der anschließt. Beliebte Fehler sind fehlende Zugentlastung, falsche Aufteilung der Phasen, vergessene Schutzerdung (PE), unzureichend angezogene Schrauben, unsachgemäß montierte Kabel, fehlende Aderendhülsen. Laien würden hingegen N mit einer Phase vertauschen, das zur Zerstörung des Herdes durch Überspannung (400 statt 230V) führt. Klar, dass bei diesen Fehlern die Versicherer auf den Gesetzgeber Einfluss nehmen. Aber glaube nicht, dass beim „Fachpersonal“ auch Fehler passieren, dann einige von ihnen sind Quereinsteiger. --Hans Haase (有问题吗) 15:39, 9. Sep. 2014 (CEST)
Bei üblichen Elektroherden kommt noch dazu, dass sie sich für verschiedene Versorgungsspannungen und Phasenzahlen umkonfigurieren lassen (230 V 1~, 230 V 2~, 230/400 V 3~, 133/230 V 3~) und dass ein Neutralleiter in jedem Fall angeschlossen werden muss, da je nach Stellung der Bedienknöpfe die einzelnen Phasen unterschliedlich belastet werden. --Rôtkæppchen₆₈ 22:11, 9. Sep. 2014 (CEST)
Wo Du die 133 V herbekommst ist mit nicht klar, alles andre teile ich. Wenn Du Deinen Elektriker kennst und er Dir vertraut, kannst Du ihn fragen, ob Du selbst anschließen kannst und er »Dein Werk« nur abnimmt. Du solltest aber warten, bis er alles geprüft hat, bevor er die Sicherungen wieder reinmacht. --Hans Haase (有问题吗) 21:26, 10. Sep. 2014 (CEST)
Ich beziehe mich da auf die alten Drehstromnetze mit damals 127 Volt in Sternschaltung und 220 Volt in Dreieckschaltung. Nach neuer Norm müssten das ungerundet 13313 Volt in Sternschaltung und Volt in Dreieckschaltung sein. Derartige Netze waren früher in Spanien verbreitet und auch in Nordostdeutschland soll es vereinzelt noch ein paar derartige Netze geben. Mein 2005 gekaufter Elektroherd (bekannter deutscher Markenhersteller) lässt sich laut Installationsanleitung an ein derartiges Netz anschließen. --Rôtkæppchen₆₈ 01:09, 11. Sep. 2014 (CEST)

Was ist Sozialindustrie

--79.255.53.101 13:18, 9. Sep. 2014 (CEST)

Das ist ein politischer Kampfbegriff scheinbar aus dem linken Milieu. Eine trennscharfe, allseits akzeptierte und noch dazu seriöse Definition, was das genau ist, wird es da kaum geben. --88.130.77.236 13:25, 9. Sep. 2014 (CEST)
Kampfbegriff und linkes Milieu halte ich jetzt mal für eine TF, da auch in seriösen journalistischen Beiträgen davon öfters die Rede ist. Es meint, dass eine Menge Leute an dem Umgang mit Empfängern von staatlichen Sozialtransferleistungen gut oder gar sehr gut verdienen können und so viele öffentliche Gelder nicht den Betroffenen zukommen, sondern einen ganzen Wirtschaftszweig der professionellen Sozialverwalter und -kümmerer ernähren. Beispiel: Der villabewohnende und maseratifahrende Chef einer Hilfsorganisation von Obdachlosen (Treberhilfe Berlin). 82.207.169.99 13:30, 9. Sep. 2014 (CEST)
Gleich auf der ersten Googletreffer-Seite das versammelte linke Millieu: Handelsblatt, Focus, CDU, FDP, mindestens ein extrem fremdenfeindlicher Blog, ... --Eike (Diskussion) 13:35, 9. Sep. 2014 (CEST)
Für manche ist selbst PI noch linkes Milieu ... 82.207.169.99 13:40, 9. Sep. 2014 (CEST)
Ja und? Handelsblatt, Focus und Co schreiben auch über die NPD und sind trotzdem nicht rechtsextrem. --88.130.77.236 13:41, 9. Sep. 2014 (CEST)
Der gesamte Begriff ist TF. Der Sache nach geht es darum, dass es Firmen gibt, die gut davon leben, dass es Menschen gibt, die immer mehr vom Staat abhängig sind. Ob die wie gerne dargestellt gegen ihren Willen entmündigt werden, oder ob sie nicht sogar ganz gerne auf Kosten des Staates leben, lass ich mal dahingestellt sein. Der Punkt ist jedenfalls, dass es Unternehmer böse Kaptalisten gibt, die doch glatt Geld damit verdienen, den Mangel für den Staat zu verwalten. Ein Beispiel, das mir einfiele, ist das Abgreifen von Geldern aus dem Bildungs- und Teilhabepaket: KiTas füllen massenhaft entsprechende Anträge aus, um zusätzliches Geld abzugreifen, scheinbar oft ohne dafür - abgesehen von der so oder so erbrachten KiTa-Leistung - auch nur irgendeine Leistung anzubieten. --88.130.77.236 13:39, 9. Sep. 2014 (CEST)
20 x bei GoogleBooks. Zusammen mit Umschreibungen des Begriffs (Sozial-industrieller Komplex) wohl ein bissl mehr als TF. GEEZER… nil nisi bene 14:05, 9. Sep. 2014 (CEST)
Ich hab ja schon beschrieben, was ich unter dem Begriff verstehe, aber ich hab keine trennscharfe, allseits akzeptierte und noch dazu seriöse Definition gefunden, was das genau sein soll. Die Betreiber der Kitas oder auch von Nachhilfeinstituten werden naturgemäß ganz anderer Meinung dazu sein, ob sie das Geld überflüssigerweise abgreifen und ob sie dafür eine Gegenleistung erbringen und sich auch dagegen wehren, begrifflich in eine entsprechende Schublade gesteckt zu werden. --88.130.77.236 14:12, 9. Sep. 2014 (CEST)
Reicht Lexikon der Politikwissenschaft ? Da könnte man einen Artikel zu schreiben ... wenn man aus der Ecke kommt... GEEZER… nil nisi bene 14:54, 9. Sep. 2014 (CEST)
Hihi, Jörg Ueltzhöffer, Axel Murswieck und Martin Jänicke als "linkes Milieu"... Ein Lacher. Tja, so ist das hier manchmal. Nicht immer, aber ... --87.149.176.147 03:22, 11. Sep. 2014 (CEST)
Sozialindustrie ist die Erledigung staatlicher Aufgaben (insbesondere Transferleistungen nach den SGB, aber auch Bildung, Hochschulen, Straßenbau usw.) durch marktwirtschaftlich organisierte Unternehmen. Die politische Ausrichtung (Stichwort Neoliberalismus) führt, u.a. durch reduzierte Steuereinnahmen durch Steuervermeidungslegalisierung für Unternehmen, zu einer Reduzierung der öffentlichen Verwaltung auf Kernbereiche (innere/äußere Sicherheit).--Wikiseidank (Diskussion) 15:58, 9. Sep. 2014 (CEST)
Hm. In deiner verlinkten Quelle kommt der Begriff "Sozialindustrie" nicht vor. Wenn du etwas behauptest, unterlasse es doch bitte wenigstens, auch noch irrelevante Quellen einzubringen. Das spart anderen Zeit. --87.149.176.147 03:26, 11. Sep. 2014 (CEST)
Ich habe zwar keine Quelle, aber fürs erste ist für mich Sozialindustrie ein Wirtschaftszweig, der kommerziell Sozialarbeit anbietet. Also sozialpädagogische Einrichtungen, Bildungseinrichtungen, Heime usw. Wobei "Industrie" abschätzig wirkt. --217.189.212.50 16:14, 9. Sep. 2014 (CEST)
Vllt. sollte darunter zusammengetragen werden, wer den Begriff für was mis-/ge-/braucht. Das würde doch ganz schön aussagekräftig informativ werden. --Hans Haase (有问题吗) 16:57, 9. Sep. 2014 (CEST)
... gilt das nicht für jeden Begriff, der etwas Nichtmaterielles bezeichnet ..? GEEZER… nil nisi bene 19:04, 9. Sep. 2014 (CEST)

Selbstverständlich findet man den Begriff "Sozialindustrie" nicht in meinem Link. Der Begriff wird in der politischen Diskussion (undefiniert) verwendet, genau so wie "links", "rechts", "...ismus", "Staatsfernsehen" usw. Er zielt - wie ich sagte - darauf ab, dass die kapitalistische (Begriff!) Marktwirtschaft (Begriff!) sogar noch mit der gesellschaftlichen Grundversorgung Gewinnmaximierung (Begriff) anstrebt.--Wikiseidank (Diskussion) 12:41, 11. Sep. 2014 (CEST)

Wie bitte? Hast du den Lexikon-Link von Geezer nicht gelesen? Ist das keine Definition? Und woher hast du den Überblick über die Verwendung des Begriffs in der politischen Diskussion, um eine so pauschale Aussage zu machen? Das ist doch Blenderei. *kopfschüttel* --212.184.134.8 16:01, 11. Sep. 2014 (CEST) (hier auch als 87.149.176.147 unterwegs)

Wiki Bilder für die Schule...?

Darf ich Bilder aus dem Wikipedia in der Schule ausgedruckt benutzen? Hilfe!

Klar, für die volle Punktzahl aber mit Angabe des Urhebers und der Lizenz (wenn sich der Lehrer auskennt). "aus Wikipedia" oder "Foto: Wikipedia" reicht streng genommen nicht. --217.189.212.50 18:04, 9. Sep. 2014 (CEST)
Siehe Wikipedia:Weiternutzung#Weiternutzung von Bildern und Mediendateien. --sk (Diskussion) 07:05, 11. Sep. 2014 (CEST)

KURT STRAUß war ein deutscher Chirurg und Prof. an der Karls-Universität Prag

Ich fühle mich durch diesen Artikel in meinen Persönlichkeitsrechten (als Tochter des Vorgenannten) äusserst verletzt. Er besteht aus groben Unwahrheit und Falschdarstellung. Ich habe die Literatur der genannten Autoren alle gelesen und kenne ihre Quellen. Ich befasse mich seit Jahren mit der Vita des Obengenannten. Ic h möchte, dass dieser Artikel gelöscht wird auf Grund grober Verletzung meiner und meiner Familie Persönlichkeitsrechten.

--Pinschen (Diskussion) 22:35, 9. Sep. 2014 (CEST)

Kurt Strauß? Wenn du belegen kannst, dass Inhalte davon falsch sind, kannst du sie entfernen - das gleiche gilt für Inhalte, die dort nicht belegt sind. Aber "das muss entfernt werden weil ich das sage" geht nunmal nicht - könnte ja jeder behaupten. Der Artikel wird auch nicht gelöscht - die Person hat nunmal existiert und ist wichtig genug, um hier einen Artikel zu haben. Inhaltliche Diskussionen zum Artikel bitte auf der dortigen Diskussionsseite (beim Artikel verlinkt). --mfb (Diskussion) 23:56, 9. Sep. 2014 (CEST)
Darüberhinaus ist das postmortale Persönlichkeitsrecht eines Menschen, der schon über 70 Jahre tot ist, schon ziemlich verblichen. Ich vermute aber, dass Benutzerin:Pinschen mit ihrer Argumentation ohnehin keine Chancen hat, denn wenn schon müsste sie sich gegen den Autor des Buches richten, das die mutmaßlich falschen Inhalte enthält, und nicht gegen eine Enzyklopädie, die dieses Buch zitiert. --Rôtkæppchen₆₈ 00:04, 10. Sep. 2014 (CEST)
Was Du fühlst oder zu kennen glaubst, ist hier irrelevant, Dein Rechtsempfinden ebenso. In der Wikipedia geht es nur darum, basierend auf Belegen anerkanntes Wissen darzustellen. Sprich: Du kannst den Artikel ändern, wenn Du Deine Änderungen mit stichfesten Belegen begründest. Ob Dir das die Mühen wert ist, mußt Du selber entscheiden. --178.6.172.6 00:25, 10. Sep. 2014 (CEST)
Zwar stimmt es, dass Wikipedia vorhandenes Wissen und damit den Inhalt vorliegender Quellen abbildet, aber das heißt nicht, dass Wikipedia ein Interesse daran hätte, Fehler weiterzuverbreiten. Auch dafür, dass die Behauptung einer anderen Quelle nicht stimmt, bedarf es dabei einer reputablen Quelle. Wie das im vorliegenden Fall aussieht, weiß ich nicht: Wenn da jemand das subjektive Bild seines Vaters aufhübschen möchte, ist das sicher nichts, woran Wikipedia sich beteiligen sollte. Sind dagegen aber tatsächlich Behauptungen im Artikel, die objektiv falsch sind, dann sollte das korrigiert bzw. zumindest auf die anderen Meinungen der anderen Quellen hingewiesen werden. --88.130.92.213 16:10, 10. Sep. 2014 (CEST)
Ich seh hier auf Anhieb erstmal nichts, was nicht zuverlässig belegt wäre. Bitte erstmal sagen, was angeblich nicht stimmt.--Mautpreller (Diskussion) 16:13, 10. Sep. 2014 (CEST)
das Problem im konkreten Fall ist, dass hier eine unbelegte persönliche Meinung eines nahen (per se befangenen) Angehörigen (wobei diese Identität nicht belegt ist) auf eine belegte Darstellung aus Aussensicht trifft. Wie nahe Angehörige eine Person sehen, weicht regelmäßig sehr von der in der Geschichtsschreibung gängigen Sicht ab. Natürlich sieht jeder Mensch seine Angehörigen gern möglichst positiv. Eine wirklich tiefgehende Reflektion, die zu einer differenzierten Sichtweise führt, scheint hier zu fehlen, hingegen Ansicht und Emotion zu überwiegen. Es bleiben 2 Möglichkeiten: Gegennachweis mit nachprüfbaren Belegen oder die Existenz der Seite ignorieren. Die oben genannte Einlassung wird sicher keine Löschung bewirken. - 23:31, 10. Sep. 2014 (CEST)
Na ja, ich wäre schon bereit, offen zu diskutieren, ob eine Einschätzung in der Sekundärliteratur sachlich korrekt ist. Aber dann müsste man auch wissen, welche Einschätzung konkret in Zweifel gezogen wird und warum. Eine allgemeine Behauptung, der Artikel sei "falsch" und zu negativ, kann man aber nicht prüfen und diskutieren.--Mautpreller (Diskussion) 10:35, 11. Sep. 2014 (CEST)
Wobei man fairerweise auch dazusagen muss, dass nicht alle Behauptungen des Artikels nachvollziehbar mit einer Quelle versehen sind. Ich hab mich nicht im Detail mit dem Thema befasst, aber im letzten Absatz z.B. stehen zwar am Ende des Absatzes drei Quellen, aber was die jetzt genau belegen, geht daraus nicht hervor. Ja, die kommen von drei unterschiedlichen Autoren, was dafür sprechen könnte, dass das, was dadrin steht, soweit es übereinstimmt, auch tatsächlich stimmt. Muss es aber nicht und was genau aus dem Absatz jetzt durch welche Quelle belegt sein soll, wird auch nicht klar. Durchaus möglich, dass zwischendrin auch Sachen stehen, die momentan völlig unbelegt oder sogar falsch sind. Der Hinweis auf WP:IK ist sicherlich gerechtfertigt und so lange wir keine konkrete Aussage des Artikels haben, die angeblich falsch sei, gibt es für uns in der Tat nichts zu tun; die allgemeine Behauptung, "der Artikel" sei "falsch", genügt nicht. --88.130.122.191 11:44, 11. Sep. 2014 (CEST)
Alle wesentlichen Aussagen dieses Abschnitts sind durch Behrendts online zugängliche Diss. belegt, der sich wiederum auf diverse Archivbestände stützt, die er auch im Einzelnen angibt. Man kann natürlich sowohl den Inhalt der Akten als auch die Bewertung Behrendts in Zweifel ziehen, ich sehe aber momentan (noch) nicht, auf welcher Grundlage das geschehen sollte. Hachtmanns Fußnote, ebenfalls mit Verweis auf Aktenbestände, bestätigt einiges davon. Mir scheint das zunächst einmal gut belegt, besser, als das meist bei vergleichbaren Biografien der Fall ist.--Mautpreller (Diskussion) 11:57, 11. Sep. 2014 (CEST)
Mir war nicht bewusst, dass man hinter dem Link zur Diss den kompletten Text einsehen kann. Vielen Dank für's Nachschauen! Halten wir also fest: Im Moment ist nicht behauptet worden, dass eine konkrete Aussage falsch sei und offensichtlich grob falsch ist "der Artikel" in seiner Gesamtheit ja scheinbar auch nicht, sonst hätte das sicher schon jemand festgestellt und ggf. korrigiert. Schon deshalb gibt es nichts zu ändern. Und selbst wenn behauptet würde, dass eine ganz bestimmte Aussage falsch sei (was es wie gesagt nicht wurde, aber gehen wir nur mal hypothetisch davon aus), dann müsste die gegenteilige Behauptung sich selbst wiederum auf eine reputable Quelle stützen (persönliche Motive reichen für eine Änderung nicht aus, siehe WP:IK). Und selbst, wenn es danach zwei Quellen mit widersprüchlichem Inhalt gäbe, heißt das immer noch nicht, dass eine der beiden Behauptungen vollständig aus dem Artikel gelöscht werden dürfte: Stattdessen gebietet das Grundprinzip des neutralen Standpunkts vielmehr auf alle halbwegs vertretbaren Meinungen in angemessener Art und Weise hinzuweisen. --88.130.122.191 12:24, 11. Sep. 2014 (CEST)

Ich denke zu dem Thema ist hier erstmal alles gesagt. Ich hab diesen Text bevor er in ein paar Tagen auf Nimmerwiedersehen im Archiv verschwindet nach Diskussion:Kurt_Strauß übertragen. --88.130.122.191 12:37, 11. Sep. 2014 (CEST)

Zinssatz EZB

Könnte die EZB den Zinsatz soweit absenken, dass er statt jetzt 0,15 % bei 0 % liegt oder gar ins Negative fällt? Wenn ja, was würde dies für die Wirtschaft und die Privatleute bedeuten? --46.114.153.190 23:18, 9. Sep. 2014 (CEST)

Ein negativer Zinssatz wäre der Burner schlechthin. Das hieße ja, wenn ich heute eine bestimmte Summe Geld von der Bank hole, müßte ich denen morgen nur eine niedrigere Summe zurückzahlen, um schuldenfrei zu sein. Dann hole ich mir heute eine Billion Euro oder so, zahle morgen den niedrigeren Betrag zurück und wäre durch den einbehaltenen Differenzbetrag sofort superreich:))) Da würde unser gesamtes geldbasierte Finanzsystem ad hoc kollabieren. Bei 0 % muß naturgemäß Schluß sein und das wäre auch schon bescheuert genug. Diese 0 % Kredite kennt man schon aus den USA und das hat deren Wirtschaft nicht gerade stabilisiert … --178.6.172.6 00:36, 10. Sep. 2014 (CEST)
Die Billion musst du aber auch geeignet lagern können, sonst hast du sie nicht lange. Negativzins ist durchaus schon vorgekommen, aber soweit ich weiß nicht als Leitzins. --mfb (Diskussion) 01:58, 10. Sep. 2014 (CEST)
<quetsch>Soweit man hört, steht Fort Knox derzeit leer. Vielleicht kann ich da einen kleinen Lagerraum anmieten für meine Billion;) --178.0.193.156 12:28, 11. Sep. 2014 (CEST)
@Ip178

So funktioniert das nicht. Kredite kosten den Kreditnehmer immer positive Zinsen. Nur der Anleger zahlt eventuell für die (vermeintliche) Sicherheit seiner Anlage eine Prämie. Letztlich sind Kontogebühren bei einem Girokonto auch eine Art Negativzins. Die Banken sollen damit veranlaßt werden, ihr Geld nicht bei der Zentralbank zu parken. Daß das einer Bankrotterklärung der "Euro(rettungs)politik" gleichkommt, erwähne ich nur am Rande. Der Zins hat ja üblicherweise eine investitionsleitende Funktion. Nur die Investition, die über dem Marktzins rentiert, ist lohnenswert und bedeutet wirtschaftlichen Fortschritt etc. Das ist ein Grund für die ökonomische Rückständigkeit islamischer Länder trotz militärischer Macht (zB osmanisches Reich), weil die Umgehung des Zinsverbots eben Kapital vernichtet.--Geometretos (Diskussion) 10:55, 10. Sep. 2014 (CEST)

Privatleute haben keinen Zugang zur EZB, den haben nur Banken. Privatleute können wieder niedrigere Zinsen erwarten (keine negativen, die Banken wollen ja auch was verdienen). Zentralbanken vergeben auch normalerweise keine langfristigen Kredite.--Antemister (Diskussion) 19:07, 10. Sep. 2014 (CEST)
Das ist schon klar, aber so ist es leichter nachvollziehbar. „Ich“ steht in dem Fall natürlich genau genommen erst mal für eine Bank, aber mir ging es nur um den Rechenweg, nicht um die Mitspieler. Vom Einfachen zum Komplizierten - Details ergänzen geht immer. --178.0.193.156 12:28, 11. Sep. 2014 (CEST)

Amerikanische Kriegsgefangene in der Sowjetunion?

Dieser Artikel der deutschsprachigen Wikipedia sagt, dass es einige Kriegsgefangene der USA in der UdSSR gab, die im Stich gelassen worden seien. Ein englischsprachiger Artikel sagt das Gegenteil. Was stimmt? --Mythographus (Diskussion) 00:35, 10. Sep. 2014 (CEST)

Messy situation. Das Buch und viele Artikel dazu erschienen in den 1990er Jahren (als auch noch 50 japanische Gefangene von dort freigelassen wurden). Messy situation. GEEZER… nil nisi bene 09:36, 10. Sep. 2014 (CEST)

Wo sagt die deutsche WP, "einige Kriegsgefangene der USA in der UdSSR" seien von ihrem Heimatland "im Stich gelassen worden"? In Kriegsgefangene#Zweiter Weltkrieg steht lediglich: In dem 1992 erschienenen Buch Soldiers of Misfortune behaupten die Autoren, dass die Sowjetunion entgegen der Vereinbarung ca. 20.000 US- und ca. 30.000 Commonwealth-Soldaten zurückhielt und dass dies der US-Regierung bewusst gewesen sei.< James D. Sanders, Mark A. Sauter, R. Cort Kirkwood: Soldiers Of Misfortune; Washington's Secret Betrayal of American POWs in the Soviet Union. National Press Books, 1992. > Die Behauptung, die Bundesregierung verrate ihre Soldaten, ist ein beliebtes Thema der politischen Rechten in USA. Sie ist z.B. Grundlage der Rambo-Filme Silvester Stallones (insbesondere von Rambo II – Der Auftrag) und von vergleichbarem Realitätsgehalt. --Vsop (Diskussion) 12:20, 10. Sep. 2014 (CEST)

Danke, auch für den Hinweis, dass ich mich verlesen habe, sollte wir dann nicht den deutschsprachigen Artikel entsprechend ergänzen? --Mythographus (Diskussion) 16:29, 10. Sep. 2014 (CEST)

Enten-Popvideo gesucht

Ungefähr Mitte der 1980er wurde im ORF-Kinderprogramm recht häufig ein Zeichentrick-Video von einem Popkonzert mit gelben Enten gespielt. Die Lyrics bestanden nur aus "qua qua qua qua..." in verschiedenen Tonlagen; dem Leadsänger kann man bezüglich Gesicht und Tonfall vielleicht eine entfernte Ähnlichkeit mit Donald Duck zubilligen (jedenfalls im Vergleich zu anderen Zeichentrickenten wie Duffy Duck oder Alfred J. Kwak). Ich habe erfolglos versucht, dieses Video wiederzufinden. Kennt es jemand, und seinen Hintergrund? Vorweg sei klargestellt, dass es mit dem Ententanz nichts zu tun hat. --KnightMove (Diskussion) 16:34, 10. Sep. 2014 (CEST)

Star Wars Jedi Knight Jedi Academy Mehrspielermodus

Ich habe seit einiger Zeit wieder Lust Star Wars Jedi Knight Jedi Academy online zu zocken aber in letzter Zeit habe ich Probleme mit den Servern dieses Spiels. Seit einigen Tagen tauchen rund um die Uhr immer die gleichen Server auf die nur Computergesteuerte Bots enthalten. Ich kann keinen Server mehr finden wo echte Spieler mitspielen. Woran liegt das? Ich habe auch den Online Modus auf den v1.01 Stand gebracht wo mehr Server sein müssten. Kann mir da jemand bitte helfen? Was ist los mit den Servern? Warum konnte ich dieses Spiel vor einer guten Woche problemlos online spielen? 91.115.11.125 17:07, 10. Sep. 2014 (CEST)

Die anderen treiben alle Sport (Tennis, Kampfsport) oder spielen Gitarre! --Eike (Diskussion) 17:35, 10. Sep. 2014 (CEST)
Es gibt auch verschiedene kleine Mods. Die Server waren vor Jahren schon dünn besiedelt, ich könnte mir vorstellen, daß es nur noch ein oder zwei Server gibt, wo Menschen spielen.--217.87.76.136 18:11, 10. Sep. 2014 (CEST)-
Nein, da waren mehr Server sogar vor ein paar Wochen. Ich weiß wirklich nicht was mit der verfluchten Serverliste passiert ist da nur Bots vorhanden sind. 91.115.11.125 18:27, 10. Sep. 2014 (CEST)
Du könntest in einem Internetforum zum Spiel nachfragen. Die findet man zum Beispiel mit Google. --Eike (Diskussion) 18:30, 10. Sep. 2014 (CEST)
Super, da wär ich nicht draufgekommen! :O Was glaubst du was ich die ganze Zeit gemacht hab? 194.166.147.236 20:27, 10. Sep. 2014 (CEST)
Und da schreibt niemand? --88.130.92.213 23:18, 10. Sep. 2014 (CEST)
Dann zeig doch mal die Links auf die Fragen, die du "die ganze Zeit" in den passenden Foren gestellt hast. --Eike (Diskussion) 08:58, 11. Sep. 2014 (CEST)

Joghurteisfrage

Joghurt wird ja nicht nur wegen seines guten Geschmacks gegessen, sondern auch wegen seiner magen- und vor allem darmwohltuenden Eigenschaften (nein, laktoseintolerante Joghurtesser sind hier nicht gemeint). Wenn man sich nun an Stelle des Verzehrs eines Bechers Joghurt beim Eisdieler seines Vertrauens die Kugel gibt, nämlich eine Kugel der hausgemachten Geschmacksrichtung "Naturjoghurt", hat das dann die gleiche darmwohltuende Wirkung? --Hummelhum (Diskussion) 19:23, 10. Sep. 2014 (CEST)

Die darmwohltuenden Eigenschaften des Joghurts sind zum größten Teil ein Phantasieprodukt der Werbung. In einem gesunden Darm findet man Milchsäurebakterien, genauso wie in Joghurt, und daraus hat man geschlossen, dass Joghurt gut für den Darm sei. Eigentlich sollten aber mit der Nahrung aufgenommene Bakterien durch die Verdauung im Magen abgetötet werden, sonst hätten wir ja alle möglichen Bakterien sofort auch im Darm, und nicht nur die, die dort nützlich sind. - Die speziell beworbenen Bakterienstämme sollen ja der Darmflora besonders ähneln und werden daher aus was gezüchtet? ... Genau! Lecker!--Optimum (Diskussion) 20:00, 10. Sep. 2014 (CEST)
Udo Pollmer nannte das Scheiße zu Gold. Fäkal- oder Joghurtkulturen müssten magensaftresistent verkapselt sein, um lebend im Darm anzukommen. Die Bakterien im Joghurt kommen nur zum allerkleinsten Teil lebend im Darm an. --Rôtkæppchen₆₈ 22:42, 10. Sep. 2014 (CEST)
Genau aus diesem Grund geht man nun auch diesen Weg. --134.3.211.71 05:10, 11. Sep. 2014 (CEST)
Vergleiche zu diesem Thema auch Probiotika und Präbiotika. --Buchling (Diskussion) 06:55, 11. Sep. 2014 (CEST)
Das leckere Naturjoghurteis wir vermutlich viel Saccharose und Fette aus gesättigten Fettsäuren enthalten, die dem Darm eher nicht so gut tun. Die Endorphinausschüttung beim genußvollen Verzehr hingegen schon, wenn auch nur indriekt. Joghurt ist trotz allem durchaus gut für den Darm, aber nicht so, wie die Werbung es suggeriert und darum lohnt es auch das teuer Geld für probiotischen Joghurt nicht, denn die extra dafür aus Babykaka gewonnenen und in Brutkammern hochgezüchteten Laktobazillen sterben einen gar grausamen Tod bereits im Säurebad des Magens. --178.0.193.156 12:55, 11. Sep. 2014 (CEST)
Wer unbedingt probiotischen Joghurt als Darmwohlfühlplacebo haben will, kann sich das schweineteure Zeugs aus der Fernsehwerbung auch sparen: Einfach einen Joghurtbereiter kaufen und normale Milch mit ein bisschen probiotischem Joghurt impfen und den Markenjoghurt preiswert nachbauen. --Rôtkæppchen₆₈ 13:00, 11. Sep. 2014 (CEST)

Körper-Normaltemperatur wiederherstellen

--84.44.169.173 14:55, 12. Sep. 2014 (CEST) wie mein Körper zur Normaltemperatur wiederhergestellt

Bei Fieber: Bettruhe, viel trinken, kühle Wadenwickel und Arztbesuch (unter Umständen als Hausbesuch, auch durch einen Bereitschaftsarzt). --BlackEyedLion (Diskussion) 14:59, 12. Sep. 2014 (CEST)
Unter Umständen sind auch Antipyretika wie Ibuprofen, Acetylsalicylsäure oder Paracetamol angezeigt. Dabei aber unbedingt Packungsbeilage und Dosierungsanleitung beachten. --Rôtkæppchen₆₈ 15:30, 12. Sep. 2014 (CEST)
Bei Untertemperatur: Geh ins Warme. S.a. Körpertemperatur. Alles weitere erklärt Dir Dein Arzt besser selber. --178.4.178.242 15:11, 12. Sep. 2014 (CEST)
Archivierung dieses Abschnittes wurde gewünscht von: 178.4.178.242 15:11, 12. Sep. 2014 (CEST)

wann ich in der usa wohne und ein loch durch det erde bored bis der andere seite ... wo komm ich denn raus ????

Wenn ich in den USA wohne und ein Loch durch die Erde bohre bis zur anderen Seite - wo komme ich dann heraus? SCNR


--91.44.54.247 18:27, 8. Sep. 2014 (CEST)

Nimm einen Globus und dann wirst du sehen, wo das Bohrloch wieder herauskommt. --Schlesinger schreib! 18:30, 8. Sep. 2014 (CEST)
Weniger destruktiv wäre, einen Kartendienst zu nehmen und die Koordinaten entsprechend anzupassen. --mfb (Diskussion) 18:32, 8. Sep. 2014 (CEST)
an jeder beliebigen stelle. außer du beschließt, auch noch den erdmittelpunkt treffen zu wollen. --Jbergner (Diskussion) 18:33, 8. Sep. 2014 (CEST)
Nirgendwo, du verbrennst bereits im oberen Erdmantel. --BHC (Disk.) 18:35, 8. Sep. 2014 (CEST)
Antipode, Das China-Syndrom --Snevern 18:36, 8. Sep. 2014 (CEST)
Nimm deinen Standort in den USA (Breitengrad a Nord und Längengrad b West). Der Antipode befindet sich am Breitengrad a Süd, und 180-b Ost. Herauskommen wird ein Standort irgendwo im Indischen Ozean zwischen Madagaskar und Australien. --Bunkerfunker (Diskussion) 19:16, 8. Sep. 2014 (CEST)
Ich erinnere mich, dass es sowas vor einger Zeit mal als Website gab (vielleicht basierend auf Google Maps?). Man gab die Koordinaten eines Punktes an (bequem durch Hinzoomen und so), drückte dann einen Button und man bekam den exakt gegenüber liegenden Punkt zu sehen. --88.130.114.41 19:20, 8. Sep. 2014 (CEST)
Ja, hier: Schönes Spielzeug - ohne selbst rechnen zu müssen: Diercke Weltaltlas Antipoden-Karte.
Von Deutschland aus käme man im Südpazifik raus, südöstlich von Neuseeland. --Snevern 19:22, 8. Sep. 2014 (CEST)
Etwas Verwirrend ist auf dieser Seite, das der Initialzustand beim Aufruf zwar eine Position in Norddeutschland als Ausgangspunkt, jedoch den Schnittpunkt von Nullmeridian und Äquator als Zielantipode angibt. --Jogo.obb (Diskussion)
Ersteres ist bei mir auch der Fall - letzteres nicht: Da wird direkt der Südpazifik angezeigt. --Snevern 19:53, 8. Sep. 2014 (CEST)
Du kannst natürlich auch die Geokoordinaten mit EW±180° und NS im Vorzeichen gewechselt umrechnen und das mit Kartenmaterial simulieren, falls Du durch den Erdmittelpunkt bohren wolltest. --Hans Haase (有问题吗) 20:02, 8. Sep. 2014 (CEST)
Hans sagte es schon: Es kommt ziemlich auf den Winkel an, in dem du die Erde anbohrst. Die meisten praktischen Bohrungen in den USA, die auch wieder irgendwo hinausführen beginnen und Enden in den USA. Yotwen (Diskussion) 21:26, 8. Sep. 2014 (CEST)
The Blue Marble: Land Surface, Ocean Color and Sea Ice
Blue Marble Antipodenkarte
Aus einer Plattkarte wie der “The Blue Marble: Land Surface, Ocean Color and Sea Ice” lässt sich mit einem Bildverarbeitungsprogramm leicht eine Antipodenkarte herstellen, auf der dann auch Laien leicht selbst Antipoden herausfinden können, ohne Koordinatenrechnerei. --Rôtkæppchen₆₈ 21:40, 8. Sep. 2014 (CEST)
Außer Argentinien und China hat fast niemand einen ordentlichen Antipoden!? Ich glaube es ist Zeit für eine Verschwörungstheorie... --92.202.108.79 23:41, 8. Sep. 2014 (CEST)
Ich schlage noch Grönland–Antarktis und Spanien–Neuseeland vor. Und von einigen südostasiatischen Inseln kannst Du Dich nach Südamerika durchbohren. Was, der Bohrer schmilzt unterwegstut unterwegs schmelzen? Mist! --Rôtkæppchen₆₈ 00:46, 9. Sep. 2014 (CEST)
Lieber einen Bohrer nehmen, der schmelzt! Tja, Landmassen auf der Südhalbkugel sind rar. Hawaii - Südafrika passt auch gut. --mfb (Diskussion) 02:21, 9. Sep. 2014 (CEST)
Es ist zwar ca. 30% der Erdoberfläche fest, aber nur von einem viel geringeren Prozentsatz (den ich aber nicht kenne) trifft man bei einer senkrechten Bohrung wieder auf Land. Wenn man aber den Ozean trifft (von unten!), müsste man dann eigentlich einen gigantischen Springbrunnen erzeugen, oder? --91.41.235.231 15:28, 9. Sep. 2014 (CEST)
Es ist aus mehreren Gründen unmöglich, die Erde zu durchbohren. Die Temperaturen sind bereits im Erdmantel so hoch, dass jedes bekannte Bohrermaterial versagen wird. Außerdem wird sich an der Bohrstelle ein Vulkan bilden, der Deinen Bohrturm zerstört. Zudem ist der innere Erdkern fest, superrotiert aber. Das heißt, dass man den Erdkern durch ein Bohrung in mehrere Stücke zerteilen muss, damit die Bohrung sich nicht sofort wieder schließt. Bei einer Bohrung am Äquator würde der innere Erdkern in zwei Halbkugeln zerteilt, bei einem anderen Bohrstandort wären es drei kegelförmige Teile. Nur wenn Du die Ende polar durchbohrst, reicht eine simple Bohrung. Deine Bohrung wäre also bereits mehrfach gescheitert, bovor das Meerwasser der anderen Seite eine Chance hätte, durch die Bohrung zu strömen. --Rôtkæppchen₆₈ 21:22, 9. Sep. 2014 (CEST)
Och, wie phantasielos. Gegen hohe Temperaturen und ein Einbrechen der Wände bräuchte es eine stabile und gut isolierende Wandverkleidung des Bohrkanals, evtl. mit zusätzlichen radiären Injektionen zur Stabilisierung durch dünne Bohrkanäle, und weiterentwickelte Bohrer - eine charmante Herausforderung für Materialwissenschaftler und Tiefbauingenieure. Laß denen noch ein paar Jahre Zeit, dann wird das schon;) Bezüglich der Superrotation des Erdkerns wäre es - sofern ich als Laie das richtig verstehe - letztlich nur eine Frage der Geschwindigkeit bzw. der zeitlichen Abstimmung; also, Mathematiker an die Front! Das mit den Halb- bzw. Drittelkugeln halte ich für ein Gerücht. Also: Good luck:) --178.6.172.6 00:03, 10. Sep. 2014 (CEST)
Da kannst du nichts abstimmen - eine dauerhafte Verbindung (so, dass Wasser durchkann) funktioniert einfach nicht, außer der Tunnel ist so stabil, dass er ein Loch in die (teilweise relativ zum Tunnel rotierenden) Erde schneiden kann. Das Wasser wäre auf beiden Seiten dann auf der gleichen Höhe relativ zum Meeresniveau, bei einem Tunnel zwischen Meer und Land also ggf. irgendwo tief unten im Bohrloch (je nach Höhe des Landteils über dem Meer). Der Tunnel ist ja nur eine kürzere Verbindung, "um die Erde herum" geschieht aber das gleiche mit den Weltmeeren. --mfb (Diskussion) 02:48, 11. Sep. 2014 (CEST)
Mal (faktenwidrig) vorgestellt, man könnte so ein Loch bohren und offenhalten: Dann würde das eindringende Wasser enorm erhitzt. Würden sich dann nicht heftige Konvektionsströme mit gewaltigen Geysiren an den Öffnungen bilden? Wäre am Nord- und Südpol doch sehr schick. Rainer Z ... 15:38, 11. Sep. 2014 (CEST)
Dazu reichen die ersten Kilometer, danach ist der Temperaturgradient gering. Und heftige Konvektion braucht wohl ein heftig großes Loch. Zwar nicht am Nordpol, aber in Island ist das schon sehr schick. --mfb (Diskussion) 15:50, 11. Sep. 2014 (CEST)
Nunja, man müßte halt schauen, daß sich die Tunnelabschnitte, die man in den Erdkern bohrt, immer wieder zusammenfinden im Zuge der Rotation, dann hätte man zwei wunderbar pulsierende Geysire. Würde sowas eigentlich (rein rechnerisch natürlich, noch haben wir den Tunnel ja nicht;) die Temperatur im Erdinneren erniedrigen können, wenn der Bohrkanal groß genug dimensioniert ist? --178.0.193.156 21:04, 11. Sep. 2014 (CEST)
Um einen künstlichen Geysir zu schaffen, musst Du gar nicht so weit bohren. In 14 Kilometer Tiefe ist es schon 300 °C warm. Aber auch in viel geringeren Tiefen reicht die Temperatur schon zur Erzeugung von Wasserdampf. Siehe auch Geothermie. --Rôtkæppchen₆₈ 21:18, 11. Sep. 2014 (CEST)

Mehrfaches Überschreiben notwendig?

Aus gegebenen Anlass: Stimmt es, dass Daten auf einer Festplatte schon nach einmaligen Überschreiben dieser nicht mehr wiederhergestellt werden können? Offizielle Empfehlungen sind ja bis zu 5 mal überschreiben. Das kann relevant sein, wenn man seinen Rechner mit Festplatte ggf. verkaufen will. --188.100.24.194 22:48, 8. Sep. 2014 (CEST)

Diese uralte Empfehlung der US-Regierung, Festplatten mehrfach zu überschreiben, bezieht sich auf eine Festplattenbauart, die seit langem nicht mehr hergestellt wird. Bei Festplatten mit GMR-Effekt genügt einmaliges Überschreiben. Bei Disketten und Magnetbändern kann mehrfaches Überschreiben aber durchaus noch sinnvoll sein. --Rôtkæppchen₆₈ 22:58, 8. Sep. 2014 (CEST)
  1. (BK)Eine Festplatte ist heute relativ billig, also vor Verkauf ausbauen und vor eine Dampfwalze werfen.
  2. POV: Vor ca. 30 Jahren hat ein mir bekannter Student als Job Festplatten mehrfach überschrieben. Begründung damals: Die Spurgenauigkeit des Kopfes ist nicht sehr genau, so das beim Überschreiben rechts oder links noch lesbare Bereiche übrig blieben. Mit entsprechenden Messgeräten waren die Daten rekonstruierbar; natürlich nicht für Otto Normalbürger. Ob das heute noch so ist, keine Ahnung. Gruss --Nightflyer (Diskussion) 23:05, 8. Sep. 2014 (CEST)
Was Heise schon vor Jahren dazu schrieb. --Snevern 23:08, 8. Sep. 2014 (CEST)
Eine Festplatte vor ne Dampfwalze werfen bringt einen doch sicherlich in Konflikt mit dem Elektronik-Recycling-Gesetz oder? --92.202.108.79 23:20, 8. Sep. 2014 (CEST)
Es hindert dich niemand daran, die überfahrene Platte von der Straße abzukratzen und im Wertstoffhof in den Elektroschrottcontainer zu werfen. Dass die WEEE-ID nicht mehr lesbar ist, soll Problem des Entsorgers sein. --Rôtkæppchen₆₈ 23:33, 8. Sep. 2014 (CEST)

Auf den Seiten des Datenwiederherstellungsexperten lese ich "aussichtslos ist eine Daten-Wiederherstellung, falls ein Sektor (...) überschrieben wurde"[9]. Gleichzeitig bietet derselbe Anbieter für überschriebene Datendateien aber auch das "Wiederherstellen [mit] speziellen Softwaretechnologien und Entwicklungsmethoden"[10] an. Wie kann das sein? Entweder man kann die Dateien wiederherstellen, oder man kann es nicht - krollontrack kann es scheinbar beides gleichzeitig. Machen die das Unmögliche möglich oder was? --88.130.114.41 23:22, 8. Sep. 2014 (CEST)

Das ist Marketing. Die machen dem Leser erstmal Panik, dass seine Daten unweigerlich im schwarzen Loch der Datenlöschung verloren sind, um dann eine erlösende Rettungsleine anzubieten. Bei sowas verdrehen sich mir die Augen von ganz allein. Es fehlt nur noch ein Spruch à la "Einmal gelöschte Daten sind unweigerlich verloren, aber unsere Experten können Ihre wertvollen Daten mithilfe von speziellen Softwaretechnologien und Entwicklungsmethoden wiederherstellen. Wenden Sie sich vertrauensvoll an uns." --78.52.76.236 23:40, 8. Sep. 2014 (CEST)
Gelöscht ist eben nicht immer gleich gelöscht. Weil üblicherweise wird nur der Sektor freigegeben und als unbelegt bezeichnet, und nicht unmittelbar überschreiben. Der Ottonormalverbraucher kommt aber an so gelöschte Dateien -besser aus dem Register gelöschte Dateien- schon nicht mehr heran. Der Fachmann unter Umständen eben schon. --Bobo11 (Diskussion) 00:05, 9. Sep. 2014 (CEST)
Ähm. PC Inspector File Recovery? Recuva? Überschrieben ist überschrieben, alles andere kann auch der Laie. Aber okay, ich lass mich gernen auch dafür bezahlen. ;) --92.202.108.79 03:00, 9. Sep. 2014 (CEST)
krollontrack spricht nicht von einfachem Löschen (etwa durch Entfernen aus der Master File Table), sondern in beiden Fällen explizit von Überschreiben. --88.130.114.41 00:08, 9. Sep. 2014 (CEST)
Zu Diskettenzeiten war das Risiko eines Diskettenfehlers beim Speichern noch etwas größer. Es hat sich deshalb durchgesetzt, dass Anwendungsprogramme zuerst in eine Temporärdatei speichern und erst dann, wenn die neue Version des Dokuments in der Temporärdatei komplett erfolgreich gespeichert ist, die alte Version zu löschen, die Temporärdatei umzubenennen und eine weitere Temporärdatei zur weiteren Bearbeitung anzulegen. Dadurch können auch bei vermeintlich anwendungsseitig „überschriebenen“ Dateien noch alte Versionen gelöscht, aber wiederherstellbar existieren. Das ist mindestens seit 36 Jahren (WordStar für CP/M) so. --Rôtkæppchen₆₈ 00:24, 9. Sep. 2014 (CEST)
(BK) Genaugenommen sagen sie zusätzlich zum durch (...) verkürzten Zitat: Der wirtschaftliche Aufwand der Wiederherstellung bei überschriebenen Sektoren ist meist unangemessen hoch. ["Was isses Dir wert?"] und Krollontrack sagt: ..überschriebene Datendateien.. --> ..zum Wiederherstellen beschädigter Daten.. ["wir bearbeiten auch überschriebene Dateien"] sowie ..Überschriebene logische Volumes..--> ..Methoden, um das ursprüngliche Volume zu lokalisieren. Nach der Zusammensetzung wird eine Liste mit allen wiederherstellbaren Dateien erstellt... ["Wir bearbeiten auch überschriebene Partitionsinformationen"] und: Überschriebene Daten --> Daten, die nur teilweise beschädigt sind, lassen sich auch ohne die überschriebenen Datenbereiche extrahieren. .. ["Von großen Dateien liefern wir Dir die paar % die nicht überschrieben wurden"].--Simius narrans 00:27, 9. Sep. 2014 (CEST)
Doch wiederherstellbar ? vor langer Zeit: Ach hätten doch en:Soft sectored noch als Artikel oder Weiterleitung. --Hans Haase (有问题吗) 08:06, 9. Sep. 2014 (CEST)
Ob was überschriebenes wieder herstellbar ist oder nicht, ist neben dem rein technischen Aspekt (wie viel Restmagnetismus der alten Datei ist noch vorhanden), auch davon abhängig ob der neue Inhalt bekannt ist. Wenn ich die aktuelle Datei kenne die über der alten liegt, ist es viel eher möglich das alte trotzdem noch mal hervor zu holen, als wenn der Sektor mit eine Zufallszahl überschrieben wurde. Heute kann man durchaus sagen ein einmaliges überschrieben mit einer Zufallszahl reicht in der Regel um eine gelöschte Datei für immer unlesbar zu machen. --Bobo11 (Diskussion) 09:15, 9. Sep. 2014 (CEST)
Das mit dem aktuellen Inhalt leuchtet mir so gar nicht ein. Ob das nun Zufallszahlen sind oder nicht - ich kann sie ja schlicht durch Einlesen rauskriegen.
Hier nochmal was Aktuelles von der c't: IT-Mythen. Mein Eindruck ist, dass die Daten heutzutage nicht mehr so "breit" gespeichert werden, dass da Platz für die alten Daten bliebe.
--Eike (Diskussion) 09:24, 9. Sep. 2014 (CEST)
@Bobo11: Bei GMR-Technik kippt immer die komplette Domäne um. Da bleiben keine Randbereiche mit alter Magnetisierung wie in der Frühzeit der magnetischen Aufzeichnung, wo ein Bit viele Domänen umfasste. --Rôtkæppchen₆₈ 09:46, 9. Sep. 2014 (CEST)
@Rotkaeppchen68: Nicht wenn die Zufallszahl als Datei lesbar hinterlegt wurde. Du also nicht weist wie viel Bit des gesamten Bites überhaupt geändert wurde. Weil wenn du jetzt 011101001011 ausliest, was ist davon neu und was Überrest von vordere Datei? Je öfters der Sektor nach dem System wieder beschreiben wurde, desto schwerer wird es dann heraus zu finden was alte Spuren sind. Es kommt dann natürlich immer bisschen auf Dateityp an der die gelöschte Datei hatte. Der Haken an der GMR-Technik all Leseschutz ist folgender. Das Bit darf nur eine Domäne gross sein, und auch keine Lücke dazwischen. Ob das heute immer garantiert werden kann ist durchaus fraglich. Ansonsten haben wir das genau gleiche Problem wie beim Magnetrad. Aber die Hauptgefahr ist immer die, dass die Datei die man für immer gelöscht haben möchte, irgendwo als nicht lesbarer -für Otto-Normal-Verbraucher versteht sich- Duplikat noch rumliegt. Darin liegt heute die Hauptgefahr, dass eben nicht alles gelöscht und überschrieben wurde, sondern nur aus der Master File Table gelöscht wurden. Dagegen hilft wirklich nur die physikalische Zerstörung der Festplatte. Oder ich überschreibe wirklich die ganze Festplatte mit Zufallszahlen/Zufalllsdateien. --Bobo11 (Diskussion) 19:06, 9. Sep. 2014 (CEST)

Ich würde anhand der Aufzeichnungstechnik der Festplatte entscheiden: Wenn es eine ältere Platte ist, die mit Longitudinal Recording arbeitet, kann sich ein zwei- bis dreimaliges Überschreiben lohnen, zumindest das Gewissen beruhigen. Zeitgenössische Platten zeichnen Perpendicular auf. Dieses Verfahren ist so fragil, dass man der Platte den Stress des mehrmaligen Überschreibens ersparen kann. --79.216.210.6 15:41, 9. Sep. 2014 (CEST)

Fragen in dieser Richtung kamen schon öfters in der Auskunft, z.B. [11][12][13][14]. Man könnte sie vielleicht mal in die FAQ eintragen. --Grip99 01:52, 12. Sep. 2014 (CEST)

Abitur mit 16

Bernd Luckes Sohn hat sein Abitur mit 16 Jahren gemacht. Wie ist das möglich, wenn er nicht mit 3 Jahren eingeschult wurde? --95.112.151.15 23:12, 8. Sep. 2014 (CEST)

Wieso sollte er dazu mit drei eingeschult worden müssen sein? Abi mit 16 ist problemlos möglich; da gibt es eine ganze Reihe von Gründen. Z.B. vorzeitige Einschulung oder Überspringen eines oder gar mehrerer Schuljahre. Ob es das oder was genau es davon in diesem Fall war, kann ich nicht sagen - je nach Geburtszeitpunkt kann aber schon nur ein einziges Jahr ausreichen. Noch dazu hat man heutzutage idR schon nach zwölf Schuljahren Abi und nicht etwa erst nach dreizehn. Schließlich kann die Information, er habe mit 16 Abi gemacht, auch schlicht falsch sein. --88.130.114.41 23:15, 8. Sep. 2014 (CEST)
Üblich ist das Überspringen von Schuljahren. Wer mit 5 eingeschult (nicht unüblich), hat im G8 mit 17 das Abitur (und kommt noch minderjährig an die Uni). Ein Schuljahr überspringen reicht da also aus, ich kenne sogar einen der zwei Schuljahre übersprungen hat und mit 15 angefangen hat zu studieren.--Antemister (Diskussion) 23:38, 8. Sep. 2014 (CEST)
Manche Hochschulen denken wohl auch schon darüber nach Elternabende zu veranstalten. --Jogo.obb (Diskussion) 09:41, 9. Sep. 2014 (CEST)
Die Unis denken nicht nur darüber nach, die machen das sogar schon: [15]. --Jossi (Diskussion) 10:29, 9. Sep. 2014 (CEST)
klar geht das, siehe bitte auch Springer (Schüler) und gleich weiter an die Uni wenn er mag [16] :). --Advanceddeepspacepropeller (Diskussion) 11:14, 9. Sep. 2014 (CEST)
Ich habe das ukrainische Abitur mit 15 gemacht. Dort wird man aber mit 7 eingeschult und lernt 10 Jahre (zweimal übersprungen, die 3. und die 6. Klasse). Half aber nichts - in Deutschland musste ich wieder in die Schule (in die 11. Klasse), obwohl vom Programm her man hier in 13 Jahren nicht mehr lernt als in der Ukraine in 10 (das, übrigens, zu den ganzen Beschwerden über "Turbo-Abi". Wenn 12 Jahre "Turbo" ist, was würden deutsche Eltern über die ukrainische Schule sagen?).--Alexmagnus Fragen? 12:45, 9. Sep. 2014 (CEST)
Ich bin mit 5 eingeschult worden und hatte bei damals üblichen G9 ohne Überspringen auch mein Abi mit 17. Bei G8 (war ja in einigen Bundesländern auch vor der Reform üblich) sollte das also kein Problem sein. 82.207.169.99 13:23, 9. Sep. 2014 (CEST)
Aufgewachsen in Winsen an der Luhe bei Hamburg übersprang er eine Klasse, gehörte zum ersten G-8-Jahrgang und machte Abitur, als er gerade 16 war. ([17]) --Grip99 01:50, 12. Sep. 2014 (CEST)

Inschrift

Ich weiss, es ist fast aussichtslos, aber fragen kann man ja... Hat irgend jemand eine Idee, wie man die Inschrift entziffern kann? Kann man das wenige, was man sieht irgendwie mit Photoshop oder so nachziehen und deutlicher machen? --Concord (Diskussion) 23:54, 8. Sep. 2014 (CEST)

Das Bild müsste zuerst mal um 90° nach links gedreht werden. --88.130.114.41 23:59, 8. Sep. 2014 (CEST)
Gedreht. --Concord (Diskussion) 03:36, 9. Sep. 2014 (CEST)
Hast du nur das Foto oder auch Zugang zu dem Ding und könntest du dort die Papierreibetechnik anwenden? GEEZER… nil nisi bene 08:16, 9. Sep. 2014 (CEST)
In Photoshop und vielen anderen Bildbearbeitungsprogrammen: Solarisation oder Scharfzeichnen wählen. --BlackEyedLion (Diskussion) 09:21, 9. Sep. 2014 (CEST)
Ich habe jetzt alle möglichen Filter drüberlaufen lassen – ich fürchte, da ist wirklich nichts zu machen. Die Kontrastunterschiede zwischen der Schrift und der Steinoberfläche sind einfach zu gering. Da kann man wohl nur am Original mit Abreiben mehr herausfinden. --Jossi (Diskussion) 10:46, 9. Sep. 2014 (CEST)
Oder mit einem Abklatsch. Geoz (Diskussion) 12:52, 9. Sep. 2014 (CEST)

Zumindest die Jahreszahl 1956 ist unten links erkennbar, darüber noch ein Die. Vielleicht wissen die Lübecker (LMG) mehr. --G-Michel-Hürth (Diskussion) 13:31, 9. Sep. 2014 (CEST)

Wenn das Ding real irgendwo rumliegt könnte man viele Fotos aus unterschiedlichen Perspektiven machen, dann mittels [18] VisualSFM ein 3D-Modell berechnen, und dann mit Phong Shading o.ä. versuchen, die Inschrift zu entziffern. Einfacher dürfte aber "hingehen und nachzeichnen" sein. --Tbhgeo (Diskussion) 14:08, 9. Sep. 2014 (CEST)
LMG? Lübecker Metallgesellschaft? Oder wieso Lübecker? Wo liegt das Teil denn? Ggf. könnte ich mal nachsehen... Gr., redNoise (Diskussion) 14:19, 9. Sep. 2014 (CEST)
Lübecker Maschinenbau-Gesellschaft. --Rôtkæppchen₆₈ 14:20, 9. Sep. 2014 (CEST)
Ja, verbibscht, die mein' ich doch! Gr., redNoise (Diskussion) 14:59, 9. Sep. 2014 (CEST)
Die Bildbeschreibung auf Commons lautet: "Ansicht eines Gedenksteines auf dem ehemaligen LMG-Gelände, nicht mehr am Origialstandort." Viel Spaß bei der Suche auf dem Gelände :-) 82.207.169.99 14:27, 9. Sep. 2014 (CEST)
Nassmachen und gegen das Licht fotografieren. --Rôtkæppchen₆₈ 14:23, 9. Sep. 2014 (CEST)
Ja, um als Amateur z.B. alte Grabsteine zu fotografieren, soll man die Inschrift nassmachen, zu verschieden Tageszeiten mit unterschiedlichem Lichteinfall hingehen, mit Blitzlicht Schatten erzeugen usw., siehe wikihow:how to photograph a tombstone. Hier sollte man zunächst den Hochlader um die Koordinaten (den Standort) bitten und die Koordinaten dann in der Bildbeschreibung hinzufügen. Ich lese links unten evtl. die Jahreszahl 1936. Man kann für ein Foto die Inschrift auch mit Wasserfarbe aus dem Schulmalkasten nachziehen. --Pp.paul.4 (Diskussion) 15:00, 9. Sep. 2014 (CEST)

Ist bekannt wo genau auf dem Gelände der Stein liegt? Die LMG ist eine ziemlich große Industriebrache in deren nicht baufälligen Bereichen sich Klein(st)unternehmer und Privatleute angesiedelt haben, die es allesamt nicht sehr gern sehen wenn man da suchend rumschlendert. Mit etwas besserer Positionsangabe kann man vor Ort rumfragen ob man da mal hin darf. Im öffentlich zugänglichen Areal liegt er jedenfalls nicht. Wenn der Stein aus ~ WKII stammt, es gibt dort einen Hochbunker, Bauweise Winkelturm der ist aber nicht aus Backstein, dort liegt er also auch nicht.--Simius narrans 15:30, 9. Sep. 2014 (CEST) edit: grade bei Jan angefragt.

Frag am besten Benutzer:JanTappenbeck oder c:User:JanTappenbeck. --Rôtkæppchen₆₈ 15:37, 9. Sep. 2014 (CEST)

Fragesteller Concord verschweigt, dass auf Portal Diskussion:Lübeck#unbekannte Gedenksteine auf dem ehemaligen LMG-Gelände bereits vorher eine Diskussion lief und dass Jan Tappenbeck dort sowohl die Koordinaten als auch Fotos des Fundortes und der noch lesbaren Inschrift auf dem „Schwesterstein“ unmittelbar neben diesem Stein verlinkt hat. --Pp.paul.4 (Diskussion) 17:03, 9. Sep. 2014 (CEST)

Ja, das wäre allerdings eine hilfreiche Information gewesen; ebenso das die Fotowerkstatt auch angefragt wurde: [19]. Hach, wie gerne macht man sich unnötige Gedanken... --Tbhgeo (Diskussion) 15:45, 11. Sep. 2014 (CEST)
Stein nach Wasserbehandlung...
Die Gedanken waren durchaus nicht unnötig, manchmal hilft es, ganz neu an eine Herausforderung heranzugehen... update: nachdem inzwischen ein neues Foto nach Wasserbehandlung hochgeladen wurde, steht die Inschrift relativ fest:

Stromausfall-Datenbank

Hi! Wie man deutlich sieht, war am 2014-08-19 zwischen 06:00UTC und 08:00UTC (8-10 Uhr Ortszeit) irgendwie der Luftstrom am Luftabzug niedriger als normal (der Sensor gibt nie 0 aus, weil ich ihn selbst gelötet hab und die zugrunde liegende Schaltung nicht richtig verstanden habe). Die Netzbetreiberin (Stadtwerke Brunsbüttel) und die örtlichen Internet-Zeitungen sagen dazu nix. Mein großer Puter, der noch nich an mein 8V bis 12,5V Notstromnetz angeschlossen ist, war plötzlich aus, aber mein TV-Fax-WWW-Server und das DSL-Router-Box-Dings und das VoIP-Telefon gingen noch. Wo kann man nachsehen, ob das ne größere Unterbrechung (z. B. die ganze Straße) war, oder ob meine Nachbarn mir und der Zwangsbelüftung die Sicherung rausgedreht haben? Thx. Bye. --Heimschützenzentrum (?) 09:59, 9. Sep. 2014 (CEST)

Frag deine Nachbarn. Radiowecker reagieren auf einen Stromausfall eigentlich immer. --Eingangskontrolle (Diskussion) 18:00, 9. Sep. 2014 (CEST)
den Nachbarn trau ich nur soweit, wie ich sie werfen kann... :-) --Heimschützenzentrum (?) 00:06, 10. Sep. 2014 (CEST)
Alle seine Nachbarn haben eine Pufferbatterie im Radiowecker. --Rôtkæppchen₆₈ 00:22, 10. Sep. 2014 (CEST)
20 Minuten Stromausfall morgens um 7 sollte eigentlich eine Zeitungsmeldung wert sein. Sehr wahrscheinlich nur ein Einzelfall.--217.87.69.211 18:51, 9. Sep. 2014 (CEST)
Ich lese aus dem Schaubild für das Ereignis eine Dauer von ca. 35 Minuten heraus. --Rôtkæppchen₆₈ 22:32, 9. Sep. 2014 (CEST)
ja... anette gute halbe Stunde kommt hin... es war aber so ungefähr von 08:50 Ortszeit bis 09:23 Ortszeit (ich hab ne Abneigung gegen Sommerzeit/Winterzeit/Zeitzonen, so dass bei mir alle Uhren auf UTC eingestellt sind)... --Heimschützenzentrum (?) 23:18, 9. Sep. 2014 (CEST)

also gibt's keine Datenbank bei der Bindesnetzagentur für die Menschen, Strom, Bahn, Telekom und Post? --Heimschützenzentrum (?) 00:06, 10. Sep. 2014 (CEST)

ist das vllt n versteckter Hinweis? --Heimschützenzentrum (?) 21:35, 11. Sep. 2014 (CEST)

Denkbar ist das. In meinem Wohnort gab es mal einen Kabelfehler zwischen den Stationen J. und K., der auch Ausfälle in anderen Teilen des Netzes, also zwischen J. und E., E. und G. und G. und R. verursachte. Damals waren alte Kabel aus den 1970er-Jahren Fehlerursache, die die hiesigen Stadtwerke dann sukzessive austauschen mussten. --Rôtkæppchen₆₈ 10:03, 12. Sep. 2014 (CEST)
das scheint hier ähnlich zu sein... das mit den alten Kabeln, meine ich... --Heimschützenzentrum (?) 14:47, 12. Sep. 2014 (CEST)

Zugriff auf ein eigentlich nicht einsehbares Buch bei GoogleBooks

Gibt es eine Möglichkeit, ein seit Jahrzehnten vergriffenes Buch in GoogleBooks lesen oder downloaden zu können, obwohl man es in Deutschland nicht einsehen kann? Ich habe zig Flohmärkte, Buchläden und das ganze Internet durchsucht und fand bisher kein einziges Exemplar (bis auf eine überteuerte, offensichtlich ungebundene Kopie die ich auch selber machen kann).

Es geht um dieses Buch ([20], [21], [22]), das offensichtlich mehrfach (auch in den USA) digitalisiert wurde.

Wenn jemand eine Möglichkeit hier ungern öffentlich kommunizieren will, gerne auch per Mail! Danke im Voraus! --hilarmont 11:10, 9. Sep. 2014 (CEST)

Schon mal mit den Tipps von wikisource probiert? Wikisource:Google Book Search --Nattr (Diskussion) 11:21, 9. Sep. 2014 (CEST)
Hier ist eine Liste von Bibliotheken in Bayern (angenommen, dass du in Bayern bist), die das Buch im Bestand haben. Der KVK kann auch Google Books und das Internet Archive durchsuchen, das hat aber keine Treffer ergeben. --Wrongfilter ... 11:28, 9. Sep. 2014 (CEST)
Bibliotheksbestände sind mir relativ egal, weil ich keine Lust habe, es zu dämlichen Öffnungszeiten abgeben zu müssen. Zudem ist es auch keine Antwort auf meine Frage, auf das gleiche komme ich auch ohne große Überlegungen. ;) hilarmont 11:32, 9. Sep. 2014 (CEST)
Noch mal auf nattrs Hinweis oben hingewiesen: die Anleitung für den US-Zugang zu Hathi und GBS findest du bei Wikisource. --Aalfons (Diskussion) 11:47, 9. Sep. 2014 (CEST)
@Hilarmont: Im konkreten Fall ist das Buch bei Google Books wahrscheinlich grundsätzlich gesperrt, auch in den USA, da viel zu neu (erschienen 1966), um frei von Urheberrechten zu sein. In den USA ist pauschal alles public domain, was vor 1923 erschienen ist; das ist der Grund dafür, dass Bücher aus dem späten 19. Jahrhundert oder frühen 20. Jahrhundert per US-Zugriff auf Google Books häufig zugänglich sind, während Google sie für Europa gesperrt hat - manchmal sogar dann, wenn sie auch hier gemeinfrei sind. Denn hier gilt ja eine Schutzfrist von 70 Jahren nach dem Tod des Urhebers, Google klärt das aber nicht im Einzelfall ab, sondern sperrt für Europa sicherheitshalber gleich alles bis ins tiefste 19. Jahrhundert zurück. - Für Bücher aus dem späteren 20. Jahrhundert wirst du dich an die Bibliothek deines Vertrauens wenden müssen :-) Gestumblindi 13:41, 9. Sep. 2014 (CEST)
PS: "weil ich keine Lust habe, es zu dämlichen Öffnungszeiten abgeben zu müssen" - häufig verschicken Bibliotheken auch gerne per Post und nehmen Rücksendungen ebenfalls auch auf dem Postweg entgegen. Gestumblindi 13:42, 9. Sep. 2014 (CEST)
Zig Flohmärkte haben noch deutlich beschränktere "Öffnungszeiten"... --Eike (Diskussion) 20:29, 10. Sep. 2014 (CEST)
Sehr hilfreiche Antwort Herr Sauer.
Das Bibliotheken Bücher auch fernleihen ist mir bekannt, aber das Buch alle 8 Wochen zurückzuschicken und dann wieder anzufordern ist mir zu blöd. hilarmont 09:37, 12. Sep. 2014 (CEST)

Autokennzeichen

Sagen wir mal ich habe ein Autokennzeichen, gibt es eine Möglichkeit den Halter feststellen zu lassen oder zu prüfen, ob das Auto(kennzeichen) gestohlen wurde? --Crosby Newton (Diskussion) 13:59, 9. Sep. 2014 (CEST)

Auskunft aus dem Fahrzeugregister: [23]. --BlackEyedLion (Diskussion) 14:11, 9. Sep. 2014 (CEST)
Deine Versicherung müsste das rausbekommen, wenn du einen Schaden meldest. Alternativ bei der zuständigen Zulassungsbehörde nach diesem Wunschkennzeichen fragen. Da siehst du dann, zumindestens ob es vergeben ist. Siehe auch hier. --sk (Diskussion) 14:14, 9. Sep. 2014 (CEST)
Nein, Deutschland ist doch Weltmeister, auch bald noch im Datenschutz, denn dann werden selbst Straftaten geheim gehalten sein. Schau mal hier und in den 2 folgenden Abschnitten nach was alles geheim gehalten wird. Hier werden noch nichteinmal geklaute Handys, die ihre IMEI samt IMSI ins Netz posaunen kassiert. Die Versicherer sind jedoch vernetzt und wissen alles über ein Fahrzeug. (siehe auch GNN) --Hans Haase (有问题吗) 14:17, 9. Sep. 2014 (CEST)
Nach einer Anfrage nach dem Wunschkennzeichen kennt er ja den Halter noch nicht. Ohne einen konkreten Grund wirst du an diese Information (auch über das Fahrzeugregister) nicht drankommen. Nach einem Unfall sicher, nchdem du das Fahrzeug z.B. aus deiner Einfahrt abschleppen lassen musstest auch. Wenn du nur wissen willst, wer z.B. dein erstes Auto heute fährt, wird's schon schwierig und ganz ohne Grund klappt das gar nicht. --88.130.77.236 14:20, 9. Sep. 2014 (CEST)

Ja gibt es. Einfach zur Polizei gehen und nachfragen ob das Auto(kennzeichen) als gestohlen gemeldet wurde. Die werden dann auch den Halter ermitteln, Dir werden sie ihn aber nicht mitteilen. --Simius narrans 15:28, 9. Sep. 2014 (CEST)

Das machen sie aber nur aus Nettigkeit und sicher nicht unbegrenzt oft. Ohne, dass du einen nachvollziehbaren Grund für deine Anfrage nennen kannst, würde ich erstmal davon ausgehen, dass die Polizei bei so einer Anfrage auch gut genau gar nichts machen kann. --88.130.77.236 16:48, 9. Sep. 2014 (CEST)
Ich hätte ja gedacht, das machen Sie um evtl. Hinweise betreffend einer Straftat zu erhalten. Der OP fragte auch nur nach einem Auto(kennzeichen) und warum man ohne nachvollziehbarem Grund davon ausgehen sollte, dass das Auto(kennzeichen) gestohlen ist...--Simius narrans 20:06, 9. Sep. 2014 (CEST)
Wenn du wüsstest, was die Leute manchmal so von einem wollen... Der Halter, die Versicherung und vielleicht noch ein Nutzer des Autos haben ein Interesse, zu efahren, ob das Auto gestohlen wurde. Ein x-beliebiger Dritter hat das einfach so aber erstmal nicht. --88.130.77.236 20:44, 9. Sep. 2014 (CEST)

Autoindex --Filzstift  15:55, 9. Sep. 2014 (CEST)

Der Autoindex ist für Schweizer Zulassungen, im Gegensatz zu Deutschland ohne jeden Grund möglich (hübsche Frau usw.) --217.189.212.50 16:19, 9. Sep. 2014 (CEST) zu "vernezt": Das ist der Gesamtverband der Deutschen Versicherungswirtschaft, die nehmen sogar stellvertretend für die Mitgliedsassekuranzen ganze Unfallmeldungen auf (telefonisch)
Der Fragesteller hat nicht explizit nach deutschen Gegebenheiten gefragt. Womöglich hilft meine Antwort aber tatsächlich nicht, könnte ja sein, dass er aus Österreich kommt. --Filzstift  21:41, 9. Sep. 2014 (CEST)

Wenn der Fragesteller zur Polizei geht und sagt, er hat den Verdacht, daß das Kennzeichen gestohlen sein könnte, wird man es sofort prüfen und ihm das Ergebnis logischerweise mitteilen. Den Halter bekommt er dabei nicht heraus. In der Frage heißt es aber auch "oder". Die Antwort lautet also: "Ja." --Tommes  09:07, 12. Sep. 2014 (CEST)

Diskrepanz Strommix Deutschland

Hallo, im Artikel Stromkennzeichnung steht ein Durchschnittswert von 58,6 % für fossile Energieträger. Wenn man allerdings die einzelnen Unternehmen in der Tabelle betrachtet, ist RWE das einzige mit einem Wert über dem Durchschnitt. Noch seltsamer ist der Wert für erneuerbare Energien von 24,2 %, über dem alle Unternehmen liegen. Zumindest diese beiden Werte können nicht stimmen, aber sowohl die Durchschnitte als auch die Einzelwerte tauchen so in den Quellen auf. Interessant finde ich das vor allem, weil jedes der Unternehmen jetzt seinen Wert mit dem zu hohen Durchschnittswert für fossile ("Wir sind besser!") und dem zu niedrigen Wert für erneuerbare Energien vergleichen kann. Wie lässt sich also dieser Unterschied erklären? --89.182.151.68 15:17, 9. Sep. 2014 (CEST)

Zur Sache kann ich nichts Substantielles sagen, aber ich kann schonmal einen Aspekt aufzeigen, wo unsere Angaben meiner Meinung nach ungenau sind (wenn mich da im Rahmen dieser Diskussion keiner belehren kann, wäre das wohl im Artikel zu korrigieren): Im Eingangssatz jenes Abschnitts wird angeblich der "Bruttostromverbrauch" betrachtet, die Quelle liefert aber überwiegend (und jedenfalls bei den von uns übernommenen Werten) Zahlen zur "Bruttostromerzeugung". Das ist schon wegen Strom-Import und -Export was anderes. --YMS (Diskussion) 15:27, 9. Sep. 2014 (CEST)
Siehe Bilanz, die Welt spricht V**A. --Hans Haase (有问题吗) 15:59, 9. Sep. 2014 (CEST)
Dazu ist zwischen Kapazität (was bei vollem Sonnenschein, viel Wind etc. theoretisch möglich ist) und realer Produktion zu unterscheiden. Gerade bei den regenerativen Energien unterscheiden sich die Werte sehr stark. --mfb (Diskussion) 17:37, 9. Sep. 2014 (CEST)
Die Tabelle umfasst nicht alle Stromerzeuger. Das wäre auch noch eine theoretisch denkbare Erklärung. --Grip99 01:49, 12. Sep. 2014 (CEST)
Oh ja, Prokon GenussVerlustrechte. --mfb (Diskussion) 02:34, 12. Sep. 2014 (CEST)

Hausratversicherung Fahrrad ohne Kaufbeleg?

Hallo, ich habe eine Hausratversicherung inkl Versicherung meines Fahrrad gegen Diebstahl abgeschlossen (schon vor längerer Zeit). Ich habe aber jetzt gemerkt, dass ich wohl den Kaufbeleg meines Fahrrads gar nicht mehr habe. Ich könnte also, wenn mein Fahrrad jetzt geklaut wird, weder den Kaufpreis noch mich als rechtmäßigen Besitzer nachweisen. Kann ich da (außer die entsprechende Option der Versicherung zu kündigen) irgendwas tun? Und würde ich im Fall des Falles ohne solchen Beleg gar nichts bekommen? Versicherung ist Allianz, falls das eine Rolle spielt. Danke! --46.115.145.188 16:12, 9. Sep. 2014 (CEST)

Du könntest im Geschäft wo Du das Fahrrad seinerzeit gekauft hast mit dieser Begründung um die Austellung einer Rechnungskopie bitten.--Simius narrans 16:17, 9. Sep. 2014 (CEST)
Und im Anschluss daran, das Fahrrad codieren lassen. Macht der Fahrradhändler gegen Gebühr und an Aktionstagen die Polizei kostenlos - zumindest in B.W. --46.5.70.241 22:19, 9. Sep. 2014 (CEST)
Falls du das Fahrrad mit Karte bezahlt hast, kannst du einen alten Kontoauszug als Kaufbeleg nehmen. Wenn du die Kontoauszüge nicht ufhebst, kannst du ganz freundlich bei deiner Bank nachfragen. --217.189.237.44 11:54, 10. Sep. 2014 (CEST)
Kaufbelege sind nicht zwingend. Man stelle sich einfach vor, die Wohnung brennt aus. Nur die allerwenigsten Menschen haben einen externen Aufbewahrungsort wie zum Beispiel ein Bankschließfach und aktualisieren dieses auch noch nach jedem Kauf eines Hausratgegenstandes! Man muss seinen Anspruch der Versicherung gegenüber glaubhaft machen zum Beispiel durch Zeugen oder Fotos. Nichtsdestotrotz würde ein Kassenzettel natürlich den Bearbeitungsvorgang beschleunigen, vor allem bei wertvollen Gegenständen. --84.172.50.109 17:55, 10. Sep. 2014 (CEST)
Das Fahrrad könnte ja auch ein Geschenk oder Erbstück sein. --Rôtkæppchen₆₈ 17:56, 10. Sep. 2014 (CEST)
Den Wert eines Fahrrades erkennt man eher am Fahrrad als am Kaufbeleg. Insofern ist es günstiger, Marke, Typ und Alter zu belegen (Foto?!) als den früheren Neupreis, der sich irgendwo im Netz oder beim Händler leicht erfragen ließe. Oder zahlt die Allianz den ehemaligen Kaufpreis voll? Eher nicht. --Tommes  09:16, 12. Sep. 2014 (CEST)

Abhängig oder unabhängig?

Ich möchte 5 Rankingverfahren vergleichen. Dazu führe ich einen Test mit einigen Querys durch und bewerte die Ergebnisse. Wie kann ich nun feststellen, ob die Unterschiede statistisch signfikant sind oder nicht? Zunächst: Woher weiß ich, ob sie abhängig sind oder unabhängig? Sind sie abhängig, wenn sie auf demselben Korpus arbeiten und dieselben Querys bei allen Rankingverfahren verwendet werden? Was ist, wenn ein Rankingverfahren nur auf einem Teilkorpus arbeitet? Wenn sie abhängig sind, könnte ich den Vorzeichentest verwenden. Wenn ich nun jedes Rankingverfahren mit jedem Rankingverfahren vergleichen will, kann ich dann einfach Verfahren 1 mit Verfahren 2, Verfahren 1 mit Verfahren 3, Verfahren 3 mit Verfahren 4 usw. vergleichen? Oder führt das zu Problemen? (nicht signierter Beitrag von 217.234.106.233 (Diskussion) 19:27, 9. Sep. 2014 (CEST))

Wenn verschiedene Rankingverfahren auf überlappende Datensätze zugreifen, dann sind ihre Ergebnisse voneinander abhängig, sonst nicht. 1 mit 2, 1 mit 3 etc. zu vergleichen ist möglich, beachte dabei aber den en:Look-elsewhere effect. Und diese Vergleiche sind dann natürlich auch voneinander abhängig ;). --mfb (Diskussion) 20:48, 9. Sep. 2014 (CEST)
Gibt es dabei einen Unterschied zwischen gepaart und abhängig? Der Vorzeichentest ist im Artikel Statistischer Test unter "Für zwei abhängige Stichproben" gelistet, in Vorzeichentest steht jedoch:
  • Die Beobachtungspaare dürfen nicht voneinander abhängen, d.h. das Wertepaar muss unabhängig vom Wertepaar sein
Ist damit gemeint, dass beim Wertepaar zwar a und b voneinander abhängig sein dürfen, jedoch nicht bspw. das erste Wertepaar vom zweiten? Ist dies bei den Rankingverfahren gegeben? --217.234.106.233 22:18, 9. Sep. 2014 (CEST)
Ja mit gepaart meint man genau das. Ich verstehe allerdings noch nicht, was du genau testen willst und wie dabei ein Vorzeichentest eingesetzt werden soll. Was ist also die Hypothese über die Rankingverfahren, die du testen willst? Zwei Rankings quantitativ vergleichen kann man z. B. mit dem Rangkorrelationskoeffizienten. Oder geht’s gar nicht um die Ranglisten an sich? -- HilberTraumd, m09:06, 10. Sep. 2014 (CEST)
Ich möchte testen, ob die Unterschiede zwischen den Rankingverfahren statistisch signfikant sind oder nicht. Ich vergleiche also zum Beispiel die x Gesamt-Bewertungen der jeweils ersten 10 Ergebnisse von x Querys eines Verfahrens mit den x Gesamt-Bewertungen der jeweils ersten 10 Ergebnisse der x Querys eines anderen Verfahrens. Die Nullhyotphese wäre also, dass die Mediane gleich sind, der Vorzeichentest sollte dann dazu führen, dass ich diese Hyptohese ablehnen kann. Das Ranking geht zwar AUCH in die Gesamt-Bewertung ein, jedoch ist es entscheidender, wie die einzelnen Ergebnisse (auf einer Punkteskala) bewertet wurden. Wenn also ein Verfahren fast immer eine bessere Bewertung bei den x Querys erhalten hat, sollte dieser Unterschied auch statistisch signifikant sein, darum der Vorzeichentest. Ich bin mir nur nicht sicher, ob der Test dafür anwendbar ist? --217.234.106.233 13:17, 10. Sep. 2014 (CEST)
So ganz habe ich’s immer noch nicht verstanden: Was sind denn die Gesamt-Bewertungen? Was heißt denn „die jeweils ersten 10 Ergebnisse“?
Ist das so gemeint: Du schaust dir die Plätze 1 bis 10 eines Rankings an und schaust dann, auf welchen Plätzen sie mit dem anderen Rankingverfahren landen? Zum Beispiel auf den Plätzen 1,4,2,3,5,6,8,9,7,10?
Wenn das so ist, dann kann man wohl keinen Vorzeichentest anwenden. Die Paare sind dann nicht unabhängig. Außerdem hat man ja – gerade wenn die Verfahren sehr ähnlich sind – oft gleiche Plätze bei beiden Verfahren. -- HilberTraumd, m18:50, 10. Sep. 2014 (CEST)

Okay, dann mal etwas konkreter, zur Vereinfachung reduziere ich jeweils die Anzahl der verschiedenen Sachen. Angenommen ich habe

  • zehn verschiedene Querys Q1, Q2, Q3, .. Q10,
  • 3 Rankingverfahren R1, R2 und R3 und
  • im Korpus 100 verschiedene Dokumente D1, D2, D3 ... D100.
  • Außerdem interessieren mich jeweils die Bewertungen der jeweils ersten drei Dokumente für jede Query.

Angenommen für die erste Query Q1 sind die Dokumente D1 und D2 sehr relevant und für D3 ein bisschen. Alle anderen Dokumente sind für Q1 gar nicht relevant. Sehr relevante Dokumente erhalten 2 Punkte, bisschen relevante Dokumente 1 Punkt und gar nicht relevante Dokumente 0 Punkte. Zusätzlich wird die Bewertung des Dokuments an der ersten Stelle mal 3 genommen und das an der zweiten Stelle mal 2 (um auch die tatsächliche Rankingpositionen mit in die Bewertung einfließen zu lassen). Folgendes Ergebnis für Query Q1 (von Platz 1 zu Platz 3):

  • Rankingverfahren R1: D1, D2, D3 => Gesamtergebnis: 2*3 + 2*2 + 1 = 11
  • Rankingverfahren R2: D1, D5, D3 => Gesamtergebnis: 2*3 + 0*2 + 1 = 7
  • Rankingverfahren R3: D5, D4, D1 => Gesamtergebnis: 0*3 + 0*2 + 2 = 2

Nun habe ich ein Gesamtergebnis für alle drei Rankingverfahren für die erste Query. Für die anderen Querys verfahre ich dann genauso, nur dass dann natürlich andere Dokumente relevant sein können. Nun möchte ich bspw. R1 mit R2 vergleichen. Mein erstes Wertepaar für Q1 wäre also (11, 7), für Q2 vielleicht (8,2), für Q3 (2,4) usw. Diese gepaarten Beobachtungen möchte ich nun vergleichen. Ich möchte also wissen, ob der Unterschied zwischen R1 und R2 bei den einzelnen Ergebnissen für die verschiedenen Querys signfikant sind oder nicht. Ich hoffe, du kannst jetzt besser nachvollziehen, was ich meine. --217.234.106.233 20:22, 10. Sep. 2014 (CEST)

Ok danke, jetzt hab ich das Problem verstanden. Ja, dann könnte man es mit einem Vorzeichentest versuchen. Wichtig ist nur, dass die 10 Querys rein zufällig gewählt werden, also unabhängig und gleichverteilt aus der Menge aller möglichen Querys. Ich weiß nicht wie einfach oder schwierig das zu bewerkstelligen ist. -- HilberTraumd, m10:07, 11. Sep. 2014 (CEST)
Okay, danke für die Hilfe! :) --217.234.106.233 16:50, 12. Sep. 2014 (CEST)

Religion in der modernen Unterschicht

Wie sieht es denn mit dem reigiösen Leben in der modernen Unterschicht aus? Die Hochfeste werden in paganisierter Form meistens mitgefeiert, ebenso wie die Sakramente (wo sich die Leute im Gottesdienst ganz gerne ziemlich daneben benehmen. Aber sonst ist das zumindest in meiner Vorstellung ein Ort der Areligiösität. Oder doch nicht?--Antemister (Diskussion) 22:49, 9. Sep. 2014 (CEST)

Was verstehst du unter der "modernen Unterschicht"? --46.114.153.190 23:52, 9. Sep. 2014 (CEST)
Und woher kommt die Idee, die Unterschicht wäre weniger religiös? Der allgemeine Trend geht eher in die andere Richtung. --mfb (Diskussion) 00:00, 10. Sep. 2014 (CEST)
Wir emmpfehlen zur Orientierung: Religiöse und kirchliche Orientierungen in den Sinus-Milieus --Concord (Diskussion) 00:19, 10. Sep. 2014 (CEST)
Boah! Ich liebe Meinungsforschung. 100 Personen werden in 10 Kategorien aufgeteilt, die dann aber regional über ganz D gestreut... und in männlich/weiblich eingeteilt.--Optimum (Diskussion) 09:07, 10. Sep. 2014 (CEST)
Sieh es als "product research". Da sind solche Gruppengrössen üblich, um ein relativ stabiles Resultat bezüglich von Tendenzen (nicht Zahlen auf Nachkommastellen) zu bekommen.
Ich sehe mir deutlich weniger als 100 Websites/GoogleBooks an, um zu einem Ergebnis zu kommen, für das ich mein Alias aufs Spiel setze.... ;-) GEEZER… nil nisi bene 09:51, 10. Sep. 2014 (CEST)
"Moderne Unterschicht" ist ein verallgemeinernder Sammelbegriff. .. Kein °Ort ° lol. --217.84.115.224 15:50, 10. Sep. 2014 (CEST)
Gut, bei Sinus habe ich angefangen, wenn auch nicht speziell bei dem PDF hier. Jetzt machen die auch wenig "Grundlagenforschung" sondern untersuchen Zielgruppen für Regierung und Industrie (bzw. hier hat die kath. Kirche), sodass ich mit dem nicht sonderlich viel anfangen konnte (es wurden ja auch scheinbar nur Leute befragt die Kontakt mit der Kirche haben). Über die moderne/neue Unterschicht (in Abgrenzung zu den alten Unterschichten der (Hilfs)arbeiter und (Klein)bauern) gibt es ja auch einen Artikel, bei Sinus die "Konsum-Materialisten"/"Prekäre"/"Hedonisten".--Antemister (Diskussion) 19:05, 10. Sep. 2014 (CEST)

hier nochmal eine andere Studie. Vielleicht hilft die weiter. --84.135.29.93 21:50, 10. Sep. 2014 (CEST)

Ob es diese pauschale Unterschicht, wie in der Fragestellung unterstellt "pagan", Religiosität (der ganzen Schicht? Im thread: 'Tendenzen'), "sich im Gottesdienst daneben benehmen", so gibt, kann man auch abstreiten. Das klingt verdächtig nach Polemik, Herabsetzung, in der Umkehr arrogantem Schubladendenken (redirect zu Vorurteil). Es gibt jeweils "eine"  Unterschicht in Sozialhierarchischen Modellen, nicht "die  Unterschicht". Es wird dann nach grundverschiedenen sozialen Merkmalen differenziert und ..
  Grundsätzlich wird die Zugehörigkeit einer Person zu einer sozialen Schicht als
  offen und änderbar angesehen (sozialer Aufstieg, soziale Mobilität), [..] 
Der Begriff moderne Unterschicht als politischer Kampfbegriff ein pseudo-beschönigendes Unwort, macht es nicht besser .. es werden aufgrund von Einzelfällen ("teilweise in der dritten Generation ohne [sozialversicherungspflichtige] Arbeit" pauschal Vorurteile geschürt. Umfrage-Institute, die derlei bestätigen, freuen sich über jeden Auftrag. Da hat man einen schönen Sündenbock für eigenes politisches Versagen (in einem der reichsten Staaten der Welt sollte so ein Kindergarten schnell wo hingebaut sein, ein bißchen Bildung an die Leute bringen, Jugendzentren, Juzze einrichten, einen Bolzplatz ohne Hausmeister mit Zaun und "Betreten verboten"-Schild dran, uvm). Dann die unter solchen Schieflagen leidenden als Verursacher verteufeln .. nice one. - Die Diskussion rückt auch zurecht: Volksverhetzung, §130 StGB ("[..] die Menschenwürde anderer dadurch angreift, dass er eine vorbezeichnete Gruppe, Teile der Bevölkerung oder einen Einzelnen wegen seiner Zugehörigkeit zu einer vorbezeichneten Gruppe oder zu einem Teil der Bevölkerung beschimpft, böswillig verächtlich macht oder verleumdet, [..] wird mit Freiheitsstrafe von drei Monaten bis zu fünf Jahren bestraft."). Da schlage ich für die Besprechung Deiner Vorstellung von diesem Ort das Café vor. Eine Wissensfrage ist es so (pauschale Schicht, "pagan", "sich daneben benehmen", "Ort der Areligiosität") eher nicht. Einer diffusen 'ganzen' Schicht eine Religion zuordnen zu wollen, 'Religiosität zu messen', ist genauso sinnlos, wie die Apfel- gegenüber Birnenessern herauszufiltern. Nur läßt sich das nicht politisch verwerten. --217.84.69.59 15:17, 12. Sep. 2014 (CEST)

Mit Windows 8.1 Datei-Explorer Textdaten auslesen?

Ich habe einen Ordnerinhalt mit mehreren Dateien. Diesen Ordnerinhalt, d.h. die Dateinamen, möchte ich in eine Textdatei kopieren (über den Zwischenspeicher, also rechter Mausklick). Mit jeder Ansicht habe ich entweder a) nur Icons oder b) Icons mit Text. Daher geht das nicht. Ich will ja nur den Text. Word Pad habe ich probiert und den Trick, das Zeug in die Browser-Zeile zu kopieren und dann zu entformatieren. Es klappt einfach nicht. Das Problem hört sich einfach an, aber ich kann es nicht lösen :-/ Die Betriebsanleitung sagt gar nichts dazu. Hat jemand einen Vorschlag? --217.189.212.50 23:54, 9. Sep. 2014 (CEST)

Nimm cmd.exe: Windows+R, dann cmd.exe eingeben, dann dir "Dein Verzeichnis". Über dir /? erfährst Du die Optionen für den dir-Befehl, also Sortierung, angezeigte Details, Attributefilter etc. Die Ausgabe kopieren mit Rechtsklick auf das Cmd-Fenster, dann Markieren, dann zu kopierenden Text mit der Maus auswählen und mit der Eingabetaste kopieren. Mit UWT kann Du Dir einen Eintrag ins Explorer-Kontextmenü zaubern, der Dir mit einem Klick ein Cmd-Fenster im richtigen Ordner zaubert. Dann musst Du nur noch dir mit Optionen eingeben und die Ausgabe kopieren mit Rechtsklick auf das Cmd-Fenster, dann Markieren, dann zu kopierenden Text mit der Maus auswählen und mit der Eingabetaste kopieren. Am Zielort wie gewohnt einfügen. --Rôtkæppchen₆₈ 00:17, 10. Sep. 2014 (CEST)
Wenn Du die Ausgabe eh' in eine Textdatei kopieren willst, kannst Du auch die Ausgabe des dir-Befehls direkt via dir > DeineDatei.txt abspeichern.--Christian Lindecke (Diskussion) 02:13, 12. Sep. 2014 (CEST)
Dann muss die Textdstei aber von Codepage 850 nach UTF-8, ISO 8859-1, Codepage 1252 oder Unicode gewandelt werden. Bei Copypaste ist das nicht nötig. --Rôtkæppchen₆₈ 11:29, 12. Sep. 2014 (CEST)

Rechtschreibhilfe

ist dieser Satz korrekt?

Schloss Neugebäude stellt trotz umfangreicher Veränderungen einen der wichtigsten Renaissancebauten nördlich der Alpen dar.

sollte es nicht heißen: eines der wichtigsten Renaissancebauten... Vielen Dank!--Hubertl (Diskussion) 11:27, 10. Sep. 2014 (CEST)

Einen. Alles andere ist schlechtes Deutsch. --Rôtkæppchen₆₈ 11:32, 10. Sep. 2014 (CEST)
Richtig. Plural von der Bau. --SCIdude (Diskussion) 11:42, 10. Sep. 2014 (CEST)
Definitiv einen, weil es ja um einen der Bauten geht und nicht um eines der Schlösser. --217.189.237.44 11:58, 10. Sep. 2014 (CEST)

Schlechtes Deutsch ist es auch mit "einen". Warum denn nicht "ist einer der wichtigsten"? Wie soll denn ein Schloss einen Bau darstellen? --Vsop (Diskussion) 12:26, 10. Sep. 2014 (CEST)

Es ist nicht ein Satz von mir, ich hätte Renaissancebauwerke verwendet, da wäre es leichter. Auch -paläste oder -schlösser wäre in diesem Fall möglich gewesen. --Hubertl (Diskussion) 13:24, 10. Sep. 2014 (CEST)
Vorschlag: „gilt als einer der wichtigsten“, wegen der Subjektivität der Aussage. Selbst das eigentlich eine zu „starke“ Aussage („stark“ ist, was einen Beleg erfordert). --Pp.paul.4 (Diskussion) 13:31, 10. Sep. 2014 (CEST)
Ich würde „wird als eines der wichtigsten Bauwerke der Renaissance gewertet“ schreiben, mit Beleg natürlich. „Darstellen“ ist Käse. --178.0.193.156 21:21, 11. Sep. 2014 (CEST)

Recht an einem Logo nach Eigentümerwechsel

Vor rund zehn Jahren habe ich eine Logo für ein anfangs kleine Hobby-Projekt, an dem ich beteiligt war, entworfen. Natürlich ohne Honorar oder Vertrag. Mit der Zeit wurde daraus ein ganz gut laufendes Nischengeschäft, mit dem ich nichts mehr zu tun hatte. Soweit kein Problem. Vor kurzem wurde es allerdings von einem Konzern der Branche übernommen. Falls das Logo übernommen wird (es sieht danach aus): Kann ich da jetzt Rechte geltend machen? Wenn ein Konzern das Logo verwendet, hätte ich natürlich gerne auch ein Honorar. Rainer Z ... 18:34, 10. Sep. 2014 (CEST)

Die erste Frage ist wohl, ob das Logo dein Eigentum oder Eigentum des Nischengeschäfts ist. Du hast die Nutzung ohne Zweifel erlaubt. Aber wie war der Vertrag? Damit meine ich - falls es das gibt - die mündliche Absprache, ein schriftliche gibt es ja nicht. Die kostenfreie Nutzung ergibt sich aus dem Sachverhalt. Kann aber das Nischengeschäft bzw. der neue Eigentümer belegen, daß du ihnen das Logo, bzw, die Rechte daran überlassen hast? Ein Kauf hat ja nachweislich nicht stattgefunden. Wahrscheinlich wäre das Beste eine Zeugenaussage des alten Inhabers des Nischengeschäfts. Ansonsten denke ich, daß das Logo dir gehört, solange du deine Urheberschaft nachweisen kannst und solange niemand nachweisen kann, daß du es veräußert hast. Die Behauptung der kostenlosen Überlassung zur Nutzung ohne Weitergabe deiner Rechte dürfte angesichts der Geschichte des Nischengeschäfts (geringe Kapitaldecke als Starter etc.) glaubwürdig sein. Und wenn keine Vereinbarung über die Rechte getroffen wurde, sind sie natürlich noch deine. --87.149.176.147 18:55, 10. Sep. 2014 (CEST) Nachsatz: Die Situation ist vielleicht vergleichbar mit einer mündlich vereinbarten kostenfreien Abdruckerlaubnis, bei der der Urheber des Textes seine Rechte ja auch nicht automatisch abgibt. Ein Eigentümerwechsel dürfte ein akzeptabler Grund sein, um die Nutzungserlaubnis für das Logo zu widerrufen. --87.149.176.147 19:08, 10. Sep. 2014 (CEST)
Hast du das irgendwann bei der VG Bild-Kunst gemeldet? Könnte hilfreich sein. Ansonsten stimme ich dem Vorredner zu. Verträge über Nutzungsgenehmigung bedürfen nicht der Schriftform. --Pölkky 22:54, 10. Sep. 2014 (CEST)
Ah - was mir noch einfällt: Du schreibst "an dem ich beteiligt war". Das ist nicht ganz klar. Wenn du das Logo in der Eigenschaft als Angestellter der Firma erstellt hast könnte die Sachlage vielleicht anders aussehen als bei einem Werk in der Freizeit für ein gemeinsames Hobby. Dazu weiß ich aber zuwenig, außer daß manche Leute recht wenig von dem haben, was sie für ihre Firma erfinden... --87.149.176.147 03:01, 11. Sep. 2014 (CEST)
Das ging damals alles sehr formlos zu. Für mich wars nur Hobby, für den Gründer wurde es schließlich zum Geschäft. Wir hatten uns dann aus den Augen verloren. Dass es das Unternehmen überhaupt noch gibt und gediehen ist, habe ich erst vor zwei Jahren mitbekommen. Dann werde ich wohl am einfachsten mal Kontakt zum Gründer aufnehmen. Viele Dank für die Einschätzung. Rainer Z ... 11:17, 11. Sep. 2014 (CEST)
Es klingt so, als ob, wie das wohl öfters der Fall ist, die Frage der Rechte garnicht thematisiert wurde, weil das in dieser Phase keine Rolle spielte. Da du offenbar auch keine Vergütung erhalten hast, scheint das Logo dein Eigentum zu sein. Wäre es Eigentum des Nischengeschäfts, wäre es von dem neuen Eigentümer mitgekauft worden und nun dessen Eigentum. Bei dem Gespräch mit deinem Kumpel von damals ist meines Erachtens weniger wichtig, wie er die Rechtssituation verstanden hat (wenn überhaupt) sondern daß hinsichtlich der Rechte an dem Logo bis auf die Erlaubnis der Nutzung keine weitere Vereinbarung getroffen wurde. --212.184.134.8 16:17, 11. Sep. 2014 (CEST) (hier auch als 87.149.176.147 unterwegs)
Stimmt, bei kleinen Projekten oder mehr oder weniger Hobbyaktionen wird so etwas oft gar nicht angesprochen. Das ist auch für mich das erste mal, dass daraus doch ein richtiges Unternehmen wurde. Rainer Z ... 13:18, 12. Sep. 2014 (CEST)

Musikstücke, deren Tempo stark beschleunigt

(keine Ahnung, wie das richtig heißt). Ich suche bekannte Stücke, die am Anfang langsam sind und dann extrem schnell werden. Mir fällt da so ein griechischer VOlkstanz ein (Zorbas). Wie heißt der richtige Begriff für die Temposteigerung a) als Vortragsanweisung, b) sozusagen als Stilelement? Man dankt! --217.189.212.50 20:14, 10. Sep. 2014 (CEST)

Hava Nagila --87.149.176.147 20:21, 10. Sep. 2014 (CEST)
Ac­ce­le­ran­do. Ungarische Tänze von J. Brahms, Slawische Tänze von A. Dvorak, Rhapsodien von F. Liszt. Auch italienische Tänze wie der Saltarello und die Tarantella werden accelerando gespielt. --Schlesinger schreib! 20:22, 10. Sep. 2014 (CEST) Nachtrag: Zorbas tanzte in dem Film einen Sirtaki ein lächelnder Smiley 
<schlaumeier> Und das war eine Komposition von Mikis Theodorakis und es war nicht Nikolaos Zorbas sondern Alexis Sorbas ("Meine Hände tun die Arbeit und der Kopf denkt für mich." und: "Boss, hast du jemals irgendwas so schön zusammenkrachen sehen?"), erdacht von Nikos Kazantzakis und gespielt von dem unvergessenen Anthony Quinn. Wobei da bisweilen auch ein Z gebraucht wird und ein s weggelassen... (Service: Die Filmszene).</schlaumeier> --87.149.176.147 20:49, 10. Sep. 2014 (CEST) ein lächelnder Smiley  Erg. --212.184.134.8 16:24, 11. Sep. 2014 (CEST) (hier auch als 87.149.176.147 unterwegs)
Die Szene ist unvergessen, wie die beiden Freunde da am Strand in der verblassenden Technicolor-Farbe Hollywoods tanzen ein lächelnder Smiley  --Schlesinger schreib! 21:00, 10. Sep. 2014 (CEST)
Siehe auch hier in Birmingham - wunderbar. --87.149.176.147 21:04, 10. Sep. 2014 (CEST) Ergänzung: Der ist auch schön, in Kanada - hopppa!
Entschuldigt meinen kleinen Wermutstropfen, aber einer bekannten Online-Enzyklopädie zufolge handelt es sich beim Sirtaki um eine erfundene Tradition *schnüff* Geoz (Diskussion) 21:42, 10. Sep. 2014 (CEST)
Kein Wermutstropfen. Du hast zweifellos recht (wobei das der guten Stimmung keinen Abbruch tut). Bei dem "kleinen Syrtos" ist es angesichts der Bedeutung des Komponisten und der Rezeption wohl nur eine Frage der Zeit. Schließlich, um ein ähnliches Beispiel heranzuziehen, firmiert - durchaus zu Recht - Das Wandern ist des Müllers Lust als Volkslied, obwohl der Text der Feder des Dichters Wilhelm Müller entstammt und die bekannteste Vertonung (Carl Friedrich Zöllner) gerade mal 170 Jahre alt ist. Wenn man, zugegeben etwas beliebig, das Bedeutungshoch des Theodorakis etwa 10 Jahre nach dem Erscheinen des Films von Michael Cacoyannis ansetzt, also um 1974, nach dem Sturz der Diktatur und seiner Rückkehr aus dem Exil, dann braucht es doch nur noch 130 Jahre und um 2144 herum werden wir den Hinweis auf die erfundene Tradition in Sirtaki kopfschüttelnd korrigieren - Hopppa!. ein lächelnder Smiley  --87.149.176.147 02:53, 11. Sep. 2014 (CEST)
Hornpipe (Tanz), Kasatschok. -- Janka (Diskussion) 20:33, 10. Sep. 2014 (CEST)
In der Halle des Bergkönigs --L47 (Diskussion)
Boléro von Maurice Ravel Da ändert sich die Lautstärke--Mauerquadrant (Diskussion) 20:57, 10. Sep. 2014 (CEST)
Wir haben sogar ein Artikelchen zum italienischen Fachausdruck: Accelerando. Ein russisches Beispiel Kalinka. Geoz (Diskussion) 21:11, 10. Sep. 2014 (CEST)
ein serbisches: Sestorka. --149.172.234.158 21:56, 10. Sep. 2014 (CEST)
Agogik (Musik) ist die Veränderung des Tempos. --Explosivo (Diskussion) 22:01, 10. Sep. 2014 (CEST)
Eisgekühlter Bommerlunder Geoz (Diskussion) 22:08, 10. Sep. 2014 (CEST)
Das rote Pferd --Eike (Diskussion) 09:39, 11. Sep. 2014 (CEST)
Eine modernere und konsequente Umsetzung des Prinzips: One (Metallica-Lied)--Suvroc (Diskussion) 23:57, 10. Sep. 2014 (CEST)

Man kann auch noch Stretta anführen.--Mautpreller (Diskussion) 11:03, 11. Sep. 2014 (CEST)

C'est bleu – von Vicky Leandros in der 2011er-Neuauflage zusammen mit Scooter. --Richard Zietz 13:26, 11. Sep. 2014 (CEST)
In Koyaanisqatsi zieht es sich massiv unterstützend als Faktor der Aussage durch den ganzen Film. --217.84.102.92 14:41, 11. Sep. 2014 (CEST)
Zigeunerweisen von Pablo de Sarasate. --Jossi (Diskussion) 15:01, 11. Sep. 2014 (CEST)
Scheint in der Klezmermusik öfters vorzukommen: => Websuche "Accelerando Klezmer". --212.184.134.8 16:33, 11. Sep. 2014 (CEST) (hier auch als 87.149.176.147 unterwegs)

Frauen- und Männerhemden, wieso links und rechts?

Wieso sind die Knöpfe bei Männerhemden in der Regel rechts und bei Frauenhemden in der Regel links angebracht? Hat das eine Bedeutung oder eine historische Herkunft oder hat es sich einfach so eingebürgert? --188.100.181.58 23:15, 10. Sep. 2014 (CEST)

Einmal die Überschrift gegoogelt: [24]. XenonX3 – () 23:25, 10. Sep. 2014 (CEST)
Das rührt daher dass die Damen in besseren Kreisen Kammerzofen hatten, die sie angekleidet haben. Somit war eine spiegelbildliche Knopfanordnung gegenüber Hemden für Herren, die sich selbst angekleidet haben, sinnvoll, denn auch die Kammerzofen waren in der Mehrzahl Rechtshänderinnen. --Rôtkæppchen₆₈ 23:32, 10. Sep. 2014 (CEST)
Waren die Kammerzofen den Herren denn eher beim Ausziehen behilflich? --178.199.1.95 06:36, 11. Sep. 2014 (CEST)
Nein. Das hätte Kammerzoff mit den dazugehörigen Damen gegeben. GEEZER… nil nisi bene 07:00, 11. Sep. 2014 (CEST)
Nein. Herren lassen nicht ausziehen, sondern ziehen selber aus. Darum hatten die Zofen Druckknöpfe. Ist einfacher, gerade auch wenn's schnell gehen soll. --92.106.184.230 20:48, 11. Sep. 2014 (CEST)

Knopf#Verwendung. --Vsop (Diskussion) 11:13, 11. Sep. 2014 (CEST)

Einen Neuen Abschnitt in einem MediaWiki nicht ganz unten einfügen?

Ich stelle diese Frage hier, weil sie zwar mit einem Wiki, nicht aber mit Wikipedia zu tun hat. Ich habe in einem Wiki das Design meiner Benutzerseite als Vorlage in den BNR ausgegliedert, sodass ich auf der Benutzerseite selbst nur den Text schreiben muss, also dort steht dann nur {{Benutzer:Mariofan13/Designvorlage|1=INHALT DER BENUTZERSEITE}}. Nun möchte ich die Designvorlage auch auf der Diskussionsseite anwenden. Sie funktioniert aber - im Gegensatz zu Div-Tags wie ich einen hier in der WP auf meiner DS verwende - nur, wenn der Vorlagen-Tag auch mit }} geschlossen wird. Nun habe ich aber das Problem, dass neue Abschnitte, die per Link "Abschnitt hinzufügen" hinzugefügt werden, ''unter'' dem schließenden Tag angelegt werden und damit das Design zerstört wird. Wie kann ich bewirken, dass neue Abschnitte ''oberhalb'' des schließenden Vorlagen-Tags }} angelegt werden und nicht darunter? Das Zauberwort <nowiki>__END__ ist leider wirkkungslos...

--Mariofan13 (Diskussion, Beiträge) 23:26, 10. Sep. 2014 (CEST)

Es geht aber darum, dass neue Benutzer, die sich mit Wikisnytax usw nicht auskennen und einfach auf "Neuen Abschnitt" klicken, diesen damit unter den den abschließenden Tag setzen würden, womit die Seite zerschossen würde. Mariofan13 (Diskussion, Beiträge) 13:40, 11. Sep. 2014 (CEST)
Die geschweiften Klammern verweisen auf eine Vorlage. Suche nach Vorlage: . Einen Abschnitt kannst Du einfügen. Wenn Du einen vorhandenen Abschnitt oder den ganzen Artikel bearbeitest, achte darauf wie Überschriften formatiert werden, siehe Hilfe:Überschrift. Gibt es einen Knopf dafür, benutze ihn, da es damit alle viel einfacher haben. --Hans Haase (有问题吗) 13:09, 12. Sep. 2014 (CEST)

Problem beim Emailempfang in Thunderbird

Hallo an alle, ich habe ein Problem mit Thunderbird. Nachdem ich mein Betriebssystem Ubuntu 14.04 installiert hatte, passiert im eingerichteten Thunderbird folgendes: Eine Mail (Versendungsdatum 28.08.) erscheint wie in einer Endlosschleife immer wieder auf's Neue, sobald ich Thunderbird öffne. Der Versender hat die Mail inzwischen bei sich gelöscht.

Habe das Programm mehrfach (über Synaptic als auch über das Software Center) gelöscht und neuinstalliert. Im Account vom Anbieter (Yahoo) passiert das nicht. Beim Googeln habe ich nichts entsprechendes gefunden, hatte vielleicht aber nicht die richtigen Stichworte. Hört sich ein bisschen komisch an, aber hat einer eine Idee?

Danke vorab.

--47.68.71.205 09:52, 11. Sep. 2014 (CEST)

Hake in den POP3-Optionen vom Thunderbird die Option an, wonach die abgeholten Mails auf dem Server zu löschen sind. --Rôtkæppchen₆₈ 09:56, 11. Sep. 2014 (CEST)
Funktioniert leider nicht. Die Mail kommt trotzdem wieder. 47.68.71.205 (10:25, 11. Sep. 2014 (CEST), Datum/Uhrzeit nachträglich eingefügt, siehe Hilfe:Signatur)
Womit überträgst du Mails, POP3 oder IMAP? Was heißt, sie erscheint immer wieder? Du löschst sie, und nächstes Mal ist sie wieder da, oder ist sie mehrfach vorhanden? Siehst du die Mail auch per Webmail? --Eike (Diskussion) 10:31, 11. Sep. 2014 (CEST)
Die Mails werden mit POP3 übertragen, und die Mail wird nach dem Löschen immer wieder geladen (mit einem Foto). Wenn ich den Thunderbird nach einiger Zeit aufrufe, wird diese Mail auch mehrfach geladen. Webmail? Hmm, weiß nicht so genau was das ist, obwohl ich schnell den Artikel gelesen habe, ich nutze nur Thunderbird und manchmal direkt Yahoo, wo das aber nicht auftritt. 47.68.71.205 (11:05, 11. Sep. 2014 (CEST), Datum/Uhrzeit nachträglich eingefügt, siehe Hilfe:Signatur)
Wenn deine Mailadresse bei Yahoo ist, dann meinte ich genau das: bei Yahoo kucken, wie das mit der Mail ist. Du siehst die Mail auf yahoo.de also überhaupt nicht? In keinem Ordner? --Eike (Diskussion) 11:17, 11. Sep. 2014 (CEST)

Bei Yahoo erscheint die Mail direkt auf der Posteingangsliste, auch an der richtigen chronologischen Stelle. Vielleicht sollte ich diese Mail auch dort löschen, habe jetzt keine Zeit mehr, muss weg vom Rechner. Fällt Dir dazu noch was ein? 47.68.71.205 (11:30, 11. Sep. 2014 (CEST), Datum/Uhrzeit nachträglich eingefügt, siehe Hilfe:Signatur)

Normalerweise schafft Thunderbird sowas auch, aber bei Yahoo löschen sollte auch helfen. --Eike (Diskussion) 11:56, 11. Sep. 2014 (CEST)
Ich hatte das Problem auch mal (mehrfach, unter Windows), es hat absolut nichts geholfen. Auch nicht loeschen der Mail beim Provider (nicht Yahoo). Irgendwann hat es dann von selbst aufgehoert. --192.91.60.11 12:03, 11. Sep. 2014 (CEST)
Die Endlos-Mail deutet auf einen defekten Ordner-index hin. Dazu TB schießen und ein Backup des versteckten .thunderbird-Orders machen. (Unter Windows: versteckte Dateien einblenden, sonst droht Datenverlsut!) Damit ist die gesamte Config samt Mailbestand gesichert. Freien Festplattenplatz prüfen. Es sollte sich der Mailbestand 2mal im Freien Platz der Platte unterbringen lassen. Auf dem betreffenden Ordner nun: Rechtsklick, Eingenschaften, rechten Registerreiter zum reparieren auswählen und mit Knopf Indexwiederherstellung starten. (Komprimieren ist etwas anderes!) Warten! Es kann mehrere Minuten dauern. Nichts erzwingen, keine Mails abrufen. Sollte der Posteingang sehr groß sein, die Archivierung beginnen oder in TB selbst Mail-Unterordner im Postfach anlegen und Mails dort hineinschieben. Ist Sichergestellt, dass kein Mailverlust aufgetreten ist, können die Ordner des Postfachs komprimiert werden. Das Empfangsproblem über POP3 kann beseitigt werden, wenn der als Backup wirksame haken in den Konten-Einstellungen "lasse abgerufene Mails auf dem Server" für einen Mailabruf herausgenommen wird. TB benutzt ein highwater-mark (Hochwassermarke) in jedem eingerichteten Mialordner eines Postfachs (=je Mailadresse) Diese Datei beinhaltet, wann zuletzt abgerufen wurde. Wurde die Datei beim Kopieren nicht übertragen, werden alle Mails auf dem Server heruntergeladen. Hier besteht der Zusammenhang mit Mails auf dem Server zu lassen. Das Programm wird im System (Programme-Ordner unter Windows) installiert. Deine Einstellungen und Mials liegen im Benutzerprofil, Stichwort %APPDATA% (WinXP), %APPDATA%\Roaming (Vista, Win7,8.x), /home/benutzer/.thunderbird (unter Linux, auch ~/ ). Unter den Lokalen Einstellungen (Temporäre Dateien = %TEMP%), unter Linux ~/.cache tummelt sich etwas Müll, der bei geschlossen und fehlerfrei beendeten Programmen oft entfernt werden kann. Er kann auch Störungsursache sein. Einige Linux-Derivate haben im Profil auch den Ordner .local mit dem im Einzelfall geprüft werden muss, was davon zu bewahren und was davon entbehrlich ist. --Hans Haase (有问题吗) 11:13, 12. Sep. 2014 (CEST)

Bestimmung eines Insektes

Welche Insektenart ist auf dem verlinkten Foto zu sehen? [25]--80.128.208.68 16:42, 11. Sep. 2014 (CEST)

Es ist jedenfalls kein Insekt, sondern ein archaisch wirkender Gliederfüßer. Insekten haben überschaubare sechs Beine. Rainer Z ... 16:50, 11. Sep. 2014 (CEST)
Als Laie würde ich Dermestidae-Larve sagen. Eine seriöse Bestimmung gibt es unter Wikipedia:Redaktion Biologie/Bestimmung. --Paramecium (Diskussion) 16:56, 11. Sep. 2014 (CEST)
Stimmt, etwas in der Art könnte es auch sein. Das Bild ist leider verdammt klein. Rainer Z ... 18:36, 11. Sep. 2014 (CEST)
Was ist das auf dem Bild? Es ist ganz klein, sieht harmlos aus und man muss es anklicken, um es größer zu sehen. Antwort: Ein Computervirus. --Pp.paul.4 (Diskussion) 03:09, 12. Sep. 2014 (CEST)

Guten Tag, wer von euch weiß, wer der Großmufti von Albanien ist? Bislang wurde der Großmufti von Tirana zum Vorsitzenden der Moslemischen Gemeinschaft Albaniens, und dieser war dann gleichzeitig der Großmufti. Sicher war dies zuletzt bei Großmufti Selim Muça. Bei Prof. Skënder Bruçaj ist es nicht mehr sicher, er ist zwar der Vorsitzende der Moslemischen Gemeinschaft, wird allerdings nicht mehr als Großmufti betitelt. Ist der Großmufti von Tirana, Ylli Gurra (oder Gura) jetzt der Großmufti von Albanien, oder existiert dieses Amt noch? Bitte jemanden um Hilfe, der evtl. Albanischkenntnisse hat. --63.141.204.102 18:04, 11. Sep. 2014 (CEST)

Man sagt, er sei auch Admin in der dt. Wikipedia. Als frage doch unter WP:AA. --81.92.99.84 10:36, 12. Sep. 2014 (CEST)
Blödsinn. Aber fragen kannst du mal Benutzer:Albinfo oder Benutzer:Alboholic, die schreiben sehr viel zu Albanien. -- sk (Diskussion) 10:54, 12. Sep. 2014 (CEST)
Nun ja, soweit ich weiß, wird der Begriff „Großmufti“ (albanisch Kryemufti/-u) in Albanien selten gebraucht, man trifft ihn eher im Kosovo und in Mazedonien. Hierarchisch ganz oben bei den Sunniten (und nicht bei allen Moslems in Albanien) ist seit 2014 Skënder Bruçaj, der Präsident der Muslimischen Gemeinschaft Albaniens (KMSH) und Großmufti von Albanien, wenn man so will. Auf der offiziellen Internetseite der KMSH ([26]) spricht man selber auch nur vom „Präsidenten“ (alb. Kryetar/-i). Ich hoffe, ich konnte helfen. Gruß--Alboholic (Diskussion) 14:15, 12. Sep. 2014 (CEST)
Was bedeutet „wenn man so will“? Also trägt er diesen Titel offiziell nicht? Gibt es das Amt des Großmuftis noch?--216.185.35.14 16:08, 12. Sep. 2014 (CEST)

Anzahl der Aleviten auf Zypern

Weiß jemand, wie viele Aleviten ungefäht auf Zypern leben? Die offizielle Webseite der Aleviten in der Türkischen Republik Nordzypern ist noch im Aufbau [27], in türkischen Alevitenforen finde ich nur wilde Spekulationen beziwhungsweise unbelegte Behauptungen.--63.141.204.102 18:04, 11. Sep. 2014 (CEST)

das wird schwierig, da wird aus meiner Sicht auch eine "offizielle" Website keine Besserung bringen. Bei der Schnittmenge Zypernkonflikt und des Trends einiger Aleviten zu einer übertriebener Selbstdarstellung (resultierend ggf. aus ihrer Lage in den Heimatstaaten) wird man eine solche Quelle wohl als "abhängig" "nicht neutral" und die Zahlen als vermutlich übertrieben einstufen müssen. Gefragt wären eher Schätzungen nicht-muslimischer Wissenschaftler, denen die innerislamischen Querelen erst mal persönlich nicht nahe gehen. - andy_king50 (Diskussion) 19:46, 11. Sep. 2014 (CEST)

Das ist keine Antwort auf meine Frage. Ich habe gesagt „ungefähr“. Du hast nur die Problematik wiederholt. Gibt es Fachliteratur oder zumindest eine große, aber verlässliche Statistik über die Aleviten-Bektaschiten auf Zypern? --63.141.204.245 20:21, 11. Sep. 2014 (CEST)

Man muss einmal zwei Gruppen von Türken unterscheiden auf Zypern, die lange ansässigen Einheimischen und die nach der Teilung vom Festland angesiedelten. Dann muss man sich fragen, von wo sind die Angesiedelten gekommen, wer wurde ausgewählt, bzw. wer meldete sich freiwillig und wollte dorthin. Dann kann man sich die Prozentzahlen der Ursprungsregionen anschauen und schätzen. Dann hat man einen ungefähren Wert für die Zugewanderten. Dann kann man sich die Fertilität und Lebenserwartung dieser Gruppe anschauen und extrapolieren. Die selbe Methode wendet man dann auch auf die Gruppe der Einheimischen an, von denen man eventuell genauer weiß, wie die religiöse Zusammensetzung vor der Teilung war (unter britischer bzw. griechischer Regierung brauchten sich die Aleviten ja nicht bedeckt halten) und extrapoliert diese Zahlen dann von den Siebzigerjahren bis heute. Da wird dann eine halbwegs gute Schätzung rauskommen. --El bes (Diskussion) 21:08, 11. Sep. 2014 (CEST)
der erste google-Treffer (Suche nach: Nordzypern Religion Aleviten) bringt doch schon ein brauchbares Ergebnis und nennt auch Vereine: [28] --Niki.L (Diskussion) 07:37, 12. Sep. 2014 (CEST)

Windows neu installieren bzw. Festplatte formatieren ohne den alten Datenträger

Weiß jemand, ob und ggf. wie es möglich ist, seine Festplatte zu formatieren und Windows 7 neu zu installieren, wenn man die alte DVD mit dem Betriebssystem etc. nicht mehr hat, sondern nur noch die drei "Rettungs-DVDs" (Recovery), die man damals anfertigen sollte? Auch die "Wiederherstellung" - "erweiterte Wederherstellungsmethoden" über den Explorerer funktioniert nicht, weil dort nach dem "Windows-Installationsdatenträger" gefragt wird...Gruß, --Gustav (Diskussion) 18:56, 11. Sep. 2014 (CEST)

Microsoft bietet die Abbilder der Windows-DVDs zum Download an. Einmal auf DVD gebrannt, hast du wieder einen Installationsdatenträger. Das einzige, was du zur Aktivierung brauchen solltest, ist dein Lizenzschlüssel. Die enthalten keinen vorinstallierten Mist, den deine Recover-DVDs evtl. enthalten, es würden aber evtl. ein paar Treiber fehlen. Wenn die Treiber in deinen Recover-DVDs drin wären, wäre das vll. ein Grund, doch die zu nutzen. --88.130.122.191 19:00, 11. Sep. 2014 (CEST)
Danke für den Hinweis, meinst Du den "Product Key", der sich auf der Rückseite befindet? Dann also diese Iso-Dateien brennen, danach formatieren und neu installieren, sehe ich das richtig? Eigenartig ist übrigens, dass ich das Problem seit dieser Aktion bzw. dem Öffnen der Betrugsnachricht habe und das System nun regelmäig abstürzt, es kann natürlich Zufall sein. --Gustav (Diskussion) 19:19, 11. Sep. 2014 (CEST)
Ja, der ist gemeint. Darauf achten genau die richtige Version zu erwischen, also home/prof und 32/64bit. Idealerweise brennst Du den Datenträger nicht auf (D)einem (eventuell) kompromittiertem System. Für so etwas (und für Partitionstabelle/Partitionierung/Formatierung) bieten sich Live-Systeme auf Linux Basis an, z-B. Knoppix. --Simius narrans 19:30, 11. Sep. 2014 (CEST) P.S.: bez der eigentlichen Frage: Ja, die drei recovery DVDs sollen eigentlich Dein Installationsmedium darstellen, sprich, die Hersteller haben aus Kostengründen keine Win7 DVDs mehr mitgeliefert, sondern die Brennaufforderung bei Erstbenutzung wälzt das auf den Nutzer ab. Ob das bei dir mit Deinen drei DVDs funktioniert (die auch nicht ewig halten) kann ich Dir nicht sagen. Du kommst im Installationssetup von Win7 aber mit Shift & F10 in die Kommandozeile, bezw grafisch bei der Auswahl der Festplatte per "erweitert" in den Dialog wo Du partitionieren sowie formatieren kannst.
Richtig, so sollte das gehen. --88.130.122.191 19:38, 11. Sep. 2014 (CEST)
Danke Euch, leider hat es nicht funktioniert, es wird ständig angezeigt, ich hätte keinen "Installationsdatenträger", obwohl ich den eben, wie oben beschrieben, runtergeladen habe, allerdings wurde da nicht nach dem Schlüssel gefragt... Ich werde es dann wohl morgen in einem Computergeschäft machen lassen, obwohl dort für die Geschichte 60 Euro berechnet werden... Nach diesem "Schock" habe ich mir übrigens einen kostenpflichtigen Virenscanner geleistet, da Avast bzw. Avira wohl manchmal nicht ausreichen...--Gustav (Diskussion) 20:21, 11. Sep. 2014 (CEST)
Was bekommst Du angezeigt, wenn Du den Datenträger ins Laudfwerk steckst? Ist das nur eine Datei X17-24288.iso o.ä. oder sind da verschiedene Ordner, u.a. sources, upgrade, support etc? Im ersteren Fall hast Du die DVD falsch gebrennt. So oder so geht es richtig. --Rôtkæppchen₆₈ 20:30, 11. Sep. 2014 (CEST)
"Ein Datenträger wurde nicht gefunden..."oä. Ja, ich habe die DVD eben erneut gebrannt, es hat wieder nicht geklappt. Ich arbeite nun parallel mit zwei Systemen, Windows 8.1 auf dem noch funktionsfähigen, neueren Computer, mit dem ich jetzt schreibe - und 7 auf dem anderen. Da mir das Ganze schlicht zu zeitintensiv und fummelig ist, werde ich es morgen wohl machen lassen...:(--Gustav (Diskussion) 21:09, 11. Sep. 2014 (CEST)
So brennt man ein iso-Image mit Windows 7. Geht sogar mit Bordmitteln. --88.130.122.191 01:14, 12. Sep. 2014 (CEST)
Die DVD war wohl nicht bootfähig oder im BIOs was das DVD Laufwerk nicht als erstes in der Bootreihenfolge angegeben. Windows installieren ist so eine einfach Sache da kann man sich das Computergeschäft sparen. Google antwortet auf Fragen wie "wie installiere ich windows von DVD" bzw die Fehlermeldungen die man bekommt sehr präzise. Wenn du 15 Minuten Googelst und dich an die Anweisungen hälst kannst du dir das Computer Geschäft sparen und hast wesentlich schneller wieder einen lauffähigen Rechner. Kaput machen kannst du ja nichts. --92.227.206.106 10:28, 12. Sep. 2014 (CEST)

Access Point als Subnetz betreiben

Hallo, folgendes Problem: Ich hab hier Internetzugang über einen Router, bei dem jedoch nicht DHCP aktiviert ist. Das kann ich auch nicht umstellen und den Internetzugang teile ich mit vielen Leuten. Ich möchte nun an meinen Internetanschluss einen Router aufstellen, der jedoch nur für mich gilt, das zentrale Netz jedoch nicht beeinflusst. Also ich möchte halt WLAN für mein Laptop etc haben. Was muss in meinem Router einstellen, um das zu erreichen? Anmerkung: Bisher war es hier so, dass der zentrale Router DHCP hatte, sodass ich bei meinem lokalen Router einfach DHCP deaktivieren konnte. Geht das weiterhin? 85.212.8.235 19:31, 11. Sep. 2014 (CEST)

Bei den meisten Routern mit Access Point steckst Du den Anschluss an das bestehende Netz an die Buchse WAN. Den Router/AP konfigurierst Du so, dass er auf der WAN-Seite DHCP-Client macht. Wenn kein WAN-seitiger DHCP-Server existiert, stelltst Du stattdessen die vom Netzverwalter bereitgestellte IP-Adresse, Netzmaske, Gateway und DNS ein. Auf der LAN-Seite des Routers aktivierst Du DHCP-Server und NAT. --Rôtkæppchen₆₈ 20:38, 11. Sep. 2014 (CEST)
Bei meinem SMC-Router kann ich unter WAN gar nichts derartiges einstellen. Da gibts nur die Reiter "ATM PVC", "Clone MAC Address" und "DNS". ATM PVC sieht am vielversprechensden aus, da gibts ein "ATM Interface". Ist das das was du meinst? Eine WAN-Buchse hat mein Router auch nicht, nur eine ADSL- und vier LAN-Buchsen. 85.212.8.235 20:48, 11. Sep. 2014 (CEST)
Diesen Router kannst Du dafür nicht verwenden, ohne den übergeordneten DHCP zu stören. Wenn Du es trotzdem tust, kannst Du den Internetzugang des gesamten Netzes lahmlegen. Tu’s nicht, Deinen Mitbenutzern zuliebe. Bei manchen Routern lässt sich LAN1 als WAN-Buchser konfigurieren, z.B. bei den Fritzboxen. Vielleicht geht das auch bei Deinem Router. Wenn Du uns Hersteller, Typenbezeichnung und Version des Routers nennst, können wir mal gemeinsam ins Handbuch schauen. --Rôtkæppchen₆₈ 20:57, 11. Sep. 2014 (CEST)
Ja, ich kenn das, war schon mehrmals der Fall, dass einzelne DAUs mit sowas das Netz geschrottet hatten, deshalb bin ich vorsichtig ;). Der Router ist ein SMC7904WBRB2. 85.212.8.235 21:00, 11. Sep. 2014 (CEST)
Ich hasbe im Handbuch leider keinen Hinweis darauf gefunden, ob einer der LAN-Ports als WAN-Port benutzt werden kann. Mein Bruder hatte ein ähnliches Problem und er hat sich gleich einen passenden Router/AP gekauft, um seine Rechner nicht für wildfremde Menschen sichtbar zu machen. --Rôtkæppchen₆₈ 21:43, 11. Sep. 2014 (CEST)
Mist. 85.212.8.235 21:51, 11. Sep. 2014 (CEST)
Wenn Du einen PC mit zwei Ethernetanschlüssen übrig hast, kannst Du auch Internetverbindungsfreigabe über den zweiten Ethernetanschluss machen und an den Deinen Router/AP mit aktiviertem DHCP anschließen. --Rôtkæppchen₆₈ 22:17, 11. Sep. 2014 (CEST)
Ist es dieser Router? Wenn ja, kannst du da OpenWrt draufspielen und die Einschränkungen der Herstellerfirmware damit umgehen. -- Janka (Diskussion) 22:32, 11. Sep. 2014 (CEST)
Nein, es ist kein "SMC-7908-ISP", sondern ein SMC7904WBRB2. 85.212.8.235 22:47, 11. Sep. 2014 (CEST)
So genau passen die Produktnummern in den Listen selten - wenn, dann bei häufiger verbastelten Geräten, bei denen die Leute die Unterschiede schon dokumentiert haben. Es lohnt sich zu gucken, ob der Router nicht baugleich ist (z.B. daran zu merken, dass die aufgelisteten Features identisch sind). -- Janka (Diskussion) 00:11, 12. Sep. 2014 (CEST)
Du willst einen Router hinter einem Router, der eine DMZ (Demilitarisierte Zone) halten soll. Stellst Du statt Deinem Client einen Router (nicht nur Accesspoint, da Dir DHCP fehlt und Du wohl nur eine IP hast), kannst Du in WAN-Seitig fest einstellen. Sein LAN wird er aber in das WAN routen, womit auch der Zugriff und damit die Beeinflussung möglich wird. Um dies zu lösen, musst Du Deinem Router den Zugriff per IP-Ausschluss auf die DMZ (das LAN, das das WAN Deines Routers ist) verbieten und nur, wenn nötig reservierten Clients (per MAC-Adresse) erlauben. Bedenke, dass längst nicht alle Router so etwas unterstützen. Sollte ein Linux drauf laufen (was es oft vorkommt), ist die Frage ob Du IPTABLES, INETD o. ä. RESET-resistent konfigurieren kannst. Einige Router bringen manche dieser Funktionalitäten im Websetup mit. --Hans Haase (有问题吗) 10:51, 12. Sep. 2014 (CEST)

Wird das Universum uns bekannter?

In kosmologischen Darstellungen ließt man immer wieder die Aussage, dass die Grenzen des uns bekannten Universums nicht die Grenzen des Universums an sich sein müssen, weil (verkürzt gesagt) es möglich wäre, dass das Licht aus den fernen Gegenden von Jenseits des bekannten Universums noch nicht zu uns gelangt ist. Meine Frage lautet: Gibt es tatsächlich Beispiele dafür, dass etwas in unser Sichtfeld gerückt ist, dass vorher noch nicht sichtbar war? Aufgrund der Endlichkeit der Lichtgeschwindigkeit würde es dann wahrscheinlich so aussehen, als ob ein Quasar (oder ähnliches) neu entsteht. (nicht signierter Beitrag von 188.100.190.212 (Diskussion) 00:09, 12. Sep. 2014 (CEST))

Wenn es stimmt, wie manche Physiker behaupten, dass die Expansionsgeschwindigkeit der Raumzeit sogar zunimmt, dann gibt es Gegenden am Rande des Universums, die wir nie zu sehen bekommen werden, weil die Raumzeit dort schneller expandiert als die Lichtgeschwindigkeit. --El bes (Diskussion) 00:52, 12. Sep. 2014 (CEST)
Es gibt ständig Dinge, die wir jetzt sehen, die vorher noch nicht zu sehen waren. Einfache Sichtweise: Das Licht das wir heute sehen, hatte uns gestern noch nicht erreicht. Das kann man zurückverfolgen bis zum ersten Licht, das noch existiert - dem kosmischen Mikrowellenhintergrund. Unser beobachtbares Universum vergrößert sich also (und das nicht nur durch seine Ausdehnung). Gleichzeitig ist es aber richtig, dass wir - sofern die beschleunigte Expansion so bleibt - manche Dinge nie zu Gesicht bekommen werden. Dazu kann man aber nicht die Expansionsrate mit einer Geschwindigkeit vergleichen, die Rechnung ist etwas komplizierter. Wir sehen durchaus Dinge, deren Abstand schneller als die Lichtgeschwindigkeit wächst. --mfb (Diskussion) 01:15, 12. Sep. 2014 (CEST)

Warum...

ist es so laut, wenn die Wasserleitung gebrochen ist?

Ich hatte eben das zweifelhafte Vergnügen den Stadtwerken austretendes Wasser auf der Straße zu melden. Mittlerweile waren sie da und es kommt zumindest nichts mehr. Was ich nicht verstehe: Der Wasserrohrbruch liegt einige Meter bevor das Wasser hier ankommt und auch bevor es das Ventil dieses Hauses erreicht. Ich bin durch lautes Fließgeräusch wach geworden und zwar kam das hier aus dem Haus. Die ganzen Leitungen haben vibriert. Warum? Die Leitungen hier im Haus sind erwiesenermaßen nicht undicht. --88.130.88.58 04:38, 12. Sep. 2014 (CEST)

An der Bruchstelle bilden sich Wirbel durch das austretende Wasser. Diese Wirbel versetzen das Wasser in den Leitungen und die Leitungen slebst in Schwingungen. Da Flüssigkeiten und Feststoffe den Schall gut leiten, pflöanzt sich der Schall auf angrenzende Anlagenteile fort. --Rôtkæppchen₆₈ 06:40, 12. Sep. 2014 (CEST)
der Wasserdruck ist auch nicht zu unterschätzen... er ist wohl so um die 4bar, glaub ich... --Heimschützenzentrum (?) 11:23, 12. Sep. 2014 (CEST)

Was muss ich mir unter Sakralenergetik vorstellen ?

Bei mir hupft grad so ein Typ rum, der ist mit einem Auto angekommen auf dem "Sakralenergetik" steht. Rausgefunden habe ich, dass es sich um eine Behandlungsmethode bei Gelenkproblemen, Knochenzeugs und so weiter handelt. Mich irritiert allerdings vor allem der Name. "Sakral" und "Energetik". Letzteres ist auch so ein Wort bei dem sich mir die Nackenhaare aufstellen. Ich befürchte, dass ist irgendwas esoterisches. Aber kann mir kein genaues Bild machen. Jemand eine erleuchtende Erklärung für mich oder einen Link wo ich mehr als nur Allgemeinplätze oder Werbung rauskriege? --Ironhoof (Diskussion) 08:28, 12. Sep. 2014 (CEST)

"Die Lehre von der heiligen Energie." Lass ihm die Luft aus mindestens einem Reifen, er wird sakralenergetisch trotzdem wegfahren können. --81.92.99.84 08:40, 12. Sep. 2014 (CEST)
Das stimmt so nicht ganz: „sakral“ meint Os sacrum (Kreuzbein), aber „Energetik“ bezieht sich auf die esoterische Feinstofflichkeit [29]. --Mps、かみまみたDisk. 08:44, 12. Sep. 2014 (CEST)
Das hat nichts mit Esoterik oder Kirche zu tun sondern mit dem Sakralgelenk, siehe dazu Iliosakralgelenk. -- Frila (Diskussion) 08:48, 12. Sep. 2014 (CEST)
Naja, die Methode besteht darin „Leitlinien“ (hier „Energielinien“) zu messen und dann neuauszurichten, was schon esoterisch ist. Zudem bietet es wohl an vorher eine Ausbildung zum „Chakren- und Auraenergetiker“ gemacht zu haben [30]. --Mps、かみまみたDisk. 09:09, 12. Sep. 2014 (CEST)
„Gesundbeter“ klang dem Typen zu wenig eindrucksvoll. -- Unfugsbeseitiger (Diskussion) 09:15, 12. Sep. 2014 (CEST)
Guck mal hier: Chakra#Sakralchakra auch Sexualchakra. --178.4.178.242 13:04, 12. Sep. 2014 (CEST)

Alternative Heilkunde: Existenz eines „Magenzahns“

Liebe Auskunft, eine alternative Heilpraktikerin erwähnte mir gegenüber eine Verbindung des ersten prämolaren Zahns unten rechts (im Zahnschema: 44) mit dem Magen-Darm-Trakt (sogenannter Magenzahn). Ist diese Verbindung in der alternativen Heilkunde, zum Beispiel in der Meridianlehre, üblich? Vielen Dank für Antworten! --BlackEyedLion (Diskussion) 14:43, 12. Sep. 2014 (CEST)

Eine Google-Suche ergibt einige Ergebnisse mit mehr oder weniger seriösen Angeboten wortwörtlich wohl doch ehr im Esoterischen Bereich. Vielleicht kann ja der Sakralenergetiker von weiter oben weiterhelfen --Jogo.obb (Diskussion) 14:50, 12. Sep. 2014 (CEST)

„Deutsche“ Umschrift des Japanischen

Guten Tag,

gibt es eine Umschrift des Japanischen, die sich nach der deutschen Aussprache richtet, so wie es sie für das kyrillische Alphabet gibt? Damit meine ich eine, bei der unter anderem nicht sh, sondern sch verwendet wird.

Grüße, --Cultor linguarum antiquarum Πράττειν διδάσκει ἡ φιλοσοφία, οὐδὲ λέγειν. 13:22, 10. Sep. 2014 (CEST)

Probier mal ISO 3602. --Rôtkæppchen₆₈ 13:49, 10. Sep. 2014 (CEST)
Konnitschi wa! Uschi deßu.
Darf ich vorstellen: Uschi, die Kuh. ;) -- HilberTraumd, m14:14, 10. Sep. 2014 (CEST)
Wie wuerde man dann Suzuki schreiben, weil wie ein deutsches z wird das z da drin ja nicht ausgesprochen. --192.91.60.10 14:24, 10. Sep. 2014 (CEST)
Wenn Du eine Romanisierung nach deutscher Laut-Buchstaben-Zuordnung willst, kannst Du es ja wie bei der Kyrillischtranskription machen: с wird zu ss, außer am Wortanfang und з wurd zu s. Die Koreaner haben mit Doppelkonsonanten am Wortanfang auch kein Problem, also wäre Ssusuki wahrscheinlich das Ergebnis. --Rôtkæppchen₆₈ 14:39, 10. Sep. 2014 (CEST)
So wirklich ist kein System das Goldene vom Ei, siehe hier. --Cultor linguarum antiquarum Πράττειν διδάσκει ἡ φιλοσοφία, οὐδὲ λέγειν. 14:48, 10. Sep. 2014 (CEST)
So machte es Gustav Langenscheidt
Ich hatte mal ein Lexikon, das eine der Methode Toussaint-Langenscheidt angelehnte Lautschrift benutzt hat. Da stand dann ganz zwanglos ß auch am Wortanfang für ein stimmloses S. Zusammen mit dem neuartigen großen ß ließe sich auch etwas machen: ẞusuki. --Rôtkæppchen₆₈ 23:55, 10. Sep. 2014 (CEST)
Sieht doch schön aus. --Altsprachenfreund Facere docet philosophia, non dicere. 10:57, 11. Sep. 2014 (CEST)
Meine Meinung: Wenn sich schon jemand mit einer Fremdsprache beschäftigt, kann er sich auch die Mühe machen, die dazugehörige Schreibung zu lernen, es gibt ja Umschrift für japanisch. --Hans Eo (Diskussion) 18:34, 13. Sep. 2014 (CEST)
Sicherlich. Wenn ich aber Japanisch wirklich lernte, wäre mir eine deutsche Umschrift lieber als eine englische. --Altsprachenfreund Facere docet philosophia, non dicere. 21:06, 13. Sep. 2014 (CEST)
Archivierung dieses Abschnittes wurde gewünscht von: --Altsprachenfreund Facere docet philosophia, non dicere. 17:12, 13. Sep. 2014 (CEST)

Möglichkeiten bei Einbrecher-Kontrollanruf

Heute früh wurde in der Wohnung meiner verreisten Freundin, in der ich übernachtet hatte, von einer unterdrückten Telefonnummer angerufen (in Österreich). Der Anrufer behauptete in radebrechendem Englisch, von Microsoft zu sein und eine Frage zu ihrem Computer zu haben. Ich glaube zu 99%, dass das ein Einbrecher auf Auslotung war, ob die Wohnung leer ist und die Bewohner verreist sind. Was kann man in solchen Fällen sinnvolles tun? Wenn man sein Telefon überwachen lässt - könnte die Polizei eine unterdrückte Nummer zurückverfolgen? Hätte es einen Sinn, solche Anrufe irgendwie aufzuzeichnen (um die Stimme zu dokumentieren)? Sonst etwas? --Anonyme Frage (Diskussion) 13:55, 10. Sep. 2014 (CEST)

Das sind wahrscheinlich keine Einbrecher: http://www.zdnet.de/88180915/lka-warnt-betrugsanrufen-angeblicher-microsoft-support-mitarbeiter/ --Eike (Diskussion) 13:57, 10. Sep. 2014 (CEST)
@„könnte die Polizei eine unterdrückte Nummer zurückverfolgen?“ Lass Dir von Deinem Telefonanbieter das Leistungsmerkmal MCID freischalten, dann kannst Du das auch selbst. Du musst dafür aber eine stichhaltige Begründung liefern, warum Du das brauchst. Die Polizei nutzt auch nur MCID. Dieses Merkmal ist z.B. bei Notrufnummern standardmäßig aktiviert, muss bei gewöhnlichen Anschlüssen aber vorher freigeschaltet werden. --Rôtkæppchen₆₈ 14:07, 10. Sep. 2014 (CEST)
Habe noch nie etwas von einem Einbrecher gehört, der bei fremden Leuten vorher anruft und denke auch, das wäre ganz schön blöd von dem Einbrecher. Der Link von Eike dürfte da weit eher in die richtige Richtung gehen. --178.0.193.156 21:25, 11. Sep. 2014 (CEST)
Solche Anrufe gehören zum Standardrepertoire von Einbrechern, siehe als Beispiel hier. Aber es dürfte stimmen, dass die dann (immer?) sofort auflegen, wenn man abhebt. --Anonyme Frage (Diskussion) 12:06, 12. Sep. 2014 (CEST)
Hier ([31]) ist auch solch ein Beispiel. --Lkl13:29, 13. Sep. 2014 (CEST)

Unterirdische Landebahn bei Posen

Vor Jahren habe ich mal (bei Wolf Jobst Siedler?) gelesen, dass die Wehrmacht/Organisation Todt bei Posen eine 5 km lange unterirdische Landebahn gebaut hatte, die bis heute längste der Welt. Im Internet finde ich dazu so gut wie nichts, nur Focke Wulf, Flugzeugbau und Bunker. Hat jemand eine Idee?--Mehlauge (Diskussion) 15:50, 10. Sep. 2014 (CEST)

Hilft das hier weiter? --88.130.92.213 16:02, 10. Sep. 2014 (CEST)
Unterirdische Landebahn ist irgendwie bisschen unlogisch. Es gibt Startbahnen die im Berg liegen, die sind aber Funktion her vergleichbar mit einem Katapult eines Flugzeugträger. Du willst im Erstfall die Flieger vor allem zuerst mal hoch bringen. Beim runter kommen hast du viel mehr Alternativen, denn das muss nicht zwingend dort sein wo zu Beginn der Kampfhandlungen das Flugzeug stand. --Bobo11 (Diskussion) 21:18, 10. Sep. 2014 (CEST)
Es gab diese Startbahn und die Länge zeigt an, dass sie für größere Fliegerverbände gedacht waren, also für militärische Nutzung. Lange Bahnen braucht man auch für Bomber, die bis ans Limit Ladung dabei haben, aber ich bezweifle, dass die tatsächlich 5 km lang war. Das bezieht sich vermutlich auf die Länge der Stollen und auf die Stellflächen für die Flieger, die Wartungsbereiche, Lager und Versorgung, die ja auch nötig sind. Die unterirdischen Startbahnen sind natürlich in erster Linie zur Sicherheit, denn Flugzeuge sind am Boden extrem verwundbar und mit wenigen Bombentreffern kann man eine Flugbahn unbenutzbar machen und Starts und Landungen verhindern. Es ist auch sinnvoll das Teil nur auf Flugbasen zu landen und nicht ungeschützt sonstwo, denn dort ist es fraglich, ob du Ersatzteile oder Sprit bekommst und jemals wieder abheben kannst, außerdem drohte feindlicher Beschuss.--Giftzwerg 88 (Diskussion) 21:32, 10. Sep. 2014 (CEST)
Der Schutz brauchst du aber eigentlich nur beim Start. Der erfolgt in der Regel auch wenn der Flughafen -und damit die Startbahn- angegriffen wird. Die Landung sollte eh nur zu kampf-freier Zeit erfolgen. Und dann kannst du eben auf die unbeschädigte Ladebahn ausweichen. Während es unklug während eines Angriffes zig Kilometer auf dem Taxiway herum zufahren, bis du bei der unbeschädigten Startbahn bist. Das man nach der Landung unmittelbar von der Landebahn in etwas verbunkertes kommt ist natürlich schon praktisch. Aber es ist von Vorteil wenn die Landebahn um ein vielfaches länger ist als das Flugzeug zum Landen braucht, weil dann kann sich das Flugzeug auf denn Kilometer unbeschädigte Landebahn ausweichen, dass es zum landen braucht. Aber auch bei den Eingänge gilt je mehr desto besser. Weil ob ich jetzt 1 Eingang der Landebahn oder 10 Eingänge zu den Unterständen zerstören muss, um den Flughafen unbrauchbar zu machen, macht schon ein Unterschied aus.--Bobo11 (Diskussion) 07:08, 11. Sep. 2014 (CEST)

Das hatten wir doch gerade letzte Woche Kavernenflugplatz oder ähnlich. Zum Landen braucht man auf jeden Fall eine offen sichtbare Landebahn. Wer mal die Bilder von den waidwunden Bombern im Landeanflug gesehen hat, weiss warum. --Eingangskontrolle (Diskussion) 22:18, 10. Sep. 2014 (CEST)

na ja, die defekten hätte man dann sicher ausserhalb landen lassen zur Not geopfert, aber was sich mir nicht erschließt, welches Ufo damals 5 km Landebahn brauchte. Selbst das Space shuttle hatte nur 4,5 km lange Landebahnen. - andy_king50 (Diskussion) 23:10, 10. Sep. 2014 (CEST)
Der Großteil einer Start/Landebahn wird ja zum Beschleunigen/Abbremsen gebraucht, es hätte wohl gereicht, wenn z.B. 500m im Freien gewesen wären und der Rest hätte dann im Berg sein können. @Andy king50: die Deutschen hatten ja im 2. Weltrkrieg recht viele Projekte zur Entwicklung für neue/experimentelle Flugzeuge am Laufen, vielleicht wollten sie nur sichergehen, dass dort jedes zukünftige Flugzeug starten kann. In Area 51 gabs ja sogar mal eine befestigte Landebahn mit >8km Länge (und in der Edwards Air Force Base gibts eine u8nbefestigte Landebahn mit fast 12km Länge), also 5km kommt mir nicht so übertrieben vor... --MrBurns (Diskussion) 03:48, 11. Sep. 2014 (CEST)
Zum Durchstarten wäre es aber hilfreich, wenn nach den 500m offene Landebahn nicht direkt ein Berg kommt. Soll ja mal vorkommen, dass eine Landung nicht im ersten Anlauf glückt. --FGodard||± 12:14, 11. Sep. 2014 (CEST)
Das Space-Shuttle hatte noch ein paar mehr List of space shuttle landing sites auch welche mit 11000m. Zur Ausgangsfrage: Im Zuge der U-Verlagerung wurden vermutlich einige Anlagen für den Raum Posen geplant. Ob es dann auch gebaut wurde, ist eine andere Frage. Siehe auch da.--Advanceddeepspacepropeller (Diskussion) 10:16, 12. Sep. 2014 (CEST)
Die Space-Shuttle-Piloten konnten nach der Landung auch nicht durchstarten (weil Space Shuttles ohne Antrieb landeten). Daher wenn man an diesem Stützpunkt nur speziell geschulte Piloten landen lässt (außer bei Notfällen), sehe ich kein großes Problem. Die Sicherheitsstandards im militärischen Bereich sind ja auch niedriger als im zivilen, vor allem wenns einen großen Krieg gibt... --MrBurns (Diskussion) 01:56, 13. Sep. 2014 (CEST)

1 jahr auf einer kanadischen schule

Wie mach ich es wenn ich aus deutschland komme und für 1 jahr auf eine kanadische schule gehen will da ich dort verwandte habe bei denen ich wohne? wie melde ich mich an der schule an ? denn ich mache nun die 10 klasse an einer realschule und möchte nach meinem realschulanschluss für 1 jahr nach kanada.

--95.112.162.123 (20:55, 10. Sep. 2014 (CEST), Datum/Uhrzeit nachträglich eingefügt, siehe Hilfe:Signatur)

Frag deine Verwandten, die kennen sich dort aus und können leichter rauskriegen, welche Schule geeignet ist und was du dort machen musst und an Qualifikationen für die Aufnahme kennen musst. Eventuell musst du nämlich einen Test machen.--Giftzwerg 88 (Diskussion) 21:41, 10. Sep. 2014 (CEST)


Das haben die kanadischen (und auch die us-amerikanischen) Steuerzahler gern. Warum erheben den die Austauschprogramme teilweise recht hohe Gebühren? Sicher nicht für die Gastfamilien. Auf jeden Fall braucht man ein spezielles Visum. --Eingangskontrolle (Diskussion) 22:25, 10. Sep. 2014 (CEST)

Wikipedia bietet Schulsystem in Kanada, die Botschaft von Kanada in Deutschland bietet Schulbesuch in Kanada (sogar auf Deutsch!). --Pp.paul.4 (Diskussion) 22:36, 10. Sep. 2014 (CEST)
Nur waren die Kanadier bei PISA stets besser. --Hans Haase (有问题吗) 18:35, 12. Sep. 2014 (CEST)

Wieso wissen die meisten Menschen was sie am 11. September 2001 gemacht haben und wo sie waren?

46.115.142.103 07:18, 11. Sep. 2014 (CEST)

Selektive Wahrnehmung eines besonderen (emotionalen) Ereignisses (Aber Vorsicht bei flashbulb memories! (Blitzlichterinnerungen). Die Reiferen hier werden sich auch noch erinnern, wo sie von Kennedys Ermordung gehört haben oder wo sie mit wem waren, als der erste Mensch auf dem Mond landete oder wo Müller nochmal nach hinten angelte und die Niederländer weinend zurückliess.... GEEZER… nil nisi bene 07:25, 11. Sep. 2014 (CEST)
Es ist höchst fraglich, ob die meisten Menschen es wirklich noch wissen. Anlässlich des Challenger-Unglücks wurden seinerzeit zahlreiche Menschen befragt, wo und wie sie erstmals davon erfuhren. Nur die wenigsten hatten es live erlebt, die meisten hatten es nachträglich in den Nachrichten gesehen. Wenige Jahre später wurden genau dieselben Personen nochmals befragt (wohl wissend, dass ihre ersten Antworten aufgezeichnet worden waren!), und jetzt erinnerten sich die meisten daran, das Unglück live im Fernsehen gesehen zu haben. Die Kraft der Bilder ist sehr stark, und die menschliche Erinnerung ist ein schlechtes und für Beeinflussungen jeder Art sehr empfindliches Beweismittel (unter Juristen gilt es sogar als das schwächste der prozessual zur Verfügung stehenden Beweismittel).
Ich selbst war übrigens damals im Büro, wo sonst niemals Radio oder Fernsehen lief. Bei einem Telefonat erfuhren wir damals von den Terroranschlägen und ließen den restlichen Tag das Radio laufen. --Snevern 08:17, 11. Sep. 2014 (CEST)
Weil da die Anschläge vom 11. September 2001 stattgefunden haben. --Eike (Diskussion) 08:23, 11. Sep. 2014 (CEST)
Es ist noch viel schlimmer. In Experimenten hat man gezeigt, dass sich Leute an Details von Ereignissen erinnerten, die nie stattgefunden hatten. Wenn der "soziale Druck" der Mitmenschen gross genug ist (beispielsweise in einem Gerichtsprozess) erinnert sich Karl-Heinz an Dinge, die er nie erlebt und sinnlich wahrgenommen hat... Ich sag' nur Menschen... GEEZER… nil nisi bene 09:11, 11. Sep. 2014 (CEST)
Karl-Heinz Rummenigge ? --RobTorgel 09:21, 11. Sep. 2014 (CEST)
... und vermutlich denkt sich jeder von uns grad: Ich weiß natürlich noch, was ich da gemacht habe, aber von den anderen haben bestimmt viele falsche Erinnerungen! --Eike (Diskussion) 09:23, 11. Sep. 2014 (CEST)
Ne, ich denke grad: Ich weiß noch so ungefähr, was ich gemacht habe, als im Radio die erste Meldung von einem Flugzeugabsturz über New York kam, und bald darauf von einem zweiten. Ich weiß aber nicht mehr, was ich gemacht habe, als zum ersten Mal das Wörtchen "Anschlag" erwähnt wurde. Und was ich den ganzen Tag davor und danach gemacht habe? Keine blassen Schimmer... Geoz (Diskussion) 09:48, 11. Sep. 2014 (CEST)
WO kriegt ihr euer Geld her ? Ich war in der Arbeit, wie fast jeden Dienstag --RobTorgel 09:55, 11. Sep. 2014 (CEST)
Ich war auf dem Heimweg vom Einkaufen. --Rôtkæppchen₆₈ 10:15, 11. Sep. 2014 (CEST)
Am Nachmittag um 3 ? Schichtler oder Bäcker ? --RobTorgel 10:21, 11. Sep. 2014 (CEST)
Teilzeitkraft. --Rôtkæppchen₆₈ 10:34, 11. Sep. 2014 (CEST)
Ich hatte damals an dem Tag die Schulbücher fürs neue Schuljahr bekommen, und weil ich schon immer ein Streber war, habe ich im neuen Physikbuch geblättert (und mich davon auch nicht durch Meldungen zu irgendwelchen Flugzeugen irgendwo in Amerika abbringen lassen). --132.230.1.28 10:54, 11. Sep. 2014 (CEST)
Ich hatte vorlesungsfreie Zeit und kam beim Rumzappen bei CNN vorbei und dachte mir 'Toll, hat CNN es jetzt auch schon noetig den neusten bescheuerten Katastrophenfilm zu promoten. *augenroll*' Beim zurueckzappen wurde mir dann klar, dass war keine neuste Hollywoodkreation und dann krachte auch schon das zweite Flugzeug in den anderen Turm. --192.91.60.11 12:00, 11. Sep. 2014 (CEST)
Ruhestand - vor dem TV. Habe Madame angerufen: "Gerade ist ein Flugzeug ins WTC geflogen." Dann aufgehängt. Ein paar Minuten später nochmal angerufen: "Gerade ist noch ein Flugzeug ins WTC geflogen." "Ja, hast du doch schon gesagt." "Nein, ein zweites Flugzeug..." "Wieso denn zwei..?" etc. etc. GEEZER… nil nisi bene 15:45, 11. Sep. 2014 (CEST)

Ganz normale Konditionierung oder auch lernen. Wenn man immer wieder gezwungen wird etwas zu beantworten, manifestiert es sich. Ob es stimmt oder nicht (Passt nicht ganz, aber: Stell dir ein großes kühles Glas schneeweißer Milch vor und sage jetzt ganz schnell, was Kühe trinken).--Wikiseidank (Diskussion) 12:32, 11. Sep. 2014 (CEST)

Ich nehme an, du meinst kleine Kühe... --Snevern 13:06, 11. Sep. 2014 (CEST)
Bei schneeweißer Milch handelt es sich auch sicher um ein dubioses chemisches Produkt. Das Weiß der Milch hat einen leichten Gelbton und unterscheidet sich erheblich vom Weiß eines frisch gefallenen Schnees. Andererseits verstehe ich, daß "milchweiße Milch" doch etwas unglücklich klingt und daher weiteres Feilen an der Formulierung vonnöten ist.--212.184.134.8 16:42, 11. Sep. 2014 (CEST)
Er meint Kühe!!! Was trinken Kühe? --Pp.paul.4 (Diskussion) 16:45, 11. Sep. 2014 (CEST)

Also ich kann mich ganz genau erinnern, ohne mediale Konditionierung. Ich war damals mit einem Freund auf Urlaub in Portugal, in einem Ferienort wo viele englischsprachige junge Touristen waren. Tagsüber waren wir immer am Strand, am Abend sind wir durch die Lokale gezogen, wo wir einen Haufen Engländer, Iren, Kanadier, Australier und US-Amerikaner kennen gelernt haben und so richtige Urlaubsfreundschaften geschlossen haben (Portugiesen, Brasilianer und Angolaner waren auch darunter). Am 11. September 2001 waren wir am frühen Nachmittag in einer Strandbar, wo so elektronische Chil-Musik vom DJ aufgelegt worden ist. Wir sind mit einem Kanadier osteuropäischer Abstammung zusammengesessen, den wir schon ein paar Tage kannten. Der war schon etwas älter, so Ende 40 und Uniassistent und von der Einstellung so akademisch "liberal" wie man drüben sagt. Mit dem haben wir beim verspäteten Frühstück angeregt politisiert, über den Unterschied zwischen Europa und Amerika und über den Unterschied zwischen Kanada und den USA, über den Vietnamkrieg, den US-Imperialismus in Lateinamerika bis zu den Waffengesetzen. Plötzlich kommt die englische Kellnerin daher und sagt es ist was ganz schlimmes passiert, New York ist angegriffen und komplett zerstört worden, die F16 fliegen schon und haben den Luftraum gesperrt. So ihre Worte. War natürlich ein wenig übertrieben, aber so hat sie es wohl im portugiesischen Fernsehen verstanden, das nebenbei in der Bar lautlos lief. Wir waren natürlich geschockt und wollten unbedingt mehr erfahren. So haben wir gezahlt und sind von der Strandbar zurück in die Stadt spaziert. Am Weg haben wir ein paar andere englischsprachige Zechkumpanen der vorigen Abende getroffen, die auch alle ganz aufgeregt waren, aber auch nichts genaueres wussten. Portugiesische Zeit war es da so etwa 16:00, dh. in New York so etwa 11:00. Mit zwei von den Engländern sind wir dann eine Bar suchen gegangen, wo es CNN gab und dort sind wir dann stundenlang vor dem Fernseher gesessen und haben die Berichterstattung verfolgt, mit allen Falschmeldungen und Vermutungen die am Anfang verbreitet wurden. Nach und nach sind in der Bar immer mehr von den anglophonen Touristen aufgetaucht, die auch alle CNN sehen wollten. Ich und mein Reisefreund waren interessiert, was da passiert ist, aber die Engländer, Kanadier und Amis waren emotional alle komplett geschockt und durch den Wind. Die hat das viel näher berührt und irgendwie kannten alle von denen irgendwen, der in New York lebt. Eine junge Engländerin, die da irgendwie in der Clique dabei war, hat gerade ein Ferialpraktikum in New York hinter sich gehabt, wo sie im World Trade Center gearbeitet hat. Ihr Vater arbeitete fix dort, bei einer von den neuen Handynetz-Firmen, ich glaub es war Verizon. Sie versuchte ihn ständig von der Bar aus anzurufen, ihr Handy ging nämlich nicht mehr, weil eben dieser Handyanbieter seine Zentrale im World Trade Center hatte. Sie hat ihren Vater nicht erreicht, eventuell war er einer von denen die dort umgekommen sind. Genaueres habe ich nie erfahren. Am Abend sind wir mit dem Kanadier portugiesisch Essen gegangen, Calamari, und dann wieder in eine Bar, wo CNN lief. Mitlerweile waren auch die portugiesischen Einheimischen dort versammelt, weil die auch mehr erfahren wollten, als in den portugiesischen Abendnachrichten gesagt wurde. Jedenfalls war der Abend noch sehr emotional, die Amis und Engländer waren alle sehr geschockt, einige Mädchen weinten ganz am Boden zerstört in der Bar, die Burschen tranken ein Beruhigungsbier und alle diskutierten mit jedem, was denn da passiert ist und wie das zu deuten ist. Auch Leute die sich gar nicht kannten, versanken in lange Diskussionen, einfach weil das Ereignis so erschaudernd war. Als in der Nacht dann schon von einem Vergeltungsschlag gegen Afghanistan die Rede war, gabelte sich die Stimmung ein wenig. Wir und der Uni-Kanadier fanden das komplett idiotisch, jetzt gleich ohne Beweise und Zusammenhang Marschflugkörper nach Afghanistan zu feuern, ein Teil der anderen anglophonen Bargäste fand das aber ganz gut. Die Diskussionen gingen jedenfalls weiter bis mindesten drei Uhr morgens, immer vorm Barfernseher, den wir aufmersamst verfolgten. Am nächsten Tag zerstreute sich dann die Runde von Urlaubsbekanntschaften, die einen buchten ihren Flug um, um schnell nach Hause zu kommen, die anderen zogen weiter an andere Küstenorte. Wir blieben noch zwei Tage und mussten dann ebenfalls Richtung Flughafen, weil unser Urlaub zu Ende war. Diese spontane familiäre Stimmung vom Abend des 11. September, wo jeder mit jedem diskutierte und alle sich betroffen fühlten, sich gegenseitig trösteten und Mut zusprachen, war dann bald wieder weg. Der Flug war übrigens ganz normal, portugiesischer Provinzflughafen, ganz gemütlich, noch keine besonderen Sicherheitschecks. Zu Hause angekommen zog ich mir dann die deutschsprachige Berichterstattung intensiv rein, war aber verwundert, dass da offensichtlich schon eine bestimmte Linie vorgegeben war, wie darüber zu berichten sei. Von den Spekulationen und Falschmeldungen der ersten Stunden hörte man gar nichts mehr, alles war schon klar: Es war Al-Kaida und die sitzen in Afghanistan und die Airforce bombardiert schon und die Nordallianz schießt sich nach Kabul durch. Aufgefallen ist mir auch, dass die Leute daheim das Ereignis nicht ganz so emotional berührt hatte. Da waren die anglophonen Jungtouris in Portugal ganz anders drauf, denen ging das viel, viel näher. So lange Rede kurzer Sinn, ich erinnere mich ganz genau an den Tag. --El bes (Diskussion) 13:17, 11. Sep. 2014 (CEST)

Wie ich diese pauschale Herabwürdigung aller Medien verabscheue, wie sie derzeit in Mode ist... "offensichtlich schon eine bestimmte Linie vorgegeben war". Klar. Vermutlich vom Propagandaminister in Zusammenarbeit mit der Fed. Ist doch offensichtlich. Wer das nicht merkt, ist blöd oder CIA-gesteuert oder beides. --Eike (Diskussion) 13:28, 11. Sep. 2014 (CEST)
... oder ist einfach ein späterer Jahrgang und war damals noch zu jung. Jedenfalls waren die ersten Stunden der Berichterstattung höchst spannend, eben weil es noch keine offizielle Interpretation gab und ich bin sehr froh, dass ich das live mitverfolgt habe. Damals war CNN übrigens noch besser, nicht so ein Trash wie heute. --El bes (Diskussion) 13:37, 11. Sep. 2014 (CEST)

Ich weiß es auch noch genau. Ich war in den Dolomiten wandern. Als ich in der FeWo den Fernseher anschaltete, um ein geeignetes Programm zu finden, wunderte ich mich, dass auf allen Kanälen der vermeintlich gleiche Film gezeigt wurde. Mein erster Gedanke war, dass der Empfang vielleicht durch die Berge und Wetterlage schlecht sein könnte, dann realisierte ich, dass es sich um eine brisante Nachricht handelt. Erinnerungsprägend ist das Verwundern über den gleichen "Film" auf allen Kanälen, was ja der Alltagserfahrung nicht entspricht .--Belladonna Elixierschmiede 13:29, 11. Sep. 2014 (CEST)

Nach dem ersten Flugzeugabsturz war überhaupt nicht klar, was da vor sich ging. Immerhin gibt es zwei große, internationale Flughäfen in unmittelbarer Nähe, und es hätte sich auch um einen besonders merkwürdigen, tragischen Unfall handeln können. Nach dem zweiten Absturz war die Wahrscheinlichkeit für einen Zufall zwar erheblich gesunken, aber die noch immer unklare Situation hinterließ ein diffuses Gefühl der Bedrohung oder zumindest den Hinweis auf ein ungelöstes Rätsel, denn Selbstmordattentäter dieser Kategorie waren bis dahin unbekannt. Dazu kam dann noch das "gebrochene Versprechen" Hollywoods, das uns in zahlreichen Katastrophenfimen (Flammendes Inferno) gezeigt hatte, dass brennende Hochhäuser stehen bleiben, während diese schon nach kurzer Zeit komplett in sich zusammenfielen, anscheinend ohne Überreste. - Dies und die Berichterstattung mit zahllosen Sondersendungen über den ganzen Tag verteilt haben sich sicherlich stärker eingebrannt, als ein singuläres abgeschlossenes Ereignis. --Optimum (Diskussion) 13:41, 11. Sep. 2014 (CEST)
Ganz genau. In Europa hat es wegen der Challanger-Katastrophe nie so eine intensive Berichterstattung gegeben. Das wurde eben in den Abendnachrichten gesagt und fertig, aber da war nie ein stundenlanger Live-Feed (eigentlich tagelanger). Die einzige Ereignisse die medial ähnlich groß waren, war Tschernobyl 1986 und der Fall des Eisernen Vorhangs/der Berliner Mauer 1989. Fast jeder der Zeitgenossen kann sich daran erinnern, wo und wann genau er das erste Mal von der Tschernobyl-Katastrophe erfahren hat. --El bes (Diskussion) 13:47, 11. Sep. 2014 (CEST)
Unter uns Primaten: Weil es ein Überlebensvorteil ist, lebensbedrohliche Ereignisse gut erinnern zu können: Wenn man irgendwo gebissen oder angegriffen wurde, wird man "einen Bogen um den Ort machen", wenn eine Frucht ungenießbar ist, wird man sie zukünftig "links liegen lassen" usw. --Pp.paul.4 (Diskussion) 14:29, 11. Sep. 2014 (CEST)

Mir scheint, es wird klar, was der entscheidende Punkt ist: am stärksten erinnert man sich an Ereignisse, die starke Emotionen hervorgerufen haben. Leider sind es genau diese starken Emotionen, die die Erinnerungen auch verfälschen können. So kann es kommen, dass Leute Stein und Bein schwören, sie hätten mit eigenen Augen gesehen, wie Menschen bei der Selektion auf der Rampe in Auschwitz innerhalb von Minuten graue Haare bekommen haben. So eine Erinnerung ist nun keineswegs ein Beweis dafür, alle Geschichten über die Rampe in Auschwitz seien frei erfunden, wie man das ja manchmal irgendwo lesen muss. Eher für die emotionale Wucht des Erlebnisses. Seltsamerweise finde ich von dieser überragenden Rolle von Emotionen für die Erinnerung nicht viel beim Durchblättern unserer "Erinnerungsartikel". Selbst Geezer hat in seiner ersten Antwort (emotional) ja nur in Klammern gesetzt... Geoz (Diskussion) 14:29, 11. Sep. 2014 (CEST)

Na, "besonders" (= ungewöhnlich" + emotional gehört irgendwie zusammen). Wenn Jopi Heesters mal sterben wird, so ist das erwartungsgemäss. Wenn irgendwo ein Flugzeug abstürzt, ist das auch zu erwarten - wenn das aber in 40 km Entfernung geschieht und ein Freund sitzt drin, ist das besonders. 9/11 war besonders, weil es nicht zu erwarten war (selbst CIA und FBI und wer sonst noch haben es nicht erwartet). Deshalb war das Beispiel mit dem Fussballspiel (50:50 Chance) auch nicht so gut. GEEZER… nil nisi bene 15:55, 11. Sep. 2014 (CEST)
Tut mir leid, dass ich Dir das jetzt sagen muss, aber Jopi weilt schon lange nicht mehr unter uns. --Optimum (Diskussion) 16:00, 11. Sep. 2014 (CEST)
Das hatte ich in eiskalter Planung absichtlich geschrieben, damit ich jetzt schreiben kann: Das hatte ich erwartet. Und ohne nachzusehen, erinnert sich keiner mehr, wann das war. Bei John war das wiederum anders... GEEZER… nil nisi bene 17:59, 11. Sep. 2014 (CEST)
Elvis lebt - und Jopi nicht weniger! Weiß doch nicht nur der Tagesspiegel: „Frack, Zylinder, Stock und Einstecktuch, das war seine Arbeitskleidung.“ Und noch heute kennt man keine Bühne, auf der nicht spät in der Nacht, wenn die Vorstellung schon lange beendet und jegliches Licht gelöscht ist, ein Zylinder, ein weißer Seidenschal und ein Stock elegant durch den Bühnenraum schweben, begleitet von einem unbestimmten Gesang; „Da geh’ ich zu Maxim...“ --212.184.134.8 17:27, 11. Sep. 2014 (CEST)
Ereignisse, bei denen ich mich deutlich daran erinnern kann, wie und unter welchen Umständen ich sie erfahren habe, sind der Mauerfall, 9/11 und die Nachricht vom Tod einer ganzen Reihe Leute: ein Jugendfreund, meine Eltern, Kennedy, A. S. Neill, Arno Schmidt, Luke Kelly und (ja, ist peinlich, aber trotzdem) Lady Di. An die Mondlandung erinnere ich mich natürlich auch noch, aber der fehlte das Schockhaft-Plötzliche, man schaltete vorher dafür den Fernseher ein, weil man es ja schon wusste, und dann zog sich das endlos hin. Ausschlaggebend für die „eingebrannten“ Erinnerungen dürfte die eigene emotionale Betroffenheit in Kombination mit dem Überraschungseffekt sein. --Jossi (Diskussion) 14:32, 11. Sep. 2014 (CEST)
@Jossi: Musst dich nicht schämen wg. Diana, ich weiß es auch. Ebenso, ach, Michael Jackson.--Antemister (Diskussion) 23:04, 11. Sep. 2014 (CEST)
+1 Meine erste Reaktion bei den ersten Radiomeldungen war: "Oh-oh! Flugzeugabsturz in New York! Das hört sich aber gar nicht gut an...", war also nicht stärker, als beim letzten Ebola-Ausbruch, oder so. Meine Erinnerungen sind daher auch nicht stärker. Für Leute, die emotional mehr betroffen waren (wie ElBes Urlaubsbekanntschaften) war danach dann womöglich wirklich nichts mehr so, wie vorher. Geoz (Diskussion) 14:54, 11. Sep. 2014 (CEST)
Tun sie gar nicht. Nur den Anschlag. Weil er außergewöhnlich war, verstehst Du? Erinnerung, Langzeitgedächtnis. --217.84.102.92 14:57, 11. Sep. 2014 (CEST)
Ich war damals Student im 15.Semester und hatte daher reichlich Zeit: Ich war mit zwei WG-Mitbewohnerinnen auf dem Weg zu H&M zwecks Shoppen (!). Ich saß hinten im Auto, als mich ein Kumpel auf dem Handy anrief und mir sagte, die Palästinenser (!) drehten durch und hätten ein Flugzeug ins WTC gesteuert. Ich war bestürzt und teilte den Damen auf den Vordersitzen die Nachricht mir, die mir sagten, für sowas hätten sie jetzt keinen Nerv. Was ich nicht mehr weiß, ist, warum wir zum ca. 700m entfernt liegenden, innerstädtischen H&M mit dem Auto gefahren sind, statt zu latschen. --Krächz (Diskussion) 16:01, 11. Sep. 2014 (CEST)
Damit ihr die Einkaufsbeute nicht manuell nach Hause schleppen musstet. --192.91.60.11 16:14, 11. Sep. 2014 (CEST)
Das mit den Palänstinensern rührt daher, dass es zunächst Meldungen gab, die PFLP habe sich zu dem Anschlag bekannt. Das stellte sich später als Ente heraus; dass aber in der oben beschriebenen unklaren Situation auch diese Meldung über die TV-Bildschirme ratterte, daran erinnere ich mich auch noch. --slg (Diskussion) 16:48, 11. Sep. 2014 (CEST)
Es war die DFLP. Selten hat eine Gruppe so schnell einen Anschlag dementiert. Um 14:57 kam die erste Meldung vom World Trade Center, um 15:37 die Reutersmeldung mit dem angeblichen Bekenntnis der DFLP, und schon um 16:02 die Meldung mit dem Dementi. (Ich war damals taz-Meinungsredakteur, also Kommentarverantwortlicher. Um 15 Uhr war meine eigene Kommentarseite weiter hinten in der Zeitung soeben "zu", als die Unruhe im Haus begann. Wir hatten vier Stunden Zeit bis zum Redaktionsschluss für den Hauptkommentar vorne ... der deswegen Oussama bin Laden nur streifte, das angeblich brennende US-Außenministerium erwähnte und vor allem den Unterschied zu Pearl Harbour erläuterte. Hauptthese unserer US-Korrespondentin Andrea Böhm war aber, dass die USA das Gefühl ihrer Unangreifbarkeit verloren hatte, und das war richtig...) --Aalfons (Diskussion) 17:09, 11. Sep. 2014 (CEST)
Wow, vielen Dank für deine Erinnerungen. Das von dir beschriebene Zeitfenster passt aber sehr gut zu meinen Erinnerungen, da ich die genanne Meldung etwa zwischen 15:30 und 16:00 angesetzt hätte, danach (kurz vor dem Einsturz des ersten Turms, wie sich später herausstellte) haben wir den Fernseher erst mal ausgemacht, ich war 14 Jahre alt und musste zum Kieferorthopäden ..... ;) --slg (Diskussion) 17:16, 11. Sep. 2014 (CEST)
Dramatische und traumatische Ereignisse halten sich hartnäckig im Gehirn. Bei Versuchen mit Ratten (oder Mäusen?) wurden traumabedingte Verhaltensmuster weitervererbt. Im Zeitalter der Handschlagsgeschäfte brachten Bauern ihre Kinder mit, wenn es um den Verkauf von Ackerland ging. Nachdem man sich über den Verkauf durch schütteln der Hände geeinigt hatte, erhielten die Kinder eine Ohrfeige ihrer Väter, damit sie sich an der Ver-/Kauf (als Zeugen) erinnerten. Nicht Bestandteil der Forschung ist es, wie möglicherweise der Unfug vor 1945 sich in das Hirn der Deutschen gefressen hat und was die Bundesagentur für Arbeit draus macht. Man kann sich nur fragen, ob es vermieden wird festzustellen, ob der Unterschicht der Landsleute die Rattengene angeknipst werden sollen. Rattenfänger gibt es bereits.[32] --Hans Haase (有问题吗) 11:33, 12. Sep. 2014 (CEST)
Ich ignoriere jetzt mal den zweiten Teil deines Postings und frage lieber, ob du für den genannten ländlichen Brauch mit den Kindern als Zeugen für ein Handschlagsgeschäft irgendeine verlässliche Quelle hast - das würde mich interessieren. --Snevern 11:59, 12. Sep. 2014 (CEST)
Ich suche schon. --Hans Haase (有问题吗) 12:41, 12. Sep. 2014 (CEST)
Dasselbe Verfahren wurde beim Aufstellen von Grenzsteinen genutzt. Die Grenzzeugen erinnerten sich so gut an den Grenzstein. --Rôtkæppchen₆₈ 13:40, 12. Sep. 2014 (CEST)
So stehts bei Wikipedia, Ohrfeige#Ohrfeigen als Gedächtnisstärkung, --Bremond (Diskussion) 14:33, 12. Sep. 2014 (CEST)
(BK mehrfach) und hier: Die Entstehung des Blutritts; und hier: Früher gab's für jeden Grenzstein Ohrfeigen, ud hier: Heute werden auch in Pfeffingen keine Grenzsteine mehr versetzt. Reputabel klingt(!): Otto von Corvin, Die Geißler, S. 291 laut GoogleBooks (Nachdruck, auch bei gutenberg.de); allerdings vermischt der Mann häufig Legenden und Wahrheiten, gewöhnlich in persuasiver Absicht, --Bremond (Diskussion) 14:47, 12. Sep. 2014 (CEST)
Danke für die Links! --Snevern 18:18, 12. Sep. 2014 (CEST)
Siehe auch: en:Beating_the_bounds Whipping the boys by way of remembrance, and stopping their cry with some half-pence --Concord (Diskussion) 04:01, 13. Sep. 2014 (CEST)
Noch ein interessanter Link, danke. Einerseits ein Hinweis darauf, dass es den Brauch auch außerhalb des deutschen Sprachraums und Rechtskreises gibt, andererseits auf die von mir schon lange vermutete medizinische Wirksamkeit von Geld. --Snevern 08:51, 13. Sep. 2014 (CEST)

Keine Frauen im Mithraskult?

Welche Quellen belegen die Aussage, daß Frauen der Zugang zum Mithraskult verwehrt war? WHOPP

Hier oder hier ... und sicherlich noch mehr. GEEZER… nil nisi bene 17:51, 11. Sep. 2014 (CEST)
Siehe en:Mithraic mysteries#cite_note-geden_1-5. --Rôtkæppchen₆₈ 17:54, 11. Sep. 2014 (CEST)
In der Fernsehserie Rome veranstaltet die weibliche Hauptfigur Atia (Mutter des Augustus) einmal eine Mithraszeremonie mit Schlachtung eines Stiers, inkl. Baden in dessen Blut. Wie historisch korrekt dies ist, kann ich nicht sagen, aber ganz aus der Luft gegriffen wird es auch nicht sein. --El bes (Diskussion) 21:51, 11. Sep. 2014 (CEST)
Benutzer:El bes dort nicht Mithras, sondern Kybele, vgl. auch Taurobolium. --HHill (Diskussion) 14:11, 12. Sep. 2014 (CEST)
Laut Mithraismus#Stieropfer: „Die in der älteren Forschung weit verbreitete Ansicht, im Mithraskult sei ein Stier geopfert (oder das Taurobolium vollzogen) worden, ist durch die Archäologie widerlegt worden. Die Knochenfunde, die bisher analysiert wurden, enthalten keine Stierknochen.“ --Rôtkæppchen₆₈ 18:16, 12. Sep. 2014 (CEST)

Sichtbares Gas bei Kaltgetränken

Entschuldigung, dass ich euch damit belästige, aber ich habe durch Google und Wikipedia nichts gefunden: Wenn man eine sehr kalte Flasche beispielsweise mit Cola öffnet, entweicht oftmals ein sichtbares Gas. Woran liegt das? Ich habe nachgelesen, warum beispielsweise Brillengläser bei Kälte beschlagen, aber es scheint nicht dasselbe Phänomen der Kondensierung zu sein? Wird die Kohlensäure bei Kälte sichtbar, wenn sie entweicht, handelt es sich um die Kondensierung der Flüssigkeit in der Flasche, oder ist es ein anderes Phänomen? --Constructor 19:36, 11. Sep. 2014 (CEST)

nö, in der Flasche befindet sich im Wasser gelöste Kohlensäure. Durch das Öffnen der Flasche entweicht der Druck. Das Gleichgewicht der Reaktion CO2+H2O <-->C2CO3 ist dann gestört. Somit fängt die Kohlensäure an sich vermehrt in Kohlendioxid und Dihydrogeniummonoxid zu zersetzen, das CO2 perlt aus und das Dihydrogeniummonoxid bleibt im Getränk. Bei der Brille ist es Kondensation. - andy_king50 (Diskussion) 19:44, 11. Sep. 2014 (CEST)
Wenn Du eine Getränkeflasche sehr schnell öffnest, entweicht ebenfalls Kohlenstoffdioxid. Duch die schnelle isentrope Expansion kühlt das Kohlenstoffdioxid stark ab. Die umgebende Luft wird mit abgekühlt, sodass die Luftfeuchtigkeit für einen Moment kondensiert. Das sieht man als grauen Nebel in der Nähe der Flaschenmündung. --Rôtkæppchen₆₈ 20:44, 11. Sep. 2014 (CEST)
Sehr gut, vielen Dank für die Erklärungen! --Constructor 23:43, 11. Sep. 2014 (CEST)
dann ist das, was man da sieht also kein Gas, sondern ne Flüssigkeit...? also ne Art Nebel? oder Aerosol? --Heimschützenzentrum (?) 14:49, 12. Sep. 2014 (CEST)
Das, was man sieht, ist strenggenommen weder die Flüssigkeit, noch das Gas, sondern die Lichtbrechung und Reflexion an den Phasenübergängen zwischen Flüssigkeit und Gas. Tauche einfach mal ins Schwimmbecken ein, egal ob mit oder ohne Taucherbrille: Siehst Du das Wasser? Nö. sobald aber Grenzschichten zwischen Wasser und Luft vorhanden sind, siehst Du da etwas, nämlich die Lichtbrechung und Reflexion an der jeweiligen Grenzschicht. Das trifft auf Nebel, den Wasserstrahl der Dusche oder die Wasseroberfläche im Gartenteich gleichermaßen zu. Reines Wasser sieht man nur in dicken Schichten vor hellem Hintergrund, z.B. in Luft- oder Unterwasserbildern von Freibädern oder Tropenlagunen. --Rôtkæppchen₆₈ 23:53, 12. Sep. 2014 (CEST)
hmm... man sieht doch ohnehin immer nur Reflexion und nich das Ding selbst... Nebel erscheint nunmal grau und Chlor grün (jedenfalls bei Radioactive Man) und n Feuerwehrauto rot... :-) --Heimschützenzentrum (?) 01:15, 13. Sep. 2014 (CEST)

Schnittfläche dreier Kreise

Wie groß ist die Schnittfläche mehrerer, z.B. dreier Kreise? --46.115.173.233 06:13, 12. Sep. 2014 (CEST)

Das errechnet man anhand der bekannnten Formeln für Dreiecke und Kreissegmente. Es gibt dafür keine allgemeingültige Lösung. Im vorliegenden Fall liegen die Mittelpunkte der kreise nicht auf den Kreislinien der jeweils anderen Kreise. --Rôtkæppchen₆₈ 06:43, 12. Sep. 2014 (CEST)
Bilden die Mittelpunkte der drei gleichen Kreise ein gleichseitiges Dreieck und sind Radius und Abstand der Mittelpunkte zueinander bekannt, dann verläuft die Höhe des Dreiecks durch einen Eckpunkt der gesuchten Schnittfläche (aller drei Kreisflächen). Diese Höhe besteht aus - je nachdem, ob die Seiten des Dreiecks in oder außerhalb der gesuchten Schnittfläche verlaufen - mehreren Abschnitten a) vom gewählten Kreismittelpunkt zum (nächstgelegenen) Eckpunkt der gesuchten Schnittfläche, gesucht b) von dort weiter mittig durch diese Schnittfläche bis zum (einem der drei die Schnittfläche mitbildenden) gegenüberliegenden Segmentbogen, gesucht, mit Unterabschnitten, b1) Eckpunkt der Schnittfläche bis zur gegenüberliegenden Sekante, gesucht b2) Höhe des (durchschnittenen) Segments, gesucht !, c) Radius minus Höhe des Dreiecks der Kreismittelpunkte (Vorzeichen umkehren je nach größer/kleiner), trigonometrisch gegeben.
Durch den Schnittpunkt letzterer Höhe-des-Dreiecks-durch-die-Kreismittelpunkte mit dem (nächstgelegenen) Eckpunkt-der-gesuchten-Schnittfläche steht eine Strecke-senkrecht-auf-einer-Seite-des-Dreiecks-der-Kreismittelpunkte, deren Länge abhängig ist vom Abstand zweier Kreismittelpunkte zueinander. Diese Strecke bildet mit ihren Endpunkten zum nächstgelegenen Kreismittelpunkt hin verbunden ein kleines rechtwinkliges Dreieck: a) Kathete: Abstand der Kreismittelpunkte minus Radius, gegeben b) Kathete: die Strecke selbst, gesucht, c) Hypotenuse: Strecke Kreismittelpunkt bis Eckpunkt der Schnittfläche. Die Strecke durch den Eckpunkt hat als Gegenkathete b zum Winkel beim Kreismittelpunkt also den Wert   tan(30°) x ( Abstand der Kreismittelpunkte - Radius).
Da durch den Eckpunkt der Schnittfläche eine Sekante parallel zur Seite des Dreiecks aus den Kreismittelpunkten läuft, findet diese Strecke auch den Schnittpunkt der Höhe des Kreismittelpunkte-Dreiecks mit der Sekante der Schnittfläche bzw die Höhe des Segments:   Höhe_Segment = verschobene_Strecke - ( Höhe_Dreieck_Kreismittelpunkte - Radius ).   (Liegt die Seite des Dreiecks aus den Kreismittelpunkten auf der Tangente vom (senkrecht zur Höhe vom) Segment, ist die Strecke gleich der Segment-Höhe)
Drei Segmente bilden mit dem einbeschriebenen Dreieck (aus den Eckpunkten der Schnittfläche) die Schnittfläche. --217.84.70.97 15:46, 13. Sep. 2014 (CEST)

(Edit: Bei Symmetrie, wenn die Mittelpunkte genau auf den Kanten liegen)--46.115.173.233 07:13, 12. Sep. 2014 (CEST)

Was meinst du mit "Kanten"? Willst du es analytisch oder numerisch gelöst haben? --81.92.99.84 08:42, 12. Sep. 2014 (CEST)
Ein nettes Paper findest du unter http://www.dtic.mil/dtic/tr/fulltext/u2/a463920.pdf --81.92.99.84 09:27, 12. Sep. 2014 (CEST)
Für den Fall, dass die Mittelpunkte auf den Kreislinien der anderen Kreise liegen und alle Kreise gleich groß sind, bekommst Du aus den Formeln für das gleichseitige Dreieck und das Kreissegment mit Mittelpunktswinkel 60° folgendes Ergebnis: . --Rôtkæppchen₆₈ 13:59, 12. Sep. 2014 (CEST)

Eine lustige Software dazu gibt es bei http://www.benfrederickson.com/calculating-the-intersection-of-3-or-more-circles/ --Plastit (Diskussion) 14:21, 13. Sep. 2014 (CEST)

DVD/CD-Laufwerk wird nicht (mehr) erkannt

Mein Notebook (Windows 7) erkennt das LW nicht mehr. Was kann ich tun? Das Schweigen der Lemma (Diskussion) 09:04, 12. Sep. 2014 (CEST)

Schauen, ob es das BIOS noch erkennt (oder ob es da vielleicht abgeschaltet ist). Wenn ja, vielleicht ein Live-Linux booten und kucken, ob auch das es erkennt. Wenn ja => Windows-Problem. Wenn nein => eher Hardware-Problem. --Eike (Diskussion) 09:33, 12. Sep. 2014 (CEST)
Die Bauform der Laufwerke ist inzwischen einheitlich. Einige Laufwerke lassen sich ohne Werkzeug, manche nur mit einer Schraube entfernen. Versehentlich gelöst? Stecker im Fach verstaubt. Neu einstecken. Diese Laufwerke haben ihre Elektronik überwiegend in der Schublade unter dem Laser und nur wenig bis nichts im Gehäuse. Das Flexboard (Flachbandleitung) auf Beschädigungen prüfen! --Hans Haase (有问题吗) 10:42, 12. Sep. 2014 (CEST)
Besten Dank! @Eike: Was bedeutet "Live-Linux" und wo bekomme ich das her? Das Schweigen der Lemma (Diskussion) 10:44, 12. Sep. 2014 (CEST)
Siehe Live-System#Linux. Du bekommst sie z.B. von Knopper.net (Knoppix) oder ubuntu.com (Ubuntu). --Rôtkæppchen₆₈ 10:48, 12. Sep. 2014 (CEST)
Sehr gut das hier auf alle Windows Frage eine Linux Lösung vorgeschlagen wird. Windows hat nämlich keinerlei Bordmittel zur Fehleranalyse. Es gibt nur einen Startknopf auf den man klicken kann und für alles andere muss man Linux verwenden. --92.227.206.106 12:40, 12. Sep. 2014 (CEST)
Dass das bei allen Fragen so wäre, ist Unfug, und warum es hier vorgeschlagen wurde, wurde ja begründet: Weil man so gut zwischen Soft- und Hardwarefehler unterscheiden können sollte - indem man eine möglichst komplett andere Software verwendet. (Wie es dann weitergeht, ob mit Hardware Anschauen oder mit Windowsmitteln, entscheidet sich dann daran.)
Aber vielleicht möchtest du ja auch was Konstruktives beitragen?
--Eike (Diskussion) 12:47, 12. Sep. 2014 (CEST)
Der Erste Schritt bei Hardware Problemen ist die Problembehandlung Systemsteuerung\Alle Systemsteuerungselemente\Problembehandlung und nicht jemanden der nicht mal weiß was eine linux live cd ist zu empfehlen zu versuchen mit einem wildfremden System herauszufinden ob das DVD Laufwerk noch da ist. Der Witz bei dieser Empfehlung ist im Übrigen wenn er irgendeine bootfähige CD oder DVD einlegt und dieses bootet ist schon mal das CD/DVD Laufwerk nicht kaputt. Das kann er genau so gut mit der Windows DVD die er bereits hat machen. Da gibt es dann auch nochmal eine Systemreperatur die Ihm vorgeschlagen wird und Ihm eventuell weiterhilft. Mit dem DVD Laufwerk eine Linux DVD zu booten um dann im System nachzusehen ob das selbe Laufwerk mit dem man gebootet hat angezeigt wird, ist die sinnloseste Empfehlung ever. --92.227.212.29 13:37, 12. Sep. 2014 (CEST)
Fein, es geht also auch teil-konstruktiv. Dir sind ein paar Sachen entgangen, zum Beispiel, dass keiner vorher wusste, ob der Fragesteller Live-DVDs kennt und dass meine Empfehlung mit dem BIOS anfängt und dass heutzutage viele Rechner ohne echte Windows-DVD ausgeliefert werden... Ich bin mir aber tatsächlich nicht sicher, wieviel nach dem Booten von DVD da noch schiefgehen könnte. Es kann sein, dass der Tipp für DVD-Laufwerke weniger Sinn macht als für andere Hardware-Probleme. --Eike (Diskussion) 13:48, 12. Sep. 2014 (CEST)
Mit einer Linux DVD booten um im System festzustellen ob das DVD Laufwerk kaputt ist mit dem man gerade bootet hat? Ach hör auf das war ein guter Vorschlag. Ich fahre jetzt mit meinem Auto zu Fahradmechaniker um nachsehen zu lassen ob mir alle vier Reifen geklaut wurden. --92.227.212.29 14:13, 12. Sep. 2014 (CEST)
Von DVD hat ursprünglich niemand gesprochen. Das Live Linux automatisch optischer Datenträger bedeutet hast Du impliziert (auch wenn das die häufigste Darreichungsform ist). Mit einem anderen OS auf das Laufwerk zuzugreifen ist eine schnelle und zuverlässige Methode um zwischen Soft und Hardwarefehler einzugrenzen. Zu Zeiten als Windows noch eine GUI war konnte man das schnell mit A-Z: erledigen heute geht das komfortabel mit einem OS das von Haus aus gut unterstützt.--Simius narrans 14:25, 12. Sep. 2014 (CEST)
Ich hatte leider tatsächlich an eine Silberscheibe gedacht. Stimmt, man könnte einen USB-Stick nehmen. Wobei ich nie einen auf Lager hab, der einfach gelöscht werden kann, und das ist da ja glaub ich meist Voraussetzung, oder? --Eike (Diskussion) 14:28, 12. Sep. 2014 (CEST)
Die 'one click' Lösungen latschen einfach drüber und nehmen die entsprechenden Einstellungen vor, stimmt. Damn small Linux live langen >128 MB, irgendein oller wird da doch noch rumliegen? 08/15 Linuxe laufen ab 1 GB, sinnvoll ist für ein breites Einsatzspektrum aber gerade die "alles wird unterstützt" Knopix DVD. Speicherkarten funktionieren auch, wenn das Bios sie als Bootmedium unterstützt. Wer eine Pinguinphobie hat macht das mit nem Microsoft Windows PE, das ist aber nicht klickibunti zu konfigurieren (afair).--Simius narrans 15:17, 12. Sep. 2014 (CEST)
Nimm Bart PE, wenn Du Linux nicht magst. --Rôtkæppchen₆₈ 15:24, 12. Sep. 2014 (CEST)
Standardprozedur bei nicht erkannten Laufwerken: Schuhe und Strümpfe ausziehen zwecks Vermeidung elektrostatischer Aufladung. Stecker raus, Kiste aufmachen, alle Kabel zum Laufwerk 3x abziehen und wieder aufstecken, Steckverbindungen zum Netzteil oder Motherboard abziehen und neu aufstecken. Achten, dass alles richtig eingerastet ist und kein Kabel verpolt ist. (Flachbandkabel: farbig markierte Seite ist da wo Pin 1 ist, was immer irgendwie gekennzeichnet ist). Bei der Gelegenheit gleich alle Lüfter, Kühlkörper und Gitter von Staub befreien. Dann Probleauf bei offenem Gehäuse, anschließend zumachen. Ich hatte schon PCs, die nicht mehr booteten, der Händler meinte, da könnte man nichts mehr machen, entweder eine neue Platte oder einen neuen PC, dabei hat sich nur ein Kabel gelockert oder es gab irgendwo eine Oxidschicht. Ich schätze mal in 80 bis 90% der Fälle ist das die einfache Lösung. Insbesondere dann, wenn die Kiste in letzter Zeit mal transportiert wurde. DVD und CDs können u. U. ziemlich vibrieren, was den selben Effekt haben kann.--Giftzwerg 88 (Diskussion) 14:59, 12. Sep. 2014 (CEST)
Zweites Wort im OP: Notebook. "In letzter Zeit transportiert" soll da häufiger mal vorkommen ;o) --Simius narrans 15:17, 12. Sep. 2014 (CEST)
Lass Schuhe uns Strümpfe an, schließ den Rechner an eine abschaltbare Steckdosenleiste an, schalte diese ab und sichere sie gegen unbeabsichtigtes Einschalten. Sollte das PC-Netzteil einen Schalter haben, so schalte auch diesen aus, denn doppelt hält besser. Bevor Du ein Teil des Rechners berührst, immer erst das PC-Gehäuse anfassen, um Potentialausgleich herzustellen. --Rôtkæppchen₆₈ 15:21, 12. Sep. 2014 (CEST)
Der Vorteil an der Barfußmethode ist, dass sich auch während des Rumschraubens nichts mehr aufladen kann, egal was für Schuhe und Bodenbelag, außerdem benötigt diese Methode kein weitergehendes technisches Verständnis oder irgendwelche technischen Voraussetzungen. Wir können nicht wissen welche Voraussetzungen der Leser hat (DAU). Bei allen Schaltern gibts immer wieder Billigprodukte oder schlaue Bastler, die nur eine Zuleitung abklemmen, so dass das Gerät zwar aus ist, aber theoretisch irgendwo am Gerät immer noch volle Spannung anliegen kann. Daher empfehle ich anderen Personen die 100%-Sicher-Lösung durch Ausstecken des Geräts. Klaro, wer häufiger rumbastelt, öffnet auch mal im Betrieb den Deckel, aber derjenige weiss dann hoffentlich was er tut und ob und wo er was anfasst und das mit dem Potentialausgleich funktioniert womöglich bei Kunststoffgehäusen nicht.--Giftzwerg 88 (Diskussion) 19:04, 12. Sep. 2014 (CEST)
Profis sind beim Schrauben mit dem Handgelenk über ein Kabel geerdet damit Sie nicht die Statistik einholt. Für nicht Profis reichts problemlos aus mal kurz Metall anzufassen bevor man in den Rechner greift. --92.227.212.29 23:49, 12. Sep. 2014 (CEST)

Roter Milan

Ich möchte wissen ob ein Milan evt. auch ein kleiner Hund von 3 1/2 Kilo als Beute ansehen könnte, und greifen würde. Vielen Dank Anja

--95.33.129.18 09:05, 12. Sep. 2014 (CEST)

Ein Blick in Rotmilan#Nahrungsspektrum lässt ahnen, dass Tiere mit 3½ kg außerhalb des Milan-Menüs liegen. Wenn ich nicht irre, ist das größte und schwerste Landsäugetier, das in dem Abschnitt erwähnt wird, der Feldhamster, der nichtmal ein einziges Kilo hinbekommt. —[ˈjøːˌmaˑ] 09:22, 12. Sep. 2014 (CEST)
...und selbst das ist schon einiges bei einem Eigengewicht des Vogels von knapp über oder unter einem Kilogramm. --Snevern 09:45, 12. Sep. 2014 (CEST)
Wenn jetzt die Wildtiere immer mehr in die Städte drängen, muss man auch mit solchen Ereignissen rechnen: Riesiger Steinadler packt Kleinkind.--Optimum (Diskussion) 11:07, 12. Sep. 2014 (CEST)
Du hast das zu Ende gelesen, ja? Ich darf mal zitieren:
"Nachtrag: Um 20.23 Uhr am selben Tag stellt sich heraus, dass es sich bei dem Video um einen Trickfilm handelt. Das staatliche Zentrum für Animation und Design (NAD) in Montréal, das Ausbildungsgänge für derartige Trickfilm-Techniken anbietet, teilte am Nachmittag (Ortszeit) mit, das Video mit dem Adler sei von vier NAD-Studenten für eine Prüfung in dreidimensionaler Animationstechnik erstellt worden. Lesen Sie hier nach, wie es zu der Falschmeldung kam."
Abgesehen davon bezweifle ich, dass "die Wildtiere" immer mehr in die Städte drängen. --Snevern 11:32, 12. Sep. 2014 (CEST)
Am Ende des Artikels wird offenbart, dass es sich um das Werk von Animationstechnikern handelt. Eigentlich eine Frechheit, solch einen Hinweis ans Ende statt an den Anfang zu setzen. --::Slomox:: >< 11:34, 12. Sep. 2014 (CEST)
Ach herjee, anscheinend handelt es sich hier um Animationstechnik. Aber dieses Video ist wirklich wahr: Adler packt Fußball.--Optimum (Diskussion) 12:16, 12. Sep. 2014 (CEST)
Der wiegt aber auch nicht mehr als ein kräftiger Hamster... --Eike (Diskussion) 12:22, 12. Sep. 2014 (CEST)
kräftiglol. --217.84.69.59 15:42, 12. Sep. 2014 (CEST)
Adler packt Fußball, oder etwas ganz ähnliches, ist jedenfalls auch echt. --Aalfons (Diskussion) 15:41, 12. Sep. 2014 (CEST)

Der Nabu schreibt: Aktiv jagen kann er nur Tiere bis zu einem Gewicht von etwa 500g. --Optimum (Diskussion) 13:00, 12. Sep. 2014 (CEST)

So Kleinvieh wie Hamster und Mäuse passen auch einer Krähe in den Schnabel. Ich sah mal eine, der hing in Flug das Hinterteil einer weißen Maus oder eines Hamsters noch heraus. Da hats wohl einer gut gemeint und das Tierchen in einer oben offenen Box an die frische Luft gestellt.--Giftzwerg 88 (Diskussion) 18:39, 12. Sep. 2014 (CEST)

Haushaltswaren: Angebotsvielfalt innerhalb einer Marke

Liebe Auskunft, große Hersteller von Haushaltswaren bieten für ein Produkt mehrere Artikel mit verschiedenen Spezial- und Zusatzeigenschaften an. Zum Beispiel gibt es Zahnpasta der Marke Colgate für glänzende Zähne (dafür gleich mit mehreren Artikeln: Colgate MaxWhite und Sensation White), für einen frischen Atem und für eine besondere Pflege des Zahnfleischs. Spülmaschinentabs der Marke Somat gibt es einerseits mit verschieden vielen Funktionen, aber auch mit zehn Funktionen als Somat 10 und mit zehn teilweise anderen Funktionen als Somat Gold. Kann man davon ausgehen, dass sich diese Artikel tatsächlich in ihren Eigenschaften unterscheiden oder handelt es sich um eine Taktik zur Erhöhung der Verkaufszahlen? --BlackEyedLion (Diskussion) 14:31, 12. Sep. 2014 (CEST)

Die unterscheiden sich schon, aber ob das im Ergebnis einen Unterschied macht, kann man nicht pauschal beantworten. Marketing ist sicherlich ein großer Faktor, aber die Konsumenten fordern auch exakt auf ihre Wünsche zugeschnittene Produkte und diese Forderung wird damit bedient. --178.4.178.242 15:06, 12. Sep. 2014 (CEST)
Produkttreue kommt vor Markentreue. Wenn es ein gewohntes Produkt gibt, wird der Verbraucher das eher kaufen als etwas neues auszuprobieren. Also kann man bestehende Produkte nicht einfach ändern, das endet normalerweise im Fiasko. Da man andererseits aber auch den Bestandskunden Innovationsfähigkeit demonstrieren muss, müssen zumindest ab und zu neue Produkte auf den Markt geworfen werden, die dann natürlich auch ihre Liebhaber finden. So entstehen die Produktzoos. -- Janka (Diskussion) 15:59, 12. Sep. 2014 (CEST)
Andererseits kann die Produkttreue auch ins Lächerliche gehen: Meister Proper als Waschpulver, Tempo als Klopapier oder Mars als Eis am Stiel braucht doch kein Mensch. Das wäre ja so, als wenn man sein Geschirr mit Sofix spült oder seine Nase mit Hakle putzt. --Rôtkæppchen₆₈ 16:05, 12. Sep. 2014 (CEST)
Für die einen ist es nur Klopapier, für die anderen das längste Taschentuch der Welt.--Optimum (Diskussion) 16:49, 12. Sep. 2014 (CEST)
Grundsätzlich ist das mindestens eine "Taktik zur Erhöhung der Verkaufszahlen". Sonst würden die das ja unterlassen, falls sie sich nichts davon versprechen. Diese Taktik nennt sich übrigens Produktdifferenzierung (Siehe auch: Produktpolitik). Es ist ja auch naheliegend einen bekannten Markennamen für ein neues Prdukt zu verwenden. Ob es auch wirklich unterschiede zwischen den Proudkten gibt kann ich nicht beurteilen. Probier es einfach aus und schau ob du einen Unterschied merkst. --Der-Wir-Ing (Diskussion) 17:09, 12. Sep. 2014 (CEST)
Gundsätzlich soll der Kunde mit der Marke zufrieden sein. Darum gilt es Varianten für die gebräuchlichsten Bedarfe verfügbar zu halten. Unterschiede gibt es da schon. Bei Limonade im Bezug auf Zucker oder Ersatzstoffen wie Süßstoff und Stevia, bei Reiniger für Bad und Küche, die sich in ihrer Eigenschaft der Löslichkeit von Fett und Kalk unterscheiden. Bei Zahnpasta der Anteil von Bleichmitteln, Scheuermitteln und Fluoride. --Hans Haase (有问题吗) 18:29, 12. Sep. 2014 (CEST)

Aurecht - was ist das?

ich hab gerade in einem Gespräch erfahren, dass zu einem Grundstück (Bayern) auch ein sogennntes Aurecht dazugehört. Dies berechtigt/verpflichtet in regelmässigen Abständen (in Gemeinschaft mit anderen) ein Stück Auwald zu pflegen bzw. abzuholzen und das Holz selber zu nutzen. Weitere Details konnte ich nicht in Erfahrung bringen. Wer weiss mehr darüber? Bapperl auch gleich gesetzt ;-)

--91.39.103.126 16:29, 12. Sep. 2014 (CEST)

So wie Du das beschreibst, könnte es etwas ähnliches wie eine Holzgerechtsame sein. In Meyers Großes Konversations-Lexikon von 1905 wie jecoch etwas anderes beschrieben: [33]. 217.230.70.42 16:44, 12. Sep. 2014 (CEST)
Bei Adelung wird auf Auenrecht und Angerrecht. verwiesen. Hier ergibt sich, daß das Auenrecht zumindest in Schlesien auch das Nutzungsrecht des Gutsbesitzers war, "über die nicht zu den wegen nötigen freien plätze innerhalb der dorflage zu verfügen". Hieraus ergibt sich, daß es in Bayern ein sehr altes Recht ist (siehe nachfolgende Hinweise zu den Suchproblemen, wenn Google aus einem n ein u liest). Allerdings ist hier in der bayrischen Rechtsdatenbank für Aurecht, Auenrecht und Angerrecht kein Treffer zu finden. Ich assoziiere Wasserrecht, ein anderes an das Grundstück gebundenes Nutzungsrecht. --212.184.138.49 17:23, 12. Sep. 2014 (CEST) Nachtrag: Ich vermute einen anderen im Recht verwendeten Begriff. Der könnte sich irgendwo im Umwelt- oder Forstrecht verbergen. Wenn sich der Suchbegriff klärt, wäre diese Seite vielleicht auch hilfreich. --212.184.138.49 17:35, 12. Sep. 2014 (CEST) Streichung --212.184.141.109 09:38, 13. Sep. 2014 (CEST)
Der Link im 3. Satz ("Hieraus") liefert falsche Suchergebnisse (B>aurecht<, >Anrecht<). 217.230.70.42 17:39, 12. Sep. 2014 (CEST)
Hm. Ich habe bei GBS mit: "Aurecht" Bayerisches Recht gesucht. Aber Google versuchte mir vorher auch sehr hartnäckig ein zusätzliches B zu verkaufen. (Ein Grundproblem von Programmen, die angebliche "helfen", faktisch aber Normalitätsalgoritmen festschreiben - irgendwann wird mir der Navi verbieten, in einen Stau zu fahren und mein Kühlschrank morgens die Milch für den Kaffee verweigern, weil ich mir noch nicht die Zähne geputzt habe...). Das Suchergebnis bei GBS gibt einige Uraltbücher aus, in denen Aurecht in dem gesuchten Sinn vorkommt. --212.184.138.49 17:58, 12. Sep. 2014 (CEST)
In den Frakturtexten lese ich meistens "Anrecht". u und n sind in Fraktur sehr ähnlich und werden hier offensichtlich nicht korrekt erkannt. Und ja: Computer können auch nicht alles. Wenn sie trotzdem solche Vorschläge machen, ist das mindestens lästig. Man sollte auch nicht alles ungeprüft glauben und z.B. dem Navi mitten in den Wald folgen, obwohl man offensichtlich nicht mehr auf befestigten Straßen fährt. 217.230.70.42 18:34, 12. Sep. 2014 (CEST)
Ein schönes Beispiel, um wieder einmal zu erfahren, was für eine Katastrophe die Google-Digitalisierung ist. Man wird mit Hunderten von Lesefehlern zu „Anrecht“, „zu Recht“ oder „aufrecht“ überschüttet. Der einzige Nachweis, den ich finden konnte, dass so etwas existiert(e), ist dieser Schnipsel. Um so merkwürdiger, dass der Begriff so total unter dem Suchradar bleibt. --Jossi (Diskussion) 20:01, 12. Sep. 2014 (CEST)
Ich habe den Begriff immer in Anführungszeichen gesetzt. (Aber nicht geprüft, ob das was hilft.) Nach meinem Eindruck finden sich einige Treffer zum historischen Recht, aber aktuell könnte das Recht wohl unter einem anderen Begriff gefaßt sein. Diesen Begriff gilt es zu finden. Am Montag kann man z.B. bei einem Grundbuchamt seine Vertrauens anrufen... (vorzugsweise in Bayern) --212.184.138.49 23:18, 12. Sep. 2014 (CEST)
Ich vergaß zu erwähnen, dass ich nach "Aurecht Bayern" gesucht habe. Über das Aurecht (Auenrecht, Angerrecht) in Schlesien (Recht der Grundherrschaft an den offenen Flächen im Dorf) findet sich einiges, aber das dürfte wohl kaum das sein, was der Fragesteller meinte. Wahrscheinlich ist das örtliche Grundbuchamt wirklich die beste Anlaufstelle. --Jossi (Diskussion) 01:11, 13. Sep. 2014 (CEST)
Sicher, dass es nicht um "Haurecht" (Holzschlagrecht) geht? Grüße Dumbox (Diskussion) 09:46, 13. Sep. 2014 (CEST)

deutsches Abitur = muttersprachliches Niveau C2 gemäß Gemeinsamen Europäischem Referenzrahmen?

Entspricht das deutsche Abitur mit Grundkurs Deutsch dem Niveau C2 gemäß dem GER? Für mein Rechtsverständnis schon. Habt Ihr entsprechende Rechtsquellen? Also irgendwelche Verordnungen? Für Englisch habe ich schon herausgefunden dass ein GK Englisch B2 und LK Englisch C1 im Abitur entspricht, sofern mind. 5 Punkte erreicht wurden. Aber wie sieht es mit Deutsch aus? Normalerweise spricht man doch von einem Muttersprachler, sofern er mind. 9 Jahre in der jeweiligen Sprache erfolgreich den Unterricht genossen hat. Freue mich auf eure Gedanken --DerHandelsreisende (Diskussion) 17:05, 12. Sep. 2014 (CEST)

Ist denn die Person, um die es geht, sowieso ein Muttersprachler? Ich habe Englisch regulär ab der 5. Klasse gehabt (ab der 2. haben wir bereits ein bisschen den Grundwortschatz gelernt) und von der 11-13. dann LK. Im TOEFL-Test nach dem Abi wurde ich als C2 eingestuft (wenn man die Punkte entsprechend umrechnet), allerdings nicht in allen Kategorien (mündlich nur C1). Und ich würde mich demnach nicht als Muttersprachler einschätzen, sondern als near-native. Muttersprachler wäre D1. XenonX3 – () 17:14, 12. Sep. 2014 (CEST)
D1 gibt es nicht. Wenn überhaupt C2+. Und es geht um Deutsch und nicht um Englisch. Also Ja, ich suche nach der Definition "Muttersprachler". --DerHandelsreisende (Diskussion) 17:21, 12. Sep. 2014 (CEST)
Muttersprachler entspräche aber D1, wenn es das gäbe (denn es ist das nächsthöhere Kompetenzniveau). Der Rahmen bezieht sich ja explizit auf Personen, die eine Fremdsprache lernen, unabhängig davon, welche. Die Niveaustufen sind auf alle Sprachen anwendbar. XenonX3 – () 17:36, 12. Sep. 2014 (CEST)
Wenns D1 gäbe, dann gäbe es auch D2 :D.--DerHandelsreisende (Diskussion) 20:20, 12. Sep. 2014 (CEST)
Ich versteh ehrlich gesagt die Frage nicht: Wenn derjenige Muttersprachler ist, was interessiert ihn dann dieser Referenzrahmen? Als Muttersprachler ist das das Niveau Muttersprachler, wie auch immer der Referenzrahmen das bezeichnet. --88.130.88.58 17:17, 12. Sep. 2014 (CEST)
Ab wann ist man Muttersprachler? Wenn man als Schlesier in Polen von seiner Mutter deutsch gelernt hat? Oder wenn man X-Jahre auch eine deutsche Schule erfolgreich besucht hat? --DerHandelsreisende (Diskussion) 17:21, 12. Sep. 2014 (CEST)
Wann, wo und (trotz der Bezeichnung Muttersprache) auch von wem man die Sprache gelernt hat, dürfte höchstens ein Indiz sein, ist für sich genommen aber egal. Laut Muttersprache ist jemand dann Muttersprachler, wenn er die jeweilige Sprache "weitgehend automatisiert beherrscht". Darüber, wann genau das der Fall ist, herrscht wahrscheinlich - wie bei praktisch allen geisteswissenschaftlichen Themen - ein heilloser Streit. --88.130.88.58 17:29, 12. Sep. 2014 (CEST)
Laut Muttersprache "Als Muttersprache bezeichnet man die in der frühen Kindheit ohne formalen Unterricht erlernte Sprache." Dem würde ich mich anschließen, also ja zum Schlesier, sofern man in Polen geboren und aufgewachsen ist und Polnisch und Deutsch von Anfang an gelernt hat (mehrere Muttersprachen sind möglich, laut Artikel bis zu 3). Nein zur Schule, da man die ja nicht in der frühen Kindheit begonnen hat. Mag sein, dass man eine Sprache auf ein muttersprachliches Niveau durch die Schule bringen kann, es wäre aber dann keine Muttersprache. XenonX3 – () 17:36, 12. Sep. 2014 (CEST)
Darauf, wo derjenige geboren wurde, kommt es dabei aber genau so wenig an wie darauf, wo er aufgewachsen ist. Der Artikel ist auch wie ich finde nicht ganz eindeutig: Einerseits sagt er, dass die Muttersprache diejenige sei bzw. diejenigen seien, die man ohne Unterricht erlernt habe, andererseits sagt er aber auch, dass bis zur Pubertät auch andere Sprachen den Status einer Muttersprache erreichen können. Ich wüsste nicht, warum dem Erreichen dieses Status (zusätzlicher) Unterricht in der jeweiligen Sprache im Wege stehen sollte, ganz im Gegenteil: Wenn man eine Sprache schon ohne Unterricht auf einem muttersprachlichen Niveau kann, dann kann ja zusätzlicher Unterricht, der das Niveau ja noch verbessert, dieses Niveau ja nicht nehmen. --88.130.88.58 17:46, 12. Sep. 2014 (CEST)

Was mir gerade noch auffällt, ist, dass der GeR nur bis C2 reicht, was "annähernd muttersprachliche Kenntnisse" bezeichnet. Ein "D1" wäre vielleicht denklogisch das nächste Niveau, aber das gibt es in dem Rahmen nicht. Für Muttersprachler selbst ist dieser Referenzrahmen gar nicht gemacht. Wenn jemand also Muttersprachler ist, dann fällt er aus diesem Rahmen raus; es wäre insofern auch sinnlos, ihn die entsprechenden Tests machen zu lassen, weil er da ja höchstens C2 erreichen kann, was nicht seine tatsächlichen Kenntnisse abbildet. Er erhielte eh ein falsches Ergebnis. --88.130.88.58 18:03, 12. Sep. 2014 (CEST)

Der Gemeinsamer Europäischer Referenzrahmen beurteilt doch nach tatsächlichen Fähigkeiten. Selbst bei Abitur in Deutschland kann es dann vorkommen, daß C2 nicht erreicht wird. Z.B. wird man bei Einreise im Kindesalter, dann deutsche Schule und danach wieder Ausreise und wenig Kontakt mit deutscher Sprache Jahrzehte später vielleicht Schwierigkeiten bei der Einstufung C2 haben. Um die eigene Einstufung abschätzen zu können, sind daher die Beschreibungen der Gemeinsamen Referenzniveaus besser geeignet als möglicherweise ältere Abschlüsse. 217.230.70.42 18:22, 12. Sep. 2014 (CEST)
Es stimmt natürlich, dass sich das Niveau, das man nach dem GeR erreicht, verändern kann; genau das ist ja auch das Ziel, wenn man eine Sprache lernt. Insofern ist es mMn auch möglich, dass man, nachdem man erst eine Sprache als Muttersprache erlernt hatte, sie durch jahrzehntelanges Nichtbenutzen wieder zumindest teilweise verlernt, so dass man innerhalb des GeR "nur" auf das "normale" C2-Niveau kommt, das auch Nicht-Muttersprachler erreichen können, oder dass man sogar darunter liegt. Diese Einstufung ändert aber doch nichts daran, dass die jeweilige Sprache in der frühen Kindheit erlernt und weitgehend automatisiert gekonnt wurde, so dass sie die Muttersprache ist. Ansonsten gäbe es nachher den Fall, dass es Menschen ohne Muttersprache gibt - das kann ja nicht sein. Für die Ausgangsfrage hilft das alles aber nicht wirklich weiter.
Die Information, dass jemand als Schlesier in Polen von seiner Mutter deutsch gelernt hat, lässt keinen sicheren Rückschluss darauf zu, ob derjenige nun ein Niveau erreicht hat, dass er Muttersprachler ist oder nicht. Schulbesuchsjahre sind (genau wie, was mit den Eltern gesprochen wurde) sicher ein brauchbares Indiz für ein gewisses Niveau, aber auch das sagt nicht, wie gut genau die Sprachkenntnisse waren. Um zu sagen, ob derjenige nun Muttersprachler ist oder nicht brauchen wir mehr Informationen. Mit dem Abitur und GK/LK erreicht man wohl etwas, das als B2/C1-gleichwertig angesehen wird (wobei auch die dazu im Detail nötigen Leistungen von Bundesland zu Beundesland variieren können). Ob das dann aber auch im Einzelfall anerkannt wird, ist wohl unterschiedlich. Es kommt wohl drauf an, wer es anerkennen soll. Ich schätze, es würde die Sache vereinfachen, wenn man ein Blatt hätte, auf dem explizit draufsteht, dass Niveau X nach dem GeR erreicht wurde. Damit kann man sich nervige Diskussionen sparen.
Ich lerne aus diesem Diskurs (korrigiert mich, wenn es falsch sein sollte): Scheinbar muss man dazwischen unterscheiden, ob jemand eine bestimmte Sprache als Muttersprache hat und ob er sie (selbst wenn er sie als Muttersprache hat) auf muttersprachlichem Niveau spricht. --88.130.88.58 19:20, 12. Sep. 2014 (CEST)

Gibts denn eine verbindliche Definition für den Begriff Muttersprache? --DerHandelsreisende (Diskussion) 20:20, 12. Sep. 2014 (CEST)

Laut Muttersprache ist jemand dann Muttersprachler, wenn er die jeweilige Sprache "in der Kindheit erlernt hat" und er sie "weitgehend automatisiert beherrscht". Darüber, wann genau das der Fall ist, herrscht wahrscheinlich - wie bei praktisch allen geisteswissenschaftlichen Themen - ein heilloser Streit. --88.130.88.58 22:51, 12. Sep. 2014 (CEST)
Muttersprachliches Niveau hat, wer in der Sprache denkt. Wer nur sehr schnell übersetzt, beherrscht die Sprache nicht auf muttersprachlichem Niveau.
Das hat mit einem "Akzent", also einer von der jeweiligen Hochsprache abweichenden Aussprache nichts zu tun.
Und auch nicht mit der sonstigen Qualität der Sprache. Ein Vierjähriges hat (normalerweise) eine Muttersprache, beherrscht diese aber eben auf einem altergemäßen Niveau. Ein erwachsener sehr ungebildeter Mensch denkt auch in seiner Muttersprache, aber der schnellübersetzende Nichtmuttersprachler spricht sie vermutlich auf einem wesentlich gebildeteren Niveau. Das Erlernen in der Kindheit erleichtert die Sache sehr, vermag aber kein Ausschlusskriterium darzustellen. Hummelhum (Diskussion) 23:42, 12. Sep. 2014 (CEST)
Wenn man Abitur macht, ist diese Stufeneinteilung für die Sprache Deutsch irrelevant. Ich glaube kaum, dass es für ausländische Bewerbungen nötig ist. Wie ist man sonst durchs Abitur gekommen, wenn man Deutsch nicht auf einem muttersprachlichen Niveau beherrscht? Viel Fließtext schreiben ist auch in anderen Fächern erforderlich. In Baden-Württemberg zum Beispiel ist es absolut nicht möglich, nur Grundkurse in Fremdsprachen abzuschließen und in Deutsch erst recht nicht, da es Pflicht ist, darin Abitur zu machen. Das Konzept Grundkurs-Leistungskurs gibt es auch nicht in allen Bundesländern. In Baden-Württemberg sind Kernfächer das Äquivalent zu Leistungskursen. Sie sind nur vierstündig, aber davon gibt es fünf. Eine abgewählte Fremdsprache, die man seit der 5. Klasse hat, bringt hier B1 und ein Abitur in der Fremdsprache ergibt auch nur B2, höchstens C1, wenn man noch weitere Zertifikate abschließt. Deshalb kann ich nicht nachvollziehen, warum anderswo ein Grundkurs schon B2 bringt. Die Definition Muttersprachler mit 9 Jahre erfolgreichem Unterricht in der jeweiligen Sprache ist sehr vage. Die Begriffe "Jahre" (wie intensiv?), "Unterricht" (wie oft?) und "erfolgreich" (gerade noch so geschafft?) sind an sich schon subjektiv. Alle Abiturienten müssten dann auch englische Muttersprachler sein. --Explosivo (Diskussion) 04:33, 13. Sep. 2014 (CEST)

Logarithmische Funktion durch drei Punkte

Ich habe drei Punkte, aus der ich in eine logarithmische Funktion ableiten möchte, die diese Punkte (zumindest näherungsweise) durchläuft. Wie kann ich so etwas näherungsweise machen (bzw. wo kann man das ein Programm wie WolframAlpha machen lassen?). Google war natürlich meine erste Wahl, was hilfreiches konnte ich aber nicht finden, vielleicht suche ich auch nur nach dem Falschen (Interpolation (Mathematik) hilft mir dabei leider auch nur wenig). --217.234.106.233 17:33, 12. Sep. 2014 (CEST)

Was hast du denn schon versucht? Ich würde einfach eine allgemeine Funktionsgleichung aufstellen . Wenn man die drei Punkte einsetzt bekommt man ein Gleichungssystem mit drei Gleichungen und drei Unbekannten (a,b und c). Das löst man und erhält die gesuchte Funktion. --Der-Wir-Ing (Diskussion) 17:51, 12. Sep. 2014 (CEST)
Ein Beispiel mit WolframAlpha: [34]. Es hängt aber natürlich davon ab, was genau unter „logarithmische Funktion“ verstanden werden soll. -- HilberTraumd, m18:00, 12. Sep. 2014 (CEST)
(BK)Hier ist es sinnvoll, nicht , sondern die äquivalente Funktion zu verwenden. Dann kann man substituieren , was die Gleichung zu vereinfacht. --Rôtkæppchen₆₈ 18:04, 12. Sep. 2014 (CEST)
Deswegen gibt es nur zwei unabhängige Parameter. Da eine Gleichung mehr vorliegt als (unabhängige) Parameter vorhanden sind, ist eine Approximation, z.B. nach der Methode der kleinsten Quadrate nötig. Oder man wählt zwei Punkte aus und ist zufrieden, wenn der dritte einigermaßen paßt. 217.230.70.42 18:07, 12. Sep. 2014 (CEST)
Super, die Antworten haben mir schon geholfen. Dass bzw. wie man mit WolframAlpha so eine Funktion bestimmt, wusste ich auch nicht, das Ergebnis reicht mir aber schon. Vielen Dank euch! --217.234.106.233 19:33, 12. Sep. 2014 (CEST)

Unterschiede Farbübertragsformate?

Worin unterscheiden sich die Übertragungssysteme NTSC, PAL und SECAM voneinander? Wo sind die größten Unterschiede? Wie wirken sich diese Unterschiede auf die Digitalumstellung aus? --188.101.69.80 12:28, 13. Sep. 2014 (CEST)

Das kannst Du in den Einzelartikeln nachlesen. Da die Digitalumstellung beim Fernsehen mittlerweile abgeschlossen ist, haben die alten Farbsysteme nur noch bei alten Analogkonserven Bedeutung. Framegrabber erkennen die verwendete Farbnorm in den allermeisten Fällen automatisch, auch geschieht die Verarbeitung durch DSPs, sodass es dem Grabber piepegal ist, was da für ein Videorecorder angeschlossen ist. Die digitale Weiterverarbeitung geschieht in Formaten, die mit den alten Analogfarbnormen nichts mehr zu tun haben. Die immer noch gemachte Unterteilung nach „PAL“- „SECAM“- oder „NTSC“-kompatibel hat mit den genannten Farbnormen nicht zu tun, sondern bezieht sich auf eine Kompatibilität mit den davor benutzten Schwarzweißnormen, also RS-170A und die Gerbernorm, aus denen die verschiedenen Analogfarbnormen entwickelt wurden. --Rôtkæppchen₆₈ 12:41, 13. Sep. 2014 (CEST)

Elternbrief wegen NPD Wahl meines Sohnes

Hallo, Ich in Vater eines 13 jährigen Sohnes, derzeit ist in unserem Bundesland Wahlkampf, der auch in der Schule Thema war, in der Klasse erfolgte eine Art Probe Wahl. Mein Sohn und sein freund erlaubten sich einen Spaß, und wählten NPD. Mein Sohn hat mir versichert, dass sie einfach nur einen Spaß machen wollten und sich Nix dabei gedacht haben. Der Lehrer, stand aber hinter Ihnen und hat zufälligerweise mitbekommen wie sie die Partei gewählt haben. Er hat sofort eingegriffen und sie von der Wahl ausgeschlossen, sowie weitere Sanktionen angekündigt. Eine ist dieser Eltern Brief. Ich soll mit meiner Frau zu eine. Gespräch in der Schule bzw. Bei dem Lehrer erscheinen. Meine Frage ist das rechtens? Drohen mir als Vater ebenfalls strafen wegen Aufsichtspflicht und so? Habe zwar nix darüber gefunden dass die NPD verboten ist aber was soll ich nun tun?--84.59.222.215 14:29, 12. Sep. 2014 (CEST)

Beschwer Dich bei der Schulleitung über den Lehrer. Wenn in der Schule schon Landtagswahl geprobt wird, sollte sie schon wie die echte Landtagswahl sein. Der Lehrer hat bei einer derartigen Reaktion selbst nicht begriffen, was allgemein, unmittelbar, frei, gleich und geheim bedeutet. Sollte sich das Ereignis durch eine schlechte Note manifestieren, so sollten die Eltern schriftlich auf eine Streichung dieser Note bestehen, denn sie ist nicht gerechtfertigt. --91.51.254.132 14:45, 12. Sep. 2014 (CEST)
Einerseits ist es Ausdruck demokratischen und liberalen Handelns, alle zur Wahl stehenden Parteien tatsächlich wählen zu können. Im Dritten Reich und in der DDR war das nicht so: Zwar ging man zur Wahl, konnte sich tatsächlich aber nur für eine Wahlmöglichkeit entscheiden. Andererseits ist es ein Auftrag der allgemeinbildenden Schulen, die Schülerinnen und Schüler unter anderem zu demokratischen Bürgerinnen und Bürgern zu erziehen. Eine Strafe halte ich deshalb für ungerechtfertigt, ungerecht und ausgeschlossen, ein besonderes Gewicht auf eine entsprechende Erziehung dieser Schüler dagegen für zulässig und geboten. --BlackEyedLion (Diskussion) 14:48, 12. Sep. 2014 (CEST)


Eine Bitte um ein Elterngespräch ist keine Strafe und auch sonst nichts Gefährliches, sondern im besten Fall Zeichen für das Bemühen des Lehrers, gemeinsam mit dir das Beste für dein Kind tun zu wollen. Unwahrscheinlich, dass er das Gespräch nur wegen dieses Kreuzchens sucht. Geh hin, erklär ihm deine Sicht der Dinge, hör dir aber auch an, was er zu sagen hat. Grüße Dumbox (Diskussion) 14:49, 12. Sep. 2014 (CEST)

Ich halte die Darstellung der IP für äußerst fragwürdig. Der Lehrer, stand aber hinter Ihnen und hat zufälligerweise mitbekommen wie sie die Partei gewählt haben. Er hat sofort eingegriffen und sie von der Wahl ausgeschlossen, sowie weitere Sanktionen angekündigt. Das halte ich für glatt gelogen. Wahrscheinlicher Hergang: Die Jungs haben ihre Wahl mit entsprechenden Bemerkungen kommentiert, die ein pädagogisches Eingreifen dringend erforderlich erscheinen lassen. Wenn die Story überhaupt einen Kern hat, dann diesen. ME ein rechter Trollbeitrag. --JosFritz (Diskussion) 14:52, 12. Sep. 2014 (CEST)

Ups, ja: In Hessen, wo die IP verortet wird, ist zurzeit auch nicht viel Wahlkampf. Archivieren? Dumbox (Diskussion) 14:57, 12. Sep. 2014 (CEST)
Upperupps: Gewählt wird übermorgen in Thüringen. Das ist, anders als der Name vermuten lässt, heute auch ein deutsches Bundesland. Und es liegt neben Hessen. Wenn nun der Vater des provoziert habenden Sprösslings gerade an seinem Schreibtisch in Hessen sitzt und wikipittert, heut abend aber wieder ins rechtslastige Thüringen heimfährt (die historisch-wirtschaftlichen Hintergründe solch thür-hessischen Pendlertumes könnten bekannt sein), dann muss er nicht deswegen ein Troll sein. Hummelhum (Diskussion) 15:14, 12. Sep. 2014 (CEST)
Jo, muss nicht. Dreieich, Landkreis Offenbach, Regierungsbezirk Darmstadt, ist aber wirklich nicht Ex-Zonenrandgebiet. Dumbox (Diskussion) 16:18, 12. Sep. 2014 (CEST)
Ganz schlechter Pädagoge. Ob solche Probewahlen grundsätzlich sinnvoll seien, darüber kann man natürlich diskutieren. Sie werden aber seit Jahrzehnten oft im Unterricht, manchmal auch für alle Schüler der Schule gemeinsam durchgeführt, um den Kindern / Jugendlichen die Möglichkeit zu geben, sich in die Lage des Wählers (der sie ja wenige Jahre später sein werden) hineinzuversetzen, sich ihre eigenen Gedanken zu machen, Wahlentscheidungen kritisch zu hinterfragen etc. Selbstverständlich muss man dabei von den Prinzipien ausgehen, denen auch die echte Wahl unterliegt. Und da ist nun mal eines der wichtigsten, wenn nicht das (gefühlt) wichtigste das Wahlgeheimnis. Der Idiot (um mal kein deutlicheres Wort zu verwenden) von Lehrer hat mit seiner krankhaften Neugier das didaktische Instrument "Wahlsimulation" versaut, indem die Schüler eben gerade nicht erfahren durften, wie sich das (verantwortliche) Wählen anfühlt; er hat sich das Vertrauen der Schüler verscherzt (und wird diesen im weiteren Unterricht so nötigen Kredit auch nicht wiedererlangen können); er hat schließlich die didaktische Aufbereitung des "Wahlergebnisses" unmöglich oder sinnlos gemacht (die Klasse hätte erarbeiten können, warum zwei NPD-Stimmen dabeiwaren; die gibt es ja auch in der Realität - sind es vielleicht Proteststimmen?). Und er hat allgemein den Schülern das Gegenteil von Demokratie beigebracht. Jeder Bürger sollte wissen, dass diese NPD nicht verboten ist und daher genauso gewählt werden darf wie jede andere Partei; ein Lehrer muss das auch erklären können. Für ihr Verbot zu sein und dafür tätig zu werden, steht auf einem anderen Blatt.
Dass man mit dem Anschwärzen extremistischer Parteien denen gerade Protest- oder Trotzwähler in die Arme treibt, wusste schon Hitler (und nutzte dieses Wissen rhetorisch).
A propos rhetorisch: Das alles wäre diesem völlig ungeeigneten Lehrer und dessen Vorgesetzten (Schulleitung, Schulamt) anzukreiden, was allerdings auch sprachlich (vgl. von Dir gewählte Rechtschreibung) einer gewissen Form bedürfte. Hummelhum (Diskussion) 14:57, 12. Sep. 2014 (CEST)

Hört sich alles sehr komisch an, aber tun wir mal so, als sei es so wie beschrieben passiert: Es ist Aufgabe des Lehrers Schülern Werte zu vermitteln. Einer dieser Werte ist der Wert der Demokratie in unserem Land. Bei Missbrauch durch Spaß-Wahl, denn aus Jux und Dollerei irgendeinen Müll zu wählen, ist nichts anderes als Missbrauch, ist es das Einzig-Richtige, pädagogische Maßnahmen zu ergreifen. Wenn du sicher bist, dass diese Wahl deines Sohnes wirklich nur ein Scherz war, dann ist ja ok: Er ist noch nicht 16, er darf noch nicht wählen und hat von Politik vll. auch noch nicht so die Ahnung, aber ein ernstes Gespräch mit Sohnemann würde ich trotzdem führen. Allerdings ist es auch so, dass die NPD trotz aller abstruser Forderungen, die diese Partei so aufstellt, immer noch nicht verboten ist. Man darf diese Partei wählen. Insofern hat dein Sohn nichts Verbotenes gemacht - aber etwas Dummes. Wenn du dir sicher bist, dass dein Sohn nicht dabei ist, ins Rechtsextreme Millieu abzudriften, dann ist das einzige Problem, dass er nicht erkannt hat, welchen Sinn und welche Bedeutung eine demokratische Wahl hat. Ich wüsste nicht, dass du zur Schule gehen müsstest, nur weil die das gerade will. Schlauer wäre es aber wahrscheinlich, doch hinzugehen - auch damit der Lehrer von deinem Sohn und eurer Familie kein falsches Bild bekommt. --88.130.88.58 15:03, 12. Sep. 2014 (CEST)

Diese simulierten Wahlunterlagen gibt es als Unterichtsmaterial für Schulen schon seit längerer Zeit und das wird in den Schulen auch durchgeführt. Die Frage ist durchaus legitim und verwundert in keiner Einzelheit der Darstellung. Wer die Unterichtsmaterialien kennt wundert sich auch nicht darüber, dass es grundsätzlich ein paar Spaßvögel gibt die das wählen wovor so ausdrücklich gewarnt wurde. Wer hier falsch handelt sind in diesem Fall die Lehrer. Die Eltern bestellt man nicht zur Gesinnungsprüfung in die Schule weil der Schüler "falsch" gewählt hat. So was gabs in der DDR. Wohingegen anzunehmen ist das die Eltern nicht in die Schule zitiert worden wären, wenn die Schüler die drei mal umbenannte SED statt der einmal umbenannten nsdap gewählt hätten. Ich würde in diesem Fall am Besten zum Gespräch gehen und erklären das eine politische Bildung durchaus in die Schule gehört, eine politische Erziehung dafür nicht im Geringsten. So falsch das sein mag die NPD oder die Linke zu wählen, das Recht dazu hat man in Deutschland ohne Sanktionen befürchten zu müssen. --92.227.212.29 15:04, 12. Sep. 2014 (CEST)
Wir haben damals als Fünfzehnjährige in der Woche nach der Landtagswahl in Baden-Württemberg 1984 im Gemeinschaftskundeunterricht selbst eine „Landtagswahl“ mit hektographierten Originalstimmzetteln durchgeführt. Die Grünen haben damals die meisten Stimmen bekommen. --Rôtkæppchen₆₈ 15:14, 12. Sep. 2014 (CEST)
Das dürft bei den meisten 15 Jährigen so ausgehen. In dem Alter glaubt man dass die Grünen niedliche Robbenbabies retten und so ein Zeug. --92.227.212.29 15:19, 12. Sep. 2014 (CEST)
1983
2006
Was mag in Leuten vorgehen, die die Grünen von 1984 und die von 2014 in einen Topf werfen? Ich will's gar nicht wissen. --Eike (Diskussion) 15:25, 12. Sep. 2014 (CEST)
Es ist ein bisschen unklar wen du gerade gemeint hast, da weder Rotkäppchen noch ich irgendjemanden in irgendeinen Topf geworfen hat. Rotkäppchen hat nur bestätigt das es diese simulierten Wahlen in Schulen sehr lange gibt und ich habe angemerkt das die Grünen unter 15 Jährigen wohl auch heute noch sehr hohe Stimmanteile bekommen. Was Sie wenn man sich die veröffentlichten Ergebnisse dieser Schulwahlen ansieht auch zutrifft. --92.227.212.29 15:29, 12. Sep. 2014 (CEST)
Das mit den Robbenbabies war schon 1984 so. Damals gab's noch echte unbesetzte "grüne" Themen wie Friedensbewegung, Atomwirtschaft und Umweltschutz allgemein, die Wahrnehmung war allerdings schon damals, dass "Grüne" im Parlament vor allem durch Stricken, Säuglinge mitbringen und Turnschuhtragen auffielen. Diese Wahrnehmung war schon damals von den Grünen so gewollt, alles sollte hauptsächlich "alternativ" sein. Heutzutage ist das totale Gegenteil der Fall, die Grünen sind im System angekommen, zehren aber immer noch von diesem "Alternativ"-Image. Das wird halt gepflegt. -- Janka (Diskussion) 15:36, 12. Sep. 2014 (CEST)
Dass das '84 so war, ist völlig klar, aber ich bezweifle, dass die Grünen von heutigen Jugendlichen als wahnsinnig alternativ wahrgenommen werden. Zufällig gestern in einem Interview gesehen, wo eine Grüne über die AFD und die etablierten Parteien gesprochen hat - zu denen selbst bei ihr, nicht mehr ganz jugendlich, selbstverständlich die Grünen zählten. Ne, das, was in meiner Generation die Grünen waren und vielleicht in der davor ein Stück weit die SPD ("Wer hat uns verraten"), das besetzten heute längst andere Parteien. Man muss bedenken, für heutige Jugendlichen sind die Grünen welche, die früher, als sie klein waren, in der Bundesegierung saßen. Und Hartz IV verabschiedet haben. Nicht die mit den Strickpullis. --Eike (Diskussion) 15:39, 12. Sep. 2014 (CEST)
(BK)In Baden-Württemberg gibt es sogar schon eine grünen Ministerpräsidenten. Gerade die Regierung Kretschmann zeigt tagtäglich, wie sich die Grünen von ihren alten Idealen entfernt haben. Ideale sind schön und gut, Macht ist besser. --Rôtkæppchen₆₈ 15:47, 12. Sep. 2014 (CEST)
Mutti hat ihnen die Wetterzauberei und das Atom weggenommen, die EU nahm ihnen die Gene und das mit dem Frieden haben Sie für sich selbst erledigt. Da bleibt Ihnen ja nichts mehr außer einfach nur Macht haben. Die werden wohl auch früher oder später den Weg der FDP gehen. Die waren am Ende auch für absolut gar nichts mehr gut. --92.227.212.29 15:54, 12. Sep. 2014 (CEST)
Wer die Ideale seiner Jugend verrät, wird mit vorzeitigem Altern bestraft. (Theodor W. Adorno zugeschrieben) --212.184.138.49 16:08, 12. Sep. 2014 (CEST)

Bei Missbrauch durch Spaß-Wahl, denn aus Jux und Dollerei irgendeinen Müll zu wählen, ist nichts anderes als Missbrauch, ist es das Einzig-Richtige, pädagogische Maßnahmen zu ergreifen. Welche pädagogischen Maßnahmen wurden denn ergriffen, nachdem Martin Sonneborn ins EU-Parlament gewählt wurde? --Optimum (Diskussion) 15:52, 12. Sep. 2014 (CEST)

Welche Maßnahmen hätte man ergreifen müssen? Das Titanic-Abo abbestellen? Uncool. Aber du, Optimum, bis ja eh hinlänglich bekannt für deinen fehlenden Humor.--Hubertl (Diskussion) 16:39, 12. Sep. 2014 (CEST)
Welchen Humor. Sonneborn und Titanic leben davon das Sie durchschnittlich alle zwei Jahre einen gehaltvollen Scherz machen und zwischenzeitlich unlustigen Blödsinn fabrizieren. Der Sonneborg im EU Parlament ist halt ein Indikator dafür wie wurscht und hoffnungslos dieses Thema für alle inzwischen geworden ist. --92.227.212.29 16:50, 12. Sep. 2014 (CEST)
Wenn ich Dich anspreche, dann wirst du es merken - oder auch nicht, was ziemlich egal wäre. Oder kannst du nicht lesen? Das ging an jemanden anderen. Bis dann, dann!--Hubertl (Diskussion) 16:53, 12. Sep. 2014 (CEST)
Na jetzt hab ichs aber vom Wikipedia Humorbeauftragten gesagt bekommen. Werde mich zukünftig zum Thema Humor nicht mehr melden, bis ich offiziell die Erlaubnis erteilt bekomme. Tüdelü ... --92.227.212.29 17:08, 12. Sep. 2014 (CEST)
Alle wollen in die Mitte, wir bieten die klare Alternative! "Ja zu Europa, Nein zu Europa" - damit decken wir beide Ränder ab! -- Janka (Diskussion) 16:03, 12. Sep. 2014 (CEST)
„Trolle bitte nicht füttern!“
so ist es! --Hubertl (Diskussion) 16:39, 12. Sep. 2014 (CEST)
Ich halte die Frage für eine Trollfrage. Das liegt nicht nur an den Schreibfehlern. Ich kann mir auch nicht vorstellen, daß ein Lehrer so blöd ist, alles, was er zum Wahlgeheimnis und Wahlrecht gesagt hat (bzw. haben muß) ad absurdum zu führen. Ich denke jedoch nicht, daß die Zielrichtung ist: Jaja, diese Ossis, wollen immernoch die Wahlen manipulieren... Nein. Es liegt nahe, mit so einer Trollfrage angesichts der bevorstehenden Wahlen die NPD ins Gespräch zu bringen (nicht verboten und so, auch: ist das rechtens...). Es führt ja zu der spontanen Reaktion: Natürlich muß der Junge da auch NPD wählen können - und das ist wohl die Reaktion, die provoziert werden sollte. Ich bin mir auch nicht sicher, ob Einladungen zu einem Elterngespräch fordern, daß beide Elternteile kommen sollen, das können die Lehrer unter uns wohl besser sagen. Zu den vielen Diskussionsbeiträgen denke ich: Freiheit bedeutet auch, daß man das Falsche tun kann. Das ist (wie die Unsicherheit) ein Preis der Freiheit und nur begrenzt durch die Regel: Keine Freiheit für die Feinde der Freiheit. --212.184.138.49 16:23, 12. Sep. 2014 (CEST)
+ Ich auch, nachdem ich vorschnell ernsthaft zu antworten versuchte. Bez. Einladung der Eltern: Diese ergeht an den/die Erziehungsberechtigte(n); ob bei zwei Erziehungsberechtigten einer oder beide kommen, liegt völlig in deren Ermessen. Und es ist eine Einladung, keine Vorladung. Wenn keiner kommt, ist es auch gut. Grüße Dumbox (Diskussion) 16:30, 12. Sep. 2014 (CEST)
Es ist zwar nicht besonders klug, aber nicht verboten, die NPD anzukreuzen oder anderen Unsinn auf den Wahlzettel zu schreiben. Allerdings fehlt diesem Lehrer eine grosse Portion Didaktik (wurde ja alles schon gesagt). Daher sollte dieser Fall schon von elterlicher Seite an die Schulleitung herangetragen werden (Das Fass hat er ausgemacht. Er haette es ja auch bei einem Gespraech mit dem Schueler belassen koennen). Ich hatte (laaang ists her) einen Ethik- und Sozialkunde-Lehrer, der haette die selbe Aktion bringen koennen. Der war eine reichlich gestoerte Person, er demuetigte wo er nur konnte. Beschwerden der Schueler (und leider viel zu selten der Eltern) haben nichts gebracht, irgendwie war sein Verhalten es immer noch so im Rahmen. Nach meiner Schulzeit ging er dann aus Altersgruenden in Pension und kurze Zeit spaeter war er dann fuer sehr lange Zeit in einer Nervenklinik. --Nurmalschnell (Diskussion) 16:40, 12. Sep. 2014 (CEST)
Die IP kann auch weit entfernt sein, denn GEO-IP ist nicht GPS. Das kann abweichen. Es gilt das Wahlgeheimnis. Da die DDR das nicht kannte, hatte sie auch keine Wahlkabinen. Auch Unternehmensberater markierten Zettel von Umfragen, die an Mitarbeiter ausgegeben wurden. Das ist dasselbe, wenn der Name oder die Identifizierung nicht gefragt wird. Ich halte beides für einen Straftatbestand. Wenn es die Politik und dazu gehören auch Schul- und Familienpolitik nicht so gestalten kann, dass unser Nachwuchs auch diese Ideen kommt, braucht sich über die Trollfütterung nicht wundern. Eine Wahl zu trainieren und dabei zu erklären und warum was wie gehandhabt wird, ist durchaus wertvoll. Sollten entsprechende Stimmen dabei sein, kann ja noch der Film Die Welle (Roman) gezeigt werden. Da das schon 1981 verfilmt wurde, sollte es heute in jeder Schule aufführbar sein. Sollte das Bundesland das anders handhaben, freuen sich schon Presse und Medien über Wahlergebnis samt Zustand. --Hans Haase (有问题吗) 18:00, 12. Sep. 2014 (CEST)
Doch Hans, das Wahlgeheimnis kannte die DDR schon, siehe ihre Verfassung (z. B. in der Fassung von 1974, Art. 54): "Die Volkskammer besteht aus 500 Abgeordneten, die vom Volke auf die Dauer von 5 Jahren in freier, allgemeiner, gleicher und geheimer Wahl gewählt werden." Wahlkabinen gab es übrigens auch, nur konnte es verdächtig sein, wenn man da hineinging.--IP-Los (Diskussion) 18:40, 12. Sep. 2014 (CEST)
Was wir so locker-flockig-trollig und lange aus der Schule hier beplaudern ist für die Beteiligten nicht so lustig, wenn der besagte Lehrer einem das nachträgt und Gerüchte durch Schulhof und sozialen Netzwerke wabern. Anyway, der Herr ist als Lehrer ein Dilletant und als Regierungsanhänger übereifrig. Der muss wissen dass es immer diese Scherzkekse gibt die NPD ankreuzen oder Sonneborn oder die APPD, ohne das was dahinter steht.--Antemister (Diskussion) 23:53, 12. Sep. 2014 (CEST)
Ich würde es eher so sehen: Bevor Urteile gefällt werden (von denen es hier schon manche gibt): Audiatur et altera pars.--IP-Los (Diskussion) 13:37, 13. Sep. 2014 (CEST)
Doch nicht, wenn's um Lehrer geht: Da reicht ein hinger*tzter Vorwurf, und ab geht die Party! Gar nicht erst ignorieren, hätte Karl Valentin gesagt. Grüße Dumbox (Diskussion) 14:00, 13. Sep. 2014 (CEST)

Für alltäglichen Sexismus, Heterosexismus, Rassismus und Antisemitismus im Klassenzimmer ist der Lehrer wahrscheinlich blind, aber wenn dann (was egtl. zu erwarten ist) die vom Lehrer aufgestellte Aufgabe/Unterrichtsaktivität nicht ernstgenommen wird, in Bezug auf dieses Sakrosanktum den impliziten, seiner Autorität entspringenden Forderungen nicht nachgekommen wird, ja dann greift er auch gerne argumentativ auf die besondere Problematik dieser Partei zurück. --Chricho ¹ ² ³ 14:06, 13. Sep. 2014 (CEST)

Seufz. Einmal versuch ich's noch. Wenn die Story nicht überhaupt fiktiv ist, hat ein Lehrer um ein Elterngespräch gebeten. Zusatzarbeit für ihn. Sohnemann hat seine Version dessen, was vorgefallen ist, dem Papa vorgetragen, und Papa hat entschieden, dass seinem Kind Unrecht geschehen ist. Hm, und jetzt? Hingehen und die Sache klären, oder? Grüße Dumbox (Diskussion) 14:12, 13. Sep. 2014 (CEST)
Im Übrigen warne ich vor dem Argument „nur ein Spaß“: Auch ein Spaß ist eine Aussage, es ist eine Aussage, nicht von der Idee komplett angewidert zu sein, gerade mit der NPD den Spaß zu treiben, es ist eine Aussage, den Spaß eben nicht mit den Violetten oder den Grauen Panthern zu machen. Angemessene Reaktion ist da aber nicht eine Skandalisierung. Ansonsten +1 an Dumbox. --Chricho ¹ ² ³ 14:32, 13. Sep. 2014 (CEST)

Ist schon interessant, wenn man die Diskussion bei Wikimedia so liest....Fazit: Der arme Lehrer hat alles richtig gemacht und war natürlich nicht so dumm, die beiden Jungs von der Wahl auszuschliessen und hat natürlich auch nicht mit Konsequenzen gedroht.....so ein Schwachsinn, denn der Elternbrief wurde ja geschrieben, es hat diese Konsequenz gegeben! Also wird der liebe Lehrer wohl tatsächlich sein Demokratieverständnis gezeigt haben: Alles ist gut außer nationales Gedankentum. Schon das der Lehrer bei dieser Wahl, wenn er sie schon durchführt, auf die Wahlzettel "zufällig" schaut und damit eine freie, persönliche Abstimmung ad absurdum führt ( es schaut ja auch keiner in die Wahlkabine), er reagiert auch noch hysterisch! Sicher gibt es demokratiefreundlichere Parteien als ausgerechnet die NPD, aber sie ist nicht verboten und damit eine Partei wie jede andere, die nicht verboten ist, obs dem Pauker nun passt oder nicht, er hätte dies akzeptieren müssen. (nicht signierter Beitrag von 89.182.57.234 (Diskussion) 11:15, 14. Sep. 2014 (CEST))

Na, daß sich das "nationale Gedankentum" (ein Euphemismus für braunen Dreck im Hirn?) hier nochmal meldet und schnell noch ein bißchen Wahlwerbung macht, war ja zu erwarten. Dacht ichs mir doch (vgl. meinen Beitrag unten als 212.184.138.49, 16:23, 12. Sep. 2014): Es geht drum, euch ins Gespräch zu bringen, offene Systeme wie dieses zu mißbrauchen... --212.184.132.111 12:05, 14. Sep. 2014 (CEST)

Ich glaube, dass hier alles möglicherweise Nützliche gesagt ist. Der Verdacht der fingierten NPD-Werbung liegt nahe, ist nicht beweisbar, und löschbar ist das Ganze sowieso nicht mehr. Aber Schluss zu machen scheint angebracht. --KnightMove (Diskussion) 20:22, 14. Sep. 2014 (CEST)

Archivierung dieses Abschnittes wurde gewünscht von: KnightMove (Diskussion) 20:22, 14. Sep. 2014 (CEST)

Irgendein besonderer Tag heute ?

Wie ich gerade feststelle, weht heute auf dem Karlsruher Schloss die Deutschland-Flagge. Das ist nur zu besonderen Anlässen der Fall. Was ist am heutigen Datum besonders? Ist es, weil heute 1972 die Erfeindschaft zwischen Deutschland und Frankreich beendet wurde? Oder war doch noch irgendwas bedeutenderes? Kann man vielleicht irgendwo nachlesen, zu welchen Anlässen das Protokoll vorschreibt, dass bei staatlichen Einrichtungen eine Flagge zu hissen ist? 129.13.72.197 14:09, 14. Sep. 2014 (CEST)

Tag der Heimat, lese ich beispielsweise hier: [35]. Dumbox (Diskussion) 14:22, 14. Sep. 2014 (CEST)
Europäischer Tag der Jüdischen Kultur, Tag des offenen Denkmals --79.240.0.7 14:23, 14. Sep. 2014 (CEST)
Ministerpräsident Kretschmann hat gem. der Verwaltungsvorschrift des Staatsministeriums Baden-Württemberg zur Beflaggung der Dienstgebäude vom 23. August 2011 (Az.: I / Prot. 0224.1, GABl. Nr. 9 vom 26. Oktober 2011, S. 526) die Beflaggung für die Dienstgebäude des Landes für den Tag angeordnet, an dem die Feiern zum Tag der Heimat stattfinden (Sonntag, 14. September 2014). --Snevern 14:25, 14. Sep. 2014 (CEST)
Archivierung dieses Abschnittes wurde gewünscht von: --37.49.102.64 23:51, 14. Sep. 2014 (CEST)

Minidrohne im Urlaub

Minidrohne (Flugmodelle), ausschließlich privat und nur zum Zweck des Flugs selbst eingesetzte Modelle mit Kamera mit in den Urlaub nach Spanien mitzunehmen ist wohl nicht ratsam, denn neben den Sanktionen dürfen die zuständige Beamten Anzeige stellen und das Gerät in Sicherheitsgewahrsam nehmen. Gibt es dazu weitere Informationen?

--E.Rivas (Diskussion) 16:12, 14. Sep. 2014 (CEST)

google:drohne+spanien liefert die gesuchten Informationen. --Rôtkæppchen₆₈ 16:16, 14. Sep. 2014 (CEST)
Archivierung dieses Abschnittes wurde gewünscht von: Googeln vergessen. --Rôtkæppchen₆₈ 16:16, 14. Sep. 2014 (CEST)

Weißes Andreaskreuz auf grünem Hintergrund?

Flagge Schottlands

Vor noch gar nicht langer Zeit (April/Mai) hatte ich mir bei Ikea ein Tablett mit einem weißen Andreaskreuz auf grünem Hintergrund gekauft. Seit dem rätsle ich, ob das vielleicht (auch unbeabsichtigt) eine Flagge sein könnte. Sieht aus wie die Flagge Schottlands, nur eben in grün. Ein Freund sagte mir, dass das genauso aussieht, wie die Flagge des russischen Zolls. Könnte das hinkommen? --Lkl21:47, 14. Sep. 2014 (CEST)

Das mit dem Zoll scheint gar nicht weit hergeholt zu sein, denn anscheinend hatten die Chinesen der Quing-Dynastie schon beinahe die gesuchte Variante in Gebrauch: [36]. --Lkl21:53, 14. Sep. 2014 (CEST)
Flag of the Federal Customs Service, 1994
Doof. Jetzt konnte ich mir die Frage selber beantworten... ([37]) - Aber danke für Eure Geduld! :) --Lkl21:58, 14. Sep. 2014 (CEST)
Archivierung dieses Abschnittes wurde gewünscht von: --37.49.102.64 23:50, 14. Sep. 2014 (CEST)

Tatort "Der Wüstensohn"

Welchen Sportwagen fährt der Prinz im o. a. Tatort? (Der Wagen ,der am Anfang zu sehen ist). Der Film ist online abrufbar unter http://www.daserste.de/unterhaltung/krimi/tatort/videos/der-wuestensohn-video-tgl-ab-20-uhr-100.html --79.224.212.82 23:07, 14. Sep. 2014 (CEST)

Ich tippe auf einen Lamborghini Aventador. --King Rk (Diskussion) 23:13, 14. Sep. 2014 (CEST)
(BK) Lamborghini Aventador --2A02:810D:10C0:6F4:9090:99FD:D9AB:6260 23:14, 14. Sep. 2014 (CEST)

Danke für die schnelle Antwort! 79.224.212.82 23:14, 14. Sep. 2014 (CEST)

Lamborghini Aventador
Es ist ein Lamborghini Aventador. Zum Vergleich: Tatort-Bildergalerie Bilder 2 bis 4. --Rôtkæppchen₆₈ 23:20, 14. Sep. 2014 (CEST)
Archivierung dieses Abschnittes wurde gewünscht von: --37.49.102.64 23:50, 14. Sep. 2014 (CEST)

Wiener Straßenbahnnetz

Guten Tag, ich wollte fragen, warum die Wiener Straßenbahnen so langsam sind. Es könnten doch einige Linien als Schnellstraßenbahn geführt werden, mit deutlich geringeren Kosten als der U-Bahn-Bau.

MfG, G. --89.144.214.211 17:24, 10. Sep. 2014 (CEST)

Sagt Straßenbahn Wien was dazu? --88.130.92.213 17:38, 10. Sep. 2014 (CEST)

Mich würden hier konkrete Vorschläge interessieren - welche Linien würden sich denn deiner Meinung nach eignen für schnellere Garnituren? --KnightMove (Diskussion) 17:42, 10. Sep. 2014 (CEST)


Ich würd sagen die Linien 1, 2, D, 6, 25, 26 (statt U7), 31 (statt U6-Verlängerung), 67, 71. Südlich von Alterlaa und nördlich vom Rennbahnweg sind die bestehenden U-Bahnen fast leer (deshalb begrüße ich den U5-Bau in den innneren Bezirken - dort braucht man wirklich eine U-Bahn). Ansonsten ist zwar keine Voll-U-Bahn nötig, die veralteten langsamen Straßenbahnen reichen aber keinesfalls aus (die klapprigen ULF können zwar schnell beschleunigen, stehen dann aber genauso lang an der nächsten Ampel/Haltestelle). Für die Schnellstraßenbahn würde ich einen Betrieb mit Fahrzeugen wie WLB 400 (ähnlich T der U6) vorschlagen, sind nicht so engräumig wie die ULF und vom Fahrgefühl erheblich besser. Statt alle E6-c6 Garnituren zu verkaufen, hätte man sie auch für solche Schnellstraßenbahnen einsetzen können, sie sind immerhin moderner als die E1, und durch den Gelenkbeiwagen bieten sie mehr Platz. Außerdem haben sie in beiden Seiten Türen, da hätte man sich die Gleiskreuzung beim Übergang von Rechts- auf Linksverkehr auf der Hochtrasse beim Gewerbepark Stadlau sparen können und trotzdem Mittelbahnsteige einbauen können. Die Ausgestaltung der Stationen denke ich, ähnlich den USTRAB-Stationen. Was planen die Wiener Linien wirklich?, abgesehen von der Verlängerung in Stadtentwicklugsgebiete (O-Nordbahnhof, D-Südbahnhof). Dass der ULF nur sehr langsam die alten Duewag-lizenz-Garnituren ersetzt, zeugt nicht wirklich von seiner Qualität. Die Technik vom ULF ist ja auch schon bald 20 Jahre alt. MfG--89.144.214.211 18:04, 10. Sep. 2014 (CEST)

da gibt es offenbar schon ein paar kleine Initiativen für eine Verbesserung der Ampelphasen für die Bims: [38],[39] ja der 1 ist unfassbar langsam und unzuverlässig und das war mir auch neu [40]--Advanceddeepspacepropeller (Diskussion) 13:36, 12. Sep. 2014 (CEST)
Der 5er ist auch extrem langsam und unzuverlässig, vor allem zwischen Westbahnhof und Lange Gasse. Wobei das mit der Unzuverlässigkeit eher an falschparkenden Autos und LKWs, die wegen Parkplatzmangel zum Ein/Ausladen auf der Straße stehenbleiben, liegt, beides passiert insbesondere in der Kaiserstraße. mMn brauchts da einfach mehr absolute Halteverbote ausgenommen Ladetätigkeit, aber damit macht man sich als Politiker halt bei den Anrainern unbeliebt. Außerdem gibts bei der Haltestelle Blindengasse die engste mir bekannte Straßenbahnkurve, was dazu führt, dass dort immer Straßenbahnen liegen bleiben, wenn mal 5cm an einem Tag schneit. Zusätzlich fahrt der 5er ziemliche Umwege zwischen den Haltestellen Lerchenfelder Straße und Lange Gasse, was aber wohl daran liegt, dass Straßenbahnlinien recht breite Straßen brauchen. --MrBurns (Diskussion) 01:23, 14. Sep. 2014 (CEST)

Aktuelles Lied mit Hund gesucht.

Wie heisst das Lied, was jetzt immer oft im Radio kommt, wo der Hund im Hintergrund jault. "Wouw Wooouuuuuwwww, Wouw Wooouuuuuwwww ..."

--Linkenwolf (Diskussion) 21:57, 11. Sep. 2014 (CEST)

Einfach wenn das Lied läuft auf die Uhr schauen und dann auf der Internetseite des Radiosenders nachschauen. --Rôtkæppchen₆₈ 22:22, 11. Sep. 2014 (CEST)
Zwar weder Hund noch aktuell, aber: Duck Sauce - Big Bad Wolf? --King Rk (Diskussion) 00:35, 12. Sep. 2014 (CEST)
Von denen ists keines? --Chricho ¹ ² ³ 17:25, 13. Sep. 2014 (CEST)

Doppelwürfel in Englisch

Es gibt ein Verschlüsselungsverfahren das sich "Doppelwürfel" nennt. Wie nennt sich das denn in Englisch. Einen 1zu1 Übersetzung bringt keine Ergebnisse. Fall jemand ein Verschlüsselungsprogramm kennt das mit dem Doppelwürfelverfahren arbeitet wäre das noch besser. --92.227.206.106 22:22, 11. Sep. 2014 (CEST)

Übchi, Ubchi, double columnar transposition cipher, double transposition. --Rôtkæppchen₆₈ 22:31, 11. Sep. 2014 (CEST)
Danke. Programme finde ich trotzdem keine für Linux. Frage mich warum. Das soll doch angeblich unentschlüsselbar sein. --92.227.206.106 22:44, 11. Sep. 2014 (CEST)
Die OHL ist bereits am 18. November 1914 zu einem anderen Verfahren gewechselt, weil Übchi viel zu leicht zu knacken war. --Rôtkæppchen₆₈ 22:50, 11. Sep. 2014 (CEST)
Sicher? http://scienceblogs.de/klausis-krypto-kolumne/2013/09/13/top-25-der-ungelosten-verschlusselungen-platz-5-die-doppelwurfel-challenge/ --92.227.206.106 22:52, 11. Sep. 2014 (CEST)
Hat nicht Simon Singh schon drüber berichtet? --Pölkky 23:19, 11. Sep. 2014 (CEST)
Hier ist ein Programm, das einen Known-Plaintext-Angriff auf den Doppelwürfel ausführt, hier ist die dazugehörige Masterarbeit von 2011. Hier berichtet der Focus über einen erfolgreichen Angriff 2013, bei dem ausgenutzt wurde, dass die Schlüssellängen und der Geheimtext zwar Teilerfremd waren, der Rest aber klein war. Ich habe keinen Hinweis auf einen Angriff gefunden, der mit den Mitteln, die 1914 zur Verfügung gestanden haben, ausgeführt werden könnte. --Hjaekel (Diskussion) 23:43, 11. Sep. 2014 (CEST)
Das Verfahren wurde 1914 von den Franzosen geknackt, das ist eine historische Tatsache. Allerdings lag das nicht an der fehlenden Sicherheit der Verschlüsselungsmethode, sondern an der unsicheren Art und Weise, wie sie eingesetzt wurde. So wurden die Schlüsselwörter nicht häufig genug gewechselt, und der Klartext enthielt zu oft leicht zu erratende Phrasen. Nachdem die deutsche Seite erfuhr, dass man in Frankreich die deutschen Meldungen mitlesen konnte, stellte sie die Verwendung von Übchi ein. Es hätte allerdings genügt, das Verfahren richtig anzuwenden, denn selbst wenn die Verschlüsselungsmethode bekannt ist, kommt man ohne Kenntnis der jeweiligen Schlüssel kaum weiter. Das wäre zweifellos besser gewesen, denn die anstelle von Übchi eingesetzten Folgeverfahren wurde von der französischen Dechiffrierabteilung erneut binnen weniger Wochen geknackt. --Snevern 23:54, 11. Sep. 2014 (CEST)
Ja das mit dem Israelischen Hacker kenne ich. Das war glaube ich eine kurze Schlüssellänge und ein Teil des Textes war Ihm bekannt. Ich gehe aktuell davon aus das bei langen Schlüssel und unbekanntem Text das Verfahren nicht zu entschlüsseln ist. (da ich nichts gegenteiliges gefunden habe) Die CIA hat angeblich mal bekannt gegeben das Sie das Verfahren entschlüsseln könnte aber Desinformation ist deren Geschäft. :) Danke für die Links die lese ich mir durch. --92.227.206.106 23:57, 11. Sep. 2014 (CEST)
Ordentliche Verfahren sollten gegen Known-Plaintext-Angriffe und sogar gegen Chosen-Plaintext-Angriffe gewappnet sein! --Eike (Diskussion) 13:54, 12. Sep. 2014 (CEST)
Ich bin offen für Vorschläge und lerne zu diesem Thema gerne dazu. Ich suche eins das bei richtiger Anwendung zu 100% sicher ist und bei dem Doppelwürfel bin ich davon ausgegangen das er bei richtiger Anwendung 100% sicher ist. --92.227.212.29 21:34, 12. Sep. 2014 (CEST)
Die gute Nachricht ist: Es gibt sichere Verschlüsselungsverfahren.
Die schlechte Nachricht ist: Sie sind in der Praxis völlig untauglich. --Snevern 23:10, 12. Sep. 2014 (CEST)
Sichere gibt's schon in praxistauglich, nur perfekte halt nicht.
@92.227.212.29: Mein erster Ansatz wäre immer etablierte Verfahren in etablierter Software, bei mir GnuPG. Spricht da was dagegen?
--Eike (Diskussion) 15:07, 13. Sep. 2014 (CEST)
"Sicher" ist nach deiner Interpretation relativ; ich meinte mit "sicher" das, was du als "perfekt" bezeichnest.
Aus meiner Sicht spricht nichts gegen etablierte Programme wie GnuPG, für die meisten Fälle dürfte das völlig ausreichend sein. --Snevern 19:29, 13. Sep. 2014 (CEST)

Sorbischsprachiges Wahlplakat

Wäre ganz interessant, womit die AfD gezielt Sorbischsprachige ködern will - die Information suchte ich auf der Bildbeschreibungsseite vergeblich.--Vinimontanus (Diskussion) 02:16, 12. Sep. 2014 (CEST)

Das heißt „Recht auf Heimat“. Das Sorbengesetz kennt ein prawo na narodnu a etnisku identitu (Recht auf völkische und ethnische Identität) und die NPD Sachsen wirbt mit dem Slogan „DOMIZNU ŠKITAĆ – HEIMAT SCHÜTZEN“. --Pp.paul.4 (Diskussion) 03:05, 12. Sep. 2014 (CEST)
Womit werben die anderen Parteien?--Geometretos (Diskussion) 11:05, 12. Sep. 2014 (CEST)
Die Linkspartei wirbt in Brandenburg (hier: Cottbus) mit folgendem Plakat: Minderheitenrechte stärken! Za pšawa Serbow!
--svolks (Diskussion) 21:39, 12. Sep. 2014 (CEST)
Aber eine ungünstige Silbentrennung haben die! X) --Eike (Diskussion) 23:25, 13. Sep. 2014 (CEST)

Gegenseitige Abhängigkeit EU/Gazprom

Die Tagesschau schreibt hier: „Es droht ein neuer Streit um russische Gaslieferungen. Diese als politische Waffe einzusetzen, hat lange Tradition.“ Angesichts der neuesten Sanktionen erscheint das nicht ausgeschlossen und manche munkeln schon, es könnte ein kalter Winter werden. Darum meine Frage: Wem würde es wohl mehr schaden, wenn Gazprom den Gashahn zudreht - der EU oder Rußland/Gazprom? --178.4.178.242 13:17, 12. Sep. 2014 (CEST)

Russland kann länger durchhalten als der Westen. --Optimum (Diskussion) 13:44, 12. Sep. 2014 (CEST)
So eindeutig lese ich das da nicht heraus. Wir können ja durchaus trotzen und massig Geld für überteuertes Gas von anderen Lieferanten zahlen, hastig unrentable Flüssiggasspeicher bauen usw. Die Frage ist, wann uns das wirtschaftlich in die Knie zwingen würde? --178.4.178.242 14:59, 12. Sep. 2014 (CEST)
Was meinst Du mit "wirtschaftlich in die Knie zwingen"? Der russische Anteil des in D verbrauchten Gases ist 39%, und diese Menge muss irgendwie ersetzt werden. Dazu könnten die anderen Zulieferer ihre Kapazitäten erhöhen, man könnte Großverbraucher, wie z.B. Gaskraftwerke abschalten und stattdessen Atomstrom aus Frankreich kaufen, vermehrt Biogas produzieren. Das wird natürlich Geld und Arbeitsplätze kosten, möglicherweise ist der höhere Gaspreis für manche gar nicht mehr bezahlbar. Wenn ausreichend viele Leute den Winter über in ihren Wohnungen gefroren haben, wird vielleicht im nächsten Jahr Fracking als Rettungstechnologie gefeiert. - Dass totalitäre Regime unbegrenzte Zeit mit Boykotten überleben können, sieht man an Kuba. Am Ende hat die ganze Sache sowohl Russland als auch Deutschland nur geschadet und der Ukraine nicht einmal geholfen. Man müsste mal herausfinden, wer denn an diesen Boykotten verdient. --Optimum (Diskussion) 16:40, 12. Sep. 2014 (CEST)
Damit hast Du Deine Frage an mich weitgehend selber beantwortet:) Wenn wir bis auf das wirtschaftliche Niveau von Kuba absinken würden, würde ich das schon als in die Knie gezwungen bezeichnen. Daß da jemand dran verdient, also im geldwerten Sinne, glaube ich gar nicht mal. Eine Verschwörung der Frackingfreaks (ich spinne da den Faden mal weiter) fände ich etwas zu weit hergeholt, wenngleich das ein Superthema für den nächsten Bondfilm wäre;) Für mich sieht das am ehesten nach einer Frage defizitärer Persönlichkeiten auf Seiten der Entscheidungsträger aus, die sich da mal wieder in etwas verrannt haben. Alte Feindbilder stecken einem halt noch lange in den Knochen. Mich interessieren tatsächlich mehr die Zusammenhänge auf der Ebene der VWL, was uns dieser Egotrip am Ende kosten wird, an welcher Stelle die Entscheidungsträger gezwungen wären einzulenken. --178.4.178.242 20:44, 12. Sep. 2014 (CEST)
Bei "Kuba" hatte ich eher an Russland gedacht, denn ich würde Deutschland eigentlich nicht als totalitäres Regime bezeichnen :). Außerdem ist ja Russland das boykottierte Land, nicht wir. Wir haben nur eine Zeit lang zu wenig Gas. Zum "Einlenken" wäre ein Abkommen zwischen Russland, dem Westen und der Ukraine über deren weitere Zukunft nötig. D allein wird den Boykott kaum beenden können. Wie sollte man das den USA erklären (die allerdings ein viel geringeres Handelsvolumen mit Russland haben)? Oder dem Baltikum und Polen (die aufgrund ihrer schlechten Erfahrungen schon jetzt die Luft anhalten)? Und Putin wird nicht einseitig einlenken, weil er dann als jemand dasteht, der dem Druck aus dem Ausland nachgibt. Wahrscheinlich ist jetzt Geheimdiplomatie gefragt. --Optimum (Diskussion) 22:30, 12. Sep. 2014 (CEST)
Narf, das ist ja alles schön und gut, führt aber völlig von meiner Frage wech. Mir geht's hier nur ums Geld, nicht um Politik, auch wenn man das aktuell kaum noch trennen kann … --178.4.178.242 23:24, 12. Sep. 2014 (CEST)
Wirklich beantworten kann es nur ein Fachmann aus der Branche, aber ich sage mal: Europa. Gazprom, ohnehin ein Staatsbetrieb, muss sich eben für eine Saison auf weniger Gewinn einstellen (und die die Sanktionen, die erfahrungsgemäß ohnehin nach zwei, spätestens drei Jahren wieder aufgehoben werden, einfach aussitzt), die europäischen Abnehmer müssten massiv gestiegene Gaspreise zahlen, was die Armen in (Ost)europa übermäßig betrifft. Nebenbei, bzgl. der Fracking-Geschichte: Ich kenne schon lange die Geschichte das die Anti-Fracking-Bewegung vom "KGB", "Gazprom", "Putin" unterstützt, weil die wiederum ihre Gewinne nicht nur kurzfristig, sondern dauerhaft gefährdet sehen.--Antemister (Diskussion) 23:43, 12. Sep. 2014 (CEST)

Ein paar Zahlen: Die EU hat ein Bruttoinlandsprodukt von rund 17 Bio. Dollar, Russland von 2 Bio. Auch wenn das nur eine der wichtigen Größen ist, dürfte Russland wesentlich empfindlicher sein. Obendrein ist das BIP pro Kopf in der EU ca. dreimal so hoch. In Russland wird es wohl wesentlich schneller wirklich eng werden. Allerdings ist es fraglich, ob Wirtschaftssanktionen generell die gewünschten Effekte zeigen. Die bisherigen Erfahrungen sprechen eher dagegen. Business as usual wäre derzeit aber auch fragwürdig. Rainer Z ... 14:43, 13. Sep. 2014 (CEST)

Ich tendiere auch zu Europa, wie Antemister, weil ich annehme, daß wir hier mehr eine Show abziehen und dann sobald die öffentliche Aufmerksamkeit nachläßt ganz schnell wieder Business as usual machen. Die Bevölkerung wird unter den Sanktionen leiden müssen - in Rußland vermutlich noch mehr als bei uns, aber die haben ja auch weniger Wahlmöglichkeiten als wir und sehen außerdem eher eine Notwendigkeit, sich gegen die wehrmachtsähnlichen Angriffe seitens der NATO zur Wehr zu setzen. Wir im Westen werden da sicherlich weit weniger Leidensbereitschaft zeigen. Aber teuer wird es zweifellos und ich wüßte gerne, wie teuer das wohl wird, denn zahlen muß erfahrungsgemäß immer die einfache Bevölkerung, hüben wie drüben. Daß das ganze der Frackingindustrie Aufwind verleihen wird, ist so eine Art Gratis-Arschtritt hinten drauf. Für die ist das wie ein vorgezogenes Weihnachten.
BTW: Wenn unsere Regierungsvertreter wirklich dazu beitragen wollen würden, den Konflikt zu lösen, würden sie Interessensvermittlung in den Vordergrund stellen, anstatt den Konflikt hoch zu puschen. --88.68.82.17 21:10, 13. Sep. 2014 (CEST)

Looping mit einen großraum-Passagierflugzeug

was würde eigentlich passieren, wenn man versuchen würde, mit einem modernen Großraumflugzeug, das für den Passagiertransport gebaut und eingerichtet ist, und nicht modifiziert wurde, einen Looping zu fliegen? Ich gehe zwar davon aus, dass das auf alle Fälle keine Gute Idee ist, aber es würde mich trotzdem interessieren, was passieren würde.--134.91.145.99 17:18, 12. Sep. 2014 (CEST)

Kommt das nicht drauf an, wie groß der Looping ist? Je weiter der Durchmesser, desto geringer dürften (zumindest nach meinem Physikverständnis), die Kräfte sein, die auf das Flugzeug wirken. Übrigens hat die Lufthansa Loopings verboten ;) XenonX3 – () 17:24, 12. Sep. 2014 (CEST)
Stimmt. Andererseits gillt aber auch: Je größer der Looping desto schneller kommt die Erde... ;-) --88.130.88.58 17:31, 12. Sep. 2014 (CEST)
Netter Scherzartikel, Lufthansa verbietet Piloten von Passagierflugzeugen, Loopings zu fliegen wäre etwas für den 1. April Ich kennen kein Großraum-Passagierflugzeug welches ein Kunstflugzulassung hat. --Raymondpilot (Diskussion) 17:37, 12. Sep. 2014 (CEST)
Loopings kann man aber auch ohne Kunstflugzulassung fliegen. Ob man das sollte, steht auf einem anderen Blatt. In einer Piper PA-34 saß ich schon beim Looping und das ist ganz sicher kein Kunstflugzeug. Aber auch keine Großraum-Passagiermaschine.... --Pölkky 17:45, 12. Sep. 2014 (CEST)
Mit großen Passagierflugzeugen lassen sich Parabelflüge durfchführen (Kotzbomber). Da sollten Loopings kein Problem sein. Beim Microsoft Flight Simulator bin ich früher immer mit dem Learjet schnell in große Höhen gestiegen, hab dann mitten im Flug auf die Sopwith Camel umgeschaltet und bin dann munter Loopings geflogen. --Rôtkæppchen₆₈ 17:53, 12. Sep. 2014 (CEST)
schon der Versuch einen Looping zufliegen unterbinden alleine die Bordcomputer in den modernen Großraum-Passagiermaschine, Du wirst es auch nicht ohne Kunstflugzulassung schaffen! --Raymondpilot (Diskussion) 17:49, 12. Sep. 2014 (CEST)
Die strukturelle Belastung würde wohl auch eine Boeing 747 aushalten. Kritischer Punkt ist jedoch die zu Beginn des Loops notwendige Mindestgeschwindigkeit für ein Flugzeuge dieses Gewichts (s. hier). Gruß --Quezon Diskussion 18:04, 12. Sep. 2014 (CEST)
(BK)Klar die Elektronik wird da rein pfushen, und schon so denn Looping verhindern. Zumindest bei der Elektronik wirst du nicht um ein Eingriff herum kommen, damit du einen bestimmten Steigwinkel überschreiten kannst. Von der technischen Sicht müsste es eigentlich gehen. Die Flugzelle sollte die Kräfte aushalten, und das Flugzeug sollte auch die notwendige Geschwindigkeit erreichen. Bei einem Looping entstehen die grössten Kräfte unten beim einleiten und beim beenden, oder andersherum da wo man ein Flugzeug auch aus einem steilen Sinkflug abfängt (Ist allerdings nicht wirklich gut für die Struktur wenn man es zu oft macht). Und der sogenannte Notabstieg muss ja mit einem grossen Verkehrsflugzeug bei Druckabfall in der Kabine gemacht werden können. Ich persönlich würde das jetzt unter der Kategorie möglich aber nicht ratsam einordnen.--Bobo11 (Diskussion) 18:10, 12. Sep. 2014 (CEST)
Flugzeuge, die für den Kunstflug zugelassen sind, haben Besonderheiten was die Versorgung mit Betriebsstoffen angeht. Alleine die Treibstoffpumpen würden bei einem Großraumflugzeug trocken laufen, wenn die Schwerkraft den Treibstoff nicht mehr zu den Pumpen befördert. Das gilt auch für die Schmiermittelpumpen und die hydraulische Anlage. --93.132.132.121 19:17, 12. Sep. 2014 (CEST)
Bei einem richtig geflogen "normalen" sprich Innenlooping gibt es aber keine negativen G's. Das die negativen G's supotimal sind, wenn das Flugzeug nicht darauf eingestellt ist bestreitet niemand. Blöd ist es einfach dann wenn du es versucht der Schwung aber nicht reicht, dann hast du ein wirkliches Problem wenn beim Flugzeug der Antrieb keine negativer Beschleunigung verträgt. Unkontrolierte Fluglage plus ausgefallener Motor fällt glaube ich unter worst-case Szenario. Aber wenn ich nur Positive G's habe, ist das mit der Technik nicht ganz so problematisch. --Bobo11 (Diskussion) 20:19, 12. Sep. 2014 (CEST)
Wer wills mal ausprobieren, ob die Flügel dranbleiben? --Giftzwerg 88 (Diskussion) 20:57, 12. Sep. 2014 (CEST)
Der Hacken an der Sache ist ja der, dass die Flieger nicht gerade billig sind, um damit einmal einfach so mal einen Looping zu fliegen. Ohne gewisse Modifikation -oder zumindest Tests- würde ich das jedenfalls nicht machen (was natürlich der Anfangsfrage wieder sprechen würde). Ein genügend verrückten Testpiloten müsst man dann natürlich auch noch finden. Erst wenn dieser Test mit einem modifizierten Flugzeug gelungen wäre, würde ich es mit einem Serienflugzeug versuchen. Irgend wie hängt man doch an seinem Leben, und so ein Flugzeug hat ja in der Regel keinen Schleudersitz. Und ich vermute mal, dass zumindest eine Schnellaustiegsmöglichkeit (wie sie auch bei den Erstflügen grosser Jets üblich sind) zu den gewünschten Modifikationen des Testpiloten gehören würde. --Bobo11 (Diskussion) 21:35, 12. Sep. 2014 (CEST)
Das meine ich aber auch. Die großen Flieger sind dafür gedacht wie Kreuzfahrtschiffe schön gerade aus zu fahren ohne spektakuläre Manöver. Man steigt damit auf, fliegt sanfte Kurven, die meiste Zeit fliegt man gerade aus und wenn es mal zu einem Luftloch kommt halten Sie das auch aus. Mit einer 60 Meter langer Röhre mit gleicher Spannweite einen Looping zu fliegen ist sicherlich keine gute Idee. Schon gar nicht wenn der Flieger mindestens 250 km/h drauf haben muss um überhaupt sicher in der Luft bleiben zu können. Die Hersteller veröffentlichen sehr detaillierte Listen zu den Flugzeugen ab welchen Geschwindikeiten pro Höhe und Steigwinkel das Flugzeug stabil bleibt. Ich würde darauf wetten das ein Looping zu den Manövern zählt in dem ein Airliner schlichtweg in seine Einzelteile zerfällt. --92.227.212.29 21:58, 12. Sep. 2014 (CEST)
Meines Wissens sind die tragenden Teile eines Flugzeuges (Rumpf, Flügel, etc.) 8-fach überdimensioniert. Sprich, man nimmt den ungünstigsten Fall des Normalbetriebs im weitesten Sinne (evtl. Notlandung) schaut wie starkt die Teile sei müssen und baut sie so dass sie 8-mal mehr aushalten. Zum Vergleich: diese Sicherheitsfaktoren bewegen sich je nach Anwendung normalerweise zwischen 1,5 und 3 oder 4. Er wird übrigens von der Versicherung des Herstellers gefordert, nicht vom Konstrukteur. Ich habe auch mal von einem Fall gehört, bei der ein Flugzeug mehrere Kilometer tief gefallen ist bevor es wieder in eine kontrollierte Fluglage kam und dabei einem vielfachen der laut Handbuch zulässigen Belastung ausgesetzt war. Die Piloten selbst waren überrascht, dass ihr Flugzeug das überstanden hat. Die Struktur sollte das also aushalten, ich weis aber nicht ob z.B. die Schubkraft ausreicht. --Der-Wir-Ing (Diskussion) 22:22, 12. Sep. 2014 (CEST)
[[41]] Der erwähnte beinahe-Absturz. --Der-Wir-Ing (Diskussion) 22:27, 12. Sep. 2014 (CEST)
Mal eine grobe Abschätzung: Flugzeug fliegt vorher 900km/h, soll im höchsten Punkt noch 300km/h haben und der Antrieb ist recht schwach gegenüber der Gewichtskraft des Flugzeugs, sodass ich ihn hier vernachlässige. Damit kann der Looping maximal etwa 3km hoch sein. Bei einer (unsinnigen) perfekten Kreisform gäbe das unten eine Kraft von 5g und oben -0.5g, bei einer besseren Form lässt sich die maximale Belastung aber auf 2-3g reduzieren (2g ist typisch für die Parabelflüge). Dann muss das Flugzeug nur noch oben schnell genug im Kreis fliegen können - bei 450km/h weniger als 3km Kurvenradius. Ist das realistisch? --mfb (Diskussion) 23:06, 12. Sep. 2014 (CEST)
JA, das würde ich sagen. Der Looping-Durchmesser muss oben keine 3 Kilometer sein. In der Höhe wird man die 3 Kilometer die du ausgerechnet hast vermutlich schon ausreizen müssen. Der Punkt ist natürlich der, dass man so ein Manöver nicht mit vollgetanktem Flugzeug machen sollte. Denn wenn man das Manöver mit leerem Flugzeug und fast leeren Tanks macht, dann kann man schon mal über die Hälfte des Gewichtes einsparen. Auch die strukturellen Belastung, nehmen dann rapide ab. Bzw. ich hab dann eben Reserven die ich mit benutzen kann. Es sind zwar nicht die doppelte G's die ein Bauteil aushält wenn ich nur noch das halbe Gewicht habe, die es dann aus hält aber es macht eben schon was aus wenn der Flügel um einiges leichter ist. Vom besseren Beschleunigunsverhalten reden wir besser nicht. Denn ich sehe das zu lösende Problem eher beim Gewicht und der Treibwerksleistung (und ggf. beim Treibwerk selber), und nicht bei der strukturellen Integrität des Flugzeuges. Das eben die Modifikation heisst „ich brauch mehr Power“, und zwar im Looping, damit mir die Geschwindigkeit beim Hochfliegen nicht zu fest abfällt.--Bobo11 (Diskussion) 23:34, 12. Sep. 2014 (CEST)
Die (http://www.airbus.com/tools/search/?tx_indexedsearch[sword]=looping&tx_indexedsearch[submit_button]=Go) Suche bei airbus.com nach "looping" ergibt "no results found". Die Kontakdaten für das Airbus Press Department sind: 1, Rond Point Maurice Bellonte, 31707 Blagnac Cedex France, Hotline +33 (0)5 61 93 1000 und media@airbus.com. (Falls jemand Lust hat, einfach mal nachzufragen...) --212.184.138.49 23:45, 12. Sep. 2014 (CEST)
Ein Testpilot ist mit einer Boeing Dash 80 Fassrollen [42] geflogen. Einen Looping hat vermutlich noch niemand versucht...im X-Plane funktionierts.;) --Advanceddeepspacepropeller (Diskussion) 09:44, 14. Sep. 2014 (CEST)
Die Fassrolle ist auch mit weniger Risiko behaftet. Die Fluglage ist um einiges besser beherrschbar. Das ist eigentlich das Problem beim normalen Looping dass wenn die Anfangsgeschwindigkeit falsch eingeschätzt wurde, und du zu wenig Speed hast, bleibt dir das Flugzeug vor erreiche des höchsten Punkt "stehen". In der Folge hat man dann auch keine positiven G's mehr. Da sehe ich das Problem -das man die Anfangs-Geschwindigkeit richtig einschätzen bzw. ausrechnen kann-, nicht ob das Verkehrsflugzeug die Kräfte aushalten kann, die bei einem normalen Looping entstehen. Aber generell wird die Elektronik denn Looping verhindern wollen, von daher ohne Modifikationen bzw. abschalten gewisser Teile der Fluglage-Überwachung wird das nicht gehen. --Bobo11 (Diskussion) 10:03, 14. Sep. 2014 (CEST)

Eigentumsrecht am Grund unterirdisch

Im Artikel Luftraum steht seit 25. Oktober 2008, 11:59 Uhr:

"Theoretisch reicht ein privates Grundstück unendlich in die Höhe und in die Tiefe bis zum Erdmittelpunkt."

Welche "Theorie" ist hier gemeint? Reicht auch das Gebiet eines Staates und reicht auch das Gebiet einer Gemeinde bis zum Erdmittelpunkt? Es geht bei den Fragen nicht um Nutzungsrechte z.B. an Bodenschätzen, oder am Grundwasser, sondern um das bloße Eigentumsrecht, Rosenkohl (Diskussion) 23:24, 12. Sep. 2014 (CEST)

Etwaige Nutzungsrechte lassen sich (nach deutschem Recht) von der Eigentumsfrage nicht trennen. "Theoretisch" bezieht sich drauf, dass der zu einem Grundstück gehörende Bereich als ein vom Erdmittelpunkt ausgehender Kegel mit theoretisch unendlicher Höhe gesehen wird. Der Weltraumvertrag grenzt diesen Bereich insofern ein, als dass der Weltraum von vornherein ausgenommen ist (obwohl der Staat ihn theoretisch schon nutzen könnte, siehe auch Staatsgebiet#Umfang_des_Staatsgebiets). So kann der Staat an dem Kegelstumpf, der im Weltraum liegt, auch keinem Privaten Eigentum verschaffen, selbst wenn er wollte. Der Grundstückseigentümer kann nun Beeinträchtigungen seines Eigentums verbieten. Das allerdings insofern nicht, wie er nach allgemeiner Anschauung kein Interesse daran hat, sie zu verbieten. Und da kommen die Bodenschätze ins Spiel: Nach unten sind daher Erdöl, Kohle usw. ausgenommen, nach oben kann er z.B. nichts gegen Flugzeuge, Satelliten udgl. tun. Für diese Überlegungen ist es egal, wer Eigentümer eines Grundstücks ist: Ob das ein Privater oder in irgendeiner Form der Staat selbst ist (z.B. eine Kommune), spielt dafür keine Rolle. --88.130.88.58 23:47, 12. Sep. 2014 (CEST)
In der Regel besitzt man nur ein Grundstück, also ein Stück Erdoberfläche. Ganz böse gesagt, ohne das darunter und das darüber. Wie viel vom Drunter und Drüber vom Besitzer benutzbar sind, ist in der Regel in der Bau- bzw. Zonenordnung festgelegt. Die Einflusssphäre des Staates reicht so gesehen schon Kegelförmig bis zum Erdmittelpunkt. Das hat aber nichts mit Eigentum zu tun, da sich das Eigentum auf die Erdoberfläche begrenzt. Alles andere sind "nur" Nutzungsrechte. --Bobo11 (Diskussion) 23:58, 12. Sep. 2014 (CEST)
Wenn "Eigentum" sich tatsächlich nur auf die Erdoberfläche und nichts anderes beschränken würde, dann dürftest du als Eigentümer noch nicht mal eine kleine Blume pflanzen: Die Wurzeln gehen ins Erdreich - unterhalb der Erdoberfläche - und die Pflanze selbst ist an der Luft - oberhalb der Erdoberfläche. Ohne eine irgendwie geartete Nutzungsmöglichkeit ist ein Grundstück nicht viel wert. --88.130.88.58 00:06, 13. Sep. 2014 (CEST)
(Einsschieb) „Ein Baum pflanzen“ ist aber Nutzung. Und das Nutzen als landwirtschaftliche Fläche ist eigentlich eines der üblichsten Nutzungsrechte bei einem Grundstück. Aber es gibt tatsächlich Grundstücke wo vom Gesetzgeber her verboten ist ein Baum zu pflanzen (Wenn die Wiese zum Beispiel zwischen Schützenhaus und Scheibenstand liegt). Dann ist das Grundstück eben mit einem andern Nutzungrecht belegt, dass ein Pflanzen von Bäumen verbietet. Das ist ja der Punkt der gerne Vergessen wird. Eigentum ist unabhängig von denn Nutzungsrecht. Und nicht jedes Nutzungsrecht ist schriftlich festgelegt.--Bobo11 (Diskussion) 10:10, 13. Sep. 2014 (CEST)
Was genau Eigentum umfasst, weiß kein Mensch. Schon bei den Römern war nicht klar, wo genau die Befugnis endet, die man als Eigentümer des fundus erlangt. Im Römischen Recht war es z.B. so, dass der "Himmel" frei bleiben sollte und daher nicht mit erfasst war. Letztlich ist die Frage, wo Nutzung (die eingeschränkt bzw. verboten werden kann) anfängt und wo Eigentum selbst aufhört so alt wie die Zeit. Eine definitive Antwort haben die Römer nicht gefunden und in den tausenden Jahren darauf auch sonst niemand. Da können wir jetzt lange philosophieren; das Dilemma werden wir nicht mal eben in 5 Minuten lösen können. ;-) --88.130.95.148 10:41, 13. Sep. 2014 (CEST)

Besser? --88.130.88.58 00:33, 13. Sep. 2014 (CEST)

Es wird sofort spassig, wenns um die Nutzung von Grundwasser und Bodenschätzen geht.--Giftzwerg 88 (Diskussion) 04:43, 13. Sep. 2014 (CEST)
Yepp, man sollte das auch im Kontext mit dem Bergregal und dem Bergrecht und der Problematik von Gewinnbeteiligungen und Setzungsschäden begreifen. Hier ein aktueller (= 2006) Kommentar (auch wenn es da um Tagebau geht). --212.184.141.109 09:51, 13. Sep. 2014 (CEST)
Um unterirdische Einschränkungen geht es in dem Artikel (Luftraum!) ja gerade nicht. Ich freu mich, dass mein Vorschlag bzgl. überirdischer Einschränkungen scheinbar ok ist. --88.130.95.148 10:41, 13. Sep. 2014 (CEST)
Ähm, Giftzwerg 88 antwortet auf Bobo11 (23:58, 12. Sep. 2014) und 212.184.141.109 auf Giftzwerg 88. Zu dem, was du schreibst äußert sich da niemand. --77.12.1.233 11:39, 13. Sep. 2014 (CEST)
Ach so, ich dachte es ginge darum, die von Rosenkohl gestellte Wissensfrage zu beantworten. --88.130.95.148 12:32, 13. Sep. 2014 (CEST)

Die Frage, welche Theorie gemeint sei bezieht sich darauf, welches Gesetz, welcher Vertrag oder welche andere Quelle mit diese "Theorie" gemeint ist. Zudem ist ein Grundstück z.B. auch ein Wertgegenstand, der gekauft und verkauft werden kann; daher wäre interessant, wie weit ins Erdinnere sich das Grundstück erstreckt, was dort gehandelt wird. Zudem betrifft es z.B. auch geographische Angaben, z.B. wenn ein unterirdisches Erdbeben stattfindet und man sagen möchte, in welcher Kommune sich das Epizentrum befindet, Rosenkohl (Diskussion) 10:38, 13. Sep. 2014 (CEST)

Für Deutschland ist das z.B. § 905 BGB mit der zugehörigen Kommentierung. Wer als erster Bereiche des Erdbodens in Kegelstümpfe eingeteilt hat, ist eine interessante Frage. Denklogisch setzt das ja erstmal (nur) voraus, dass man schon wusste, dass die Erde nicht platt, sondern eine Kugel ist. Ich schätze, danach kommt man relativ schnell auf die Idee... --88.130.95.148 10:47, 13. Sep. 2014 (CEST)
Die Kugelform der Erde ist seit der Antike bekannt und die Vorstellung, im Mittelalter hätte man geglaubt, die Erde sei eine Scheibe, ist eine Fiktion des 19. Jahrhunderts. --Rôtkæppchen₆₈ 10:53, 13. Sep. 2014 (CEST)
(BK)Die Frage betreffend geographische Angabe ist einfach zu beantworten. Die "Grenzen" verlaufen Senkrecht in Richtung Erdmittelpunkt. Das ist ja genau der Grund warum man mit Winkelangabe arbeitet. Denn bei der Angabe 6 Grad 15 Minuten 10 Sekunden östlicher Länge und 50 Grad 30 Minuten 23 Sekunden Nördliche Breite ist es so was von egal, in welcher Höhe sich der Punkt befindet. Der Punkt kann einem Gemeinde/Staat zugewiesen werden.--Bobo11 (Diskussion) 10:53, 13. Sep. 2014 (CEST)
Um die Frage zu beantworten, ist zunächst mal zu klären auf welches Land es sich bezieht. In der Schweiz haben Grundstücksbesitzer beispielsweise mehr Rechte als in Frankreich. Das war beim Bau des LEP wichtig, der teilweise unter der Schweiz, teilweise unter Frankreich verläuft. In Frankreich kein Problem, in der Schweiz musste das (im Prinzip) mit jedem Eigentümer ausgehandelt werden. --mfb (Diskussion) 18:17, 13. Sep. 2014 (CEST)
Ich sehe schon Nachbarschaftsstreitigkeiten ungeahnten Ausmaßes. Am Erdkern: »Das ist mein Eisen!« – »Nein, das ist meins!« Im Weltall: »Nehmen sie den Jupiter aus meinem Grundstück! – »Passen Sie erst mal auf Ihren Satelliten auf!« Rainer Z ... 19:41, 13. Sep. 2014 (CEST)
Zum Glück gibt's den Weltraumvertrag. --88.130.95.148 20:36, 13. Sep. 2014 (CEST)
Das »theoretisch« könnte auch einfach darauf verweisen, dass es praktisch gar nicht möglich ist, sein Eigentumsrecht bis zum Erdmittelpunkt und entgegengesetzt bis in die unendlichen Weiten auszuüben. Man kommt einfach nicht hin und dran. Rainer Z ... 13:58, 13. Sep. 2014 (CEST)
Da ist es ganz nett beschrieben.--Advanceddeepspacepropeller (Diskussion) 16:29, 13. Sep. 2014 (CEST)
Praktische hat diese Frage ab einer gewissen Höhe oder Tiefe keine Bedeutung mehr, da man - wie Rainer Z schon geschrieben hat - das Eigentumsrecht nicht unendlich weit ausüben kann.
Aber selbst in der Theorie ist die Abgrenzung nicht so einfach, wie hier mehrfach mit Hinweis auf den Erdmittelpunkt erläutert wird. Auf der Erdoberfläche orientieren wir uns an der Lotrichtung. Mauern stehen dann senkrecht, wenn sie senkrecht auf einer Äquipotentialfläche stehen. Es liegt nahe, so eine senkrecht stehende Mauer nach oben und unten zu verlängern, um das Grundstückseigentum zur Seite abzugrenzen. (Würde man eine Gerade durch einem Fußpunkt der Mauer und den Erdmittelpunkt legen, liegt die Maueroberkante i.a. nicht auf dieser Geraden, d.h. die Maueroberkante würde auf das Nachbargrundstück ragen oder einen gewissen Grenzabstand haben.) Da die Erde im Vergleich zu einer Kugel abgeplattet ist, geht die Verlängerung der Lotlinien nach unten i.a. jedoch nicht durch den Erdmittelpunkt.
Nun könnte man sagen, dann reicht das Eigentum eben soweit, bis sich die verlängerten Lotlinien schneiden. Da sich der Krümmungsradius der Erde (bei Annahme eines Rotationsellipsoids) vom Äquator zu den Polen vergrößert, ist im Bereich um die Pole der Abstand der Schnittpunkte von der Erdoberfläche größer als der Erdradius. Das hieße, daß sich die Eigentumsbereiche von Nord- und Südhalbkugel im Bereich des Erdmittelpunktes überlappen. Noch schwieriger wird es, wenn man nicht ein gleichmäßiges Rotationsellipsoid (davon gibt es ja auch mehrere) zugrundlegt, sondern auch die Lotabweichung berücksichtigt.
Man könnte natürlich auch überlegen, das Eigentum an den Lotlinien zu begrenzen. Das scheitert aber an der praktischen Unmöglichkeit im Erdinnern. Nicht nur die Ausübung des Eigentumrechts, sondern auch die Abgrenzung bleibt eine theoretische Überlegung... 217.230.103.56 17:45, 13. Sep. 2014 (CEST)

Was 217.230.103.56 hier so detailliert beschreibt ist das Dilemma aus Sicht eines Geographen. Schließlich könnten wir jetzt noch aus Sicht eines Mathematikers ausrechnen, welchen Unterschied diese Methoden für die nutzbare Größe des Eigentums haben: Der Erdradius ist etwa 6371 Kilometer und ein hohes Gebäude kann sagen wir mal 500m hoch sein. Wenn ich jetzt dieses Gebäude senkrecht hochmaure, dann ragt es senkrecht vom Erdboden aus 500m in die Höhe. Würde ich dagegen der Kegelkante folgen, dann würde es etwas mehr nach außen gehen, heißt, nach 500m Höhe wäre ich ein Stück weiter "drüben". Die Frage ist: Wie weit?

Ich schätze, das kann man berechnen, wenn man im Dreieck Bodenpunkt Mauer, Höchstpunkt senkrechte Mauer, Höchstpunkt Kegellinien-Mauer den Winkel kennt, der unten am Boden zwischen den beiden Mauern besteht. Wie kann man diesen Winkel ermitteln? --88.130.95.148 18:14, 13. Sep. 2014 (CEST)

<Quetsch> Das macht bis zu etwa 0,2° aus, d.h. auf 500 m etwa 1,7 m, auf 20 m schon 7 cm. So etwas kann man schon messen. Aber Geographen beschäftigen sich wohl weniger mit solchen Details. 217.230.103.56 18:39, 13. Sep. 2014 (CEST)
Für die Aussage müssen wir die Grundstücksbreite kennen, dann ist es einfacher Dreisatz. Bei 1km Breite (was sicher nicht ein einziger Wolkenkratzer wird) haben wir 1/6370, bei 500m Höhe also nur etwa 10cm. --mfb (Diskussion) 18:43, 13. Sep. 2014 (CEST)
Nein, mfb, hier geht es nicht um höhenabhänge Streckenreduktionen, sondern um den Winkel zwischen der Geraden Oberflächenpunkt-Erdmittelpunkt und der Ellipsoidnormalen. Der ist breitenabhängig, aber nicht abhängig von der Grundstücksgröße. Ungeprüfte Näherungsformel: 0,2° * sin(2*Breite). 217.230.103.56 18:51, 13. Sep. 2014 (CEST)
Du sagst, der Winkel sei 0,2°. Wie genau kommst du dadrauf? Das ist mir irgendwie nicht klar. --88.130.95.148 20:37, 13. Sep. 2014 (CEST)
Hier ist ein Rechenbeispiel, in dem die benötigten Größen vorkommen. Der Winkel ist die Differenz zwischen (Breite) und (Winkel zwischen Äqutorebene und Gerade Erdmittelpunkt-Oberflächenpunkt). In dem Beispiel beträgt die Differenz bei etwa 49° Breite rund 0,2°. Am Äquator und an den Polen ist die Differenz 0°, bei etwa 45° hat sie den größten Betrag. 217.230.67.181 10:27, 14. Sep. 2014 (CEST)
Komme für eine Ellipse mit Abplattung f auf eine ähnliche Größenordnung für den ungefähren Winkel zwischen Bogennormaler und Strecke zum Mittelpunkt: f * 360°/(2*pi) * sin(2*Breite); insbesondere für die Erdabplattung von f=1/298,256 ergibt sich ein Winkel von 0,1921° * sin(2*Breite).
Zu beachten ist allerdings, daß die Lotlinien keine Geraden sind, sondern gekrümmt sind. Somit wird eine auf der abgeplatteten Erde entlang der Lotrichtung gemauerte Mauer an keinem Punkt gerade, sondern überall leicht gebogen. Würde man genügend weit in den Weltraum hochmauern, müßte man sich zunehmend schließlich der Fliehkraft anpassen und annähernd parallel zur Äquatorebene weitermauern. Die Erde als Rotationsellipsoid angenommen geht zwar die lineare Verlängerung einer Lotlinie von der Erdoberfläche nach unten i.a. nicht durch den Erdmittelpunkt; die Lotlinien selbst treffen sich aufgrund ihrer Krümmung jedoch durchaus im Erdmittelpunkt, Rosenkohl (Diskussion) 11:26, 14. Sep. 2014 (CEST)
Darum, daß es wenig sinnvoll ist, einen (geometrisch definierten) Kegel mit der Grundfläche des Grundstücks und dem Erdmittelpunkt als Spitze zu betrachteten, geht es ja hier. Und gekrümmte Lotlinien kann man im Erdinnern nur bis zu einer gewissen Tiefe tatsächlich messen und theoretisch nicht exakt beschreiben, da dafür die Massenverteilung innerhalb der Erde bekannt sein muß.
Nach oben hast Du den interessanten Aspekt der Fliehkraft für erdgebundene Massen angesprochen. Eine erdgebundene, an einer Lotlinie entlang gebaute Mauer müßte ab einer gewissen Höhe (35.800 km am Äquator, s. Geostationärer Satellit) mit eigener Kraft festgehalten werden, da die Fliehkraft größer als die Gravitation wird. Aber schon in weit tiefer liegenden Atmospärenschichten ist es wegen des Luftwiederstandes nicht möglich, Mauern bis in großen Höhen zu bauen. Der tatsächliche Einfluß hoch über und tief unter der Erdoberfläche ist also physikalisch begrenzt. 217.230.67.181 12:46, 14. Sep. 2014 (CEST)

Deutsche Behörden beschneiden (durch gaaaaaaaanz vieeeeeeeele Gesetze und Verordnungen bestärkt) in unzähligen Bereichen die Rechte eines Grundstückseigentümers unterhalb und oberhalb der Erdoberfläche. Beispiele gefällig, was erforderlich ist bzw. sein könnte?

  • Baum pflanzen: Aufforstungserlaubnis
  • Baum mit Wurzeln entfernen: Rodungserlaubnis
  • Größere Grube ausheben: Abgrabungserlaubnis
  • Größere Grube auffüllen: Na klar: Auffüllungserlaubnis
  • Gebäude errichten? Weiß jeder: Baugenehmigung
  • Quelle fassen / Brunnen graben: Wasserentnahmeerlaubnis
  • Wasser in Bach einleiten: Ergibt sich von selbst: Einleitungserlaubnis
  • Ein Grundstück außerhalb eines Bebauungsplanes einzäunen: Einfriedungsgenehmigung
  • Schacht graben: Muss natürlich genehmigt werden, den Namen kenn ich leider nicht
  • Flugdrachen höher als 100 Meter fliegen lassen: Weiß ich nicht, ist aber flugtechnisch im Sinne derjenigen geregelt, die über das Privateigentum fliegen wollen und somit dürfen.
  • Und hunderte oder tausende weitere Einschränkungen könnten folgen...

@Rainer Z: »Natürlich ist das mein Eisen!«, »Jupiter gehört nicht auf mein Grundstück!« und natürlich »Satelliten weg im Luft- und Weltraum über mir!« Ich hänge jetzt noch schief, so habe ich gelacht. Danke! (Fast) ernst gemeint von --46.78.27.108 21:27, 13. Sep. 2014 (CEST)

Wieso braucht man für einen internationale Führerschein

(ein richtig unmoderner Lappen) ein biometrisches Paßfoto z.B. http://www.stadt-koeln.de/service/produkt/fuehrerschein-internationaler-fuehrerschein ? --93.134.130.224 06:07, 13. Sep. 2014 (CEST)

Frage dort. --178.194.131.35 06:52, 13. Sep. 2014 (CEST)
Haben denn biometrisches Fotos irgendwas mit der Frage Lappen oder Plastikkarte zu tun? --Eike (Diskussion) 08:00, 13. Sep. 2014 (CEST)

Vielleicht steht ja was im Artikel Internationaler Führerschein? Ansonsten würde ich sagen, gilt der IF ja immer nur in Verbindung mit dem normalen Führerschein. Und damit man es den ausländischen Behörden einfacher macht, lässt sich ein Passbild halt besser vergleichen, als den Text (Name, Vorname). Gruß kandschwar (Diskussion) 08:35, 13. Sep. 2014 (CEST)

Man braucht das, weil es in § 25a Absatz 2 der Fahrerlaubnis-Verordnung so steht ("Dem Antrag sind ein Lichtbild, das den Bestim­mungen der Passverordnung entspricht, und der Führerschein beizufügen."). Und das steht dort, weil es mit der Vierten Verordnung zur Änderung der Fahrerlaubnis-Verordnung und anderer straßenverkehrsrechtlicher Vorschriften vom 18. Juli 2008 so eingefügt wurde. --Snevern 08:40, 13. Sep. 2014 (CEST)
OK, die Frage: Warum reicht ein normales Paßbild nicht aus. Millionen von Leuten haben noch graue oder rosa Führerscheine, ein Vergleich mit dem anderen Paßbild ist damit hinfällig (Kandschwar). Das mit dem "unmodernen Lappen" habe ich erwähnt, weil das Teil nicht maschinenlesbar ist. --93.134.130.224 10:27, 13. Sep. 2014 (CEST)
Ich hatte einen rosa Führerschein und als ich den IF beantragte hieß es ich muss dann den in den Scheckkartenformat umwandeln lassen. Also geht aktuell nur die Kombi Scheckkartenführerschein und IF. Die alten "Lappen" gehen nicht. Aber wie Snevern geschrieben hat, gibt es da gesetzliche Grundlagen für. Gruß kandschwar (Diskussion) 11:18, 13. Sep. 2014 (CEST)
Ich kann Kandschwar bestätigen: Ich habe seit 9 Jahren alle 3 Jahre einen neuen Internationalen, der Scheckkartenführerschein war die Voraussetzung. Gruß Gruenschuh (Diskussion) 12:11, 13. Sep. 2014 (CEST)
Nachdem der Fragersteller nicht bei der Behörde fragen wollte, die das verlangt, haben wir jetzt die Antwort, dass es so im Gesetz steht. Bevor die Wikipedia jetzt auch noch beantworten soll, was sich die Abgeordneten dabei gedacht haben, sollte der Fragesteller den nächstbesten Abgeordneten fragen, was er sich dabei gedacht hat. --178.194.131.35 10:39, 13. Sep. 2014 (CEST)
Jedes Foto ist machinenlesbar. --Eike (Diskussion) 15:11, 13. Sep. 2014 (CEST)

Als ich 1991 von Spanien aus dienstlich längere Zeit in den USA war (El Paso), habe ich mir vorsorglich einen internationalen Führerschein besorgt. In Amerika war es dann aber so, daß mein DDR-Führerschein in den USA und auch Mexico anstandslos anerkannt wurde, den internationalen wollte niemend haben. Damals war es so, daß dich die DDR in Sachen Straßenverkehr aus irgendeinem Grund an der USA orientiert hat (Stopschild, Rechtsabbiegepfeil, Fahrerlaubnisklassen...). So war es den amerikanischen Beamten leicht möglich, anhand de Buchstaben die Klassen zu identifizieren. Die BRD hatte jedoch vollkommen abweichende Klassen, das hätte niemand verstanden, deshalb die internationalen Lappen. Unser Kollege aus Westdeutschland brauchte den dann auch. Ich habe meinen ersten internationalen Führerschein in Barcelona übrigens auf der Basis des deutschen Dokuments erhalten, also ein spanisches Dokument, was nur in Verbindung mit dem Dokument der nicht mehr existierenden DDR galt. --Pölkky 11:44, 13. Sep. 2014 (CEST)

Wie komme ich als Privatmann an die Amtliche Begründungen zu Gesetzes- und Verordnungswerken des Bundes (ohne Geld für den Bundesanzeiger ausgeben zu müssen)? Denn darin dürfte der Grund stehen, zumindest ein Verweis auf entspr. EU-Recht. --93.134.130.224 12:15, 13. Sep. 2014 (CEST)
Die Verordnung ist hier: http://www.bgbl.de/banzxaver/bgbl/start.xav?startbk=Bundesanzeiger_BGBl&jumpTo=bgbl108s1338.pdf aber ohne amtliche Begründung, weil Verordnung und nicht Gesetz. --178.194.131.35 12:42, 13. Sep. 2014 (CEST)
Aus der (amtlichen) Begündung:
"Mit Nr. 13 werden die bestehenden Anforderungen an die Beschaffenheit des beizubringenden Lichtbildes für den Führerschein den Anforderungen der Verordnung zur Bestimmmung der Muster von Reisepässen der Bundesrepublik Deutschland angeglichen. Die Anwendung dieser Vorschriften erleichtert die Beurteilung geeigneter Lichtbilder durch das Personal der Verkehrsbehörden und führt auch beim Bürger zu einer Verfahrenserleichterung. (...) Der Führerschein dient dem Inhaber als Nachweis über den Besitz der entsprechenden Fahrerlaubnis. Dies bezieht auch die Personendaten mit ein, da es keine Verpflichtung gibt, neben dem Führerschein einen Personalausweis oder Reisepass mitzuführen. Bei einer Kontrolle muss erkennbar sein, dass es sich bei dem Fahrzeugführer auch tatsächlich um den Inhaber der Fahrerlaubnis handelt. Hierzu ist ein aussagekräftiges Lichtbild unerlässlich. Zum Zwecke der weitgehenden Einheitlichkeit werden die Anforderungen zugrunde gelegt, die zur Sicherstellung der Biometrietauglichkeit von Passbildern entwickelt wurden. (...) Die Anwendung dieser Vorgaben aus den Pass- und Personalausweisvorschriften erleichtert die Beurteilung geeigneter Lichtbilder duch das Personal der Verkehrsbehörden erheblich. Fotografen und Bürger kennen die Anforderungen an Lichtbilder aus eben diesen Vorschriften. Diskussionen um Lichtbilder, die diesen Forderungen nicht genügen, werden mit der Gleichstellung der Vorschriften vermieden. (...) Durch die §§ 25a (...) werden die Vorschriften der IntKfzVO in die FeV übernommen. In diesem Zusammenhang wurden die Vorschriften zur Beschaffenheit des Lichtbildes angepasst." (VkBl. Heft 19 2008, S. 564 f., 568 f.)
"Nr. 13" der Verordnung bezieht sich auf die Einführung biometrischer Fotos für den nationalen Führerschein. IntKfzVO ist die Verordnung über internationalen Kraftfahrzeugverkehr, FeV die Fahrerlaubnis-Verordnung. --Snevern 18:51, 13. Sep. 2014 (CEST)
Beim nationalen Führerschein braucht man aber kein Passfoto, weil einfach das Foto vom Personalausweis genommen wird. Wieso man das nicht auch beim internationalen Führerschein machen kann, ist mir unbekannt. -- Liliana 18:55, 13. Sep. 2014 (CEST)
Von wann ist denn dein Führerschein, Liliana? Nach der aktuellen Fahrerlaubnis-Verordnung gilt folgendes:
"§ 21, Antrag auf Erteilung einer Fahrerlaubnis: (...) Dem Antrag sind folgende Unterlagen beizufügen: (...) ein Lichtbild, das den Bestimmungen der Passverordnung vom 19. Oktober 2007 (BGBl. I S. 2386) entspricht (...)."
Die FeV enthält übrigens dem bemerkenswerten Zusatz "Die Fahrerlaubnisbehörde kann Ausnahmen von der in Satz 1 Nr. 2 vorgeschriebenen Gestaltung des Lichtbildes zulassen." - was allerdings nur für den nationalen, aber nicht für den Internationalen Führerschein gilt.
Ich selbst habe meinen grauen Lappen erst kürzlich gegen eine Scheckkarte getauscht und dabei das Foto verwendet, das auch für meinen Pass und für meinen Ausweis verwendet wurde - aber die Behörde hat sich das nicht selbst aus der Akte gezogen, sondern ich habe es dem Antrag beigefügt (und danach wieder zurückbekommen). --Snevern 19:22, 13. Sep. 2014 (CEST)
Das war bei mir zunächst 2009 so (Rollerführerschein) und danach 2011 (Erweiterung auf Autoführerschein). Beidesmal hat man auf das Foto vom Personalausweis zurückgegriffen, ich musste gar nichts machen und hab den Führerschein einfach in die Hand gedrückt bekommen. -- Liliana 19:48, 13. Sep. 2014 (CEST)
Praxisnah und sinnvoll - und wohl auch nicht verboten, obwohl es nicht den Vorgaben der Verordnung entspricht (wie oben zu lesen). --Snevern 19:55, 13. Sep. 2014 (CEST)
Danke an alle! Die amtlichen Begründungen erfährt man wohl in den Bekanntmachungen der Fachministerien, in diesem Fall das Verkehrsblatt. --93.134.130.224 21:26, 13. Sep. 2014 (CEST)

Innerschweizer Kulturpreis

Auf der Hauptseite zu Innerschweizer Kulturpreis wird im Jahr 1974 nur Herr Leo Lienert als Preisträger geführt. 1974 hat auch die Gesellschaft für Geistliche Spiele, Welttheatergesellschaft Einsiedeln den Innerschweizer Kulturpreis erhalten. Als Preisträger wird die Welttheatergesellschaft nur im Web Link (Preisträger 1953 - 1999 ) geführt, jedoch nicht auf der Hauptseite. Könnte man dies dort ergänzen. Besten Dank. Welttheatergesellschaft Einsiedeln

--62.202.6.178 13:58, 13. Sep. 2014 (CEST)

Das Wörtchen Auswahl hinter Preisträger ist schon aufgefallen? An sonsten Sei mutig.--Mauerquadrant (Diskussion) 16:26, 13. Sep. 2014 (CEST)
Auswahl ist an dieser Stelle aber unenzyklopädisch. --Eingangskontrolle (Diskussion) 16:37, 13. Sep. 2014 (CEST)
Der Artikelautor hat offensichtlich nur die Personen genommen und ausgezeichnete Organisationen weggelassen. --Mauerquadrant (Diskussion) 17:27, 13. Sep. 2014 (CEST)

Ich suche nach einem Kunstmaler der früheren DDR Namens: Ernst Rentsch, habe einige Bilder die von ihm gemalt wurden im Jahre

1938 bis 1944, Wahrscheindlich von Dresden gebürtig?

--Benutzer:85.0.216.80 (Diskussion) 16:33, 13. Sep. 2014 (CEST)

http://www.mv-naumburg.de/fritzrentsch ? Is sich nich Ernst, is sich Fritz, aber genau genommen Friedrich Ernst Rentsch, passt aber sonst.--Mautpreller (Diskussion) 19:14, 13. Sep. 2014 (CEST)

See also http://www.zeller.de/en/katalog/auction-82-oct-04/auktionsartikel/fritz-ernst-rentsch-1867-dresden-1946-naumburg/ --Mautpreller (Diskussion) 19:17, 13. Sep. 2014 (CEST)

Hotelzimmer "RIA" ?

Tag! Was bedeutet "RIA" bei der Beschreibung von Hotelzimmern? Beispiele: [43] --84.119.205.246 19:30, 13. Sep. 2014 (CEST)

Klingt nach Zimmerkategorisierung im dertour-Katalog oder sowo. Guckstu die Print- bzw. pdf-Ausgabe an, etwa so eine. Irgendwo im Kleingedruckten ist es meist zu erschließen. --Xocolatl (Diskussion) 19:47, 13. Sep. 2014 (CEST)
Kategorisierung kann mMn gut hinkommen. Wenn, dann findet man das eigentlich immer in Verbindung mit "Superior" als "Superior (RIA)". Außer "RIA" findet man auch noch "RIB", "RIC" und "OIM". Was jetzt wie gut ist, weiß ich nicht. --88.130.95.148 20:30, 13. Sep. 2014 (CEST)
*Rofl* Ich hab grade sogar "RIP" gefunden. --Xocolatl (Diskussion) 20:57, 13. Sep. 2014 (CEST)
"Unsere Rest-in-Piece-Suiten sind geräumige Ein-Mann-Appartements und liegen allesamt in einer Tiefe von angenehmen 2 Metern. Erdbedeckung und saisonale Bepflanzung gehören genau so zu unseren Standards wie musikalische Begleitung bei Ihrem Einzug." --88.130.75.128 12:34, 14. Sep. 2014 (CEST)
So einfach ist es offenbar nicht. Es geht tatsächlich um Kategorien für Zimmer und ich finde keine gescheiten Übersichten, vor allem auch nicht bei den Anbietern. Ich sehe, daß R offenbar für Room steht und A möglicherweise für All inclusive und ich lese folgende Kategorien (wobei unsicher bleibt, ob die allgemeingültig sind):
RI (Standard)
RI (Garden/Mountain View)
RIM (Ocean View)
RX (Oceanfront)
RC (Standard Breakfast)
RX1 (Partial Ocean View Breakfast)
RCM (Oceanfront Breakfast)
unklar: Deluxe (RJX), Vista (RJM).
--212.184.141.109 21:27, 13. Sep. 2014 (CEST) Nachtrag: Der von Xocolatl verlinkte Katalog schlüsselt z.B. nichts auf und weiß nur auf S. 337: "Die Unterscheidung in z. B. Deluxe, Superior, Standard richtet sich nicht ausschließlich nach Größe und Ausstattung, sie kann auch auf die Lage zurückzuführen sein" ----212.184.141.109 21:38, 13. Sep. 2014 (CEST)
Ich hatte immer gedacht, Größe und Ausstattung wären Wurscht? --Cookatoo.ergo.ZooM (Diskussion) 22:08, 13. Sep. 2014 (CEST)
R heißt nicht "Room", sondern steht für eine Zimmerkategorie, in dem verlinkten Katalog gibt es da auch noch andere. RIA habe ich z.B. auf Seite 46 gefunden. Der letzte Buchstabe steht in dem Zusammenhang wohl für die Lage an der Außenanlage. -- Janka (Diskussion) 22:34, 13. Sep. 2014 (CEST)
Ich glaub schon, dass R was mit Room zu tun hat. Suiten oder Apartments oder solche Sachen scheinen nämlich regelmäßig mit einem anderen Buchstaben anzufangen, meiner Erinnerung von gestern nacht nach z. B. mit W. Wenn ich die oben aufgeführte Liste anschaue, habe ich auch den Eindruck, dass der mittlere Buchstabe eine römische Zahl sein könnte (I, X, C), was auch wieder eine "Rangordnung" ergäbe. Als ich zuletzt mit einem Reisebüromenschen zu tun hatte, dieses Frühjahr, konnte der übrigens auch nicht alles aufschlüsseln, aber da ging's um andere Abkürzungen. Trotzdem sollte der Fragesteller vielleicht mal einen solchen kontaktieren. --Xocolatl (Diskussion) 11:12, 14. Sep. 2014 (CEST)
Auch ordinäre Zimmer fangen in dem verlinkten Katalog oft mit anderen Buchstaben an. "R" ist daher allenfalls eine bestimmte Art von Zimmer. -- Janka (Diskussion) 14:01, 14. Sep. 2014 (CEST)

türkisches Pfund 1915

Wieviel war das etwa wert, so real beschrieben? --Quetsch mich aus, ich bin ein Schreibsklave itu (Disk) 20:08, 13. Sep. 2014 (CEST)

Bis 1914 galt der Goldstandard, laut en:Ottoman lira 1 Goldlira = 6,61519 Gramm Feingold, entsprechend 18,4564 Goldmark. Im ersten Weltkrieg wurden dann Papierlira eingeführt. 1920 entsprachen neun Papierlira einer Goldlira. --Rôtkæppchen₆₈ 22:20, 13. Sep. 2014 (CEST)
Ok, danke, also wohl etwa die 6,6 gramm Gold. Hintergrund der Frage war übrigens das Gerichtsprotokoll zu Soghomon Tehlirian. --Quetsch mich aus, ich bin ein Schreibsklave itu (Disk) 23:32, 13. Sep. 2014 (CEST)
Ich lese grad dort "... türkische Pfund, ein Pfund entspricht ungefähr einem goldenen Zwanzigmarkstück. " (Berlin 1915) --Quetsch mich aus, ich bin ein Schreibsklave itu (Disk) 01:16, 14. Sep. 2014 (CEST)
In der im Artikel verlinkten englischen Fassung des Gerichtsprotokolls stehts auch so: “The amount was 4800 Turkish gold pounds; one Turkish pound is worth 20 gold marks.” --Rôtkæppchen₆₈ 09:26, 14. Sep. 2014 (CEST)
.Wobei die einfache Umrechnung auf Basis des Goldpreises heute, wo es schon lange keinen Goldstandard mehr gibt und Gold ein frei handelbares Gut ohne monetäre Bedeutung ist mit entsprechend schwankenden Preisen ist nicht wirklich sinnvoll ist. Besser wäre der Vergelich mit dem damaligen BIP pro Kopf/Durchschnittskommen eines Normalbürgers. Aber aus dem osmanischen Reich gibts keine VGR.--Antemister (Diskussion) 15:02, 14. Sep. 2014 (CEST)
Keine der „naheliegenden“ Umrechnungen ist wirklich sinnvoll. 4800 Goldlira sind eben nicht 4800 Türkische Lira von heute, das wären ca. 1700 EUR. Es sind auch nicht ca. 89000 DEM oder 45000 EUR. 4800 Goldlira entsprechen ca. 32 Kilogramm Gold, nach heutigem Goldpreis wären das ca. 94000 US-Dollar. Die Wahrheit liegt irgendwo dazwischen. --Rôtkæppchen₆₈ 15:26, 14. Sep. 2014 (CEST)
Hier findet sich ein Pro-Kopf-BIP-Wert für 1913 und Türkei: 1213 “1990 International Geary-Khamis dollars”. Nach Goldstandard entspricht ein Golddollar 1,504475172816 Gramm Gold. 4800 Goldlira entsprechen also ca. 21000 Golddollar oder dem 17,4fachen des Pro-Kopf-BIP der Türkei von 1913. --Rôtkæppchen₆₈ 15:43, 14. Sep. 2014 (CEST)

Thc Abbau im Körper

Wird THC schneller im Körper abgebaut wenn Mann in kurzer zeit viel abnimmt? Dar THC ja im Fett eingelagert wird! und wenn dieses vom Körper verbrannt wird, wird dann auch das Eingelagerte THC mit verbrannt bzw. Schneller abgebaut? --46.115.4.90 00:08, 14. Sep. 2014 (CEST)

es könnte auch sein, dass die Leber einfach die THC Konzentration im Fettgewebe erhöht, so dass man dann wieder den gleichen Verlauf hätte... z. B. kann man viel Quecksilber im Fettgewebe haben, aber auch wenig... --Heimschützenzentrum (?) 08:25, 14. Sep. 2014 (CEST)
Es wird kein THC eingelagert, sondern nur die (nicht psychotropen) Abbauprodukte. Ändert im übrigen nichts an der Nachweisbarkeit, denn die testet auf *kleinste* Mengen diese Abbauprodukte, die immer zurückbleiben. Praktisch sofort landen sie in Haaren und Nägeln und sind da auch nicht mehr rauszubekommen. In den Nägeln kann man das Zeug daher noch viele Monate später nachweisen. -- Janka (Diskussion) 14:28, 14. Sep. 2014 (CEST)

Was ist ein "Brennrecht"?

im Artikel Verschlussbrennerei steht: Verschlussbrennereien können mit oder ohne Brennrecht betrieben werden. Der Begriff Brennrecht ist dabei nicht als Handlungserlaubnis zu verstehen, sondern als das Recht, Branntwein unter finanziell besonders günstigen Bedingungen herzustellen.[1] Im Rahmen des Brennrechts kann der Alkohol zu einem über dem Marktpreis liegenden Preis an die Bundesmonopolverwaltung für Branntwein abgeliefert werden. Brennrechte können ohne weiteres 100.000 Liter Alkohol pro Jahr und mehr betragen.

Fragen dazu: 1) Der dort erwähnte Begriff "Brennrecht" hat kein Artikel in der Wikipedia. Was versteht man darunter? 2) Warum kauft die Bundesmonopolverwaltung für Branntwein Alkohol über dem Marktpreis? Sind dort gehbehinderte Alkoholiker versammelt, die es nicht mehr in den nächsten Schnappsladen schaffen? --77.3.140.70 00:50, 14. Sep. 2014 (CEST)

1: Verschlussbrennerei: Der Begriff Brennrecht ist dabei […] als das Recht [zu verstehen], Branntwein unter finanziell besonders günstigen Bedingungen herzustellen. Im Rahmen des Brennrechts kann der Alkohol zu einem über dem Marktpreis liegenden Preis an die Bundesmonopolverwaltung für Branntwein abgeliefert werden. Brennrechte können ohne weiteres 100.000 Liter Alkohol pro Jahr und mehr betragen.
2.: Bundesmonopolverwaltung für Branntwein#Rechtliche Grundlage: Das Branntweinmonopol [ist] eine nationale Teilmarktordnung für Ethanol mit agrar- und sozialpolitischen Zielsetzungen. --BlackEyedLion (Diskussion) 01:00, 14. Sep. 2014 (CEST)
Das ist eine Subvention, die das Schwarzbrennen verhindern soll. Weil, machen tun's die Bauern sowieso. -- Janka (Diskussion) 06:28, 14. Sep. 2014 (CEST)
Ich glaube, da werfen die Autoren zwei Dinge zusammen: 1. Das Recht, den erzeugten Alkohol zu subventionierten Preisen an die BMonVerw. abzugeben und 2. Das Recht, überhaupt eine gewissen Menge an Alkohol zu erzeugen. Traditionelle Brennrechte haben landwirtschaftliche Erzeuger z.B. Obstbauern, diese alten Rechte , die höher sind, als die heute bei neu zugelassenen Brennereien, werden vererbt. Und dann gibt es noch die Kleinbrennereien und die Hobbybrenner. All dies wird in den Artikeln nicht deutlich genug beschrieben und auseinander gehalten. Gilt auch für die Situation außerhalb D. Im Elsaß , meine ich mich zu erinnern, laufen diese Rechte mit dem Tod des letzten Inhabers aus. Aber nichts genaues weiß man (Wiki) nicht.--G-Michel-Hürth (Diskussion) 12:43, 14. Sep. 2014 (CEST)

Benutzerkonto gelöscht?

Habe versucht neues Passwort anzufordern, bekomme keine Mail. Wusste nicht mehr, über welche Mail ich das angemeldet hatte, deswegen hab ich zwei versucht. Habe dann mein BN Konto gesucht und nicht gefunden. Ist es gelöscht? Hatte ein paar Wochen keine Zeit, also was das Konto inaktiv. Wenn es weg ist, kann man es wieder herstellen? Kann mich jetzt ja nicht anmelden - es geht um das Benutzerkonto Tulfy --178.213.246.34 07:50, 14. Sep. 2014 (CEST)
Hinweis: Für Fragen speziell zur Wikipedia gibt es ein eigenes Diskussionsforum: Fragen zur Wikipedia.

user:Tulfy scheint es noch zu geben... vllt mal bei WP:FZW versuchen? --Heimschützenzentrum (?) 08:19, 14. Sep. 2014 (CEST)
Es gab in Zuge der WMF-globalen Zusammenführung von Konten (WP:SUL), damit jeder in jedem Projekt arbeiten kann eine Umbenennung von Benutzernamen, wenn gleiche Namen in verschiedenen Projekten aktiv waren. Dies ist mehr als ein paar Wochen her. Erinnere Dich an eine Deiner Bearbeitungen und sehe Dir die Versionsgeschichte des Artikels an. Melde als der Benutzer an, der die Änderung tätigte (aber nicht wiederholte, im Fall eines Editierkrieges auch Editwar genannt). Bevor Du anmeldest, rufe die Beiträge des Nutzers ab und vergleiche, ob es Deine sind. --Hans Haase (有问题吗) 10:47, 14. Sep. 2014 (CEST)
Die Angabe ("hatte ein paar Wochen keine Zeit") passt zu Spezial:Beiträge/Tulfy. "Habe versucht neues Passwort anzufordern" ist vieldeutig. Ist das alte Passwort noch bekannt? --Pp.paul.4 (Diskussion) 12:05, 14. Sep. 2014 (CEST)
Das funktioniert nur, wenn eine Email-Adresse hinterlegt wurde. --Hans Haase (有问题吗) 13:57, 14. Sep. 2014 (CEST)

Dauer der Regierungsbeteiligung der FDP im Bund

Im Artikel der FDP steht, dass diese länger als jede andere Partei an der Bundesregierung beteiligt war. Nun frage ich mich, ob das korrekt ist. Und ich frage hier, weil mir die Tragweite einer Änderung des Artikels zu groß erscheint, auch weil diese Behauptung regelmäßig in den Medien wiedergegeben wird.

Zu den Fakten: Über den Artikel Liste der deutschen Bundesregierungen und den einzelnen Kabinetten, ergeben sich, sofern das korrekt ist, folgende Daten: (vorzeitige Ausstiege der FDP aus den Regierungen habe ich berücksichtigt)

FDP

  • 20.09.1949-26.06.1956 = 2.471 Tage
  • 14.11.1961-28.10.1966 = 1.809 Tage
  • 22.10.1969-17.09.1982 = 4.713 Tage
  • 04.10.1982-26.10.1998 = 5.866 Tage
  • 28.10.2009-17,12.2013 = 1.511 Tage
Summe: 16.370 Tage

CDU

  • 20.09.1949-01.12.1969 = 7.377 Tage
  • 04.10.1982-26.10.1998 = 5.866 Tage
  • 22.11.2005-08.09.2014 = 3.212 Tage
Summe: 16.455 Tage

Insofern ergäbe sich aktuell eine längere Regierungsbeteiligung der CDU von 85 Tagen. Kann das sein? Habe ich richtig gerechnet. Ich glaube eher an einen Fehler meinerseits als dass ich der erste sein soll, dem das auffällt. Wäre toll, wenn mal jemand meine Berechnungen überprüft.--87.148.104.178 23:22, 8. Sep. 2014 (CEST)

Überprüft hab ich die Zahlen nicht, aber die FPD ist vor mehr als 85 Tagen aus der Bundesregierung ausgeschieden; die CDU regiert dagegen die ganze Zeit weiter. Insofern kann es rein logisch also durchaus sein, dass die Behauptung im Artikel FPD mittlerweile falsch, weil schlicht veraltet ist. --88.130.114.41 23:37, 8. Sep. 2014 (CEST)
Du meinst sicher die FDP. Die FPD ist was anderes. Deren Artikel ließt sich allerdings wie eine bitterböse Satire auf die FDP: "Die FPD ist eine deutsche Kleinpartei." --Der-Wir-Ing (Diskussion) 07:36, 9. Sep. 2014 (CEST)
Natürlich. Der Schreibfehler kommt wahrscheinlich von meiner immensen Tippgeschwindigkeit. ;-) --88.130.77.236 13:15, 9. Sep. 2014 (CEST)
Diese Aussage steht bereits seit 1. Februar 2008 im Artikel. Seit 15. Juni 2014 ist sie aber veraltet. --Rôtkæppchen₆₈ 23:39, 8. Sep. 2014 (CEST)
(BK)Also, nach deiner Rechnung war die Angabe im Artikel bis vor drei Monaten korrekt...--Antemister (Diskussion) 23:41, 8. Sep. 2014 (CEST)
Basierend auf welchen Informationen hast du das ausgerechnet?
@ IP: Hast du Abweichungen berücksichtigt, die dadurch entstehen, dass einige Jahre Schaltjahre waren? --88.130.114.41 23:43, 8. Sep. 2014 (CEST)
Basierend aus den Regierungsbeteiligungen die sich aus den Artikeln der einzelnen Kabinette ergeben. Ich habe die Tage nicht selbst errechnet, sondern diesen Tagesrechner verwendet. Selbst wenn der Rechner Schaltjahre nicht beücksichtigt, so waren es wohl kaum mehr als 85. --87.148.104.178 00:00, 9. Sep. 2014 (CEST)
Ja, die Aussage wird einfach veraltet sein und kann (und sollte) daher entfernt werden. --mfb (Diskussion) 02:09, 9. Sep. 2014 (CEST)
Der Tagesrechner berücksichtigt tatsächlich keine Schaltjahre. Allerdings kommen nach meiner Zählung bei beiden Parteien 11 Schaltjahre vor, so dass sich die Fehler aufheben. --Grip99 01:10, 12. Sep. 2014 (CEST)
Hinweis: Es geht um den Artikel Freie Demokratische Partei, dort den zweiten Absatz der Einleitung und den Abschnitt Freie Demokratische Partei#Geschichte. --Rôtkæppchen₆₈ 07:09, 9. Sep. 2014 (CEST)
Die Information, dass es nicht mehr so ist, würde ich übrigens eher dem Geschichtsabsatz als der Einleitung überlassen. --Eike (Diskussion) 08:35, 9. Sep. 2014 (CEST)

Rotkäppchen und Eike haben das im Artikel FDP entsprechend korrigiert. --88.130.77.236 13:21, 9. Sep. 2014 (CEST)

Bitte auch für die CSU überprüfen. Die müsste ja genauso lange wie die CDU an der Regierung beteiligt sein. Folglich wäre die momentane Aussage bei CDU sie wäre länger als jede andere Partei and der Regierung beteiligt, nicht korrekt. --Der-Wir-Ing (Diskussion) 07:36, 9. Sep. 2014 (CEST)

Ich hab mal in die oben verlinkte Liste geschaut, die CSU war immer mit dabei. Soweit ich sehen kann, werden die Nachfolger in diesem Rekord, CDU und CSU, im Artikel aber gar nicht mehr erwähnt. --Eike (Diskussion) 12:28, 10. Sep. 2014 (CEST)
Ich hab das mal in beiden Artikeln eingefügt. --Der-Wir-Ing (Diskussion) 23:26, 14. Sep. 2014 (CEST)
Archivierung dieses Abschnittes wurde gewünscht von: Dann ist wohl alles klar. :o) --Eike (Diskussion) 13:42, 15. Sep. 2014 (CEST)

Drohne von Wikipedia

Stimmt es, dass Wikipedia einen Oktokopter (Achtflügler) mit Kamera einsetzt und Aufnahme im Wikipedia-Projekt verwendet? Wo kann man die Bilder finden? --E.Rivas (Diskussion) 16:21, 14. Sep. 2014 (CEST)
Hinweis: Für Fragen speziell zur Wikipedia gibt es ein eigenes Diskussionsforum: Fragen zur Wikipedia.

Die Seite hatte ich so verstanden, dass es sich um Fragen zur Bearbeitung der Wikipedia handelt. Deshalb hatte ich die Auskunft gewählt, da es sich ja um keine Bearbeitungsfrage handelt. --E.Rivas (Diskussion) 17:26, 14. Sep. 2014 (CEST)
Dann hast Du das falsch verstanden. Deine Frage wäre dort am richtigen Ort. --178.4.111.125 18:18, 14. Sep. 2014 (CEST)
Archivierung dieses Abschnittes wurde gewünscht von: E.Rivas (Diskussion) 11:12, 15. Sep. 2014 (CEST)

Nachfolger von Schmidt

Bekanntlich hatte Helmut Schmidt 1977 im Fall des Scheiterns der Erstürmung der Lufthansa-Maschine in Mogadischu seinen Rücktritt als Bundeskanzler geplant. Gibt es irgendwelche Anhaltspunkte dafür, wie in einem solchen Fall der weitere Ablauf gewesen wäre und wen Schmidt als seinen Nachfolger im Auge hatte? Die Sache hätte ja tatsächlich genauso gut schiefgehen können und ich kann mir kaum vorstellen, dass Schmidt für diesen Fall nur bis zu dem Zeitpunkt seiner Rücktrittserklärung und nicht darüber hinaus gedacht und geplant hat. Gab es dazu von ihm irgendwelche Äußerungen? --Proofreader (Diskussion) 19:13, 14. Sep. 2014 (CEST)

Nach dem Rücktritt hätte der Bundestag einen neuen Kanzler gewählt. Das wäre vermutlich derjenige der sich innerhalb der SPD durchgesetzt hätte, nicht zwingend derjenige den Schmidt oder sonstwer vorgeschlagen hätte. --Der-Wir-Ing (Diskussion) 19:37, 14. Sep. 2014 (CEST)
Das ist wahr; Schmidt selbst ist ja nach dem Rücktritt seines Vorgängers so in sein Amt gekommen. Streng genommen ist das natürlich keine Wissensfrage, aber welche Kandidaten wären in einem solcen Fall denn in Frage gekommen und hätten die besten Chancen gehabt? --Proofreader (Diskussion) 19:58, 14. Sep. 2014 (CEST)
Hans-Jochen Vogel, alles andere wäre reine Spekulation. --Studmult (Diskussion) 20:11, 14. Sep. 2014 (CEST)
Archivierung dieses Abschnittes wurde gewünscht von: --112.198.82.38 23:02, 15. Sep. 2014 (CEST)

Noch eine Frage zur Halogenbirne

Es gibt von Osram eine Niedervolt g6.35 Halogenbirne für die industrielle Anwendung, welche bei gleicher Leistung (50W) die doppelte Helligkeit erzeugt wie eine Standardbirne. Dafür wir dide Lebensdauer mit nur 50 Stunden angegeben im Vergleich zu 2000 Stunden der normalen Birne. An meiner Schreibtischlampe habe ich zwei Stufen, wobei die erste Stufe dazu führt, dass die Birne mit nur 35 Watt betrieben wird. Bekannt ist, dass eine Absenkung der Spannung um wenige Prozent die Lebensdauer eines Leuchtmittels vervielfachen kann. Meine Lampe habe ich fast immer in Stufe 1 betrieben und daher hält sie auch seit 10 Jahren bei täglich mehrstündiger Betriebsdauer. Ich frage mich nun was passiert, wenn ich mir die Speziallampe kaufe und diese auf Stufe 1 betreibe, also mit geringerer Spannung. Die Lebensdauer der Lampe wird sich dadurch vervielfachen und dabei vielleicht an die Lebensdauer der normalen Lampe herankommen, würde ich diese bei 50 Watt betreiben. Allerdings dürfte die Lichtausbeute doppelt so hoch sein als die der Standardbirne, wenn ich diese bei 35 Watt betreibe. Anstatt dass meine auszutauschende Birne wieder 10 Jahre hält wird sie dann nur noch vielleicht 1 Jahr halten, dabei aber viel effizienter sein. Bei einem Leuchtmittelpreis von 3 EUR zu würde sich das rechnen. Sehe ich das richtig? Spezialampe: Osram 64655, Normallampe: HALOSTAR ST 50 W 12 V GY6.35. Hier ein interessantes Diagramm dazu http://de.wikipedia.org/wiki/Gl%C3%BChlampe#mediaviewer/File:Gluelebensdauerrp.png 35 Watt sind zu 50 Watt 30% geringerere Spannung. --77.3.187.182 22:53, 10. Sep. 2014 (CEST)

Die Speziallampe wird bei Dir drei Monate halten (Dreisatz). --Rôtkæppchen₆₈ 23:11, 10. Sep. 2014 (CEST)
Du hattest glaube ich mal geschrieben, dass ich nicht auf eine 35 Watt Lampe ausweichen darf, weil der Trafo dies nicht vertragen würde. Auf dem Trafo steht leider nichts. Neben dem Glas steht aber, dass ich nur 50W verwenden darf. Warum? Was passiert, wenn ich eine 35W Birne verwende? Es ist ein schwerer Trafo also kein Scheltnetzteil. --77.3.187.182 23:16, 10. Sep. 2014 (CEST)
Hab deinen Text wiedergefunden. Ich kopiere ihn hier mal rein: "Noch ein wichtiger Hinweis: Beim Austausch des Leuchtmittels musst Du unbedingt das Typenschild des Halogentrafos beachten. Hat das neue Leuchtmittel zu wenig Leistung, so steigt die Betriebsspannung und das Leuchtmittel stirbt vorzeitig durch Überspannung. Hat das neue Leuchtmittel zu viel Leistung, so steigt der Strom durch den Halogentrafo. Bei zu großem Strom kann dieser den vorzeitig durch Hitze sterben"
Das würde ja bedeuten, dass ich mir einfach eine 35 Watt Birne kaufen und einsetzen kann und mit einem Multimeter dann nachprüfen, dass die 12 V nicht überschritten werden. Dann müsste es gehen?--77.3.187.182 23:21, 10. Sep. 2014 (CEST)
Das kannst Du ausprobieren. Und wenn die Spannung in Stufe zwei zu hoch ist, kannst Du immer noch Stufe eins nehmen. Oder hol Dir ein LED-Retrofit-Leuchtmittel. --Rôtkæppchen₆₈ 23:45, 10. Sep. 2014 (CEST)
Danke. --77.3.187.182 23:57, 10. Sep. 2014 (CEST)
Derzeitige LED-Leuchtmittel benötigen rund 20% der Energie einer Glühlampe. Die Leerlaufspannung dürfte damit in den Griff zu bekommen sein. Viele LED-Leuchtmittel regeln intern, was sie wiederum tolerant für geringe Überspannung macht. Beeinflusst wird dies, wenn die Leuchte "dimmbar" ist, also für den Betrieb hinter Dimmern ausgelegt ist. Das sind längst nicht alle! Auch der Transformator muss dazu ausgelegt werden, siehe Datenblätter und Spezifikationen. Eine (maximale) Betriebsspannung wird nahezu jedem elektrischen Bauteil spezifiziert. Bei LED-Leuchten ist ein Spannungsbereich üblich. --Hans Haase (有问题吗) 11:48, 12. Sep. 2014 (CEST)
LED für den angegebenen Zweck sind alles durch die Reihe Funzeln mit zusätzlich schlechter Farbwiedergabe. Alleine die Lichtstärke der retrofit Lampen wird schwer enttäuschen. --93.132.132.121 19:31, 12. Sep. 2014 (CEST)
Die oben verlinkte LED-Lampe mit 550 lm leuchtet so hell wie eine Allgebrauchslampe mit 49,1 Watt oder eine Halogenlampe mit ca. 30 Watt. Die Werbung, dass diese Lampe hell wie eine 50-Watt-Lampe leuchte, ist eine glatte Lüge. Das ist aber bei vielen Retrofitleuchtmitteln so. Ich hab mir mit diesen LEDs selber eine LED-Leuchte gebaut mit 2,4 Watt 196 lm. Da weiß ich, dass sie fast so hell wie eine 25-Watt-Allgebrauchslampe (220 lm) leuchtet. Eine normale 50-Watt-GY6.35-Halogenlampe hat 910 lm. Mit 33 dieser LEDs (plus 11 Vorwiderstände) erreicht man ca. 924 lm bei ca. 9,24 Watt. --Rôtkæppchen₆₈ 21:23, 12. Sep. 2014 (CEST)
Mit 5000 K hat die ein recht frisches Weiß, warmes Weiß liegt um die 2700 K. Nach Datenblatt, Seite 10, liegt die mit Abstand hellste Farbart bei 454 nm, was eine blaue LED ausmacht, die mit einer Leuchtschicht versehen ist, die ihre Resonanz im Grünbereich Richtung gelb hat. Tiefes Rot und Blaugrün gibt sie nur schwach wieder. Was sich technisch nicht korrigieren lässt, kann mit RGB-LEDs erreicht werden. Dabei wird das Licht blauer, grüner und roter LEDs gemischt, was über additive Farbmischung weiß ergibt. Sind die LEDs richtig ausgesucht, kommen die Farben dem Original sehr nahe. Das gibt es auch geregelt und es existieren einige Patente darüber, das Weiß weiß zu halten. --Hans Haase (有问题吗) 20:41, 14. Sep. 2014 (CEST) Da müssen die Waschmittel schon sehr früh aufstehen, um das so gut hinzubekommen.
Mit RGB-LEDs erreicht man zwar fast jedes Weiß, aber nur niedrige Farbwiedergabeindizes Ra von ungefähr 60, vergleichbar mit billigen Dreibandenquecksilberdampfleuchtstofflampen. Blau- oder UV-LEDs mit Leuchtstoffen könen aber wie gute Quecksilberdampf-Leuchtstofflampen einen Farbwiedergabeindex Ra von 90 erreichen. --Rôtkæppchen₆₈ 09:04, 15. Sep. 2014 (CEST)

gleichung mit zwei unbekannten

1.) 2 a + 3 b = 140

2.) 4 a + 2 b = 160

a=?

b=?

--Yannick1999 (Diskussion) 07:12, 11. Sep. 2014 (CEST)

Seufz. --81.92.99.84 07:52, 11. Sep. 2014 (CEST)
und der Lösungsweg:
a) Rechtschreibung ist eine Tugend.
b) dividiere Gleichung (2) durch 2 und erhalte 2a + b = 80 und somit b = 80 - 2a
c) setze das in Gleichung (1) ein und erhalte 2a + 240 - 6a = 140
d) löse das nach a auf und setze die Zahl in b = 80 - 2a ein, um auch den Wert von b zu erhalten
--81.92.99.84 08:02, 11. Sep. 2014 (CEST)
Oder auch:
  1. Gleichung (1) mit −2 multiplizieren
  2. beide Gleichungen addieren
  3. Summengleichung nach b auflösen
  4. b in (1) oder (2) einsetzen und nach a auflösen.
--Rôtkæppchen₆₈ 09:37, 11. Sep. 2014 (CEST)
Man bilde eine Matrix und transformiere sie zu einer Einheitsmatrix.
But Neo can't help.--1970gemini 12:45, 12. Sep. 2014 (CEST)
Dann kann man das ja gleich mit dem Determinantenverfahren lösen. Hat jemand noch größere Kanonen für noch kleinere Spatzen? --Rôtkæppchen₆₈ 18:20, 12. Sep. 2014 (CEST)
GMRES? ;-) -- HilberTraumd, m14:10, 13. Sep. 2014 (CEST)
Mit dem Dachprodukt und Cramer Regel: a = (140e1+160e2)^(3e1+2e2)/((2e1+4e2)^(3e1+2e2)), b = (2e1+4e2)^(140e1+160e2)/((2e1+4e2)^(3e1+2e2))--2.162.82.162 18:30, 14. Sep. 2014 (CEST)

4d

Gibt es im 4d-Raum eine geometrische Figur, für die es keine Entsprechung im 3d-Raum gibt und die man hier deshalb auch nicht darstellen kann? Zum Vergleich: Ein 4d-Würfel hat den 3d-Würfel oder das Quadrat als Entsprechung. Eine 4d-kugel hat die Kugel oder den kreis als Entsprechung. --88.73.198.112 18:38, 12. Sep. 2014 (CEST)

Kugel und Würfel sind sehr regelmäßige Körper. Für unregelmäßige Körper gibt es keine direkte Entsprechung, denn man erhält unterschiedliche Ergebnisse, je nachdem welche Dimension man zusammenfallen läßt. --Optimum (Diskussion) 18:47, 12. Sep. 2014 (CEST)
24-Zeller--Der-Wir-Ing (Diskussion) 18:57, 12. Sep. 2014 (CEST)
Der 24-Zeller ist übrigens ein schönes Beispiel für einen Körper, den es NUR in 4 Dimensionen gibt. In mehr als vier Dimensionen gibt es nicht mal alle Entsprechungen zu platonischen 3D-Körpern (in 4D hingegen alle fünf plus eben der 24-Zeller)--Alexmagnus Fragen? 13:32, 13. Sep. 2014 (CEST)
Es gibt noch etwas lustigeres. Das Kreuzprodukt in Koordinatenform ist nur für 3D und 7D definiert. Das liegt unter anderem an der Eigenschaft von 3D und 7D das dort ein Bivector einen dualen Pseudovector hat. Für beliebige Dimensionen nimmt man aber andere Formeln dann. Gruß--78.51.231.191 21:27, 14. Sep. 2014 (CEST)

DDR - Colasirup sinello

Colasirup sinello

Ich hab hier ein Foto von "Colasirup sinello" aufgenommen im DDR-Museum Pirna. Das schwarze Gebräu ist laut Aufschrift "zum Selbstberetten(sic!) von Colamilch". Außerdem ist es Coffeinfrei. Da die Allwissende Müllhalde das scheinbar noch nicht kennt, meine Frage an Euch: Was ist das? Meine Vermutung: ein Sirup mit dem man eine Colaähnliches Getränk herstellen kann. Aber Colamilch hab ich noch nie gehört. Links unten steht noch "Verdünnung: 1 Teil Sirup 12 Teile Milch". Im Artikel Nordhausen hab ich auch nichts über dieses Produkt finden können, obwohl es beim VEB Fruchtsaftgetränk Nordhausen hergestellt wurde. Weiß jemand mehr dazu? --sk (Diskussion) 22:26, 12. Sep. 2014 (CEST)

*quetsch* Das Druckbild ist wahrscheinlich nicht fehlerhaft und es heißt doch "zum Selbstbereiten" und nicht "Selbstberetten". Wenn man das Foto in voller Vergrößerung anschaut, kann man einen kleinen, hellen Schmutzfleck bzw. Abrieb erkennen, der die schwarze Farbe im Zwischenraum zwischen I und dessen Punkt verdeckt und so aus größerer Entfernung den Eindruck einer komplett durchgehenden t-Linie erweckt. scnr. --178.24.34.96 16:46, 13. Sep. 2014 (CEST)
Ja, da hast du recht. Es heißt tatsächlich "Selbstbereiten".--sk (Diskussion) 17:15, 13. Sep. 2014 (CEST)
Ich dachte immer, in der DDR gab es nur Club-Cola.--Optimum (Diskussion) 22:33, 12. Sep. 2014 (CEST)
Nein, da gab es mehrere, z.B. auch die mMn total eklige Vita Cola. --88.130.88.58 22:47, 12. Sep. 2014 (CEST)
Cola in der DDR - es gab reichlich Sorten. Nur nicht gleichzeitig und oft nur regional. --an-d (Diskussion) 20:03, 13. Sep. 2014 (CEST)

Cola und Milch scheint nicht zu klappen, siehe Experimenthier. andy_king50 (Diskussion) 19:12, 13. Sep. 2014 (CEST)

Sinello war aber keine Cola-Brauselimonade, sondern Cola-Sirup. Mir ist irgendwie unklar, was hier unklar ist ;) Sirup mit Colageschmack wird mit Milch gemischt, dann hat man süsse Milch mit Colageschmack, was man Colamilch nennen kann. Wo ist das Mysterium? ;) --King Rk (Diskussion) 20:37, 13. Sep. 2014 (CEST)
Danke für die Erklärung. Dieses Produkt ist mir zu DDR-Zeiten nie untergekommen. Nur Himbeer-Sirup und die war eigentlich mit Wasser zu verdünnen. Deswegen bin ich bei Colasirup auch von Wasser als zweiten Bestandteil ausgegangen. --sk (Diskussion) 07:11, 14. Sep. 2014 (CEST)
Cola-Sirup (wie überhaupt Sirup weit verbreitet war - und man damit nicht von den Zwangsmischungen der Herrsteller abhängig war) gab es bis zum Schluss (keine Ahnung mehr wie es hieß, allerdings war die Flaschenform eine andere). Cola war übrigens nicht immer "Coke/Pepsi mit viel Eis". Bei Werner Holt wurde ein "Topf Cola gekocht".--Wikiseidank (Diskussion) 07:36, 15. Sep. 2014 (CEST)

ftp geht nicht

Hallo, meine Kumpels und ich haben uns einen vServer gemietet, wo Debian Wheezy drauf läuft, nur irgendwie bekommen wir es nicht hin, dass wir mit Filezilla Dateien hochladen können. Gibt es irgendwo ein brauchbares how-to, mit dem wir Schritt für Schritt einen Webserver (Apache) mit PHP5, Mysql und eben FTP konfigurieren können? (nicht signierter Beitrag von 79.195.56.254 (Diskussion) 12:39, 13. Sep. 2014 (CEST))

Ja, das gibt es irgendwo. --Blauer Berg (Diskussion) 13:08, 13. Sep. 2014 (CEST)
Naja, soweit waren wir auch schon... allerdings gab es dann Probleme mit den Rechten, da obwohl wir in Filezilla die richtigen Rechte gesetzt hatten, Joomla sich beschwerte, dass es im Ordner nicht schreiben kann. (nicht signierter Beitrag von 79.195.56.254 (Diskussion) 17:34, 13. Sep. 2014 (CEST))
Dann sind die Rechte *eben nicht* richtig gesetzt. Vermutlich hat der Ordner den falschen Besitzer/falsche Gruppe. Welche die richtige ist, sieht man anhand der Webserver-Konfiguration, meist ist das wwwrun/wwwrun oder wwwrun/www. -- Janka (Diskussion) 18:49, 13. Sep. 2014 (CEST)
Wer schon nicht in der Lage ist, sein Problem zu erkennen und nachvollziehbar zu beschreiben, der dürfte mit dem Betrieb eines vServers vermutlich eh überfordert sein. Mein Ratschlag an 79.195.56.254: bezahle jemanden dafür, der sich mit der Thematik auskennt, und für die Ergebnisse seiner Arbeit auch entsprechend haftet. --Blauer Berg (Diskussion) 08:53, 14. Sep. 2014 (CEST)
Ist die Firewall dafür offen? Hort der Rechner auf den Port? Läuft der Daemon ps -A  ? Was sagt netstat? mit telnet den port getestet? ist in den /var/log/messages etwas zu finden? --Hans Haase (有问题吗) 14:06, 14. Sep. 2014 (CEST)
...oder hör auf jemanden, der lsof -i TCP:21 nicht kennt. --Blauer Berg (Diskussion) 14:30, 14. Sep. 2014 (CEST)
Du weißt ganz genau, dass das alles nicht passieren wird. Leider ist es heutzutage so, dass man ohne minimale Kenntnisse einen LAMP in der Standardkonfiguration aufsetzen kann und deshalb denken die Leute, dass sie das auch tun müssten. Die Probleme kommen dann fünf Minuten später, aber da ist das Ding schon bestellt und bezahlt. -- Janka (Diskussion) 15:11, 14. Sep. 2014 (CEST)
Vielleicht hilft es ja, zu Hause einen Rechner mit Debian Wheezy, Apache, MySQL, PHP5 und ftpd aufzusetzen und diesen als Testrechner für den vServer zu verwenden: Wenn auf dem Testrechner alles klappt, kann die Konfiguration auf den vServer übernommen werden. --Rôtkæppchen₆₈ 00:37, 15. Sep. 2014 (CEST)

anogenitale venerische warzen

A poster from the Office for Emergency Management. Office of War Information, 1941-1945 - Bedröppelt guckt er, der Matrose.

Hallo liebe Wikipedia Gemeinde, im Nachhinein an unseren Unterricht in der Berufsschule stieß ich auf eine Frage. Und zwar ging es im speziellen um die Virologie und HPV. Nach ICD 10 sind ja die daraus resultierenden Condylome als A63.0 "Anogenitale (venerische) Warzen" bezeichnet. Anogenital ist verständlich, aber was haben die Warzen mit den Venen zu tun? Desweiteren gibt es diese Warzen auch im Körperinneren (Gebärmutterhals etc), konnte dazu leider keine Bilder finden vielleicht kennt ja jemand eine gute Bilderquelle? Vielen Dank --2003:5F:E34:1400:45A7:C4C4:C87D:E184 16:28, 13. Sep. 2014 (CEST)

"Venerisch" hat nichts mit Venen, sondern mit der guten Venus, Göttin der Liebe und des Beischlafs, zu tun. Grüße Dumbox (Diskussion) 16:31, 13. Sep. 2014 (CEST)
Siehe auch im Wiktionary unter "venerisch": "Bedeutungen: [...] [3] medizinisch, über Krankheiten: sexuell übertragbar". --Buchling (Diskussion) 18:26, 13. Sep. 2014 (CEST)
VD (Veneral Disease) nennt man jetzt STD (Sexually Transmitted Disease). Aber die alten Poster sind lustig. Tipp: Venerologie! --Lkl22:32, 14. Sep. 2014 (CEST)
Auf Deutsch heißen diese Krankheiten auch STI. Dazu zählen nicht nur die „klassischen“ Geschlechtskrankheiten, sondern auch Feigwarzen, Gebärmutterhalskrebs, HIV/Aids, diverse Hepatitiden etc. --Rôtkæppchen₆₈ 22:45, 14. Sep. 2014 (CEST)

Verstärker für UMTS

Ich habe in der Wohnung schlechten Empfang, wenn ich zum Fenster gehe, ist es deutlich besser. Gibt es ein Gerät, die ich dort hinstellen könnte, das das Signal verstärkt? Die SIM-Karte ist im Rechner, die will ich nicht jedes Mal herausnehmen. --Pölkky 18:57, 13. Sep. 2014 (CEST)

Ein UMTS-Router sollte da helfen. --Eike (Diskussion) 18:59, 13. Sep. 2014 (CEST)
Das funktioniert. Aber da müßte ich ständig die SIM aus dem Rechner nehmen. Die Schublade dafür ist mir schon mal zerbrochen und ist schweineteuer. Ich suche was, bei dem die Karte im Rechner bleiben kann. --Pölkky 19:25, 13. Sep. 2014 (CEST)
Anbieter fragen, ob Zweit-SIM angeboten wird ? andy_king50 (Diskussion) 19:55, 13. Sep. 2014 (CEST)
irgendwas mit ner Bi-Quad-Antenne? --Heimschützenzentrum (?) 20:23, 13. Sep. 2014 (CEST)
Könnte es nicht sein, dass ein Wlan oder eine anderes Handy überstrahlt und von daher der sehr schlechte Empfang in der Wohnung resultiert? --2.242.47.239 20:33, 13. Sep. 2014 (CEST)
Normalerweise ist das eher das Wand- und Fassadenmaterial, das dämpft. WLANs nutzen einen anderen Frequenzbereich und andere Handys und UMTS-Adapter senden auf anderen Zeit- und Frequenzslots, stören also nicht. Es gibt Indoor GSM-Repeater. Ob die auch für UMTS taugen, weiß ich nicht. --Rôtkæppchen₆₈ 21:51, 13. Sep. 2014 (CEST)
@Pölkkyposkisolisti: Gib mal umts repeater bei Amazon ein. --Rôtkæppchen₆₈ 22:52, 13. Sep. 2014 (CEST)
Danke, da habe ich was gefunden: [44] - allerdings ist das deftig teuer. Ich suche mal weiter. --Pölkky 05:33, 14. Sep. 2014 (CEST)

Bitte § 149 TKG in Verbindung mit § 55 Absatz 1 TKG beachten! --Blauer Berg (Diskussion) 08:32, 14. Sep. 2014 (CEST)

@Pölkkyposkisolisti: Da wäre ein mobiler UMTS-Router wohl besser. Wenn Du die SIM-Karte nicht ständig umbauen willst, musst Du den allerdings dauert mit Deinem Rechner rumschleppen. Viele Smartphones lassen sich auch als UMTS-Router konfigurieren (Tethering). Wenn Dein Mobilfunkvertrag Geschwindigkeit und Datenvolumen hergibt, wäre das eine Alternative. --Rôtkæppchen₆₈ 08:56, 14. Sep. 2014 (CEST)
Die Bi-Quad-Antenne ist eine Richtantenne. Sie würde Zellenwechsel unterbinden. Für Deutschland genügt das bei LTE bekanntlich. In Ländern ohne »kastriertem« LTE wäre davon abzuraten. Nur die Position der Mobilfunkantenne sollte geändert werden. Ob, USB-Stick am USB-Kabel, UMTS-Router ist egal. Ob Tetering oder über die Luft per Passwort »ausgeliehene« SIM-Karte wie im Auto ebenso. WLAN ist jedoch sehr hungrig nach Energie, was Akkubetrieb angeht. --Hans Haase (有问题吗) 11:02, 14. Sep. 2014 (CEST)
Ich benutze die SIM ausschließlich im Rechner. Ich weiß nicht, ob die auch telefonieren kann, dazu brauchwe ich sie nicht. Ich hab 10 GB pro Monat bei 1&1 Vodafone. Es ist ja gerade angenehm, daß ich keinen Stick oder Router herumschleppen muß. So ein kleines flaches Ding für 8 Teilnehmer habe ich. --Pölkky 13:18, 14. Sep. 2014 (CEST)

Ich habe ein ähnliches Problem in meiner Wohnung, wo ich am Schreibtisch gar keinen Empfang habe. Ich habe mir überlegt, ob es mit eine Bluetoothgerät entsprechend der Freisprecheinrichtung in meinem Auto gehen würde. Wäre das realistisch? --Eingangskontrolle (Diskussion) 20:17, 14. Sep. 2014 (CEST)

Bluetooth reicht halt leider nur ca. 10 Meter weit. Ich hab schon ein Problem, wenn ich nur mit dem Bluetoothkopfhörer aufs Klo gehe: Der Bluetoothadapter (2,4-GHz-Band) reicht nicht durch zwei Stahlbetonwände. Der Analogfunkkopfhörer im 860-MHz-Band schafft das aber. --Rôtkæppchen₆₈ 22:53, 14. Sep. 2014 (CEST)

Rezept

[45] war ziemlich ekelhaft. Ich glaube fast, Wein, Essig und Sauere Sahne vertragen sich nicht besonders? --85.180.142.79 20:22, 13. Sep. 2014 (CEST)

Inwiefern entsprach es nicht Deinem Geschmack? Eigentlich normales Burgunderbraten-Rezept auf Schwein übertragen, ich hab ähnliches oft mit Rind gekocht. Kann es ggf. sein, dass Du zuviel Marinade mit in den Bratenfond getan hast? Oder aus Ersparnisgründen irgend einen ungeeigneten "Pennertrost" als Rotwein genommen? - andy_king50 (Diskussion) 20:36, 13. Sep. 2014 (CEST)
Ich vermute mal das „Immer wieder mit Marinade übergießen“ wurde fehlinterpretiert. Später im Bratvorgang sollte man grundsätzlich den Sud im Topf zum übergiessen benutzen. Also hast du mehr als eine Tasse Marinade im Backofen verbraucht? Ansonsten wäre eben die falsch abgeschmeckte Sauce durchaus auch drin. Beim Kochen mit Wein gilt grundsätzlich, benutze keinen Wein zum Kochen, denn du nicht auch im Glas konsumieren würdest. --Bobo11 (Diskussion) 21:02, 13. Sep. 2014 (CEST)
Nach kurzer Recherche beim Chefkoch ist das ein Sauerbratenrezept, bei dem die Zubereitung der Sauce mir fragwürdig erscheint. Bitte beschreibe etwas genauer, in welcher Form das „ekelhaft“ war, dann ließe sich das Problem eingrenzen. --88.68.82.17 22:50, 13. Sep. 2014 (CEST)
Ich habe das „Immer wieder mit Marinade übergießen“ tatsächlich so interpretiert, dass immer mehr von der Marinade genommen werden soll. Das Fleisch war vom Geschmack gewöhnungsbedürftig, aber möglicherweise diskutabel. Die Sauce war meines Erachtens aber jedenfalls ungenießbar, weil furchtbar sauer (mehr als meine Geschmacksnerven vertragen) und sehr unangenehm nach Essig riechend. ... Und es handelte sich um guten Rotwein, nicht um einen aus dem Tetrapack. --85.180.142.79 02:58, 14. Sep. 2014 (CEST)
Faustregel beim Sauren: Nicht mehr als ein Löffel. Und es muss auf jeden Fall wiederholt abgeschmeckt werden, allein nach Rezept kann man nicht kochen! -- Janka (Diskussion) 06:59, 14. Sep. 2014 (CEST)
Die Marinademenge selbst scheint mir nicht zu groß. Allerdings enthält sie ein Drittel Essig, das ist mehr als bei einer Vinaigrette. Wenn man dass beim Kochen noch einreduziert, muss das grausam werden. Ich würde den Essigaanteil mindestens halbieren und entsprechend Wasser ergänzen. Nach dem Einlegen nur einen Teil der Marinade zum Fleisch geben und ggf. Wein nachgießen, wenn die Flüssigkeit verdampft. – Oder gleich ein gescheites Rezept suchen ;-) Wobei ich auch glaube, dass so ein Rotwein-Sauerbraten aus Rind grundsätzlich besser ist. Rainer Z ... 11:19, 14. Sep. 2014 (CEST)
Sowas hatte ich vermutet. Die ganze Marinade in die Sauce zu kippen ist nicht gut, da müßte man ewig den Essig rauskochen. Ich würde den Bratensatz eher mit Wasser und Rotwein ablöschen und in jedem Fall abschmecken vor dem Essen. --178.4.111.125 18:11, 14. Sep. 2014 (CEST)
Essig kannste nicht rauskochen. Die Essigsäure bleibt in der Sauce, wird also beim Kochen weiter konzentriert. drum sollte man am Anfang, so wie auch beim Salz, vorsichtig sein, weil bei solchen Schmorgerichten ein erheblicher Teil der Flüssigkeit verdampft. Rainer Z ... 13:11, 15. Sep. 2014 (CEST)

Wir speichern Cookies, bitte "OK" klicken!

Hallo, ich frage mich warum immer mehr Webseiten einem jetzt erzählen, dass sie Cookies nutzen und möchten, dass irgendein Button gedrückt wird. Gabs rechtliche Änderungen, die das nötig machen? Falls nicht, was ist der Vorteil für die Betreiber der Seiten, dass Surfer mit dieser unnützen Information belästigt werden? (Achja, Datenschutz und gezielte Information der Nutzer kann ja nicht der Grund sein: wenn 99,999% aller häufig besuchten Webseiten Cookies nutzen, wäre eher eine solche Information sinnvoll: "Wir nutzen keine Cookies, sind wir nicht toll? [Ja] [Nein] [Weiß nicht]") -- 77.64.189.189 23:01, 13. Sep. 2014 (CEST)

http://ec.europa.eu/ipg/basics/legal/cookies/index_en.htm#section_2 --88.130.95.148 23:08, 13. Sep. 2014 (CEST)
Das ist mal wieder ein schönes Beispiel, wie eine Regelung ins Leere läuft. Alle nützlichen Cookie-Anwendungen *außer* dem Internet-Stalking müssen nämlich nicht angekündigt werden. Natürlich wollen die dich weiterverfolgen, deshalb holen sie sich am Anfang dein Einverständnis dazu und dürfen dann wieder schalten und walten, wie sie wollen. Gibst du das Einverständnis nicht, kannst du den Dienst nicht nutzen. Richtig wäre es stattdessen gewesen, auch (und gerade) durch Firmen betriebenes Internet-Stalking zur Straftat zu erklären und an einem Werbenetzwerk ein Exempel zu statuieren. -- Janka (Diskussion) 14:17, 14. Sep. 2014 (CEST)
Tja, solange das Wort »Datenschutz« in der Digitalen Agenda nicht mal vorkommt und Oettinger (»wird Bildschirmschoner« TAZ) in der EU für dieses Internet und so zuständig ist, wird sich auch nichts tun. Rainer Z ... 14:34, 14. Sep. 2014 (CEST)
Welche Digitale Agenda meinst du? Die "Digitale Agenda für Europa" oder die "Digitale Agenda 2014 – 2017" der Bundesregierung? Und hast du eine von beiden gelesen? Das Wort "Datenschutz" kommt nämlich in beiden sehr wohl vor - und zwar jeweils gleich mehrfach.
Was natürlich an der Berechtigung der von dir und Janka geäußerten Kritik nichts ändert. --Snevern 20:01, 14. Sep. 2014 (CEST)

Berühmte Drehbuchautoren

Schauspieler haben eine große Fangemeinde. Regisseure können auch eine gewisse Beliebtheit erlangen und auch für Filmkomponisten gibt es Kreise, die sich speziell für ihr Werk interessieren. Nur von Drehbuchautoren hört man so gut wie nichts. Gibt es überhaupt Kreative, die durch das Schreiben von Drehbüchern berühmt geworden sind? Bisher sind mir nur berühmte Theaterautoren bekannt. --188.101.73.16 11:44, 14. Sep. 2014 (CEST)

Ethan und Joel Coen sind so clever, einfach alles zu machen. --Mikano (Diskussion) 11:47, 14. Sep. 2014 (CEST)
Öffentliche Berühmtheit erlangen Drehbuchautoren tatsächlich fast nie. Branchenintern gibt es allerdings durchaus Stars. Bei vielen Filmen werden Drehbücher von verschiedenen Autoren immer wieder umgeschrieben, bis es »passt«. Wer dann im Abspann steht, ist Verhandlungssachen, manchmal auch ein Streitfall.
Mir fällt gerade etwas auf: Nicht nur bei Theater, auch bei klassischer Musik steht der »Erfinder« an erster Stelle, der »Interpret« dahinter. Film und z. B. Popmusik werden originär medial vermittelt, über »Konserven«. Dort stehen die Interpreten an erster Stelle (Regisseur, Hauptdarsteller, Sänger, Band), Autoren und Komponisten im Hintergrund. Das spiegelt sich auch darin, dass es jeder für selbstverständlich hält, dass Opern und Theaterstücke neu inszeniert, Klavierkonzerte interpretiert werden. Beim Film nennt sich das »Remake«, bei Popmusik »Cover« und hat einen eher zweifelhaften Ruf. Rainer Z ... 12:01, 14. Sep. 2014 (CEST)
Mir fällt da William Goldman ein. Allein schon durch Die Brautprinzessin in der er in der Rahmengeschichte über sich selbst und Butch Cassidy and the Sundance Kid schreibt. --Mauerquadrant (Diskussion) 12:19, 14. Sep. 2014 (CEST)
Billy Wilder war ein hochangeseher Drehbuchautor, u.A. Mitarbeiter von Ernst Lubitsch, bevor er selbst Regisseur wurde. Sein Partner I. A. L. Diamond war ein Drehbuchautor der allerhöchsten Reputation. Dann fällt mir noch Ben Hecht ein, vor allem als Autor einiger Hitchcock-Filme. Aber stimmt schon, Drehbuchautoren werden in der Regel nicht so bekannt. Vielleicht auch deshalb, weil viele Top-Regisseure ihre Drehbücher selbst schreiben. Und auch Regisseure, die nicht selbst Drehbuchautor waren, Hitchcock, Lubitsch, griffen gerne in die Gestaltung des Drehbuchs ein, so nach dem Motto: "Schreib mir mal so etwas...". -- Gerd (Diskussion) 22:21, 14. Sep. 2014 (CEST)
Anders berühmt geworden, aber OT-erwähnenswert: Charlie Chaplin, u.a. zwei Oscar-Nominierungen für das beste Drehbuch. --212.184.132.111 00:51, 15. Sep. 2014 (CEST) Nachsatz: Ich denke mittlerweile, daß - wenn überhaupt - es ganz ganz selten ist, daß jemand ausschließlich über seine Arbeit am Drehbuch Berühmtheit erlangte. Ich habe gerade nochmal beispielhaft Oscar/Bestes adaptiertes Drehbuch durchgesehen. Die geläufigen Namen haben ihren Ruf immer durch andere Arbeiten erlangt. Mir ist als erstes Charlie Kaufman eingefallen, aber dessen Bekanntheit über die Branche und die Cineasten hinaus dürfte auch eher gering sein. --212.184.132.111 01:10, 15. Sep. 2014 (CEST)

Fränkische Flagge am Rathausmast obwohl verboten - wie das?

In Bayern ist es soweit ich weiß verboten die Frankenfahne an Masten von Behörden zu hissen, einzige Ausnahme: Tag der Franken am 2. Juli. In meiner Heimatgemeinde jedoch (und in vielen andren Orten habe ich das auch beobachtet) ist die fränkische Flagge bis zum heutigen Tag am 14. September am offiziellen Mast des Rathauses gehisst. Wieso? --217.225.118.121 20:44, 14. Sep. 2014 (CEST)

Der von dir verlinkte Artikel Frankenfahne gibt doch schon die Antwort - demnach ist es nämlich nicht verboten. --Magnus (Diskussion) 20:55, 14. Sep. 2014 (CEST)
Weiter oben wurde heute schon ein Tag der Heimat erwähnt, dürfte daran liegen. Auch wenn es nicht verboten ist bräuchte es ja einen Grund sie aufzuziehen.--Antemister (Diskussion) 21:17, 14. Sep. 2014 (CEST)

Kurze rechtliche Frage über Rundfunksbeitragsservice und Zahlung

Was passiert wenn ich auf "Vorbehalt bezahle" wegen bestimmten GG-Verletzungsgründen und in 2 Jahren vor dem Verfassungsgericht rauskommt, dass das Unternehmen tatsächlich gegen diese GG-Paragraphen verstößt. Bekomme ich mein Geld auch dann zurück, wenn dieser Beitragsservice Insolvenz anmeldet. Danke--Tronkenburger (Diskussion) 21:10, 14. Sep. 2014 (CEST)

Der Beitragsservice ist keine rechtsfähige Gesellschaft, auch keine solche des öffentlichen Rechts, sondern Teil der Rundfunkanstalten. Solange also die Rundfunkanstalten nicht Pleite gehen, kann der Beitragsservice das auch nicht. --Snevern 21:24, 14. Sep. 2014 (CEST)
Für mich hörte sich "Beitragsservice" wie die freundliche Umschreibung für eine Inkassogesellschaft an. In diesem Fall also: Wenn die Insolvenzmasse den Zahlungsverpflichtungen des Unternehmens entspricht, ja. Aber wenn es nur ein Beitragsservive ist, wird es wohl im Pleitefall an Insolvenzmasse mangeln. Dann also nicht. (IANAL) --Aalfons (Diskussion) 21:33, 14. Sep. 2014 (CEST)
Ich vermute stark: Wenn ein Staatsvertrag rückwirkend als verfassungswidrig erklärt wird, dann hat der Beitragsservice die Beiträge von sich aus (ohne Antrag) zurückzuzahlen; egal ob jemand die Zahlungen unter Vorbehalt geleistet hat oder nicht ."Beitragsservice" könnte ein Euphemismus sein, ähnlich Entsorgungspark. Der Service bezieht sich übrigens auf die Rundfunkanstalten, nicht auf den Bürger. --93.134.130.224 21:56, 14. Sep. 2014 (CEST)
Und was ist mit all denjenigen, die selbst keinen Beitrag zahlen, sondern sich unter Angabe der Teilnehmernummer der Mitbewohnerin oder des Mitbewohners abgemeldet haben? Bekommen die alle automatisch einen Aufhebungs- und Beitragsnacherhebungsbescheid von ihrer Landesrundfunkanstalt oder dem Beitragsservice? Müssen juristische Personen nachträglich alle vorhandenen Rundfunkempfänger und ans Internet angeschlossene Rechner einzelnachweisen, statt wie seit dem 1. Januar 2013 pauschal zu zahlen? Was ist, wenn die juristische Person im Vertrauen auf die Wirksamkeit des Rundfunkbeitragsstaatsvertrag ihren Beschäftigten der Rundfunkkonsum am Arbeitsplatz erlaubt war, der vorher wegen der Pflicht zur Anmeldung jedes einzelnen Rundfunkgeräts per Betriebsvereinbarung verboten war? Hat die juristische Person dann Schadenersatzansprüche gegen den jeweiligen Landtag, der den verfassungswidrigen Rundfunkbeitragsstaatsvertrag ungeprüft ratifiziert hat und so der juristische Person finanziellen Schaden zugefügt hat? --Rôtkæppchen₆₈ 23:53, 14. Sep. 2014 (CEST)
Habe ich keinen Beitrag gezahlt, sondern mich am Beitrag meines Wohnungsgebers beteiligt und dieser bekommt seine Beiträge in zwei Jahren zurück, habe ich ihm gegenüber einen zivilrechtlichen Erstattungsanspruch.
Sollte das BVerfG die Regelung für verfassungswidrig erklären und sie rückwirkend außer Kraft setzen, ohne sich zu Rückabwicklung oder zu Übergangslösungen zu äußern, ist mit einer Welle von Klageverfahren zu rechnen, in denen die mit dem dann eintretenden Beitragschaos auftretenden Details geklärt werden müssen.
Schadensersatzansprüche gegen den jeweiligen Landtag aufgrund der angeblich ungeprüften Ratifizierung eines später für verfassungswidrig erklärten Vertrages kann ich mir allerdings bei aller Phantasie nicht vorstellen. --Snevern 11:18, 15. Sep. 2014 (CEST)

Verkauf älterer Kunstdrucke

Meine Tante, die gerne auch mal nicht mehr gebrauchte Bücher verheizt, wollte dass ich einige ererbte Kunstdrucke mit teilweise (subjektiv) recht schönen Rahmen zum Sperrmüll bringe. Da habe ich mich geweigert, aber loswerden will muss die Dinger trotzdem, nachdem ich auch keinen Bedarf an so was habe. Jetzt ist das natürlich nichts von Wert, aber zum Wegwerfen doch zu schade. Gibt es Händler die so was kaufen (auch umsonst mitnehmen)? Wo muss ich da zu suchen anfangen?--Antemister (Diskussion) 22:49, 14. Sep. 2014 (CEST)

Sicher, dass die Sachen nix wert sind? Neben den neuzeitlichen Möglichkeiten wie Ebay - probiere es mal beim Antiquitätenhändler oder Trödler um die Ecke, ab und zu nehmen Galeristen so etwas ab oder inhabergeführte Geschäfte, die Bilder, Rahmen ... verkaufen. Wenn Du dazu Zeit und Lust hast, kann Flohmarkt funktionieren oder Du verschenkst die Sachen einem gemeinnützigen Verein mit Flohmarkt, die sie für wenig Geld weiter verkaufen. Je nach Alter und Erhaltungszustand des Rahmens sind manchmal auch kleine Heimatmuseen an so etwas interessiert. Und da gibts im Gegenzug mit viel Glück eventuell sogar eine Spendenbescheiniung. --37.49.102.64 23:29, 14. Sep. 2014 (CEST)

Suche innerhalb einer Website

Hallo Auskunftler, gibt es eine Möglichkeit eine Auflistung aller Artikel auf einer Website, deren Domain mit einer gewissen Zeichenkombination beginnt? Bspw. die Möglichkeit alle Seiten die mit „ghanafa.org/pages/blackstars/“ beginnen oder die aus „ghanafa.org/pages/*hier zwischen den Schrägstrichen kann etwas Beliebiges stehen*/201409“ bestehen aufzulisten? Oder auch alle mit „ghanaweb.com/GhanaHomePage/SportsArchive/artikel.php?ID=“ beginnenden Seiten, die hinter „=ID“ eine Zahl zwischen 300000 und 320000 haben? Die Funktion site: bei Google kenne ich bereits. Es dankt schon mal und grüßt, --Jakob Gottfried (Diskussion) 14:03, 13. Sep. 2014 (CEST)

Für die site-Suche bei Google ist dein Vorhaben deutlich zu komplex. Überhaupt werden Google und Co dir hier kaum weiterhelfen. Wenn du für die entsprechende Seite eine Sitemap hast (laut Sitemaps-Protokoll gibt es dafür keinen standardisierten Ort, aber vielleicht wirst du in der robots.txt fündig), kannst du dort die Analyse mit ein bisschen Aufwand (grep o.ä.) vielleicht selbst machen. Ansonsten musst du die Seiten der Seite wohl selbst indizieren, hoffen dass du alle Seiten findest, und dann wiederum deinen Index entsprechend aufbereiten. --YMS (Diskussion) 14:35, 13. Sep. 2014 (CEST)
Den ersten Fall erschlägst Du bei Google mit inurl:ghanafa.org/pages/blackstars . --Rôtkæppchen₆₈ 16:41, 13. Sep. 2014 (CEST)
den zweiten Fall erschlägst Du bei Google mit inurl:ghanafa.org/pages inurl:201409 . --Rôtkæppchen₆₈ 16:43, 13. Sep. 2014 (CEST)
Und das letzte ging mit curl: curl http://ghanaweb.com/GhanaHomePage/SportsArchive/artikel.php?ID=[300000-320000]. --FGodard||± 15:49, 14. Sep. 2014 (CEST)
Vielen Dank euch Dreien für die Antworten. Da ich mit Sitemaps und Robots nicht nicht sonderlich viel anfangen kann, werde ich es mit den Antworten von Rotkaeppchen68 und FGodart versuchen. Grüße, --Jakob Gottfried (Diskussion) 11:19, 16. Sep. 2014 (CEST)

Internetseite am Aktualisieren hindern

Ich möchte eine Internetseite benutzen, die sich aber alle paar Sekunden hektisch verändert, sodass man sie nicht in Ruhe lesen kann. Gibt es einen Browser oder ein Programm, das eine Internetseite am fortwährenden Aktualisieren hindert? "Seite speichern unter" speichert eine htm Datei, die sich aber nach dem Öffnen wieder genauso hektisch aktualisiert. Ein Screenshot ist auch keine Lösung, weil man darin dann die weiterführenden Links nicht anklicken kann. --62.178.147.22 19:07, 13. Sep. 2014 (CEST)

JavaScript deaktivieren? Die Frage ist nur, ob man dann überhaupt etwas sieht. Quellcode der heruntergeladenen Seite anschauen, wenn gar nichts anderes hilft. --mfb (Diskussion) 19:10, 13. Sep. 2014 (CEST)

Ist auch keine Lösung, weil man die Seite dann nicht mehr benutzen kann. (nicht signierter Beitrag von 62.178.147.22 (Diskussion) 19:20, 13. Sep. 2014 (CEST))

Mir gehen diese Seiten auf die Nerven. Ich benutze Firefox und verwende NoScript. Damit habe ich Kontrolle über die einzelnen Skripte, die auf einer Seite ausgeführt werden. Standardmäßig habe ich alle Skripte deaktiviert. Wenn ich merke, dass mein gewünschter Seiteninhalt nicht angezeigt wird, aktiviere ich Schritt für Schritt die Skripte, die mir für den Abruf des jeweiligen Inhaltes notwendig erscheinen, bis die Seite funktioniert. Das Addon ist in meinen Augen leicht zu bedienen und spart eine ganze Menge Seitenladezeit ein, wenn im Hintergrund noch tausend andere Sachen mitlaufen. --178.24.34.96 21:11, 13. Sep. 2014 (CEST)
Mir war gar nicht bewusst, dass es noch Menschen gibt, die NoScript *nicht* benutzen. Ohne wird man doch von der allfälligen Popup-Werbung regelrecht blümerant. -- Janka (Diskussion) 22:09, 13. Sep. 2014 (CEST)
Dafür gibt's ja spezialisierte Plugins. Dadurch wusste ich lange gar nicht, dass vor Youtube-Videos Werbung gezeigt wird... --Eike (Diskussion) 22:20, 13. Sep. 2014 (CEST)
Und ich bis gerade eben ... Erklärt das die Gedenksekunden vor dem Start, wenn man Adds blockiert hat? Ich dachte immer, da wird der FIlm geladen. Rainer Z ... 11:25, 14. Sep. 2014 (CEST)
*quetsch* Während dieser Sekunde kämpfen dein Adblocker und das Script, das die Werbung anzeigen will, gegeneinander. Ein sehr kurzer Kampf, und meistens gewinnt der Adblocker. --78.52.58.20 21:53, 15. Sep. 2014 (CEST)
Ich möchte jetzt gleich mal eine Lanze brechen für all die Leute, die weder Adblock noch Noscript benutzen und sich diesen ganzen Werbemüll reinziehen (womöglich noch irgendeins der beworbenen Produkte kaufen.) Ihr sorgt dafür, dass es immer noch Idioten in der Werbebranche gibt, die denken, dass man Webseitenanbietern ohne meßbare Gegenleistung Geld in den Arsch schieben sollte. Ihr seid unsere Helden. Danke! -- Janka (Diskussion) 13:57, 14. Sep. 2014 (CEST)
Ich stimme Dir zu, Janka, aber nicht komplett. In meinen Augen muss man unterscheiden zwischen 1.) Internetseitenbetreibern, die die Möglichkeit, Werbung zu schalten, schamlos ausnutzen (nach dem Motto "Wir machen es, weil wir es können! Drauf angewiesen sind wir nicht, aber vielen Dank, ihr Trottel, für die zusätzlichen Einnahmen") und 2. Betreibern, meist private, die die Inhalte erst wegen der Werbeeinnahmen anbieten können. Beispiel sind hier viele Youtuber, Entwickler kostenloser Spiele, Nachschlagewerke usw. Da habe ich nichts gegen angezeigte Werbung, weil ich im Gegenzug messbare Gegenleistungen erhalte: Youtubevideos, die das gleiche wie Fernsehen sind (nur zum Selbstaussuchen) und damit der Unterhaltung dienen, Einträge aus Wörterbüchern, Lexika usw. Wikipedia schaltet zum Glück keine Werbung, finanziert sich aber durch Spenden und kann nur deswegen (meist) hochwertige Inhalte bereitstellen. Wenn Du so willst, ist das Akzeptieren von Werbung eine andere Art der Finanzierung, mit der ich bezahle, und für gute und für mich brauchbare Inhalte mache ich das auch gerne. Nicht allerdings, und da hast Du definitiv Recht, bei Firmen, die die Möglichkeit völlig unnötig ausnutzen. Es gibt vieles im Internet kostenlos, das ist wahr, aber alles als selbstverständlich verfügbar hinzunehmen, fände ich überheblich. --178.24.34.96 18:35, 14. Sep. 2014 (CEST)
Wikipedia schaltet auch Werbung: Da sind zum einen die in die Artikel eingeschmuggelte Schleichwerbung (POV) und zum andern die regelmäßigen Spendenaufrufe. --Rôtkæppchen₆₈ 18:40, 14. Sep. 2014 (CEST)
Ich hätte auch beinahe folgendes geschrieben: "Wenn Du so willst, ist die Werbung meine Art von Spende, die ich für die Inhalte bezahle. Bei Wikipedia heißt es Spende, bei anderen Seiten heißt es Werbung". Ich hab es nicht getan, damit sich jetzt niemand aufregt, weil ich die Spenden für das Wikiprojekt mit Werbung auf Schmuddelseiten oder so gleichsetze. --178.24.34.96 19:15, 14. Sep. 2014 (CEST)
Die nicht meßbare Gegenleistung bezieht sich darauf, dass das handelnde Unternehmen eine Agentur damit beauftragt, im Internet Werbung ständig wechselnd auf tausenden von Seiten zu schalten, so dass beim Auftraggeber letztendlich niemand mehr versteht, ob die Werbemaßnahme überhaupt erfolgreich war und wenn ja/nein, warum. Anders als mit der völligen geistigen Abwesenheit der Auftraggeber in dem ganzen Internet-Werbezirkus ist die Beliebtheit z.B. von Popup-Werbung und anderen besonders nervigen Werbeformen nicht zu erklären, denn das ist so ziemlich das schlimmste, was man dem eigenen Produkt an Werbung antun kann.
Nebenbei: NIEMALS werde ich Seitenbacher-Müsli fressen oder mein Auto von Carglass reparieren lassen. -- Janka (Diskussion) 02:41, 15. Sep. 2014 (CEST)

Uringeruch nach Konsum von Steinpilzen

Wenn ich Steinpilze gegessen habe, und seien es auch nur wenige Stückchen, selbst unter einer größeren Menge einer anderen Pilzart, merke ich das spätestens beim nächsten Gang auf die Toilette anhand einer sehr prägnanten Geruchsnote im Urin, die es von der Intensität locker mit der Asparagusinsäure aus dem Spargel aufnehmen kann. Nach ausgiebigem Konsum von Dal (aus Linsen) riecht es übrigens auch so. Meine Frage: welche Substanz ist wohl dafür verantwortlich? --Blauer Berg (Diskussion) 09:14, 14. Sep. 2014 (CEST)

Von so etwas habe ich noch nie gehört, auch Google hält sich zurück. Wenn du bei Steinpizen und Linsen den gleichen Geruch wahrnimmst, müsste in beiden die gleiche Substanz vorkommen. Auf Anhieb kommen mir da Eiweißverbindungen in den Sinn. Und möglicherweise eine Stoffwechselstörung, die für ein stark riechendes Abbauprodukt verantwortlich ist. Rainer Z ... 11:46, 14. Sep. 2014 (CEST)
Bei der Asparagusinsäure ist es letztlich der Schwefel; sehr ähnlich übrigens auch bei der als Medikament genommenen Liponsäure.
Schwefel ist in den Pilzen sicherlich drin, aber wird der nun deshalb etwas absonderlich verdaut, weil man die Pilze in bestimmten Kombinationen mit anderen Lebensmitteln gegessen hat, oder weil im Stoffwechsel des Patienten etwas anders ist als bei Otto Normalpilzesser?
Besorgniserregend ist es aber, wenn der Duft "spätestens" beim nächsten Gang auf die Toilette" auftritt - entweder ist die Schwefelverbindung auch im Schweiß vorhanden und der Patient hat sehr stark geschwitzt, oder aber er hätte (sage ich mal diskret) nicht vor dem Toilettengang, insbesondere nicht vor dem Herablassen der Beinkleider urinieren sollen... Hummelhum (Diskussion) 12:25, 14. Sep. 2014 (CEST)
Ich merke das deswegen "spätestens" auf der Toilette, weil ich es vorher ja auch schon am Geschmack der Speise erkannt haben kann, daß Steinpilze enthalten sind. Der Geruch von dem ich spreche ist deutlich anders als der nach Spargel, mein Vergleich bezog sich nur auf die Intensität. --Blauer Berg (Diskussion) 12:50, 14. Sep. 2014 (CEST)
Außer natürlich, es kommt jetzt wer, der das ist, was ich nicht bin, also Chemiker, und erklärt uns, der Schwefel komme aus anderen Resourcen und der Pilz habe nur als Katalysator fungiert (vgl. fungus). Bin aber schon wieder weg. Hummelhum (Diskussion) 12:34, 14. Sep. 2014 (CEST)
Gleicht sich denn der Geruch bei Steinpilzen und Dal bzw. Linsen? Erinnert er an einen anderen, bekannteren? Merkst du sonst noch etwas auffälligen wie Verdauungsbeschwerden o. ä.? Rainer Z ... 13:23, 14. Sep. 2014 (CEST)
Ja. Keinen, den ich benennen könnte. Nicht in dem Zusammenhang. --Blauer Berg (Diskussion) 14:01, 14. Sep. 2014 (CEST)
Ich vermute mal so eine individuelle Frage kann nur ein Wissenschaftler durch Untersuchung von Patient und Urin erklären. Häufig sind solche individuellen Besonderheiten auf Fehlen oder Vorhandensein von bestimmten Enzymen zurückzuführen. Es heißt, es gäbe vermutlich für jeden Menschen einige Enzyme, die nur bei sehr wenigen Personen oder womöglich nur bei einer einzigen Person vorkommen, dieses ist der Grund warum z. B. Medikamente nicht bei allen Menschen gleich wirken. Die Wikipedia Auskunft ist zur Klärung dieser Frage nicht in der Lage.--Giftzwerg 88 (Diskussion) 13:35, 14. Sep. 2014 (CEST)
Der letzte Satz klingt so, als könntest Du ausschließen, jemand mit Kompetenz im Lebensmittelbereich könne mir eine Substanz benennen, deren Konzentration in Steinpilzen wie auch Linsen besonders hoch ist, und die bei einigen oder allen Menschen zu etwas prägnant riechbarem verstoffwechselt wird. --Blauer Berg (Diskussion) 14:17, 14. Sep. 2014 (CEST)
Sind denn auch nachfolgende Klobesucher von dem Geruch beeindruckt? Bei Spargel ist es ja so, dass man früher die Menschheit in Spargelstinker und Spargelnichtstinker eingeteilt hat. Jetzt heißt es, es gebe nur Spargelstinker, aber Spargelgestankriecher und Spargelgestanknichtriecher, Also, vielleicht hast du ja einfach nur ein besonders feines Näschen. 89.14.143.36 13:56, 14. Sep. 2014 (CEST)
So intensiv, daß er auch nach dem Abziehen noch weiter in der Luft hängen würde, ist der Geruch nun auch wieder nicht - aber selbst wenn er dauerhaft in der Luft bliebe, empfinde zumindest ich ihn nicht einmal als sonderlich unangenehm. Mein Näschen ist übrigens tatsächlich relativ fein, zumindest in Relation zu denen meines sozialen Umfelds, denn ungewöhnliche Gerüche bemerke ich zumeist als erster. --Blauer Berg (Diskussion) 14:10, 14. Sep. 2014 (CEST)

Siehe Asparagusinsäure, Liponsäure. Auf die Menge kommt es an. -- Janka (Diskussion) 14:39, 14. Sep. 2014 (CEST)

Was genau soll ich bitte im (von mir bereits verlinkten) Artikel Asparagusinsäure bzw. unter Liponsäure sehen? --Blauer Berg (Diskussion) 19:08, 14. Sep. 2014 (CEST)
Auf die Menge kommt es nun gerade nicht an; der Spargeluringeruchskenner nimmt ihn wahr, ob er nun eine halbe Stange Spargel oder ein ganzes Pfund gegessen hat. Und mehr als das hier bisher Gesagte steht im asparuginsauren Artikel nicht; im von mir verlinkten liponsauren Artikel steht hingegen rein gar nichts; dass auch die zu wahrnehmbaren Gerüchen im Urine führe, bleibt hier bisher eine gänzlich unbequellte Behauptung von mir (die aber zutrifft). Hummelhum (Diskussion) 20:20, 14. Sep. 2014 (CEST)
Du sagtest, die Intensität sei mit der bei Spargel vergleichbar. Das halte ich für ziemlich intensiv. Zartere Unterschiede (die dir vielleicht schon deutlich auffallen) wären erst mal nicht erstaunlich, es ist ja erwartbar, dass die Nahrung Auswirkungen auf die Abbauprodukte in den Ausscheidungen hat. Seltsam ist hier, dass offenbar kaum jemand vergleichbare Erfahrungen gemacht hat. Man müsste vermutlich wirklich den Urin analysieren, um da weiterzukommen. Rainer Z ... 14:46, 14. Sep. 2014 (CEST)
Diagnosen stellen alleine durch Riechen und Schmecken des Urins ist seit Dr. Eisenbart nicht mehr Stand der Wissenschaft. Alleine mit der vagen Beschreibung "prägnant" eines Geruchs wird das noch weniger klappen. Die Frage nach gemeinsamen Substanzen in Dal und Steinpilzen löst nicht das Problem, denn wenn es die gibt, müssen sie nicht die Ursache sein und verschiedene Stoffe können auf verschiedenem Wege zum gleichen Abbauprodukt führen.--Giftzwerg 88 (Diskussion) 15:38, 14. Sep. 2014 (CEST)
Ich empfinde das ganze nach wie vor nicht als Problem. --Blauer Berg (Diskussion) 20:30, 15. Sep. 2014 (CEST)
Ich habe noch nie davon gelesen oder gehört, dass Steinpilze die Zusammensetzung/den Geruch des Harns beeinflussen. Einige PIlze verändern die Farbe des Harns (z.B. Reizker), aber bezüglich Steinpilzen ist mir nichts bekannt... kann es sein, dass deine Steinpize aus irgendeinem Grund eine abnormal hohe Schwefelkonzentration haben? Aber wie schon angemerkt, kann wahscheinlich nur ien Wissenschaftler diese Frage klären Mariofan13 (Diskussion, Beiträge) 20:26, 14. Sep. 2014 (CEST)
Nein, das wäre schon extrem unwahrscheinlich, wenn alle Steinpilze, die ich in verschiedenen Ländern gegessen habe (darunter sowohl selbst gesammelte als auch gekaufte), ausnahmslos eine abnormal hohe Schwefelkonzentration gehabt hätten. Um mal zu versuchen, den Geruch in Worte zu fassen: er erinnert mich am ehesten noch an Knoblauch (den ich sehr schätze), aber ohne die scharfe Komponente, die wohl von der Schwefelverbindung Allicin herrührt. --Blauer Berg (Diskussion) 20:30, 15. Sep. 2014 (CEST)

Toshiba Satellite Akku hin?

Hallo! Mein Laptop (4 Jahre) hat einen immer schwachbrüstigeren Akku und jetzt lädt er sich sogar weder im An- noch im Aus-Zustand auf. Im Aus-Zustand brennt die Ladeanzeige-LED in rot für 10 Sekunden und geht dann aus. Im An-Zustand zeigt die Akku-Bildschirm-Ladesymbol immer weniger an, gestern noch 44%, heute nur noch 28%, und behauptet: "Im Netzbetrieb, Akku wird nicht aufgeladen", trotz ständigen Netzkabelanschlusses. Und wenn ich das Kabel rausziehe, geht der Laptop sofort aus, als ob der Akku garkeinen Strom mehr liefert.
Gibt ja wohl nur zwei Möglichkeiten: Akku hin oder Ladevorrichtung im Laptop kaputt.
Meine Frage an die Fachleute, bevor ich mir einen neuen Akku (PA3534U-1BRS) ab 20 Euro aufwärts zulege: Wie kann ich als Laie testen, ob der alte Akku hin ist oder ob mein Läppi nicht mehr auflädt?
Im schlimmsten Fall stehe ich da, habe 2 Akkus und die laden sich nicht auf. Danke für eure Hilfe. Grüßle.

--95.116.172.122 12:07, 14. Sep. 2014 (CEST)

Ich wüsste nicht, wie du die Aufladevorrichtung des Laptop ohne zweiten Akku testen könntest. Nach deiner Beschreibung ist typischerweise der Akku hin. So ein Akku ist für eine bestimmte Anzahl Ladevorgänge ausgelegt und wenn die überschritten wird, ist seine Lebenszeit irgendwann zuende. Nach vier Jahren mit regelmäßigem Betrieb liegst du da irgendwo im Sollbereich. Vor allem der Umstand, dass der Akku erst immer noch aufladbar war und dann langsam immer schwächer wurde (auch was die maximal erreichbare Ladung angeht), spricht für einen kaputten Akku - und nicht für eine kaputte Ladevorrichtung. --88.130.75.128 12:28, 14. Sep. 2014 (CEST)
Deine Beschreibung spricht für einen Akku im Streik, soll heissen der ist hinüber. Gerade weil laut deiner Beschreibung er vorher schon geschwächelt hat. Bei einem defekten Ladegerät wäre es einem Schlag von 100% auf 0%. --Bobo11 (Diskussion) 12:44, 14. Sep. 2014 (CEST)
+1. Akkus sind Verschleißteile. Ich habe es bisher nur bei NiCd-Akkus von Akkuschraubern (Marken- wie Billigware) erlebt, dass ein alter Akku das Ladegerät hinrichtet. Lithium-Ionen-Akkus haben die Ladeschaltung eingebaut. Es ist also immer der Akku mitsamt Ladeschaltung zu tauschen. --Rôtkæppchen₆₈ 12:49, 14. Sep. 2014 (CEST)
+1 Nicht basteln, Lithium-Ionen-Akkumulatoren können platzen! Ca. 3,5 Jahre nach Herstellung sind sie alles andere als neu. Wenn die Zellen nahe genug am Gehäuse sind und das Gehäuse elastisch genug ist, verformt es sich. Das ist von ausßen ersichtlich. Dann ist der Akku garantiert hin. Bleigelakkus haben diese Eigenschaft auch. Tauschen wird mit 95% hilfreich sein. Einen Hinweis: Es gibt bei den Webshops für Batterien wohl jemand weniger zuverlässiges. Das kann ich bestätigen. Hier besser einen Blick in die Bewertungsportale werfen. Hilfreich kann im Zweifel auch das Googlen nach dem Geschäftsführer sein, bevor man kauft. Das Google Suchvorschläge löschen muss, ist in diesem Fall eine Arbeitsbeschaffungsmaßnahme für staatliche Stellen. --Hans Haase (有问题吗) 13:55, 14. Sep. 2014 (CEST)
Der Akkudürfte futsch sein, meiner am Toshiba Ultrabook hat nach 2 Jahren den Geist aufgegeben. Toshiba hat für solche Fälle einen externen Dienstleister, www.ime.de - die haben kompetent geholfen. --Pölkky 18:03, 14. Sep. 2014 (CEST)

Vielen Dank! <3 Ich habe mir also einen neuen Akku bestellt. Falls aber trotzdem das Ladegerät hin gewesen sein sollte, schicke ich den Akku an WIKIPEDIA: AUSKUNFT zurück, dann könnt IHR euch drüber ärgern... :P Gruß --77.183.146.246 14:16, 15. Sep. 2014 (CEST)

Bittesehr – ((Rechtshinweis)) mit 95% sind 5% Restrisiko der Ferndiagnose. Lad uns doch ein Bild vom Akku hoch, dass Du selbst gemacht hast. Es dürfen auch beide sein. --Hans Haase (有问题吗) 19:33, 15. Sep. 2014 (CEST)

Grundgesetz

Warum steht in vielen Gesetzesentwürfen, die man als Drucksachen beim Bundestag einsehen kann, Folgendes:

Der Bundestag hat mit Zustimmung des Bundesrates das folgende Gesetz beschlossen; Artikel 79 Abs. 2 des Grundgesetzes ist eingehalten:
Artikel 1
Änderung des Grundgesetzes
Das Grundgesetz für die Bundesrepublik Deutschland in der im Bundesgesetzblatt Teil III, Gliederungsnummer ..., veröffentlichten bereinigten Fassung,

zuletzt geändert durch Gesetz zur Änderung des Grundgesetzes (Artikel ...) vom ... (BGBl. I S. ...), wird wie folgt geändert:


Aber schlussendlich hat sich am Wortlaut des GG nichts geändert. Wie kann das sein? (nicht signierter Beitrag von 141.89.215.91 (Diskussion) 14:18, 14. Sep. 2014 (CEST))

Durch Gesetzesentwürfe ändert sich das Grundgesetz natürlich nicht. Trotzdem steht im Entwurf schon mal das, was für den Fall der Verabschiedung des Gesetzes später mal vom Bundespräsidenten unterzeichnet und im Bundesgesetzblatt veröffentlicht wird. --Snevern 14:29, 14. Sep. 2014 (CEST)
Wenn Bundestag und Bundesrat zugestimmt haben, wird das aber ja im Normalfall auch tatsächlich im Grundgesetz landen... Vielleicht wär ein konkretes Beispiel gut. --Eike (Diskussion) 14:59, 14. Sep. 2014 (CEST)
Beispiel: Entwurf eines Gesetzes zur Ergänzung des Grundgesetzes um Volksinitiative, Volksbegehren, Volksentscheid und Referendum - da steht genau das drin, was der Fragesteller zitiert (Seite 2 ganz oben), aber es ist eben ein (nie Gesetz gewordener) Entwurf, folglich hat er das Grundgesetz natürlich nicht geändert. --Snevern 15:13, 14. Sep. 2014 (CEST)
Ah, ok. Man schreibt in den Entwurf schon rein, dass alle zugestimmt haben werden würden. --Eike (Diskussion) 15:33, 14. Sep. 2014 (CEST)
Sonst müsste man ja einen Entwurf nach dieser Zustimmung noch ändern, um das wiederzugeben. --mfb (Diskussion) 16:12, 14. Sep. 2014 (CEST)
Aus meiner Sicht ist das Metainformation, die nicht in den eigentlichen Gesetzestext gehört. Der Bundestag beschließt ja so, "Der Bundestag hat [...] beschlossen". Zeitschleife? Aber na gut, so hat man's offensichtlich halt organisiert. --Eike (Diskussion) 08:17, 15. Sep. 2014 (CEST)
Man könnte das als "Meta-Information" betrachten, aber es ist tatsächlich Bestandteil des Gesetzestextes, der beschlossen und verkündet wird.
In Klageschriften formuliert man: "Ich beantrage, wie folgt zu erkennen: Der Beklagte wird verurteilt, 1000 Euro zu zahlen." Damit ist der Tenor der Entscheidung des Gerichts, falls der Klage stattgegeben wird, bereits vorformuliert; manchmal kopiert das Gericht den Klageantrag 1 zu 1 ins Urteil hinein.
So ähnlich ist es hier: Der Gesetzesentwurf sieht (mit Ausnahme des Datums) exakt so aus, wie das fertige Gesetz später aussehen soll. Und die Feststellung, dass die Beteiligungsrechte des Bundesrats beachtet wurden, gehört eben einfach mit ins Gesetz, also steht's auch schon mal so im Gesetzes-Entwurf. --Snevern 11:25, 15. Sep. 2014 (CEST)

Vielen Dank. Vielleicht könnt ihr auch meine zweite Frage beantworten. Hin und wieder finde ich in solchen Anträgen folgende Formulierung:

Gesetzentwurf der Abgeordneten Dr. Edith Niehuis, Christel Hanewinckel, […] und der Fraktion der SPD

Gelegentlich steht dort anstatt Fraktion auch Gruppe. Kann man dann davon ausgehen, dass alle SPD-Anhänger hinter diesem Antrag stehen? Warum sind einige namentlich aufgeführt? Gibt es einen definitorischen Unterschied zwischen Fraktion und Gruppe? (nicht signierter Beitrag von 141.89.215.91 (Diskussion) 19:03, 15. Sep. 2014 (CEST))

Nein, man kann keinesfalls davon ausgehen, dass alle SPD-Anhänger dahinter stehen. In der Regel kann man auch nicht davon ausgehen, dass alle SPD-Parteimitglieder hinter einem Gesetzesentwurf stehen bzw. mit ihm einverstanden, und oft stehen noch nicht einmal alle SPD-Abgeordneten dahinter. Wie es in der Demokratie so ist, geschieht auch innerhalb einer Partei oder einer Fraktion Willensbildung oft durch Mehrheitsbeschlüsse oder durch Diskussion und Kompromisslösungen. Es dürfte die Ausnahme sein, dass alle Angehörigen der Fraktion inhaltlich mit einem Entwurf einverstanden sind; dennoch steht natürlich "die Fraktion" dahinter, wenn der Entwurf vorgelegt wird. Ebenso wie "der Bundestag" das Gesetz beschließt, auch wenn einige Abgeordnete dagegen stimmen, die erforderliche Mehrheit aber dafür.
Abgeordnete, die zusammen keine Fraktion bilden können, können sich stattdessen gemäß § 10 Abs. 4 GOBT zu einer Gruppe zusammenschließen. Der Bundestag muss dieser Gruppenbildung zustimmen. Eine Gruppe hat nicht dieselben Rechte wie eine Fraktion. --Snevern 19:35, 15. Sep. 2014 (CEST)
Zu der Frage: "Warum sind einige namentlich aufgeführt?": Ich kann mir vorstellen, daß es sich dabei oft um die Abgeordneten handelt, die den Gesetzesentwurf in dem entsprechenden Ausschuß mit der Opposition verhandelt und inhaltlich erarbeitet und abgestimmt haben. Denn das ist die übliche und überwiegende Praxis. Bevor ein Gesetzesentwurf ins Parlament eingebracht wird, ist er meist im Ausschuss bereits fertig ausgehandelt. Eine medienwirksame kontroverse Auseinandersetzung im Plenarsaal findet ja nur über einen sehr geringen Teil der Entscheidungen statt. Das führt dazu, daß Abgeordnete über das, was sie abstimmen, sehr oft nicht oder nur sehr oberflächlich informiert sind und im Rahmen des (und dieses Wort muß zu Sneverns Ausführungen unbedingt hinzugefügt werden) Fraktionszwanges die Hand heben, bzw. ihr Kärtchen abgeben, weil das Verhandlungsergebnis von Bundeskanzler oder Ministern bzw. Fraktionsführung und Parteivorstand abgenickt ist. Zitat: "Es gibt noch seltsamere, für die Allgemeinheit noch weniger interessante Gesetzgebungs- und Beratungsmaterien. Über sie wird wohltuenderweise ohne eine Aussprache meist am Schluß einer Sitzung Beschluß gefaßt, wenn sich die Fraktionen über das Ergebnis einig sind und die Experten eine Hervorhebung im Bundestag für überflüssig ansehen. Dazu muß man wissen, daß über 90 v.H. der Gesetzentwürfe vom Bundestag einstimmig oder nahezu einstimmig verabschiedet werden.'' (Klaus Kremer: Präsenz im Plenum. Erwägungen über die Pflichten des Abgeordneten, an den Arbeiten des Bundestages, besonders an den Plenarsitzungen, teilzunehmen. In: Werner Blischke, Hans-Achim Roll (Hg.): Plenarsitzungen des Deutschen Bundestages: Festgabe für Werner Blischke. Duncker & Humblot, 1982, (Band 4 von Beiträge zum Parlamentsrecht), S. 22) --87.149.166.189 11:01, 16. Sep. 2014 (CEST)

Hadoques Hut aus Tim und Struppi

Kann mir jemand sagen was Hadoques Hut für einen Namen hat? Thx--Sanandros (Diskussion) 22:54, 14. Sep. 2014 (CEST)

Weiss nicht, ob das einen bestimmten Namen hat. Im Artikel Dreispitz wird als Vorläufer ein "runder Hut mit breiter Krempe der spanischen Soldaten (Achtzigjähriger Krieg und Reunionskrieg (1683–1684)) (...) bei dem die Krempe zuerst auf einer Seite (...) hochgeschlagen wurde" genannt. Das wird es vermutlich sein. --King Rk (Diskussion) 23:06, 14. Sep. 2014 (CEST)
Kapitänsmütze?--Optimum (Diskussion) 23:13, 14. Sep. 2014 (CEST)
Die trägt Haddock. Es geht aber um seinen Vorfahren Hadoque ;) --King Rk (Diskussion) 23:14, 14. Sep. 2014 (CEST)

Nun ja mit anderen Worten, was für Suchbegriffe muss ich bei Goolge eintippen damit ich den Hut rausbekomme den Hadoque trug?--Sanandros (Diskussion) 01:16, 15. Sep. 2014 (CEST)

Musketiere und Pikeniere aus der Zeit des Dreißigjährigen Krieges (Heeresgeschichtliches Museum Wien) - der Herr links...
Schlicht Schlapphut, allerdings der schwedischen Soldaten im dreißigjährigen Krieg, statt der spanischen im achtzigjährigen. !7. Jahrhundert kommt aber hin, war damals also wohl international. Geoz (Diskussion) 01:18, 15. Sep. 2014 (CEST)
Ein Fantasiegebilde? Im Franz.: chapeau à plume - also etwas Musketierartiges etwa wie der hier ... Musketierschlapphutmitfedern... GEEZER… nil nisi bene 09:49, 15. Sep. 2014 (CEST)
OK mit Filz und Schlapphut kam ich dann auf den Edelmannhut - Mittelalter.--Sanandros (Diskussion) 10:01, 15. Sep. 2014 (CEST)
Dies Modell um 1652 kommt dem des Hadoque sogar farblich recht nah.
Mittelalter? Zur Zeit des Sonnenkönigs war selbst das Spätmittelalter schon lange vorbei. Der Chevalier Hadoque trug Hüte des Barock (reim dich, oder ich fress dich!) wobei sein spezielles Exemplar eine ungewöhnlich hohe Hutkrone aufwies, womöglich also tatsächlich eher schwedische Mode, als französische, mit eher flachen oder runden Kronen. Wenn es aber nur um die Anfrage in einem Kostümverleih geht, dann kommst Du mit "Musketierhut" wohl am weitesten. Geoz (Diskussion) 12:29, 15. Sep. 2014 (CEST)
Am ehesten ein en:Cavalier hat. Laut Der Schatz Rackhams des Roten sank Hadoques Schiff 1676. Dann passt sowas hier: http://www.metmuseum.org/collection/the-collection-online/search/436616 . Das ist 1623, würde zur zeitlichen Ansiedelung uuungefähr passen. Oder der Vermeer hier Der Soldat und das lachende Mädchen, 1658, noch näher dran. --64.214.128.118 17:41, 15. Sep. 2014 (CEST)
Laut diesem Hutlexikon ist "Kavaliershut" auch im Deutschen ein (zeitlich auf Anfang bis Mitte des 17. Jahrhunderts eingegrenztes) Synonym für den Schlapphut. Ich pflege diese Info mal in unseren Artikel ein und versuche die Interwikilinks zu erstellen. Handelt es sich jetzt aber bei Kavaliersdelikten jetzt um "schlappe Delikte", oder um "Delikte von Schlappen"? Geoz (Diskussion) 22:29, 15. Sep. 2014 (CEST)
Kavaliersschmerzen sind alles andere als schlapp. --Rôtkæppchen₆₈ 23:49, 15. Sep. 2014 (CEST) *scnr*

Wäre ein Asperger-Autist ein guter Elfmeterschütze?

--79.255.51.133 13:56, 10. Sep. 2014 (CEST)

Es existieren keine Untersuchungen zu diesem Thema. --Eike (Diskussion) 14:01, 10. Sep. 2014 (CEST)

Asperger-Autist zu sein ist schwerlich hilfreich für irgendeine motorische, Geschicklichkeit und Geschwindigkeit erfordernde Tätigkeit. Sicher gibt es solche, die das genauso gut können wie "normale" Menschen... --Anonyme Frage (Diskussion) 14:08, 10. Sep. 2014 (CEST)

Verwechsle Asperger nicht mit Kanner. Bei letzteren ist die motorische Fähigkeit eingeschränkt.--Hubertl (Diskussion) 14:44, 10. Sep. 2014 (CEST)
Stimmt, Verallgemeinern liefert Überraschungen.
Es gibt Untersuchungen, welche Sportarten nicht optimal und welche besser für AAs sind. Fussball als schneller Team-Sport ist nicht so günstig (= ungünstig). Elfmeterschützen werden ja nicht extra zum Schuss hineingeholt, sondern kommen aus der Mannschaft; das nicht lesen können des Torwarts (nicht durch ihn abgelenkt werden) könnte sogar ein Vorteil sein. Trotzdem: MANNSCHAFT wird grossgeschrieben - nicht so gut.
Bei völlig isolierten "Spielen" kann ein AA high precision erreichen. Ein anderer Team-Sport, der schon mal besser als Fussball ist, ist beispielsweise Bowling: Es gibt zwar eine Mannschaft, aber vor der (präzisen) Leistung (die man allein erbringt), kann man überlegen und "der andere" muss nicht unbedingt gelesen werden. Bogenschiessen wäre vermutlich auch interessant... GEEZER… nil nisi bene 14:54, 10. Sep. 2014 (CEST)
Meine Wortmeldung entspringt durchaus nicht unfundierten Vorurteilen gegen Autisten - und diese, dass 90% der Asperger schlecht in Sport sind, ganz bestimmt auch nicht. Individuelle Ausnahmeleistungen widersprechen dem nicht. --Anonyme Frage (Diskussion) 15:33, 10. Sep. 2014 (CEST)
nicht unfundierte Vorurteile - das merke ich mir... ;-)
Ich behaupte nicht, dass sie im Sport supertoll sind, aber 80 % deutsche Doppelwhopper auch nicht.
Fussball ist sicherlich keine ideale Sportart, wenn man als Mensch mit Koordinierungsschwierigkeiten Sport machen will (Frage oben). Aber es gibt anderen Krempel, den die ausprobieren können, um z.B. Koordination zu verbessern. GEEZER… nil nisi bene 17:04, 10. Sep. 2014 (CEST)
Überheblicher Nonsens. Fußball kann genauso geeignet sein, wie jede andere Sportart. Gerade für solche Autisten, die im Bereich Koordination oder Teamplay Defizite haben, wäre Fußball klar geeignet, um diese Fertigkeiten zu trainieren – solange der betreffende Freude daran hat und die Mannschaft bereit ist, den Mitspieler zu integrieren. --178.0.193.156 12:40, 11. Sep. 2014 (CEST)
Die ursprüngliche Frage war aber nicht, ob Fußball gut für einen Asperger-Autisten ist, sondern ob ein Asperger-Autist ein guter Elfmeterschütze ist. Um ein guter Elfmeterschütze zu werden muss man normalerweise auch ein gute Fußballer sein, damit man Profi werden kann (klar kann man auch als nicht-Profi ständig Elfmeter trainieren, aber das ist dann ohne gute Anweisungen durch einen Trainer und ohne eine echte Herausforderung/Prüfung der Fähigkeiten durch einen guten Torwart). Um ein guter Fußballer zu werden, muss man natürlich gut im Team zusammenarbeiten können. Dazu reicht es nicht, dass man von der Mannschaft akzeptiert wird, man muss auch vorausahnen können, was die Mitspieler machen werden. Asperger-Autisten dürften Schwierigkeiten haben, andere Menschen zu "lesen", andererseits können sie gut analysieren, daher ist es schwierig zu sagen, ob sie z.B. die Laufwege und Pässe der Mitspieler eher besser oder schlechter erahnen als Menschen ohne irgendein psychische Syndrom. --MrBurns (Diskussion) 23:01, 13. Sep. 2014 (CEST)
Ein Asperger-Autist kann ein guter Elfmeterschütze sein, wenn er die dafür notwendigen Fähigkeiten mitbringt. Es ist vorstellbar, daß er seinen Autismus dabei zu seinem Vorteil einsetzen kann, wenn es darum geht, den Torwart auszutricksen, weil es für viele Menschen schwierig ist, Mimik und Körpersprache eines Autisten zu interpretieren. --178.0.193.156 21:26, 11. Sep. 2014 (CEST)
Typisch für Asperger ist umgekehrt die mangelnde Fähigkeit, Mimik und Körpersprache anderer Menschen zu interpretieren. In Wettstreiten um wechselseitige Beeinflussung und Bluffen haben Asperger - im Schnitt - ganz schlechte Karten. Dieser Nachteil lässt sich umso eher ausgleichen oder gar umkehren, als Zeit für eine rationale Analyse der Situation und ihrer Was-wäre-wenn-Szenarien zur Verfügung steht. Das ist beim Elfmeterschießen nicht gegeben. --Anonyme Frage (Diskussion) 10:29, 12. Sep. 2014 (CEST)
Nope, zu kompliziert gedacht. In der Elfmetersituation ist zunächst mal der Schütze der Akteur und der Torwart kann nur reagieren oder auf gut Glück ins Leere springen. Wenn der Schütze für den Torwart nicht lesbar ist in seinem Vorhaben, ist das ein Vorteil für den Schützen. Der Nachteil, den Torwart nicht lesen zu können, ist demgegenüber gering. Auf die kurze Distanz muß der Schütze auch nicht unbedingt antäuschen, es reicht, wenn der Torwart nicht vorhersehen kann, wohin der Ball gehen wird, demnach bleibt ein Vorteil für den Schützen. Es geht ja nur um einen Elfmeter - bums und rein - nicht um integratives Sozialverhalten. --178.4.178.242 15:42, 12. Sep. 2014 (CEST)
Bei einem wirklich guten Elfmeter ists wurscht, ob der Tormann erratet, in welches Eck er geht, perfekt geschossene Elfmeter sind unhaltbar. --MrBurns (Diskussion) 23:01, 13. Sep. 2014 (CEST) PS: Das Erraten vom Eck ist natürlich etwas, das einen gewissen Einfluss auf die Elfmeterstastik hat, weil keiner jeden Elfmeter perfekt schießen kann, wichtiger ist aber das Fußballerische Können und vor Allem die Nervenstärke (die meisten Fußballer könnten ohne den psychischen Druck wahrscheinlich ca. 90% ihrer Elfmeter unhaltbar schießen, die durchschnittlich Elfmetertorquote liegt ziemlich unabhängig vom Niveau des Bewerbs im Profibereich bei ca. 75-80%, beim Elfmeterschießen ist sie noch etwas niedriger und nicht jedes Elfmetertor entsteht aus einem perfekt geschossenen Elfmeter). --MrBurns (Diskussion) 23:09, 13. Sep. 2014 (CEST)
Jeder Asperger-Autist hat seine eigene Superkraft (wie bei den X-Men), von daher glaube ich das es bestimmt welche gibt die als Torwart super sind. Gruß--78.51.231.191 17:00, 15. Sep. 2014 (CEST)
Das ist eine etwas naive Annahme, wenn auch mit durchaus wahrem Kern. Allerdings sind diese "Superkräfte" selten sportlicher Natur, sondern eher im Gedächtnis, analytischen Denken und sonstigen geistigen Leistungen zu finden. Vielleicht könnten sich manche Asperger-Autisten darauf trainieren, besonders zielsichere Schützen unhaltbarer Elfmeter zu sein. Selbst dann ist es sehr gut möglich, dass sich so ein Asperger von unsportlichen Ablenkungen (wie von Tim Krul im Elferschießen gegen Costa Rica bei der WM) ganz besonders leicht irritieren lässt. --Anonyme Frage (Diskussion) 10:01, 17. Sep. 2014 (CEST)
Asperger-Autisten und analytisch denken? Keine Ahnung, was DU unter analytischem Denken verstehst, aber dein Beitrag lässt eher das Gegenteilige vermuten. Wir sind im Übrigen hier bei der Auskunft und nicht auf der Asperger-Disku.--Hubertl (Diskussion) 11:02, 17. Sep. 2014 (CEST)
Die OP-Frage und damit die gesamte Diskussion, inklusive deiner Beiträge, drehen sich direkt um die Stärken und Schwächen von Aspergern. Diese aggressive Reaktion ausgerechnet gegen meinen Beitrag, verbunden mit einer abwertenden Äußerung gegen die analytische Denkfähigkeit von Aspergern vermag ich nicht als konstruktiv zu beurteilen und rate von weiterem persönlichen Streit ab. --Anonyme Frage (Diskussion) 11:26, 17. Sep. 2014 (CEST)

Soziale Netzwerke

Hallo. ICh habe ein Problem. Ich bin eher ein Außenseiter. In meiner Klasse sind alle außer ich bei solchen sozialen Netzwerken dabei wie Facebook oder Whatsapp. Einige Mitschüler, die es gut meinen, drängen mich dazu mich ebenfalls dort anzumelden.

Hilft das wirklich was?.


glaubt ihr eher, dass sie mich dann mehr ärgern werden, wenn sie mich besser kennen und merken, dass ich anders bin. Sie sagen auch, ich soll einmal am Wochenende beim "fort gehen" mitkommen = saufen gehen. Aber das ist nichts für mich. Viele suchen dort auch nach Mädels. Wenn man am Abend in der Stadt ist, sieht man ja wie offen solche angeheiterten Mädls sind. Aber das ist leider genau die Type von Mädels, was ich nicht mag.

Eine weitere Frage: gibt es auf solchen sozialen Netzwerken eine möglichkeit, auch ein Mädel kennenzulernen. Also ein "normales" Mädel was nicht bei solchen Saufereien mitmacht. Gibt es heutzutage überhaupt noch Mädels, was nicht bei diesen sozialen Netzwerken mitmachen? Wo findet man die? Wie man sieht z.B. im Zug starren alle Jugendlichen in diesen Smarttrottel und merken gar nicht was rundherum vorgeht. Komplett zugestöpselt. Kein Wunder dass die alle so deppert sind.

Ich hoffe ein paar von euch können mich verstehen.

--89.144.229.120 11:14, 12. Sep. 2014 (CEST)

Kann es sein, dass du dasselbe vor Monaten und Jahren [46] [47] schon gefragt hast?
Wie dem auch sei... Die allermeisten jungen Menschen dürften in sozialen Netzwerken aktiv sein, und ehrlich gesagt, ich würde mich an deiner Stelle auch anmelden (auch wenn ich hiermit vermutlich Protest ernten werde). Es trägt natürlich das Risiko, dass du dann auch da zu sehr geärgert wirst - aber dann kann man sich immer noch wieder abmelden.
Aber es gibt ja offensichtlich Leute, die es gut mit dir meinen. Sagst du ja selbst. Triff dich mit denen - offline und online - und schaut, was ihr für gemeinsame Interessen habt. Selbst die saufen wohl kaum rund um die Uhr.
Der entscheidende Punkt ist aber wohl, dass du Angst hast, dass die Leute dich kennenlernen, weil du "anders" bist. Du musst damit anfangen, dich selbst zu mögen, sonst werden die anderen damit auch Schwierigkeiten haben.
--Eike (Diskussion) 11:22, 12. Sep. 2014 (CEST)


Verstehe ich richtig: Du beklagst dich darüber, dass du mit deinen Mitschülern keine gemeinsamen Interessen hast. Gleichzeitig erwägst du aber, Ihren Rat anzunehmen, an einem "sozialen Netzwerk" teilzunehmen, in dem sich eben diese Leute, mit denen du keine gemeinsamen Interessen hast, ausnahmslos tummeln. Das ergibt keinen Sinn. *Entweder* du machst da mit und triffst auf genau dieselben Leute wie im RL, was das Netzwerk superfluid macht. Oder du machst da mit und meidest diese Leute, was du ohnehin jederzeit und ohne irgendeinen Ratschlag von irgendwem tun könntest (mit offenem Ergebnis), genau wie die dritte Möglichkeit, es zu lassen. -- Janka (Diskussion) 14:40, 12. Sep. 2014 (CEST)

Dieser Abschnitt kann archiviert werden. Ewig wiederkehrende Trollerei, um die Regulars zum Labern über Beziehungsanbahnung zu animieren. Langweilig.
Ich befürchte ja Schlimmeres - dass er tatsächlich seit fünf Jahren dasselbe Problem hat... --Eike (Diskussion) 14:43, 12. Sep. 2014 (CEST)
Dem Schreibstil nach zu urteilen handelt es sich um eine Trollanfrage. Löschen wäre besser als beantworten. --Rôtkæppchen₆₈ 14:46, 12. Sep. 2014 (CEST)
(BK) Ich glaube auch nicht, daß die Frage ernst gemeint ist, antworte aber trotzdem, denn es sieht schon so aus, als hättest Du wirklich ein Problem. Die Frage „Hilft das?“ Steht im luftleeren Raum. Wobei soll das helfen? Was willst Du erreichen? Sich einfach nur anmelden und dann laufen lassen wäre denkbar naiv und riskant. Falls es Dir darum geht, kein Außenseiter sein zu wollen, Du aber andererseits nichts mit den anderen unternehmen willst, dann hülfe es auch nicht weiter, sich in sozialen Netzwerken anzumelden. Wenn es Dir darum geht, mit anderen innerhalb sozialer Netzwerke zu interagieren, dann überlege Dir ganz genau, wie das aussehen soll und passe sowohl die Benutzereinstellungen als auch Dein Nutzungsverhalten entsprechend an. BTW: Man kann auch mit anderen fort gehen, die gerne saufen und angetrunkene Mädels anbaggern, ohne mit zu saufen oder zu baggern, quasi mit dabei aber trotzdem noch anders/Du selbst. Ich an deren Stelle würde mir das allerdings überlegen, denn Du hältst die ja alle für „deppert“ und so jemand ist ein Stimmungskiller. --178.4.178.242 14:48, 12. Sep. 2014 (CEST)

Als erstes möchte ich mich entschuldigen, dass diese Frage nicht so gut rüberkommt. Die beiden anderen Fragen von vor 5 Jahren sind nicht von mir! Mir geht es nicht um Sex (das wird sich im Laufe einer Beziehung schon von selbst ergeben). Sondern ich suche ein nettes Mädel, ds Aussehen ist egal. Wichtig ist der Charakter.

"Man kann auch mit anderen fort gehen, die gerne saufen und angetrunkene Mädels anbaggern": Ich habe schon oben geschrieben, dass solche Mädels nicht mein Typ sind. Deshalb hilft mir dieser Vorschlag nicht weiter.

Da ich mich nicht wirklich gut auskenne, kann ich es nur vermuten, aber bei solchen sozialen Netzwerken sind doch nicht nur solche "Säufer"-Typen dabei. Fast alle Jugendlichen was ich sehe starren unentwegt in ein Smartphone. Vielleicht sind auch welche dabei, die ähnliche Interessen haben wie ich? Wo kann man sonst noch Mädels kennen lernen, außer durch Zufälle (z.B. selbe klasse, oder im selben Autobus), beim Fortgehen oder bei sozialen Netzwerken??

Gibt es heutzutage noch Mädels, die nicht bei solchen sozialen Netzwerken sind?

Ich danke allen, die sich die Zeit genommen haben, um mir zu antworten. --89.144.232.123 20:43, 12. Sep. 2014 (CEST)

Warum willst du keine, die bei sozialen Netzwerken sind? Du schließt damit grundlos aus dem Fingern gesogene 80% aus. --Eike (Diskussion) 21:17, 12. Sep. 2014 (CEST)



Weil auf solchen sozialen Netzwerken sind nur solche Partygirls oder Emo-Girls. Wenn man schaut jedes Profil mit so einem Handyfoto vor dem Spiegel, da wird mir schon schecht. Aber keine natürlichen Mädels, die ernsthaft eine längere Beziehung wollen. Außerdem finde ich es nicht angenehm, wenn jeder lesen kann, was ich gerade mache, ob und mit wem ich in einer Beziehung bin und der ganze Blödsinn, der nicht mal der Rede wert ist, dokumentiert zu werden (z.B. diese hunderten Selfies was alle machen heutzutage). Meine Großmutter sagt, früher war es einfacher, ein Mädel kennenzulernen.--89.144.192.153 12:31, 13. Sep. 2014 (CEST)

Woher hast du die "Information", dass da nur Party- und Emogirls wären? Das ist Quatsch.
Warum glaubst du, dass da jeder lesen könnte, was du machst? Das ist Quatsch.
--Eike (Diskussion) 15:10, 13. Sep. 2014 (CEST)

Ich weiß es wie gesagt nicht genau, weil ich nicht bei solchen sozialen Netzwerken angemeldet bin. Zitat von Herrn Janka: "Gleichzeitig erwägst du aber, Ihren Rat anzunehmen, an einem "sozialen Netzwerk" teilzunehmen, in dem sich eben diese Leute, mit denen du keine gemeinsamen Interessen hast, ausnahmslos tummeln. Das ergibt keinen Sinn."

Also habe ich angenommen, dass dort solche Typen angemeldet sind. Für mich ist "Beziehung" aber nicht ein paar Wochen mit einer "gehen" und dann ist es wieder vorbei. Sondern was längerfristiges. Und da muss halt der Charakter stimmen.

Ich denke aber, dass es irgendwo sicher ein Mädel gibt, dass zu mir passen würde, also eher Außenseiterin und nicht bei allen Partys und Saufereien dabei. Nur die Frage ist, wie ich sie je kennenlernen soll???--89.144.216.238 18:19, 13. Sep. 2014 (CEST)

wenn Du Probleme mit sozialen Kontakten hast, wir die Lösung sicher nicht im Rückzug ins Internet zu finden sein. Ich folgere aus Deinem letzten Beitrag, dass Du jede ggf. aufkommende Beziehung mit zu hohen Anforderungen killen wirst. Wenn sich zwei Menschen kennenlernen und der ein stellt (ggf. innerlich) gleich Forderungen "muss langfristig sein" "Charakter muss stimmen" ist das einer Beziehung keinesfalls förderlich. Wenn Du die Probleme schon so lange wie oben genannt hast, ggf. psychologischen Rat einholen. Davon abgesehen hat es ein Außenseiter immer schwerer, da Frauen nun mal tendenziell sozialkonform sind.- andy_king50 (Diskussion) 20:09, 13. Sep. 2014 (CEST)

Danke dass du dich über mich lustig machst. Ich habe das wirklich ernst gemeint und ich glaube auch weiterhin, dass ich mit "Charakter muss stimmen" auf Dauer Recht habe. Dass es keine weiblichen Außenseiter ist Quatsch, (um es mit Eikes Worten auszudrücken). Es gibt genauso beliebte und schüchternere Mädels. Nochmal meine Frage: Wie und wo schaffe ich es so ein Mädel kennenzulernen?? --89.144.201.59 14:45, 14. Sep. 2014 (CEST)

Vor allem klappt es nicht mit dem Vorsatz »Ich will eine Freundin finden«. Das hat noch nie funktioniert. Natürlich hilft es, sich unter Menschen zu begeben und das möglichst da, wo man im weiteren Sinne Gleichgesinnte trifft. Und dann noch was wichtiges: Der eigenen Idealvorstellung wird man nie begegnen. das wäre letztlich auch fad. Wer Freunde oder eine Freundin finden will, sollte eine gewisse Neugier und Offenheit mitbringen. Das wünscht du ja auch dir gegenüber. Auf die Frage »Wie und wo schaffe ich es so ein Mädel kennenzulernen??« gibt es kein Rezept als Antwort. Rainer Z ... 17:11, 14. Sep. 2014 (CEST)
Wie es aussieht, behauptest Du einerseits, nicht zu wissen, was Du tun sollst und hier Rat zu suchen, andererseits widersprichst Du jedem, der Dir antwortet und glaubst offenbar ständig, alles besser zu wissen. Ich vermute zu ahnen, warum Du kein Mädel mit stimmigem Charakter findest. Entweder bist Du wirklich so, wie Du es hier beschreibst, dann denk mal ernsthaft über die Antworten nach, oder Du findest es lustig, hier Unfug zu posten, was Dich als pubertären Schwachmaten ausweisen würde. Ich persönlich glaube Dir kein Wort. --178.4.111.125 18:01, 14. Sep. 2014 (CEST)
Noch ein Kind und eine Sexualmoral aus dem 19. Jahrhundert und mit ebenso angestaubt wirkender Kulturkritik aufwartend... Es gibt diverse Möglichkeiten sozialer Interaktion, in politischen Vereinigungen, bei Konzerten oder Tanveranstaltungen, in Lesekreisen oder auch online im Kreis der Autorenschaft der Wikipedia. Kannst du alles ausprobieren und bei allem können sich Beziehungen zu anderen Menschen entwickeln, ob das nun Mädels, Jungen, sonstige oder gleich zwei davon sind, mit denen du gepflegte Konversation, Geschlechtsverkehr oder langfristige Liebesbeziehungen pflegst, wird sich dann herausstellen. --78.25.123.155 18:05, 16. Sep. 2014 (CEST)

Namensrecht bei Doppelstaatsbürgerschaft

(Ja, den Hinweis zu Rechtsthemen und Haftungsausschluss habe ich zur Kenntnis genommen.)

Ich bin ein in Großbritannien wohnhafter gebürtiger Deutscher, mit sowohl deutscher als auch neuerdings britischer Staatsbürgerschaft. Wenn ich meinen Namen unter britischem Recht ändern lasse, kann ich dann einfach so beantragen, dass der Name auch in den deutschen Amtsregistern geändert wird? Oder muss ich weiterhin die gleichen Kriterien erfüllen, wie Leute, die ihren Namen einfach gleich in Deutschland ändern lassen? Ich habe Angst, dass ich, wenn ich meinen Namen hier ändern lasse, nachher mit zwei verschiedenen Namen in meinen zwei Pässen dastehe, was natürlich etwas suspekt aussieht (und auch nicht gerade praktisch ist)

Herzlichst

(pers. Info entfernt, zum Schutz des Fragestellers)

--89.243.122.37 20:13, 12. Sep. 2014 (CEST)

Oha, die Ordnungshut ist da! Also, dann wollen wir mal die Frage brav beantworten:
"Führt ein Deutscher, der auch eine ausländische Staatsangehörigkeit besitzt, nach dem Recht des ausländischen Staates, dessen Staatsangehöriger er auch ist, einen anderen Familiennamen als den, den er nach dem Recht im Geltungsbereich des Gesetzes zu führen verpflichtet ist, so kann die »hinkende Namensführung« dadurch beseitigt werden, dass der im Geltungsbereich des Gesetzes zu führende Familienname in den Familiennamen geändert wird, der nach dem Recht des anderen Staates zu führen ist. Soll dagegen der andere Familienname aufgegeben werden, so ist der Betroffene an die Behörden des Staates zu verweisen, dessen Staatsangehörigkeit er auch besitzt." (Allgemeine Verwaltungsvorschrift zum Gesetz über die Änderung von Familiennamen und Vornamen, Nr. 49).
Kurzfassung in deutscher Sprache: Ja, geht.
Zufrieden? --Snevern 21:24, 12. Sep. 2014 (CEST)
Gut gemacht. Ein Gummipunkt für Snevern. --88.68.82.17 21:19, 13. Sep. 2014 (CEST)

Gilt das beliebig oft? --Eingangskontrolle (Diskussion) 16:46, 13. Sep. 2014 (CEST)

Es steht jedenfalls nirgendwo, dass es nur genau einmal geht. Ich bezweifle allerdings, dass die Frage praktische Relevanz besitzt, denn für die Anwendung ebendieser Vorschrift müsste der Antragsteller seinen Namen "beliebig oft" in England wechseln, um dann jedesmal in Deutschland die Beseitigung der "hinkenden Namensführung" zu beantragen. Ich wäre nicht überrascht, wenn der zuständige Beamte nach dem zweiten, spätestens dritten Antrag einfach mal einen ablehnenden Bescheid erlässt und die Entscheidung damit auf die Widerspruchsbehörde bzw. das Verwaltungsgericht abwälzt. --Snevern 19:10, 13. Sep. 2014 (CEST)
Im Ergebnis stimme ich dir zu, allerdings meine ich mich zu erinnern, dass es in England sehr einfach ist, seinen Namen zu ändern, ich mein, man kann das praktisch alleine machen und somit auch beliebig oft, ohne dass man für den Namenswechsel großen Aufwand hätte. Ob es eine gute Idee ist, jede Woche anders zu heißen, steht freilich auf einem anderen Blatte. --88.130.95.148 23:03, 13. Sep. 2014 (CEST)
Die Frage war nicht, ob es in England einfach ist, den Namen zu wechseln, und wie oft das dort möglich ist - die Frage war, wie oft man nach einem solchen Namenswechsel dann in Deutschland seinen Namen anpassen lassen kann. --Snevern 23:11, 13. Sep. 2014 (CEST)
Wenn man den Namen im Ausland nicht mehr ändern kann, hat sich auch die Frage einer Anpassung erledigt. --88.130.95.148 23:14, 13. Sep. 2014 (CEST)
Man kann in England eben beliebig oft. Und dann eröffne ich jedesmal mit den neuen Papieren ein Konto und bürge für mich selbst, da ja ein Notar auch "von Person bekannt" beglaubigt und ich ihm ja meinen neuen Namen nicht unbedingt nennen muß... Eine ganz schlechte Idee, das jemand ausländisches Recht so ungefiltert die Oberhand gewinnen lässt. Das zeugt von mangelndem Selbstbewußtsein. --Eingangskontrolle (Diskussion) 23:54, 16. Sep. 2014 (CEST)

Neue Lebensformen

Wo hat Milchkefir gelebt, bevor es Milch in Gefäßen gab? Ist er erst in den letzten paar tausend Jahren entstanden? 88.73.198.112 20:39, 14. Sep. 2014 (CEST)

Du meinst die Pilzart, mit der Kefir angesetzt wird. Den Pilz gab es bestimmt schon vor den Dinos. Wo er gelebt hat (Wasser?) weiß vielleicht jemand anderes. Das mit der Kefirkultur ist sicher eine Zufallserfindung. --93.134.130.224 20:49, 14. Sep. 2014 (CEST)
Kefir lesen und nachsehen, wo die verwendeten Mikroorganismen herkommen (in der Natur vorkommen).
Analog könnte man fragen, was die Bierhefe vor dem Bier gemacht hat.
Es sind halt Mikroorganismen, die ihre ökologische Nische hatten - und haben - und die wir für unseren Nutzen ins Rampenlicht geholt und dabei tatsächlich ein bissl modifiziert (= "gezüchtet") haben. Wie Hundis, Kühe, Schweine, uns selber etc. GEEZER… nil nisi bene 21:48, 14. Sep. 2014 (CEST)
Die Bierhefe hat auch vorher schon Zucker zu Alkohol vergoren, das passiert in jedem vergammelnden Apfel. Aber Milch kommt - im Gegensatz zum Zucker - in der Natur überhaupt nicht gediegen vor. Sie ist entweder im Euter und dort durchs Immunsystem geschützt oder im Verdauungstrakt von Säugetieren und dort nur kurze Zeit noch Milch, bevor sie denaturiert und absorbiert wird. Und Milchsäurebakterien kommen auch aus dem Zerfall von Pflanzenmaterial, z.B. Sauerkraut oder Silage vergammelt beispielsweise milchsauer. Aber bei Kefir? --88.73.198.112 22:01, 14. Sep. 2014 (CEST)
Mit "Bierhefe" meinte ich, dass die derzeit verwendeten gut bekannt, charakterisiert und propagiert sind. Genauso wie die Kefir-Combo in der Kefir-Fabrik. Jeder, der selber einen Kefir-Karl-Heinz hatte - üblicherweise während der Studentenzeit, wenn man diese Liebesgabe einer Dame nur ungern ausschlägt - und nicht genau die Anweisungen befolgt, bekommt man sehr bald ein Ungleichgewicht und sehr unattraktive Resultate. GEEZER… nil nisi bene 09:37, 15. Sep. 2014 (CEST)
Was genau meint das? Dass der Kefir evolutionär insgesamt instabil ist? Würde ich verneinen, immerhin wird er ja weg geworfen, wenn er die Zusammensetzung seiner Organismen zu seinen Ungunsten verändert. Jedenfalls ist es mir bei Wasserkefir schon mehrmals gelungen, die Essigsäuregärung zurück zu drängen, weil die übrigen Organismen des Wasserkefis bereits gelernt haben, dass sie nicht überleben wenn sie sich nicht gegen die Essigsauren durchsetzen und dementsprechend in die Vorhand gehen, sobald sie die Gelegenheit haben. Ich bin also sicher, dass der Milchkefir echt stabil ist, aber er vermehrt sich eben nur seit es Milch in "gediegener" Form gibt, und das ist noch nicht sehr lange.88.73.198.112 18:03, 15. Sep. 2014 (CEST)

OT eine Stilblüte aus dem Artikel Kefir: „Die häufig anzutreffende Aussage über die angeblich lebensverlängernde Wirkung von Kefir geht auf eine probiotische Schrift des russischen Bakteriologen Ilja Iljitsch Metschnikow aus dem Jahre 1908 zurück.“ Soso, eine probiotische Schrift. Bisher kannte ich nur probiotischen Joghurt mit probiotischen Bakterien. --Rôtkæppchen₆₈ 22:59, 14. Sep. 2014 (CEST)

Vielleicht sollte man dann besser die Schrift essen als den Kefir:) --Optimum (Diskussion) 23:25, 14. Sep. 2014 (CEST)
Zurück zur Frage: Am schönsten steht es im Artikel über die Milchsäurebakterien: "(...) sind Milchsäurebakterien beinahe ubiquitär verbreitet. (...) im Darm beziehungsweise in Schleimhäuten von Säugetieren (...). Beim Menschen finden sich Milchsäurebakterien vor allem im Verdauungstrakt – inklusive Mundhöhle und Darm – sowie in der Vagina (...). Bei Schweinen, Rindern, Hunden, Mäusen, Ratten, Hühnern und weiteren Tierarten wurden sie als Bestandteil der Darmflora nachgewiesen.[2] Sogar im Magen von Honigbienen und Fröschen kommen sie vor. Weiterhin sind sie in lebenden oder sich zersetzenden Pflanzen, (...). Ähnliches gilt für Hefen. Man braucht also nur ein bisschen Glück und das in einer Umgebung, die nur ein bisschen weniger sauber sein muss als heutige Lebensmittelproduktionsstätten. Hummelhum (Diskussion) 23:05, 14. Sep. 2014 (CEST)
Im Darm von jungen Säugetieren haben die Milchsäurebakterien wahrscheinlich auch die Lactose kennengelernt, die ihnen als Nährstoff bei der Kefirproduktion dient. --Rôtkæppchen₆₈ 23:26, 14. Sep. 2014 (CEST)

@Grey Geezer: Sorry, aber, redest du wirklich von Hundis? --112.198.82.38 22:59, 15. Sep. 2014 (CEST)

Nein, Haushundis... GEEZER… nil nisi bene 23:02, 16. Sep. 2014 (CEST)

Etymologie von "durchdrehen"

Kennt jemand die Bedeutung des Wortes "durchdrehen" im Sinne von "den Verstand, die Nerven verlieren"? Ich habe mir gedacht dass es vielleicht vom Titel der Novelle Das Durchdrehen der Schraube kommt, finde dazu aber keinen Hinweis. --Cubefox (Diskussion) 21:45, 14. Sep. 2014 (CEST)

Anmerkung: Die Metapher ist sehr ähnlich konstruiert wie „ausrasten“ oder „Schraube locker haben“. Sollte das „durchdrehen“ auf die Übersetzung von Karl Nicol zurückgehen, so wäre die Frage, ob solche anderen Metaphern wiederum darauf zurückgehen. --Chricho ¹ ² ³ 21:58, 14. Sep. 2014 (CEST)
Muss neueren Datums sein. Grimm: Dt. Wörterbuch kennt durchdrehen in dem Sinne noch nicht. GEEZER… nil nisi bene 22:01, 14. Sep. 2014 (CEST)
Wo wir schon dabei sind: gibt es etymologisch eine Verbindung zum englischen "screw it!"? --Lkl22:00, 14. Sep. 2014 (CEST)
Das ist eine technische Metapher, auf deren Inhalt in jedem handelsüblichen Automobil (und nicht nur dort) der Drehzahlbegrenzer eine wirksame Antwort darstellt. Wer gar nicht Auto fährt, kann auch in dem Artikel kurz lesen, wozu (id est: wogegen) der Begrenzer da ist - eben das ist das potentiell sehr schädliche, ja sogar gefährliche "Durchdrehen" der Metapher. Hummelhum (Diskussion) 22:26, 14. Sep. 2014 (CEST)
Ich vermute, das ist eher eine andere Bezeichnung für das "Überdrehen" einer Schraube.--Optimum (Diskussion) 23:19, 14. Sep. 2014 (CEST)
Die Schraube dreht durch, wenn sie bereits überdreht ist, also das Gewinde abgeschert ist. Durchdrehen gibt es aber auch beim Fleischwolf und der Mangel. --Rôtkæppchen₆₈ 23:22, 14. Sep. 2014 (CEST)
Wenn man länger nachdenkt, ist "Durchdrehen" ja eher mit erhöhter Aktivität verbunden. Das klingt doch eher nach Motor oder durchdrehendem Rad als nach Schraube oder Mangel.--Optimum (Diskussion) 23:41, 14. Sep. 2014 (CEST)
Unter Durchdrehen versteht man normalerweise bei Fahrzeugen, dass die Antriebsräder sich drehen ohne dass das Fahrzeug sich bewegt. Durchdrehen bei den Antriebsrädern entsteht bei einem Fahrzeug, wenn die Traktion zu gering ist und die Reifen daher keinen Halt finden. Das verhindert auch ein Drehzahlbegrenzer nicht, jedes moderne Auto hat einen Drehzahlbegrenzer, trotzdem kann es z.B. auf Schnee, Eis oder matschigem Untergrund zum Durchdrehen der Räder kommen. Eine Reduktion der Drehzahl dürfte da nur insofern hilfreich sein können, dass wenn die Räder sich sehr langsam drehen die Reibung etwas höher ist (Grund: es gibt einen Übergang zwischen Haftreibung und Rollreibung, aber dafür ist eine viel niedrigere Drehgeschwindigkeit der Räder notwendig als die, die man hat, wenn man im 1. oder 2. Gang am Drehzahlbegrenzer ankommt (und mit 3./4./5. Rang anfahren dürfte recht schwer sein, jedenfalls war ich schon öfters in Autos, die wegen durchdrehenden Rädern nicht weggekommen sind, aber da hat der Fahrer in einen höheren Gang geschaltet als den 2.). Der Drehzahlbegrenzer hilft beim Auto also nicht gegen Durchdrehen, was hilft ist aber eventuell ein Allradantrieb und Differentialsperre und jedenfalls eine Traktionskontrolle (auch bekannt als Antriebsschlupfregelung (ASR)). Die Traktionskontrolle begrenzt aber nicht die Drehzahl auf einen fixen Wert, sondern reduziert die Drehzahl (der Räder) abhängig davon, wie die Traktion gerade ist. --MrBurns (Diskussion) 02:10, 17. Sep. 2014 (CEST)
Von "The Turn of the Screw" kommt es vermutlich nicht, weil der deutsche Titel "Das Durchdrehen der Schraube" wirklich erst seit der Neuübersetzung von 2001 verwendet wird. WIMRE gibt der Übersetzer Karl Ludwig Nicol (nicht identisch mit Karl Nicol) im Vorwort an, sich dabei auf genau diese Bedeutung (und auf das "Überdrehen" der Schraube) zu beziehen. --wtrsv 00:14, 15. Sep. 2014 (CEST)
Schliesse mich nach ausgiebigem Googeln den technischen Metaphern (oben) an: ausgerastet (Zahnräder), durchgedreht (Schraube, Motor), durchgeknallt (Sicherung), behämmert, schräg drauf, Schraube locker, einen Schlag weghaben (Rad, Felge), von der Rolle sein (Seilführung), verpeilt sein etc. Moderne Zeiten... GEEZER… nil nisi bene 11:24, 15. Sep. 2014 (CEST)
Das soll man erstmal alles auf seine Festplatte bekommen... --Optimum (Diskussion) 19:14, 15. Sep. 2014 (CEST)
Klapsmühle verweist hinsichtlich des Durchdrehens auf Röhricht, der wiederum zitiert s.v. Mühle Goethes Faust (1790): "Mir wird von alledem so dumm, als ging mir ein Mühlrad im Kopf herum." Rund um Wörter wie "Mühle", "Tretmühle", Treträder in den Arbeitshäusern des 17. und 18. Jh., "verwirrt", "schwerfällig im Denken" gibt es ein reiches semantisches Inventar, von dem sich einiges in der aufkommenden industriellen Revolution auf mechanische Vorgänge übertragen haben dürfte. --Aalfons (Diskussion) 11:49, 15. Sep. 2014 (CEST)
Kleine Modifikation: Durchgedreht im psychischen Sinn hat bei GBS einen frühen Nachweis 1914, im mechanischen Sinn reichen die Nachweise sehr viel weiter zurück, bis an den Beginn des 18. Jh. Vllt erweiterte sich die Bezeichnung für die mechanische Bewegung auf das psychische Problem, weil letzteres bereits ähnliche Verwandte aus dem Mühlenbereich hatte. --Aalfons (Diskussion) 12:48, 15. Sep. 2014 (CEST)
Kannst dus zitieren? Ich kanns gerade nicht nachvollziehen. --Chricho ¹ ² ³ 17:41, 15. Sep. 2014 (CEST)
Auf der GBS-Seite im Suchfeld "durchgedreht" eingeben: Seeliger, Das sterbende Dorf (München: Müller 1914), S. 186: "Sowas ist mir ja mein Lebtag noch nicht vorgekommen, der muß ja ganz durchgedreht sein." "So arg ist's gerade nicht!" lenkte Liese ein"... --Aalfons (Diskussion) 22:01, 15. Sep. 2014 (CEST)
Hier auch der Link: [48] --Cubefox (Diskussion) 21:39, 16. Sep. 2014 (CEST)
Vielleicht kann man das deshalb nicht zitieren, weil es frei erfunden ist? Mühlen (Wasser- und Wind-) drehen nicht durch wie ein Verbrennungsmotor, weil sie im Fall zu hoher Antriebsenergie (also bei extremem Hochwasser bzw Sturm) rechtzeitig gebremst und festgemacht werden.
Die Verbindung von "Mühlrad im Kopf" nach "Durchdrehen" wird oben behauptet, nicht belegt. Und ist in der Sache nicht ein bisschen plausibel. Hummelhum (Diskussion) 20:39, 15. Sep. 2014 (CEST)
Das mit der Mühle finde ich auch nicht so plausibel. Die vielen ähnlichen Wörter (wie von Geezer erwähnt) weisen darauf hin, dass es grob ums "kaputt gehen" geht, also um technische Metaphern zum "die Nerven verlieren". --Cubefox (Diskussion) 21:39, 16. Sep. 2014 (CEST)
Ich habe eine noch etwas ältere Quelle gefunden, "Etymologisches Wörterbuch der deutschen Seemannssprache" von 1902: [49] Da wird über "durchdrehen" in einem anderen Sinne (technischer Fehler beim Segeln) gesprochen, aber auch die psychologische Bedeutung erwähnt. Es wird allerdings kein etymologischer Zusammenhang zwischen beiden Bedeutungen hergestellt. --Cubefox (Diskussion) 21:39, 16. Sep. 2014 (CEST)
Danke für den Service, Cubebox. Ich zitiere Goedel hier mal: "Durchgedreht" sagt der Seemann auch von einem, der einen Sparrn zuviel hat, oder den er im Ärger für verrückt erklärt, ohne daß der Betroffene wirklich geisteskrank zu sein braucht." Der Vorbesitzer meines Exemplars hat noch angemerkt: aufgeregt, Kopf verlieren... Pastor Goedel ist zwar, was Etymologien angeht, unzuverlässig, aber eine so simple Aussage dürfte stimmen. Imho kristallisieren sich zwei mögliche, vllt auch verwandte Bedeutungen heraus: durchgedreht als "dysfunktional, kaputt", und durchgedreht als "ungehemmt, ungebremst, unkontrolliert, ungehindert". Über Analogien ist hier noch niemand hinausgekommen, daher möchte ich meinen Hut einstweilen im Ring lassen. --Aalfons (Diskussion) 22:34, 16. Sep. 2014 (CEST)